You are on page 1of 291

Telegram @unacademyplusdiscounts

Join Us on Telegram for More Such Books

https://telegram.me/unacademyplusdiscounts

Join us from the above link or search ''unacademyplusdiscounts'' in Telegram


EBD_7839
• Corporate Office : 45, 2nd Floor, Maharishi Dayanand Marg, Corner Market,
Malviya Nagar, New Delhi-110017
Tel. : 011-49842349 / 49842350

Typeset by Disha DTP Team

DISHA PUBLICATION
ALL RIGHTS RESERVED

© Copyright Publisher

No part of this publication may be reproduced in any form without prior permission of the publisher
and author. The author and the publisher do not take any legal responsibility for any errors or
misrepresentations that might have crept in. We have tried and made our best efforts to provide
accurate up-to-date information in this book.

For further information about the books from DISHA,


Log on to www.dishapublication.com or email to info@dishapublication.com
Contents
TREND ANALYSIS OF KVPY - STREAM SA (2014-2019) i-iv

KVPY SOLVED PAPER 2019 - STREAM SA 2019-1-20

KVPY SOLVED PAPER 2018 - STREAM SA 2018-1-24

MOCK TESTS

1. Mock Test - 1 1–8

2. Mock Test - 2 9–18

3. Mock Test - 3 19–26

4. Mock Test - 4 27–36

5. Mock Test - 5 37–46

6. Mock Test - 6 47–56

7. Mock Test - 7 57–66

8. Mock Test - 8 67–76

9. Mock Test - 9 77–86

10. Mock Test - 10 87–96

Mock Test Solutions (1-10) 97-220


EBD_7839
TREND ANALYSIS OF KVPY - Stream SA (2014-2019)
MATHEMATICS
2014 2015 2016 2017 2018 2019
Unit
Unit Name
No. Part Part Part Part Part Part Part Part
P-1 P-II P-1 P-II
I II I II I II I II
Sets, Functions and Number 3 (Q. 7, 2 (Q. 63,
I 5 2 3 1 6 4 1 1 (Q.13)
Systems 10, 13) 64)

4 (Q. 2,
II Algebra 3 1 2 1 1 2 3 3 (Q. 2, 4, 5) 1 (Q. 61) 1 (Q. 61)
3, 9, 12)

III Sequences and Series 2 1 1 2 1 1 (Q. 3) 2 (Q. 4, 15)

IV Lines and Triangles 1 1 1 1 4 (Q. 8, 9, 11, 14) 2 (Q. 5, 6) 1 (Q. 65)

V Quadrilaterals 1 1 1 1 1 1 (Q. 15) 1 (Q. 65) 1 (Q. 11)

VI Circles and Conic Sections 2 1 3 1 1 1 2 (1, 10) 1 (Q. 64) 1 (Q. 1)

VII Mensuration 1 3 3 1 1 2 1 1 (Q. 7) 1 (Q. 63) 1 (Q. 14)

VIII Statistics and Probability 1 1 2 (Q. 6, 12) 1 (Q. 8) 1 (Q. 62)

Permutations and
IX 3 2 1 1 (Q. 62)
Combinations

TOTAL NO. OF QUESTIONS 15 5 15 5 15 5 15 5 15 5 15 5

(i)
TREND ANALYSIS OF KVPY - Stream SA (2014-2019)
PHYSICS
2014 2015 2016 2017 2018 2019
Unit
Unit Name
No. Part Part Part Part Part Part Part Part Part Part Part Part
I II I II I II I II I II I II
Physical World and
I 1 1 1 1 1 (Q. 17)
Measurements

II Kinematics 1 1 2 3 2

III Laws of Motion 2 1 1 1 1 (Q. 67)

IV Work, Energy and Power 1 1 2 (Q. 23, 29)

V Rotational Motion 1 1 1 1 3 1 (Q. 22) 1 (Q. 66)

VI Gravitation 4

Mechanical Properties
VII 1 1 1 2 2 (Q. 18, 28) 1 (Q. 70)
of Solids and Fluids

VIII Heat and Thermodynamics 2 1 1 1 1 1 2 1 3 1 1 (Q. 27) 1 (Q. 69)

IX Oscillations and Waves 1 2

Light, Electricity & Magnetism, 8 (Q. 16, 19, 20,


X 9 2 6 2 8 1 8 2 8 1 1 (Q. 68)
Atoms & Nuclei 21, 24, 25, 26, 30)

TOTAL NO. OF QUESTIONS 15 5 15 5 15 5 15 5 15 5 15 5

(ii)

EBD_7839
TREND ANALYSIS OF KVPY - Stream SA (2014-2019)
CHEMISTRY

2014 2015 2016 2017 2018 2019


Unit
Unit Name
No.
Part Part Part Part Part Part Part Part Part Part Part Part
I II I II I II I II I II I II
Basic Concepts of Chemistry,
Structure of Atom, Chemical 6 (Q. 31, 37,
I 6 2 5 2 5 1 7 2 4 2 1 (Q. 73)
Bonding, Classification & Periodic 38, 39, 43, 44)
Properties of Elements

II States of Matter & Thermodynamics 1 1 2 1 1 1 1 2 (Q. 42, 45) 1 (Q. 72)

Equilibrium, Redox Reactions &


III 3 2 1 2 1 3 1 (Q. 41)
Chemical Kinetics

IV s & p Block Elements 1 1 1 1 2 (Q. 35, 36) 1 (Q. 71)

3 (Q. 32, 33,


V G.O.C & Hydrocarbons 5 3 4 2 5 2 5 1 5 2 1 (Q. 75)
34)

Analytical Chemistry, Biomole-


VI cules, Metals-Non Metals & Acid 2 2 2 1 (Q. 40) 1 (Q. 74)
-Base

TOTAL NO. OF QUESTIONS 15 5 15 5 15 5 15 5 15 5 15 5

(iii)
TREND ANALYSIS OF KVPY - Stream SA (2014-2019)
BIOLOGY
2014 2015 2016 2017 2018 2019
Unit
Unit Name
No. Part Part Part Part Part Part Part Part Part Part Part Part
I II I II I II I II I II I II
I Diversity of Living Organisms 1 1 1 1 1 2 (Q. 55, 59)

Anatomy and Morphology of


II 1 1 1
Living Organisms

III Plant Physiology 1 2 4 2 1 2 1 2 (Q. 52, 57) 1 (Q. 80)

6 (Q. 47, 48,


IV Animal Physiology 9 2 1 5 1 4 1 6 1 49, 53, 56,
58)

V Biochemistry and Cell Biology 1 2 1 1 2 2 2 4 1 2 (Q. 76, 78)

VI Reproduction 1 1 1 (Q. 60) 1 (Q. 77)

4 (Q. 46, 50,


VII Genetics and Evolution 2 5 1 4 1 2 1 (Q. 79)
51, 54)

VIII Biotechnology 1 1 1
IX Ecology 2 1 2 1 1

X Human Health and Microbes 1 3 1 1

TOTAL NO. OF QUESTIONS 15 5 15 5 15 5 15 5 15 5 15 5

(iv)

EBD_7839
Telegram @unacademyplusdiscounts

Join Us on Telegram for More Such Books

https://telegram.me/unacademyplusdiscounts

Join us from the above link or search ''unacademyplusdiscounts'' in Telegram


KISHORE VAIGYANIK PROTSAHAN YOJANA
SOLVED PAPER 2019 STREAM : SA
Time : 3 Hours Maximum Marks : 100
INSTRUCTIONS
1. There are 80 questions in this paper.
2. The question paper contains two parts; Part–I (1 Mark Questions) and Part–II (2 Marks
Questions). There are four sections; Mathematics, Physics, Chemistry and Biology in each part.
3. There are four options given with each question, only one of them is correct.
4. For each incorrect answer 0.25 Mark in Part–I and 0.5 Mark in Part–II will be deducted.

PART-I (1 MARK QUESTIONS)

MATHEMATICS 3. Let p(x) = x 2 + ax + b have two distinct real


roots, where a, b are real numbers. Define
1. Let ABC be an equilateral triangle with side g(x) = p(x3) for all real numbers x. Then which
length a. Let R and r denote the radii of the of the following statements are true?
circumcircle and the incircle of triangle ABC I. g has exactly two distinct real roots
respectively. Then, as a function of a, the II. g can have more than two distinct real roots
III. There exists a real number such that g(x)
R for all real x
ratio
r (a) Only I (b) Only I and III
(a) strictly increases (c) Only II (d) Only II and III
(b) strictly decreases 4. Let an, n 1, be an arithmetic progression with
first term 2 and common difference 4. Let Mn be
(c) remains constant the average of the first n terms.
(d) strictly increases for a < 1 and strictly de- 10
creases for a > 1 Then the sum M n is
2. Let b be a non-zero real number. Suppose the n 1
(a) 110 (b) 335
1
quadratic equation 2x2 + bx + = 0 has two (c) 770 (d) 1100
b 5. In a triangle ABC, BAC = 90°; AD is the alti-
distinct real roots. Then tude from A on to BC. Draw DE perpendicular to
1 5 1 5 AC and DF perpendicular to AB. Suppose
(a) b (b) b AB = 15 and BC = 25. Then the length of EF is
b 2 b 2
(a) 12 (b) 10
1
(c) b2 – 3b > –2 (d) b2 + <4 (c) 5 3 (d) 5 5
b2
EBD_7839
2 KVPY SOLVED PAPER 2019 STREAM : SA

6. The sides a, b, c of a triangle satisfy the (a) N1 and N2 are coprime


relations c2 = 2ab and a2 + c2 = 3b2. Then the (b) the HCF (Highest Common Factor) of N1
measure of BAC, in degrees, is and N2 is 55
(a) 30 (b) 45 (c) the HCF of N1 and N2 is 11
(c) 60 (d) 90 (d) the HCF of N1 and N2 is 33
7. Let N be the least positive integer such that
14. Let l > 0 be a real number, C denote a circle with
whenever a non-zero digit c is written after the
circumference l, and T denote a triangle with
last digit of N, the resulting number is divisible
perimeter l. Then
by c. The sum of the digits of N is
(a) 9 (b) 18 (a) given any positive real number , we can
choose C and T as above such that the
(c) 27 (d) 36
8. Let x1, x2 , ..., x11 be 11 distinct positive inte- Area(C )
ratio is greater than
gers. If we replace the largest of these integers Area(T )
by the median of the other 10 integers, then
(b) given any positive real number , we can
(a) the median remains the same
choose C and T as above such that the
(b) the mean increases
(c) the median decreases Area(C )
ratio is less than
(d) the mean remains the same Area(T )
9. The number of cubic polynomials P(x) satisfying
(c) given any C and T as above, the ratio
P(l) = 2, P(2) = 4, P(3) = 6, P(4) = 8 is
(a) 0 Area(C )
is independent of C and T
(b) 1 Area(T )
(c) more than one but finitely many
(d) infinitely many (d) there exist real numbers a and b such that
10. A two-digit number ab is called almost prime if for any circle C and triangle T as above, we
one obtains a two-digit prime number by
Area(C )
changing at most one of its digits a and b. must have a < <b
(For example, 18 is an almost prime number Area(T )
because 13 is a prime number). Then the number
of almost prime two-digit numbers is 15. The number of three digit numbers abc such
(a) 56 (b) 75 that the arithmetic mean of b and c and the square
of their geometric mean are equal is
(c) 87 (d) 90
(a) 9 (b) 18
11. Let P be an interior point of a convex quadrilat-
eral ABCD and K, L, M, N be the midpoints (c) 36 (d) 54
of AB, BC, CD, DA respectively. PHYSICS
If Area(PKAN) = 25, Area(PLBK) = 36, and
Area(PMDN) = 41 then Area(PLCM) is 16. Various optical processes are involved in the
(a) 20 (b) 29 formation of a rainbow. Which of the following
(c) 52 (d) 54 provides the correct order in time in which these
processes occur ?
12. The number of non-negative integer solutions of
the equations 6x + 4y + z = 200 and x + y + z = 100 (a) Refraction, total internal reflection, refraction
is (b) Total internal reflection, refraction, total
(a) 3 (b) 5 internal reflection
(c) 7 (d) Infinite (c) Total internal reflection, refraction, refraction
13. Let N1 = 255 + 1 and N2 = 165. (d) Refraction, total internal reflection, total
Then internal reflection.
KVPY SOLVED PAPER 2019 STREAM : SA 3

17. A specially designed Vernier calliper has the (a) I only


main scale least count of 1 mm. On the Vernier (b) I and II only
scale there are 10 equal divisions and they (c) I and III only
match with 11 main scale divisions. Then, the (d) II and III only
least count of the Vernier calliper is 20. Frosted glass is widely used for translucent
(a) 0.1 mm (b) 0.909 mm windows. The region where a transparent
(c) 1.1 mm (d) 0.09 mm adhesive tape is stuck over the frosted glass
18. A steel ball is dropped in a viscous liquid. The becomes transparent. The most reasonable
distance of the steel ball from the top of the explanation for this is,
liquid is shown below. The terminal velocity of (a) diffusion of adhesive glue into glass
the ball is closest to : (b) chemical reaction at adhesive tape - glass
interface
(c) refractive index of adhesive glue is close to
0.4 that of glass
(d) adhesive tape is more transparent than glass
0.3 21. Consider two equivalent, triangular hollow
prisms A and B made of thin glass plates and
Distance (m)

arranged with negligible spacing as shown in


0.2 the figure. A beam of white light is incident on
prism A from the left. Given that the refractive
index of water is inversely related to tempera-
0.1 ture, the beam to the right of prism B would
NOT appear white if

0
0 0.5 1 1.5 2 B
ght
Time (s) White li A

(a) 0.26 m/s (b) 0.33 m/s


(c) 0.45 m/s (d) 0.21 m/s (a)both prisms are filled with hot water (70 °C)
19. A student in a town in India, where the price per (b)both prisms are filled with cold water (7 °C)
unit (1 unit = 1 kW-h) of electricity is ` 5.00, (c)both prisms are empty
purchases a 1 kVA UPS (uninterrupted power (d)prism A is filled with hot water (70 °C) and
supply) battery. A day before the exam, 10 prism B with cold water (7 °C)
friends arrive to the student’s home with their 22. A ball is moving uniformly in a circular path of
laptops and all connect their laptops to the UPS. radius 1 m with a time period of 1.5 s. If the ball
Assume that each laptop has a constant power is suddenly stopped at t = 8.3 s, the magnitude
requirement of 90 W. Consider the following of the displacement of the ball with respect to
statements its position at t = 0 s is closest to
I All the 10 laptops can be powered by the (a) 1 m (b) 33 m
UPS if connected directly. (c) 3 m (d) 2 m
II All the 10 laptops can be powered if 23. A particle slides from the top of a smooth hemi-
connected using an extension box with a spherical surface of radius R which is fixed on a
3A fuse. horizontal surface. If it separates from the hemi-
III If all the 10 friends use the laptop for 5 hours, sphere at a height h from the horizontal surface
then the cost of the consumed electricity is then the speed of the particle is
about ` 22.50. (a) (2g(R h)) (b) (2g(R h))
Select the correct option with the true
statements. (c) 2gR (d) 2g h
EBD_7839
4 KVPY SOLVED PAPER 2019 STREAM : SA

24. The nuclear radius is given by R = r0A1/3, where (a) 1% of Mumbai’s water needs
r0 is constant and A is the atomic mass number. (b) 10% of Mumbai’s water needs
Then, (c) 50% of Mumbai’s water needs
(a) the nuclear mass density of U238 is twice (d) 100% of Mumbai’s water needs
that of Sn 119 29. A mass M moving with a certain speed V collides
(b) the nuclear mass density of U238 is thrice
elastically with another stationary mass m. After
that of Sn 119
the collision the masses M and m move with
(c) the nuclear mass density of U238 is the same
as that of Sn119 speeds V and v respectively. All motion is in
(d) the nuclear mass density of U238 is half that one dimension. Then
of Sn119 (a) V = V + v (b) V = V + v
25. The electrostatic energy of a nucleus of charge (c) V = (V + v)/2 (d) v = V + V
Ze is equal to kz2e2/R, where k is a constant and 30. Four rays, 1, 2, 3 and 4 are incident normally on
R is the nuclear radius. The nucleus divides into the face PQ of an isosceles prism PQR with apex
two daughter nuclei of charges Ze/2 and equal angle Q = 120°. The refractive indices of the
radii. The change in electrostatic energy in the material of the prism for the above rays 1, 2, 3
process when they are far apart is
and 4 are 1.85, 1.95, 2.05 and 2.15, respectively
(a) 0.375kz2e2/R (b) 0.125kz2e2/R
2 2 and the surrounding medium is air. Then the
(c) kz e /R (d) 0.5kz2e2/R
rays emerging from the face QR are
26. Two masses M1 and M2 carry positive charges
Q1 and Q2, respectively. They are dropped to (a) 4 only (b) 1 and 2 only
the floor in a laboratory setup from the same (c) 3 and 4 only (d) 1, 2, 3 and 4
height where there is a constant electric field
vertically upwards. M1 hits the floor before M2. CHEMISTRY
Then, 31. The hybridizations of N, C and O shown in the
(a) Q1 > Q2 (b) Q1 < Q2 following compound
(c) M1Ql > M2Q2 (d) M1Q2 > M2Q1 R
27. Which one of the following schematic graphs
best represents the variation of PV (in Joules) N==C==O
versus T (in Kelvin) of one mole of an ideal gas? respectively, are
(The dotted line represents PV = T.) (a) sp2, sp, sp2 (b) sp2, sp2, sp2
(a) (b) (c) sp2, sp, sp (d) sp, sp, sp2
32. The following compounds
PV (J) PV (J)

T (K) T (K) are


(a) geometrical isomers
(c) (d) (b) positional isomers
PV (J)
(c) optical isomers
PV (J)
(d) functional group isomers
33. The major product of the following reaction
T ( K)
T (K)
Br 1. excess alc. KOH
28. Mumbai needs 1.4 l012 litres of water annu-
Br
ally. Its effective surface area is 600 km 2 and Ph 2. NaNH 2
it receives an average rainfall of 2.4 m annually.
If 10% of this rain water is conserved it will meet 3. H3O+
approximately is
KVPY SOLVED PAPER 2019 STREAM : SA 5

H
H
(a) Ph H (b) Ph

concentration
Br
Br Br
H H
(c) Ph (d) Ph
Br H 0 time
34. IUPAC name of the following compound The order of this reaction with respect to the
reactant is
O
(a) 0
(b) 1
(c) 2
HO (d) not possible to determine from this plot
42. During the free expansion of an ideal gas in an
is
isolated chamber,
(a) 1 -hydroxycyclohex-4-en-3-one (a) internal energy remains constant
(b) 1 -hydroxycyclohex-3-en-5-one (b) internal energy decreases
(c) 3-hydroxycyclohex-5-en-1-one (c) work done on the system is negative
(d) 5-hydroxycyclohex-2-en-1 one (d) temperature increases
35. In water-gas shift reaction, hydrogen gas is 43. The number of moles of water present in a
produced from the reaction of steam with spherical water droplet of radius 1.0 cm is
(a) methane (b) coke [Given: density of water in the droplet = 1.0 g cm–3]
(c) carbon monoxide (d) carbon dioxide 2
(a) (b)
36. Treatment with lime can remove hardness of 18 27
water caused by 2
(c) 24 (d)
(a) CaCl2 (b) CaSO4 9
(c) Ca(HCO3)2 (d) CaCO3 44. Among the following, the correct statement
37. The most polarizable ion among the following is about cathode ray discharge tube is
(a) F– (b) I– (a) the electrical discharge can only be
+ observed at high pressure and at low volt-
(c) Na (d) Cl–
ages.
38. For a multi-electron atom, the highest energy
(b) in the absence of external electrical or magnetic
level among the following is field, cathode rays travel in straight lines.
(a) n = 5, l = 0, m = 0, s = +½ (c) the characteristics of cathode rays depend
(b) n = 4, l = 2, m = 0, s = +½ upon the material of electrodes.
(c) n = 4, l = 1, m = 0, s = +½ (d) the characteristics of cathode rays depend
(d) n = 5, l = 1, m = 0, s = +½ upon the gas present in the cathode ray tube.
39. The oxide which is neither acidic nor basic is 45. For a spontaneous process
(a) As 2 O 3 (b) Sb4 O10 (a) enthalpy change of the system must be
(c) N2O (d) Na2O negative.
40. The element whose salts cannot be detected by (b) entropy change of the system must be
flame test is positive.
(c) entropy change of the surrounding must be
(a) Mg (b) Na
positive.
(c) Cu (d) Sr
(d) en tr opy ch an ge of the system plus
41. The plot of concentration of a reactant vs. time
surrounding must be positive.
for a chemical reaction is shown below:
EBD_7839
6 KVPY SOLVED PAPER 2019 STREAM : SA

BIOLOGY (c) Glucose to lactate


46. Which one of the following is a CORRECT (d) Glucose to CO2 and H2O
statement about primate evolution? 53. Where are the proximal and distal convoluted
(a) Chimpanzees and gorillas evolved from tubules located within the human body?
macaques (a) Adrenal cortex (b) Adrenal medulla
(b) Humans and chimpanzees evolved from (c) Renal cortex (d) Renal medulla
gorillas 54. In a diploid organism, when the locus X is
(c) Humans, chimpanz ees and gorill as inactivated, transcription of the locus Y is
evolved from a common ancestor triggered. Based on this observation, which one
(d) Humans and gorillas evolved from of the following statements is CORRECT?
chimpanzees (a) X is dominant over Y
47. The crypts of Lieberkühn are found in which (b) X is epistatic to Y
one of the following parts of the human (c) Y is dominant over X
digestive tract?
(d) Y is epistatic to X
(a) Oesophagus (b) Small intestine
55. Which one of the following sequences
(c) Stomach (d) Rectum
represents the CORRECT taxonomical
48. Removal of the pancreas impairs the breakdown
hierarchy?
of
(a) Species, genus, family, order
(a) lipids and carbohydrates only
(b) lipids and proteins only (b) Order, genus, family, species
(c) lipids, proteins and carbohydrates (c) Species, order, genus, family
(d) proteins and carbohydrates only (d) Species, genus, order, family
49. Microscopic examination of a blood smear 56. Which one of the following organs is NOT a
reveals an abnormal increase in the number of site for the production of white blood cells?
granular cells with multiple nuclear lobes. (a) Bone marrow (b) Kidney
Which one of the following cell types has (c) Liver (d) Spleen
increased in number? 57. Which one of the following anatomical
(a) Lymphocytes structures is involved in guttation?
(b) Monocytes (a) Cuticle (b) Hydathodes
(c) Neutrophils (c) Lenticels (d) Stomata
(d) Thrombocytes 58. Which one of the following parts of the eye is
50. Which one of the following genetic phenomena affected in cataract?
is represented by the blood group AB? (a) Cornea (b) Conjunctiva
(a) Codominance (b) Dominance (c) Retina (d) Lens
(c) Overdominance (d) Semidominance 59. Which one of the following organisms is a
51. The mode of speciation mediated by
bryophyte?
geographical isolation is referred to as
(a) Liverwort (b) Volvox
(a) adaptive radiation
(c) Chlamdomonas (d) Fern
(b) allopatric speciation
(c) parapatric speciation 60. During oogenesis in mammals, the second
(d) sympatric speciation meiotic division occurs
52. Which one of the following metabolic (a) before fertilisation
conversions requires oxygen? (b) after implantation
(a) Glucose to pyruvate (c) before ovulation
(b) Glucose to CO2 and ethanol (d) after fertilisation
KVPY SOLVED PAPER 2019 STREAM : SA 7

PART-II (2 MARKS QUESTIONS)

MATHEMATICS
10 cm
61. Let a, b, c, d be distinct real numbers such that
a, b are roots of x2 – 5cx – 6d = 0 and c, d are

Laser
40 cm
roots of 30 cm 30 cm

x2 – 5ax – 6b = 0. Then b + d is
(a) 180 (b) 162
(c) 144 (d) 126
62. Let S = {1, 2, 3, ..., 100). Suppose b and c are (a) 8.13 s (b) 14.05 s
chosen at random from the set S. The probability (c) 16.27 s (d) 23.86 s
that 4x2 + bx + c has equal roots is 67. In an hour-glass approximately 100 grains of
(a) 0.001 (b) 0.004 sand fall per second (starting from rest), and it
(c) 0.007 (d) 0.01 takes 2 sec for each sand particle to reach the
63. Let be the set of positive integers. For all bottom of the hour-glass. If the average mass
n , let of each sand particle is 0.2 g then the average
fn = (n + l)1/3 – n1/3
force exerted by the falling sand on the bottom
1 of the hour-glass is close to.
and A = n : fn 1 fn
3(n 1)2 / 3 (a) 0.4 N (b) 0.8 N
Then (c) 1.2 N (d) 1.6 N
(a) A = 68. A student uses the resistance of a known
(b) A is a finite set resistor (1 ) to calibrate a voltmeter and an
(c) the complement of A in is nonempty, but ammeter using the circuits shown below. The
finite student measures the ratio of the voltage to
(d) A and its complement in are both infinite
current to be l l03 in circuit (a) and 0.999
64. A prime number p is called special if there
in circuit (b). From these measurements, the
exist primes p1, p2, p3, p4 such that
resistances (in ) of the voltmeter and ammeter
p = p1 + p2 = p3 – p4.
are found to be close to:
The number of special primes is
(a) 0
(b) 1
(c) more than one but finite 1 1
(d) infinite
65. Let ABC be a triangle in which AB = BC. Let X (a) (b)
be a point on AB such that AX : XB = AB : AX. If
AC = AX, then the measure of ABC equals (a) 102 and 10–2 (b) 103 and 10–3
(a) 18° (b) 36° (c) 10-2 and 102 (d) 10–3 and 103
(c) 54° (d) 72° 69. A hot air balloon with a payload rises in the air.
Assume that the balloon is spherical in shape
PHYSICS with diameter of 11.7 m and the mass of the
66. A water-proof laser pointer of length 10 cm balloon and the payload (without the hot air
placed in a water tank rotates about a horizontal inside) is 210 kg. Temperature and pressure of
axis passing through its center of mass in a outside air are 27 °C and 1 atm = 105 N/m2
vertical plane as shown in the figure. The time respectively. Molar mass of dry air is 30 g. The
period of rotation is 60 s. Assuming the water to temperature of the hot air inside is close to, [The
be still and no reflections from the surface of gas constant R = 8.31 J/K/mol]
the tank, the duration for which the light beam (a) 27 °C (b) 52 °C
escapes the tank in one time period is close to
(Refractive index of water =1.33) (c) 105 °C (d) 171 °C
EBD_7839
8 KVPY SOLVED PAPER 2019 STREAM : SA

70. A healthy adult of height 1.7 m has an average [Given: atomic weight of Ag = 108, F = 19,
blood pressure (BP) of 100 mm of Hg. The heart Cl = 35.5, Bi = 80, I = 127]
is typically at a height of 1.3 m from the foot. (a) F (b) Cl
Take the density of blood to be 103 kg/m3 and (c) Br (d) I
note that 100 mm of Hg is equivalent to 75. An organic compound X with molecular formula
13.3 kPa (kilo Pascals). The ratio of BP in the C6H10, when treated with HBr, forms a gem
foot region to that in the head region is close to. dibromide. The compound X upon warming with
(a) one (b) two HgSO4 and dil. H2SO4, produces a ketone which
(c) three (d) four gives a positive iodoform test. The compound
X is
CHEMISTRY
71. PbO2 is obtained from (a) (b)
(a) the reaction of PbO with HCl
(b thermal decomposition of Pb(NO3)2 at 200 °C (c) C (d)
(c) the reaction of Pb3O4 with HNO3
(d) the reaction of Pb with air at room BIOLOGY
temperature 76. A cell weighing 1 mg grows to double its initial
72. For one mole of a van der Waals gas, the mass before dividing into two daughter cells of
PV equal mass. Assuming no death, at the end of
compressibility factor Z at a fixed 100 divisions what will be the ratio of the mass
RT
of the entire population of these cells to that of
volume will certainly decrease if
[Given: “a”, “b” are standard parameters for van the mass of the Earth? Assume that mass of the
der Waals gas] Earth is 1024 kg and 210 is approximately equal
(a) “b” increases and “a” decreases at constant to 1000.
temperature (a) 10–28 (b) 10–3
(b) “b” decreases and “a” increases at constant (c) 1 (d) 103
temperature 77. Papaya is a dioecious species with XY sexual
(c) temperature increases at constant “a” and genotype for male and XX for female. What will
“b” values be the genotype of the embryos and endosperm
(d) “b” increases at constant “‘a” and temperature nuclei after double fertilization?
73. The correct statements among the following (a) 50% ovules would have XXX endosperm
i. E2s (H) > E2s (Li) > E2s (Na) > E2s (K)
and XY embryo, while the other 50% would
ii. The maximum number of electrons in the have XXY endosperm and XX embryo
shell with principal quantum number n is
(b) 100% ovules would have XXX endosperm
equal to 2n2.
and XY embryo
iii. Extra stability of half-filled subshell is due
to smaller exchange energy (c) 100% ovules would have XXY endosperm
iv. Only two electrons, irrespective of their and XX embryo
spin, may exist in the same orbital. (d) 50% ovules would have XXX endosperm
are and XX embryo, while the other 50% would
(a) i and ii (b) ii and iii have XXY endosperm and XY embryo
(c) iii and iv (d) i and iv 78. Solid and dotted lines represent the activities
74. An organic compound contains 46.78% of a of pepsin and salivary amylase enzymes of the
halogen X. When 2.00 g of this compound is digestive tract, respectively. Which one of the
heated with fuming HNO3 in the presence of following graphs best represents their activity
AgNO3, 2.21 g AgX was formed. The halogen X vs pH?
is
KVPY SOLVED PAPER 2019 STREAM : SA 9

(a) 79. If the gene pool of the locus X in the human


genome is 4, then what would be the highest
Activity possible number of genotypes in a large
population?
(a) 6 (b) 8
(c) 10 (d) 16
1 5 10
pH 80. Match the plant hormones in Column I with their
(b) primary function in Column II.
Column I Column II
Activity

P. Abscisic acid (i) Promotes disease


resistance
Q. Ethylene (ii) Maintains seed
dormancy
1 5 10
pH R. Cytokinin (iii) Promotes seed
germination
(c) S. Gibb ere llin (iv) Promotes fruit
ripening
Activity

(v) Inhibits leaf


senescence
Choose the CORRECT combination.
1 5 10 (a) P - iii, Q - iv, R - i, S - ii
pH (b) P - ii, Q - iv, R - v, S - iii
(c) P - v, Q - iii, R- ii, S - i
(d) (d) P - iv, Q - ii, R - iii, S - v
Activity

1 5 10
pH

ANS WER KEYS


Part-I Part-II
1 (c) 11 (c) 21 (d) 31 (a) 41 (a) 51 (b) 61 (c) 71 (c)
2 (c) 12 (c) 22 (d) 32 (d) 42 (a) 52 (d) 62 (a) 72 (b)
3 (b) 13 (d) 23 (a) 33 (a) 43 (b) 53 (c) 63 (a) 73 (a)
4 (a) 14 (a) 24 (c) 34 (d) 44 (b) 54 (d) 64 (b) 74 (c)
5 (a) 15 (b) 25 (a) 35 (c) 45 (d) 55 (a) 65 (b) 75 (d)
6 (b) 16 (a) 26 (d) 36 (c) 46 (c) 56 (b) 66 (c) 76 (c)
7 (a) 17 (a) 27 (a) 37 (b) 47 (b) 57 (b) 67 (a) 77 (d)
8 (c) 18 (b) 28 (b) 38 (d) 48 (c) 58 (d) 68 (b) 78 (a)
9 (a) 19 (c) 29 (d) 39 (c) 49 (c) 59 (a) 69 (c) 79 (c)
10 (d) 20 (c) 30 (c) 40 (a) 50 (a) 60 (d) 70 (c) 80 (b)
EBD_7839
10 KVPY SOLVED PAPER 2019 STREAM : SA

HINTS & SOLUTIONS

PART-I g(x) = x6 + ax3 + b and it is even degree


polynomial. So, g(x) x R
MATHEMATICS
Hence, I and III are correct.
1. (c) In equilateral triangle centroid (G) and 4. (a) a1 = 2, d = 4
circumcentre are coincide n
A 2a1 (n 1)d
r=2 2
Mn 2(n 1) 2 2n
r = GD, R = AG n
a a
AG : GD = 2 : 1 10 10
G Mn 2 n 110
R n 1 n 1
2
r B D C 5. (a) Here, BAC is a right angle triangle
2. (c) D > 0 (for real roots)
B
3
1 b 8
b2 4 2 0 0 D
b b F

(b 2)(b2 2b 4) A
0 1.5
b E C
0 3
b ( ,0) (2, ) AB = 15 & BC = 25

Clearly options A and B are wrong AC BC 2 AB 2 20


Let f(b) = b2 – 3b 1 1
Area of ABC BC. AD AB.AC
range of f(b) when b (– , 0) (2, ) 2 2
= (f(2), ) = (–2, ) BC. AD AB. AC
b – 3b > – 2 is correct
2
25(AD) 15(20)
So, b (– , 0) (2, ) is subset of solution AD = 12
set of b2 – 3b + 2 > 0 AEDF is rectangle then, AD = EF = 12
6. (b) c2 = 2ab ...(i)
2 1 a2 + c2 = 3b2 ...(ii)
Also D is wrong as b (0, )
b2 Substitute (i) in (ii) equation,
3. (b) Let be the roots of p(x) = 0, then we get, a2 + 2ab = 3b2 A
p( x ) ( x )( x ) (a + b)2 + 4b2 2a
a
a = b, c = 2a
g ( x) ( x3 )( x3 )
A = 45° = B
C
Let a13 and b13 and C = 90° a B
7. (a) Sum of digits of N should be divisible by 9.
g ( x) ( x3 a13 )( x3 b13 ) In this case, the resulting number will always
g(x) = (x – a1)(x – b1) be divisible if c = 1, 2, 3, 5, 6 and 9.
To make the resulting number divisible by 4,
x2 a1x a12 x2 b1x b12
7 and 8 as well N should be a multiple of
D 0 D 0 LCM of (4, 7, 8) i.e. 56.
For exactly two values of x, g(x) = 0
KVPY SOLVED PAPER 2019 STREAM : SA 11

Smallest positive integer which is multiple 12. (c) x + y + z = 100 ...(i) and 6x + 4y + z = 200 ...(ii)
of 56 and whose sum of digits is a multiple From eqn. (i) – (ii)
of 9 is 504. 5x + 3y = 100 ...(iii)
N will be 504. No. of ordered pair (x, y) satisfying eqn. (iii)
Sum of digits = 9. are (2, 30), (5,25), (8, 20), (11, 15) (14, 10),
8. (c) Let x1 < x2 < ... < x11 (17, 5), (20, 0) and for (x, y, z) using (i)
x5 x6 equation
Median of x1, x2 ,..., x10 (x, y, z) {(2, 30, 68), (5, 25, 70), (80, 20, 72)
2
x5 x6 (11, 15, 74), (14, 10, 76), (17, 5, 78), (20, 20, 80)}
Now replace x 11 by and then No. of non negative integral solution = 7
2
arrange in order 13. (d) Given,
So, new set of numbers are N1 = 255 + 1, N2 = 165 = 5 × 3 × 11
x x N1 = (25)11 + 1 = (32)11 + 1 = (33 – 1)11 + 1
x1, x2, x3, x4, x5, 5 6 , x6, x7, x8, x9, x10, = 11C0 (33)11 – 11C1(33)32 + ...+ 11C1033 –
2
11C + 1
x5 x6 11
Hence, median is x6 N1 is divisible by 33
2
median decreases Unit digit of N1 is 9
9. (a) Let P(x) – 2x = a(x – ) (x – ) (x – ) (x – r) N1 is not divisible by 5.
where r {1, 2, 3, 4} and r are HCF of N1 and N2 = 33
distinct then, for any combination of 14. (a) Let radius of circle be r and sides of be a,
r = 0, so P(x) = 2x b, c
no cubic polynomial possible. Circumference of circle = l = perimeter of
10. (d) Total numbers of two digit number = 90 triangle
In each row from 11 to 19.
2 r a b c r
20 to 29, 30 to 39,...,90 to 99 i.e. every set of 2
form {10 a + b; a = 1, 2, ....9; b = 0, 1, 2, ....., 9}
2 2
has atleast one prime number so each
Area of circle (C) = r 2
number can be reduce in to prime number 2 4
by changing atmost one digit.
Hence, required number of almost prime area of (T) = s( s a)(s b)( s c)
two-digit number is 90. where 2s = a + b + c = l.
11. (c) M C A.M. G.M.
D
1
( s a) ( s b) (s c )
N P L (s a)(s b)(s c )) 3
3
3
s
A K B ( s a )( s b )(s c )
3
Let, 1 = Area of PAK = area of PBK
[ AK = BK] 82 2
Similarly, 2 = Ar( PBL) = Ar( PCL) s( s a )( s b)( s c)
3 3 12 3
3 = Ar( PMC) = Ar( PMD)
4 = Ar( PMN) = Ar( PNA) 2
Hence, 1 + 4 = 25 ...(i)
Area of (C) 4
1 + 2 = 36 ...(ii) 2
Area of (T)
4 + 3 = 41 ...(iii)
(ii) + (iii) – (i) 2 + 3 = 52 12 3
Area (PLCM) = 52
EBD_7839
12 KVPY SOLVED PAPER 2019 STREAM : SA

18. (b) Terminal speed is the maximum speed


Area of (C) 3 3 attained by an object as it falls through a
Area (T) fluid. When the object attains terminal speed,
Hence, for given positive real number , we the net force on the object is zero.
can choose C and T such that the ratio of So acceleration a = 0
area of circle C to area of triangle T is greater If we calculate the slope of given distance
than . time graph between 0.3 m and 0.4 m then
15. (b) It is given that slope has approximately constant value, as
the steel ball acquires constant terminal
b c 1 1 1 speed in liquid some time after falling into it.
bc b c
2 2 2 4
0.4 0.3 1
(2b – 1) (2c – 1) = 1 Terminal speed = m/s =
2b – 1 = 1 and 2c – 1 = 1 or 2b – 1 = –1 and
0.3 3
2c – 1 = –1. = 0.33 m/s
b = 1 and c = 1 or b = 0 and c = 0 19. (c) Power delivered by the UPS battery is
Numbers can be of form a11 to a00 1kVA i.e., 1000 V.A = 1000W
where a = 1, 2, 3...,9 When all the laptops connected directly to
Number of three digit numbers = 18 UPS then total power requirement 90 10
= 900W,
PHYSICS
So battery (UPS) can provide power to all
16. (a) In primary rainbow, two refraction and one laptops.
TIR If all laptops are used for 5 hours, then cost
(1) Refraction of incident ray (2) TIR of electricity consumed as the cost of
(3) Again refraction when rays come out of liq- electricity is ` 5.00 per unit,
uid drops
900 5 3600
= 5 = ` 22.5
(1) 3.6 10 6
20. (c) Frosted glass is produced by coating on the
(2) surface and on its surface refractive index is
different.
Now, as a transparent adhesive tape is stuck
on the glass surface, the refractive index of
(3) the upper coating is adjusted to be same as
that of glass, so it becomes transparent.
21. (d) If refractive index of both prism parts are
In secondary rainbow, two refraction and
same, then the effects of dispersion are
two TIR.
cancelled out and white light will appear as
17. (a) From question, white light. Since R.I. of both parts will not
11 be same when parts are filled with water at
1 VSD = = 1.1 mm different temperatures, and given that the
10
refractive index of water is inversely related
1 MSD = 1 mm to temperature, hence white light will not
Least count of vernier calliper come out as white light.
L.C. = 1 VSD – 1 MSD 22. (d) Since time period is 1.5 sec, so ball takes
LC = 1.1 mm – 1 mm = 0.1 mm approximately 5.5 round in 8.3 seconds.
Displacement in 5 round = 0
KVPY SOLVED PAPER 2019 STREAM : SA 13

and in remaining 1/2 round, displacement


4 3 4 3 R
= 2R = 2m R =2 R0 i.e. R0 =
3 3 (2)1/3
Hence total displacement in 8.3 s is
approximately 2m [By volume conservation]
Since total time is 8.3 seconds so final Both nuclei separate to large distance,
remaining time is 8.3 –7.5 = 0.8 seconds So, final, or total energy
2 2 Ze
2
= t= 0.8 = 2 K
1.5 T 2 2kZ 2 e2 1/3
Ef = 2
R0 4R
16
= rad. = angle moved in last 0.8 sec
15
kZ 2 e 2 1 kZ 2 e 2
, so displacement 2R = 2m = 0.63
R 21/3 R
Change in electrostatic energy
23. (a)
kZ 2 e 2 kZ 2 e 2
= Ei – Ef = [1 – 0.63] = 0.37
R R
26. (d) If aM1 aM 2 then M1 hits the floor before M2

M1 g Q1 E M 2 g Q2 E
Block leaves contact at point P. i.e.
M1 M2
Using W.E. theorem (A to P)
Q1E Q2 E Q2 E Q1E
1 g– g >
mg(R – h) = mv2 M1 M2
2 M2 M1
M1Q2 > M2Q1
v= 2 g ( R h)
27. (a) For ideal gas,
24. (c) All nuclei have nearly the same mass PV = nRT
density. or, PV = RT [ n = 1 mole given]
Density of nucleus (approx) Slope of PV Vs T graph is R
M AmP AmP PV = 8.314 T
= So with respect to PV = T
V 4 3 4 3
R r0 A
3 3 PV = 8.314 T is having more slope so, graph
(a) correctly depicts PV versus T graph.
3mP 28. (b) 10% of total rain water is conserved.
= = constant (independent of mass
4 r03 Volume of water received annually
no. A) 10 10
V=A h = 600 2.4 = 1.4 108 m3
25. (a) Initially, electrostatic energy of nucleus = Ei 100 100
kZ 2e 2 [ A = 600 km2 = 600 106 m2 and h = 2.4 m]
= Mumbai needs = 1.4 1012 litres water
R
annually
Now it breaks into two nuclei with radii R0
= 1.4 109 m3
each
EBD_7839
14 KVPY SOLVED PAPER 2019 STREAM : SA

For all rays angle of incidence on PR is 30°.


1.4 108
% of water received = 100 For TIR at PR
1.4 109
= 10% 1 1
sin 1
< 30° or, < sin 30°
i.e., 10% Mumbai’s water needs.
29. (d) According to question, all motion is in one
dimension. 1 1
or, >2
2
M V m ; M V m
Before rest After So rays 3 ( = 2.05) and 4 ( = 2.15) will
collision collision emerge out.
For elastic collision velocities after collision
CHEMISTRY
(m1 m2 ) 2m2 31. (a) R—N==C==O
V1 = u1 u2
(m1 m2 ) m1 m2 Nitrogen has 2 sigma bond and a lone pair of
electron hybridisation is sp2. Carbon has 2
(m2 m1) 2m1 sigma bond hybridisation is sp. Oxygen
V2 = u u
(m1 m2 ) 2 m1 m2 1 has 1 sigma bond and two lone pair of
electron hybridisation is sp2.
(M m) NOTE: -bonding electron does not take part in
or, V = V ...(i)
(M m) hybridisation.
32. (d) an d
2M
and, v = V ...(ii) compounds are functional group isomers.
( M m) Functional group isomers are these the
Subtracting (i) – (ii) isomers which have same molecular formula
[( M m) 2M ]V but different functional group.
V v= 33. (a)
( M m)
Br
M m Br Excess alc. KOH Br NaNH 2
V v= V V – v = –V Ph Ph +
M m H 3O
V +V=v Ph—C CH
30. (c) For apex Q = 120° and normal incidence 34. (d)
on PQ. For a ray to pass through QR it will O
get total internal reflection on PR i.e., ray 1
6 2
will not directly go to QR.
5
3
Q HO 4
5-Hydroxycyclohex-2-en-1-one
120°
35. (c) In water gas shift reaction carbon monoxide
reacts with steam at high temperature to
produce carbondioxide and hydrogen.
36. (c) Lime can remove temporary hardness of water
which is caused due to bicarbonate salt of
90° 30° Ca and Mg.
30° 60° 30° 37. (b) As the size of anion increases polarizibility
P R
of anion increase. Among given anions I–
has maximum size.
KVPY SOLVED PAPER 2019 STREAM : SA 15

38. (d) According to (n + l) rule, maximum is the BIOLOGY


(n + l) value higher is the energy level. If two
level has same (n + l) value then the level 46. (c) According to the new genetic research-when
which has higher value of n will have higher combined with known fossils-the lineage that
energy. led to humans, chimps, and gorillas evolved
39. (c) As2O3 is amphoteric oxide. It reacts with both from a common ancestor about 10 million
acids as well base. years ago.
Sb4O10 is acidic oxide while Na2O is basic 47. (b) The crypts of Lieberkuhn are simple, tubular
oxide. N2O is neutral oxide which neither react glands which occur throughout the small
with acid nor with base. intestine between the villi and secrete
enzymes as well as mucus.
40. (a) Magnesium salts cannot detected by flame
test because magnesium has high ionisation 48. (c) The pancreas is a glandular organ. It is the
enthalpy. Flame unable to provide sufficient part of the digestive system located in the
energy to excite electron to next higher abdomen and produces insulin and other
energy level. important enzymes and hormones that help
to break down the food. The enzymes include
41. (a) The rate of zero order reaction is independent
trypsin and chymotrypsin to digest proteins,
of reactant concentration. As the reaction
amylase to break down carbohydrates and
proceeds reactant concentration decrease
lipase, to break down fats into fatty acids
with respect to time.
and cholesterol.
42. (a) Free expansion of an ideal gas in an isolated
49. (c) The correct answer will be Neutrophils. In
chamber is isothermal process. Internal
this type of granules, granulocytes has very
energy remains constant durin g free
tiny granules and nucleus is multi lobed with
expansion of an ideal gas.
lobed connected by thin strands of nuclear
43. (b) Volume of a spherical water droplet material.
4 3 4 50. (a) Codominance occurs when both alleles
= r (1)3
3 3 show dominance, as in the case of the AB
4 blood type (IAIB) in humans. It is a relationship
V= cm3 between two versions of a gene. Individuals
3
Mass of spherical water droplet receive one version of a gene, called an allele,
from each parent. If the alleles are different,
4 4
= p×V= 1 g the dominant allele usually will be expressed,
3 3 while the effect of the other allele, called
18 g of water= 1 mol recessive, is masked.
no. of moles of water in a spherical water
51. (b) Allopatric speciation, also referred to as
droplet
geographic speciation or its earlier name, the
4 2 dumbbell model, is a mode of speciation that
=
3 18 27 occurs when biological populations of the
44. (b) Cathode rays are independent of the nature same species become isolated from each
of the gas and electrodes placed in discharge other to an extent that prevents or interferes
tube. They travel in a straight line in the with gene flow.
absence of external electrical or magnetic 52. (d) Aerobic respiration uses oxygen to break
field. down glucose, amino acids and fatty acids
45. (d) For a spontaneous process, and is the main way the body generates
S = ( Ssystem + Ssurrounding) > 0 adenosine triphosphate (ATP), which
supplies energy to the muscles.The products
of this process are carbon dioxide and water.
EBD_7839
16 KVPY SOLVED PAPER 2019 STREAM : SA

53. (c) The renal tubule is a long and convoluted Volvox, Chlamydomonas are grouped under
structure that emerges from the glomerulus algae and Ferns are a group of about 20,000
and can be divided into three parts based on species of plants in the division Pteridophyta.
function. The first part is called the proximal 60. (d) A primary oocyte begins the first meiotic
convoluted tubule (PCT) due to its proximity division, but then arrests until later in life
to the glomerulus which stays in the renal when it will finish this division in a
cortex. The second part is called the loop of developing follicle. This results in a
Henle, or nephritic loop, because it forms a secondary oocyte, which will complete
loop (with descending and ascending limbs) meiosis if it is fertilized. So, the second
that goes through the renal medulla. The third meiotic division occurs after fertilisation.
part of the renal tubule is called the distal
PART-II
convoluted tubule (DCT) and this part is also
restricted to the renal cortex. MATHEMATICS
54. (d) A gene is said to be epistatic when it's 61. (c) Since, a and b are the roots of the equation
presence suppresses the effect of a gene at x2 – 5cx – 6d = 0 ...(i)
another locus, so here X is suppressing Y. Then, a + b = 5c and ab = –6d
Hence, X is epistatic to Y. Since, c and d are the roots of the
55. (a) Taxonomic hierarchy is a sequence of equation
categories in a decreasing or increasing order x2 – 5ax – 6b = 0 ...(ii)
from kingdom to species and vice versa. Then, c + d = 5a and cd = –6b
Kingdom is the highest rank followed by Now, a + b + c + d = 5 (a + c)
division, class, order, family, genus and (a + c) + (b + d) = 5(a + c)
species. (b + d) = 4 (a + c)
56. (b) In the human adult, the bone marrow And, (ab) (cd) = (– 6d) (–6b)
produces all of the red blood cells, 60-70 (ac) (bd) = 36bd
percent of the white cells (i.e., the ac = 36
granulocytes), and all of the platelets. The Since, a is the root of eqn. (i) and c is the
reticuloendothelial tissues of the spleen, liver, root of the eqn. (ii), then
lymph nodes, and other organs produce the a2 – 5ac – 6d = 0 ...(iii)
monocytes (4-8 percent of the white cells). c2 – 5ac – 6b = 0 ...(iv)
57. (b) Hydathodes are involved in the process of From (iii) + (iv),
guttation, in which positive xylem pressure (a + c)2 – 12 ac – 6(b + d) = 0
(due to root pressure) causes liquid to exude (a + c)2 – 12 × 36 – 6 × 4 (a + c) = 0
from the pores. It mainly occur in the leaves [ ac = 36 and (b + d) = 4 (a + c)]
of submerged aquatic plants. (a + c)2 – 24 (a + c) – 432 = 0
58. (d) A cataract is a clouding of the normally clear a + c = 36 or a + c = –12
lens of an eye. It often develop slowly and when a + c = 36, then b + d = 4 (a + c)
can affect one or both eyes. Symptoms may = 4 × 36 = 144
include faded colors, blurry or double vision, Hence, b + d = 144.
halos around light, trouble with bright lights, 62. (a) For equal r oots di scri mi n an t D = 0
and trouble seeing at night. b2 – 16c = 0.
2
b =4×4×c
59. (a) Bryophytes are a group of plant species that
reproduce via. spores rather than flowers or c has to be perfect square.
seeds. It is consisting of three divisions of Then, c can be any perfect square numbers
non-vascular land plants (embryophytes): the from the given sets and the perfect square
liverworts, hornworts and mosses. number in the set S =
= 1, 4, 9, 16, 25, 36, 49, 64, 81, 100
KVPY SOLVED PAPER 2019 STREAM : SA 17

Number of ordered pair (b, c) will be 10 Ratio of length of sides of triangle > 0
Required probability
5 1
10 0
= 0.001 2
100 100
( AB ) 2 ( BC ) 2 ( AC ) 2
63. (a) f n (n 1)1/3 n1/3 Vn N cos
2( AB )( BC )
Rationalising fn, we get
1 2
fn 2
cos
(n 1)2/3 (n)1/3 (n 1) 2/3 n 2/3 2
1 1 5 1
Clearly, 2/3
fn 2/3 ...(i) cos 36
3(n 1) 3n 4
1 PHYSICS
fn 1 2/3 ...(ii)
3(n 1) 66. (c) Light will come out when the angle is less
than critical angle ' C '
1
fn 1 fn n N
3(n 1)2/3
10 cm
Hence, A = N
C C
64. (b) If none, of p1, p2, p3, p4 is 2.
All p1, p2, p3, p4 are odd
40 cm

r
Since, p is also a prime number.

se
La
30 cm 30 cm
p1 + p2 + and p3 – p4 are both odd and
hence cannot be prime.
One of p1 or p2 (say p1) and p4 must be 2.
p = 2 + p2 = p3 – 2.
Above equation is satisfied only if
1
p = 5, p2 = 3 and p3 = 7. Using
Hence, only one special prime exists. sinc
65. (b) Let ABC be a triangle, where AB = BC
1 3
sin c = [ = 1.33 given]
A 1.33 4 water

c = 50° (approx)
X
C
t = 2c [ = ]
AB t
B
It is given that, 2 50
AX AB 1 2c 2 50 180
(say) t= =
180 2
XB AX
T
2
AX AB and XB AX AB
2 2 50 60
AB AX XB AB AB =
2 180
2 1 5 t = 16.67 s 16.27 s
1 0
2
EBD_7839
18 KVPY SOLVED PAPER 2019 STREAM : SA

67. (a) Velocity of sand particle just before striking 69. (c) Here, mass of the balloon and payload, m
the bottom = 210 kg
v = u + at v = 0 + 10 2 = 20 m/s Diameter of the balloon, d = 2r = 11.7 m
[ u = 0 and t = 2s given] Air pressure (outside), P = 105 N/m2
Initial momentum, pi = mV = (0.2 10–3) 20 Temperature of outside air, T0 = 27°C = 300 k
Final momentum, pf = 0 in
= density of air inside the balloon
[ final velocity = 0] o
= density of air outside the balloon
| p| = Pi Pf = 4 10–3 k m/s 210g + inVg = oVg
| p| M
Rate of change of momentum = force 210 210
t – =
exerted by falling sand
o in
V 4 3 V
r
3
| p| 4 10 3 100
favg = n =
t 1 PM PM 210 3
= [ PV = nRT]
or, favg = 0.4 N RT0 RTin 4 r3
68. (b) case – (a)
PM 1 1 210 3
I =
R T0 Tin 4 r3
1
1 1 210 3 R
=
T0 Tin P M 4 r3
I
Let, RV = resistance of voltmeter and
210× 3×8.31× 7 × (2) 3
RA = resistance of ammeter = 0.0007
105 × 30× 4× 22× (11.7)3
V IRv
= = RV(1 + RA) = 1000
A I 1 1
= – 0.0007
1 RA Tin 300
or, RV(1 + RA) = 1000 ...(i) or, Tin 107.22 °C
case – (b) 70. (c) Given: Height of man = 1.7 m
I
A
Head

1
A

Heart
I
1 + RV 1.7 m
I 1.3 m

I
Rv
V 1 Rv Rv Feet
= = = 0.999
A I Rv 1

RV = 0.999 (1 + RV) RV = 999 103 Height from heart to feet = 1.3 m


Putting this value of RV in eqn (i) and Density of blood = 103 kg/m3
solving, we get Pressure at the heart level
RA = 10 3 = 100 mm of Hg = 13.3 kPa
KVPY SOLVED PAPER 2019 STREAM : SA 19

Total pressure on the feet of man 46.78 2 x


Pfoot = Pheart + gh 2.21 100 108 x
= 13.3 + 103 10 1.3 = 26.3 kPa
x
P hea d = P hea rt – gh = 13.3 – 10 3 10 0.42335 =
(1.7 – 1.3) = 9.3 kPa 108 x
0.42335(108 + x) = x
Pfoot 26.3
= 3 45.7218 + 0.42335x = x
Phead 9.3
x = 79.3 80
Halogen present in organic compound is
CHEMISTRY
Bromine.
71. (c) Pb3O4 + 4HNO3 2Pb(NO3)2 + PbO2 + 2H2O 75. (d)
PVm
72. (b) Compressibility factor Z = Br
RT HBr

PVm Vm a Br
Z=
RT Vm b RTVm (X)
HgSO4

1 a H2SO4
= b ... (i) O H
1 RTVm
Vm
Considering the equation (i) the value of –
NaOH Na+O
compressibility factor Z decreases as the I2 + CHI3
value of ‘b’ decreases and the value of O O
compressibility factor Z decreases as the
value of ‘a’ increases at constant temperature BIOLOGY
and fixed volume.
76. (c) Division in the cell is calculated by 2n so,
73. (a) Correct statements are (i) and (ii) only. Extra
stability of half filled subshell is due to After 100th division No. of cells = 2100
greater exchange energy. In an orbital only Total Numbers of cells = 2
two electrons of opposite spin may exist. Mass of one cell = 1 mg = 10–6 kg.
74. (c) Carius method of estimation of halogen: In Total mass of cells = 2100 × 10–6 kg
this method a known weight of organic { 210 = 103}
compound is heated with fuming nitric acid 3 10 –6
= (10 ) × 10 kg = 10 kg 24
in presence of AgNO3. Halogen forms silver
halide. Carbon and h ydrogen of th e 1024
compound are oxidised to CO2 and H2O. Ratio of mass = 1
% of Halogen (X) 1024

At. wt . of X × wt. of AgX ×100 77. (d) After double fertilization when male gametes
= Mol. wt. of AgX × wt. of organic compound enter in embryo sac and fused with XX of
female and XY of male then the genotypes,
Let the atomic weight of the halogen (X) = x
we receive i.e., 50% ovules having XXX
the atomic weight of AgX = 108 + x
Substituting the value in the above equation endosperm and XX embryo, while the other
we get, 50% would have XXY endosperm and XY
embryo.
x 2.21 100
46.78 = 108 x 2
EBD_7839
20 KVPY SOLVED PAPER 2019 STREAM : SA

78. (a) In the graph (A) indicating activity of pepsin Formula – for given gene alleles
at low pH i.e., 2.8. It will be highest and Total number = n
similarly activity of salivary amylase will be
n
highest at the pH of 6.8. Possibel genotype = n +1
(B) is representing minimum activity and 2
(C) and (D) graphs show constant activity 4
of both enzyme at increasing pH. 4 1
2
79. (c) For locus 'x' total gene = 4 = 2 (5)
For example a, b, c and d = 2 × 5 = 10
Possible genotype 80. (b) ABA Maintain seed dormancy
Ethylene Promote fruit ripening
aa bb cc dd
Cytokinin Inhibit leaf sensesence
ab bc cd Gibberellin Promotes seed germination
ac bd
ad
Telegram @unacademyplusdiscounts

Join Us on Telegram for More Such Books

https://telegram.me/unacademyplusdiscounts

Join us from the above link or search ''unacademyplusdiscounts'' in Telegram


KISHORE VAIGYANIK PROTSAHAN YOJANA
SOLVED PAPER 2018 STREAM : SA
Time : 3 Hours Maximum Marks : 100
INSTRUCTIONS
1. There are 80 questions in this paper.
2. The question paper contains two parts; Part–I (1 Mark Questions) and Part–II (2 Marks
Questions). There are four sections; Mathematics, Physics, Chemistry and Biology in each part.
3. There are four options given with each question, only one of them is correct.
4. For each incorrect answer 0.25 Mark in Part–I and 0.5 Mark in Part–II will be deducted.

PART-I (1 MARK QUESTIONS)

MATHEMATICS Then,
(a) both I and II are true
1. The number of pairs (a, b) of positive real (b) both I and II are false
numbers satisfying a4 + b4 < 1 and a2 + b2 > 1 is (c) I is true and II is false
(a) 0 (b) 1 (d) I is false and II is true
(c) 2 (d) More than 2 5. The number of polynomials p(x) with integer
coefficients such that curve y = p(x) passes
2. The number of real roots of the polynomial through (2, 2) and (4, 5) is
equation x4 – x2 + 2x – 1 = 0 is (a) 0
(a) 0 (b) 2 (c) 3 (d) 4 (b) 1
3. Suppose the sum of the first m terms of an (c) more than 1 but finite
arithmetic progression is n and the sum of its (d) infinite
first n terms is m, where m ¹ n. Then, the sum of 6. The median of all 4-digit numbers that are
the first (m + n) terms of the arithmetic progres- divisible by 7 is
sion is (a) 5797 (b) 5498.5 (c) 5499.5 (d) 5490
7. A solid hemisphere is attached to the top of a
(a) 1 – mn (b) mn – 5 cylinder, having the same radius as that of the
(c) – (m + n) (d) m + n cylinder. If the height of the cylinder were
4. Consider the following two statements: doubled (keeping both radii fixed), the volume
I. Any pair of consistent linear equations in of the entire system would have increased by
two variables must have a unique solution. 50%. By what percentage would the volume have
increased if the radii of the hemisphere and the
II. There do not exist two consecutive integers, cylinder were doubled (keeping the height fixed)?
the sum of whose squares is 365. (a) 300% (b) 400% (c) 500% (d) 600%
EBD_7839
2 KVPY SOLVED PAPER 2018 STREAM : SA

8. Consider a DPQR in which the relation 13. Let S be the sum of the digits of the number
QR2 + PR2 = 5 PQ2 holds. Let G be the points of 152 × 518 in base 10. Then,
intersection of medians PM and QN. Then (a) S < 6 (b) 6 £ S < 140
ÐQGM is always
(c) 140 £ S < 148 (d) S ³ 148
(a) less than 45°
14. Let PQR be an acute-angled triangle in which
(b) obtuse PQ < QR From the vertex Q draw the altitude
(c) a right angle QQ1, the angle bisector QQ2 and the median QQ3,
(d) acute and larger than 45° with Q1, Q2, Q3 lying on PR. Then,
9. Let a, b, c be the side-lengths of a triangle and (a) PQ1 < PQ2 < PQ3 (b) PQ2 < PQ1 < PQ3
l, m, n be the lengths of its medians. Put (c) PQ1 < PQ3 < PQ2 (d) PQ3 < PQ1 < PQ2
l+m+n 15. All the vertices of a rectangle are of the form
K= . (a, b) with a, b integers satisfying the equation
a+ b+ c
(a – 8)2 – (b – 7 )2 = 5. Then, the perimeter of the
Then, as a, b, c vary, K can assume every value rectangle is
in the interval
(a) 20 (b) 22 (c) 24 (d) 26
æ 1 2ö æ 1 4ö
(a) çè , ÷ø (b) çè , ÷ø PHYSICS
4 3 2 5
16. A block of wood is floating on water at 0°C with
æ3 ö æ 4 5ö volume V0 above water. When the temperature
(c) çè ,1÷ø (d) çè , ÷ø
4 5 4 of water increases from 0 to 10°C, the change in
10. Let x 0, y 0 be fixed real numbers such that the volume of the block that is above water is
x02 + y02 > 1. If x, y are arbitrary real numbers best described schematically by the graph.
such that x2 + y2 £ 1, then the minimum value of
(x – x0)2 + (y – y0)2 is

( )
2
(a) x 20 + y 20 – 1 (b) x 20 + y 02 – 1 (a) (b)

(| x 0 | +| y 0 | –1) (| x 0 | +| y0 |)
2 2
(c) (d) –1

11. Let PQR be a triangle is which PQ = 3. From the


vertex R, draw the altitude RS to meet PQ at S.
(c) (d)
Assume that RS = 3 and PS = QR. Then, PR
equals
(a) 5 (b) 6 (c) 7 (d) 8 17. A very large block of ice of the size of a volley-
ball court and of uniform thickness of 8 m is
12. A 100 mark examination was administered to a floating on water. A person standing near its edge
class of 50 students. Despite only integer marks wishes to fetch a bucketful of water using a rope.
being given, the average score of the class was The smallest length of rope required for this is
47.5. Then, the maximum number of students who about
could get marks more than the class average is
(a) 3.6 m (b) 1.8 m (c) 0.9 m (d) 0.4 m
(a) 25 (b) 35 (c) 45 (d) 49
KVPY SOLVED PAPER 2018 STREAM : SA 3

18. A box filled with water has a small hole on its 22. One can define an alpha-volt (a-V) to be the
side near the bottom. It is dropped from the top energy acquired by an a-particle when it is
of a tower. As it falls, a camera attached on the
accelerated by a potential of 1 V. For this
side of the box records the shape of the water
stream coming out of the hole. The resulting video problem, you may take a proton to be 2000 times
will show heavier than an electron. Then,
(a) the water coming down forming a parabolic (a) 1 a- V = 1 eV/4000 (b) 1 a- V = 2 eV
stream (c) 1 a- V = 8000 eV (d) 1 a- V = 1 eV
(b) the water going up forming a parabolic
23. In a particle accelerator, a current of 500 mA is
stream
carried by a proton beam in which each proton
(c) the water coming out in a straight line
has a speed of 3 × 107 m/s. The cross-sectional
(d) no water coming out
area of the beam is 1.50 mm2. The charge density
19. An earthen pitcher used in summer cools water in
it essentially by evaporation of water from its in this beam (in C/m3) is close to
porous surface. If a pitcher carries 4 kg of water (a) 10–8 (b) 10–7 (c) 10–6 (d) 10–5
and the rate of evaporation is 20 g per hour, tem- 24. Which of the following is not true about the
perature of water in it decreases by DT in two total lunar eclipse?
hours. The value of DT is close to (ratio of latent
of evaporation to specific heat of water is 540°C) (a) A lunar eclipse can occur on a new moon
(a) 2.7°C (b) 4.2°C (c) 5.4°C (d) 10.8°C and full moon day
20. Two plane mirrors are kept on a horizontal table (b) The lunar eclipse would occur roughly
making an angle q with each other as shown every month, if the orbits of earth and moon
schematically in the figure. The angle q is such were perfectly coplanar
that any ray of light reflected after striking both
(c) The moon appears red during the eclipse
the mirrors returns parallel to its incident path.
because the blue light is absorbed in earth’s
For this to happen, the value of q should be
atmosphere and red is transmitted
(d) A lunar eclipse can occur only on a full
moon day
25. Many exoplanets have been discovered by the
q
transit method, where is one monitors, a dip in
the intensity of the parent star as the exoplanet
moves in front of it. The exoplanet has a radius
R and the parent star has radius 100 R. If I0 is the
(a) 30° (b) 45° (c) 60° (d) 90° intensity observed on earth due to the parent
21. A certain liquid has a melting point of –50°C and star, then as the exoplanet transits
a boiling point of 150°C. A thermometer is (a) the minimum observed intensity of the parent
designed with this liquid and its melting and star is 0.9 I0
boiling points are designated at 0°L and 100°L. (b) the minimum observed intensity of the
The melting and boiling points of water on this parent star is 0.99 I0
scale are
(c) the minimum observed intensity of the
(a) 25°L and 75°L, respectively
parent star is 0.999 I0
(b) 0°L and 100°L, respectively
(d) the minimum observed intensity of the
(c) 20°L and 70°L, respectively parent star is 0.9999 I0
(d) 30°L and 80°L, respectively
EBD_7839
4 KVPY SOLVED PAPER 2018 STREAM : SA

26. A steady current I is set up in a wire whose cross-


sectional area decreases in the direction of the A
flow of the current. Then, as we examine the
B D
narrowing region, (c) (d)
C
(a) the current density decreases in value
(b) the magnitude of the electric field increases 30. If a ball is thrown at a velocity of 45 m/s in
(c) the current density remains constant vertical upward direction, then what would be
(d) the average speed of the moving charges the velocity profile as function of height?
remains constant (Assume, g = 10 m/s2)
27. Select the correct statement about rainbow.
(a) We can see a rainbow in the western sky in
the late afternoon
(b) The double rainbow has red on the inside (a) (b)
and violet in the outside
(c) A rainbow has an arc shape, since the earth
is round
(d) A rainbow on the moon is violet on the
inside and red on the outside
28. Remote sensing satellites move in an orbit that
is at an average height of about 500 km from the (c) (d)
surface of the earth. The camera onboard one
such satellite has a screen of area A on which
the images captured by it are formed. If the focal CHEMISTRY
length of the camera lens is 50 cm, then the
31. The number of water molecules in 250 mL of
terrestrial area that can be observed from the
water is closest to [Given, density of water is 1.0
satellite is close to
g mL–1; Avogadro’s number = 6.023 × 1023]
(a) 2 × 103 A (b) 106 A
(a) 83.6 × 1023 (b) 13.9 × 1023
(c) 1012 A (d) 4 × 1012 A 23
(c) 1.5 × 10 (d) 33.6 × 1023
29. Letters A, B, C and D are written on a cardboard
32. Among the following, the correct statement is:
as shown in the figure below.
(a) pH decreases when solid ammonium
A chloride is added to a dilute aqueous solu-
tion of NH3
D B
(b) pH decreases when solid sodium acetate is
C added to a dilute aqueous solution of acetic
acid
The cardboard is kept at a suitable distance
(c) pH decreases when solid NaCl is added to a
behind a transparent empty glass of cylindrical
shape. If the glass is now filled with water, one dilute aqueous solution of NaOH
sees an inverted image of the pattern on the (d) pH decreases when solid sodium oxalate is
cardboard when looking through the glass. added to a dilute aqueous solution of oxalic
Ignoring magnification effects, the image would acid
appear as 33. The solubility of BaSO4 in pure water (in gL–1) is
closest to
A A [Given; Ksp for BaSO4 is 1.0 × 10–10 at 25°C.
B D
D B Molecular weight of BaSO4 is 233 g mol–1]
(a) (b) (a) 10 × 10–5 (b) 10 × 10–3
C
C
(c) 2.3 × 10–5 (d) 2.3 × 10–3
KVPY SOLVED PAPER 2018 STREAM : SA 5

34. Among the following, the incorrect statement is 39. The chlorine atom of the following compound
(a) No two electrons in an atom can have the c
same set of four quantum numbers Cl
(b) The maximum number of electrons in the d b
shell with principal quantum number, n is Cl Cl
equal to n2 + 2
(c) Electrons in an orbital must, have opposite O a
spin Cl
(d) In the ground state, atomic orbitals are filled that reacts most readily with AgNO3 to give a
in the order of their increasing energies precipitate is
35. A container of volume 2.24 L can withstand a (a) Cla (b) Clb (c) Clc (d) Cld
maximum pressure of 2 atm at 298 K before 40. Among the following sets, the most stable ionic
exploding. The maximum amount of nitrogen (in g) species are
that can be safely put in this container at this –
+
temperature is closest to
(a) 2.8 (b) 5.6 (c) 1.4 (d) 4.2 (a) and
36. The compound shown below
O NO + +
2
+
(b) and

can be readily prepared by Friedel-Craft’s – –


reaction between
(c) and
(a) benzene and 2-nitrobenzoyl chloride
(b) benzyl chloride and nitrobenzene
– +
(c) nitrobenzene and benzoyl chloride
(d) benzene and 2-nitrobenzyl chloride (d) and
37. The correct statement about the following com-
pounds 41. The correct order of energy of 2s-orbitals in H,
Li, Na and K, is
(a) K < Na < Li < H (b) Na < Li < K < H
(c) Na < K < H < Li (d) H < Na < Li < K
Br Br 42. The hybridisation of xenon atom in XeF4 is
X Y
is (a) sp3 (b) dsp 2
(c) sp d3 2 (d) d2 sp 3
(a) Both are chiral
(b) Both are achiral 43. The formal oxidation numbers of Cr and Cl in the
(c) X is chiral and Y is achiral ions Cr2O72– and ClO3–, respectively are
(d) X is achiral and Y is chiral (a) + 6 and +7 (b) +7 and +5
38. The most acidic proton and the strongest, (c) +6 and +5 (d) +8 and +7
nucleophilic nitrogen in the following compound 44. A filter paper soaked in salt X turns brown when
O exposed to HNO3 vapor. The salt X is
c (a) KCl (b) KBr (c) KI (d) K2SO4
b
N N CH3 45. The role of haemoglobin is to
a H H (a) store oxygen in muscles.
N (b) transport oxygen to different parts of the
H body.
respectively, are
(c) convert CO to CO2.
(a) Na – H; Nb (b) Nb – H; Nc
a c (d) convert CO2 into carbonic acid.
(c) N – H; N (d) Nc – H; Na
EBD_7839
6 KVPY SOLVED PAPER 2018 STREAM : SA

BIOLOGY (a) O2 + O ¾® O3 (b) O2 ¾® O + O


46. Which one of the following molecules is a (c) O3 + O3 ¾® 3O2 (d) O + O ¾® O2
secondary metabolite? 55. Which one of the following statements is true
(a) Ethanol (b) Lactate about trypsinogen?
(c) Penicillin (d) Citric acid (a) It is activated by enterokinase.
47. Lecithin is a (b) It is activated by renin.
(a) carbohydrate (b) phospholipid (c) It is activated by pepsin.
(c) nucleoside (d) protein (d) It does not need activation.
48. The water potential (Y p) of pure water at 56. Which one of the following organisms respires
standard temperature and atmospheric pressure is: through the skin?
(a) 0 (b) 0.5 (c) 1.0 (d) 2.0 (a) Blue whale (b) Salamander
49. Action potential in neurons is generated by a (c) Platypus (d) Peacock
rapid influx of 57. Which one of the following human cells lacks a
(a) chloride ions (b) potassium ions nucleus?
(c) calcium ions (d) sodium ions (a) Neutrophil
50. Erythropoietin is produced by (b) Neuron
(a) heart (b) kidney (c) Mature erythrocyte
(c) bone marrow (d) adrenal gland (d) Keratinocyte
51. Tendrils are modifications of
58. The first enzyme that the food encounters in
(a) stem or leaf (b) stem only
human digestive system is :
(c) leaf only (d) aerial roots only
(a) Pepsin (b) Trypsin
52. Which one of the following combinations of
(c) Chymotrypsin (d) Amylase
biomolecules is present in the ribosomes?
(a) RNA, DNA and protein 59. Glycoproteins are formed in which one of the
(b) RNA, lipids and DNA following organelles?
(c) RNA and protein (a) Peroxisome (b) Lysosome
(d) RNA and DNA (c) Golgi apparatus (d) Mitochondria
53. Which one of the following proteins does not 60. An example of nastic movement (external
play a role in skeletal muscle contraction? stimulus-dependent movement) in plants is
(a) Actin (b) Myosin (a) folding-up of the leaves of Mimosa pudica.
(c) Troponin (d) Microtubule (b) climbing of tendrils.
54. Which one of the following reactions is (c) growth of roots from seeds.
catalysed by high-energy ultraviolet radiation (d) growth of pollen tube towards the ovule.
in the stratosphere?

PART-II (2 MARKS QUESTIONS)

MATHEMATICS repetitions) such that the sum of any two adjacent


m
61. What is the sum of all natural numbers n such digits is odd. Then is equal to
n
that the product of the digits of n (in base 10) is
(a) 9 (b) 12 (c) 15 (d) 18
equal to n2 – 10n – 36?
63. The number of solid cones with integer radius
(a) 12 (b) 13 (c) 124 (d) 2612
and integer height each having its volume
62. Let m (respectively, n ) be the number of 5-digit numerically equal to its total surface area is
integers obtained by using the digit 1, 2, 3, 4, 5 (a) 0 (b) 1 (c) 2 (d) infinite
with repetitions (respectively, without
KVPY SOLVED PAPER 2018 STREAM : SA 7

64. Let ABCD be a square. An arc of a circle with A 69. A proton of mass m and charge e is projected
as centre and AB as radius is drawn inside the from a very large distance towards an a-particle
square joining the points B and D. Points P on with velocity v. Initially a-particle is at rest, but
AB, S on AD, Q and R on arc BD are taken such it is free to move. If gravity is neglected, then
that PQRS is a square. Further suppose that PQ the minimum separation along the straight line
area PQRS of their motion will be
and RS are parallel to AC. Then, is
area ABCD (a) e2/4pe0 mv2 (b) 5e2/4pe0 mv2
2
(c) 2e /4pe0 mv 2 (d) 4e2/4pe0 mv2
1 1 1 2
(a) (b) (c) (d) 70. A potential is given by V(x) = k(x + a) 2/2
8 5 4 5
65. Suppose ABCD is a trapezium whose sides and for x < 0 and V(x) = k(x– a)2/2 for x > 0. The
heigh are integers and AB is parallel to CD. If the schematic variation of oscillation period T for a
area of ABCD is 12 and the sides are distinct, particle performing periodic motion in this po-
then |AB – CD| tential as a function of its energy E is
(a) is 2 T T
(b) is 4
(c) is 8
(d) cannot be determined from the data (a) (b)
PHYSICS
E E
66. A coffee maker makes coffee by passing steam
through a mixture of coffee powder, milk and T T
water. If the steam is mixed at the rate of 50 g per
minute in a mug containing 500 g of mixture, then
it takes about t0 seconds to make coffee at 70° C (c) (d)
when the initial temperature of the mixture is
25°C. The value of t0 is close to (ratio of latent E E
heat of evaporation to specific heat of water is
540°C and specific heat of the mixture can be CHEMISTRY
taken to be the same as that of water) 71. Among the following, the species with identical
(a) 30 (b) 45 (c) 60 (d) 90 bond order are
67. A person in front of a mountain is beating a drum (a) CO and O2– (b) O2– and CO
2
at the rate of 40 per minute and hears no distinct 2–
(c) O2 and B2 (d) CO and N+2
echo. If the person moves 90 m closer to the
mountain, he has to beat the drum at 60 per 72. The quantity of heat (in J) required to raise the
minute to not hear any distinct echo.The speed temperature of 1.0 kg of ethanol from 293.45 K to
of sound is the boiling point and then change to liquid to
vapour at that temperature is closest to
(a) 320 ms–1 (b) 340 ms–1
(c) 360 ms –1 (d) 380 ms–1 [Given, boiling point of ethanol 351.45 K. Spe-
cific heat capacity of liquid ethanol 2.44 J g–1 k–1.
68. A glass beaker is filled with water up to 5 cm. It is
Latent heat of vaporisation of ethanol 855 J g–1]
kept on top of a 2 cm thick glass slab. When a
coin at the bottom of the glass slab is viewed at (a) 142 × 102 (b) 9.97 × 102
(c) 142 × 10 5 (d) 9.97 × 105
the normal incidence from above the beaker, its
apparent depth from the water surface is d cm. 73. A solution of 20.2 g of 1,2-dibromopropane in
Value of d is close to (the refractive indices of MeOH upon heating with excess Zn produces
water and glass are 1.33 and 1.5, respectively) 3.58 g of an unsaturated compound X. The yield
(a) 2.5 cm (b) 5.1 cm (%) of X is closest to [Atomic weight of Br is 80]
(c) 3.7 cm (d) 6.0 cm (a) 18 (b) 85 (c) 89 (d) 30
EBD_7839
8 KVPY SOLVED PAPER 2018 STREAM : SA

74. The lower stability of ethyl anion compared to number of different genotypes and phenotypes
methyl anion and the higher stability of ethyl obtained respectively would be
radical compared to methyl radical, respectively, (a) 4 and 9 (b) 6 and 3
are due to
(c) 9 and 4 (d) 11 and 4
(a) + I- effect of the methyl group in ethyl anion
78. If the H+ concentration
of an aqueous solution is
s ® p-orbital conjugation in ethyl radical
0.001 M, then the pOH of the solution would be
(b) – I - effect of the methyl group in ethyl (a) 0.001 (b) 0.999 (c) 3 (d) 11
anion and s ® s* p-conjugation in ethyl
79. Consider the following vision defects listed in
radical
column I and II and the corrective measures in
(c) + I effect of the methyl group in both cases column III. Choose the correct combination.
(d) + I- effect of the methyl group in ethyl anion
Column I Column II Column III
and s ® s* conjugation in ethyl radical.
P. Hyper- (i) Near- a. Convex
75. The F-Br-F bond angles in BrF5 and the Cl-P-Cl
metropia sightedness lens
bond angles in PCl5, respectively, are
Q. Myopia (ii) Far- b. Concave
(a) identical in BrB5 but non-identical in PCl5
sightedness lens
(b) identical in BrB5 and identical in PCl5
(a) P–ii–b (b) Q–i–b
(c) non-identical in BrB5 but identical in PCl5
(c) P–i–a (d) Q–i–a
(d) non-identical in BrF5 and non-identical in
80. Which one of the following properties causes
PCl5
the plant tendrils to coil around a bamboo stick?
BIOLOGY (a) Tendril has spines.
76. If the genotypes determining the blood groups (b) The base of the tendril grows faster than
of a couple are IAIO and IAIB, then the probability the tip.
of their first child having type O blood is: (c) Part of the tendril in contact with the
(a) 0 (b) 0.25 (c) 0.50 (d) 0.75 bamboo stick grows at a slower rate than
77. A cross was carried out between two individuals the part away from it.
heterozygous for two pairs of genes. Assuming (d) The tip of the tendril grows faster than the
segregation and independent assortment, the base.

ANS WER KEYS


Part-I Part-II
1 (d ) 11 (c) 21 (a) 31 (a) 41 (a) 51 (a) 61 (b) 71 (c)
2 (b) 12 (d) 22 (b) 32 (a) 42 (c) 52 (c) 62 (c) 72 (d )
3 (c) 13 (b) 23 (d) 33 (d) 43 (c) 53 (d) 63 (b) 73 (b )
4 (b) 14 (a) 24 (a) 34 (b) 44 (c) 54 (b) 64 (d) 74 (a)
5 (a) 15 (a) 25 (d) 35 (d) 45 (b ) 55 (a) 65 (b) 75 (d )
6 (b) 16 (a) 26 (b) 36 (a) 46 (c) 56 (b) 66 (b) 76 (a)
7 (c) 17 (c) 27 (Non e) 3 7 (c) 47 (b ) 57 (c) 67 (c) 77 (c)
8 (c) 18 (d) 28 (c) 38 (b) 48 (a) 58 (d) 68 (b) 78 (d )
9 (c) 19 (c) 29 (d) 39 (a) 49 (d ) 59 (c) 69 (b) 79 (b )
10 (a) 20 (d) 30 (a) 40 (d) 50 (b) 60 (a) 70 (b) 80 (c)
HINTS & SOLUTIONS

PART-I d
(m – n)a + (m – n)(m + n –1) = – (m – n)
MATHEMATICS 2

1. (d) We have, Þ 2a + (m + n –1) d = –2 [m ¹ n]


a4 + b4 < 1 and a2 + b2 > 1 m+n
\ Sm + n = [(2a + (m + n - 1)d ]
The graphs of x2 + y2 = 1 and x4 + y4 = 1 2
are given below:
m+n
= (- 2) = -(m + n)
2
Y
x4+y4=1
4. (b) (I) Statement I is false. Consistent Lin-
ear equations may have unique of infi-
nite solutions.
X' X (II) Statement II is also false
x2+y2=1
Q 132 + 142 = 365
5. (a) Let
P(x) = anxn + an –1xn–1 + an – 2xn–2
Y' + ...+ a1x + a0
a0, a1, a2 ...anÎI
It is clear from graph, there are many
positive real numbers Given, P(2) = 2 and P(4) = 5
(a, b) satisfying a4 + b4 < 1 and a2 + b2 > 1
\2 = an 2n + an –1 2n –1 + an – 2 2n – 2
2. (b) Given,
x4 – x2 + 2x – 1 = 0 +a1 2 + a0 ...(1)
Þ x4 – (x – 1)2 =
0
5 = an 4n + an –1 4n –1
Þ (x – x + 1) (x + x – 1) = 0
2 2

Þ x2 – x + 1 = 0 or x2 + x – 1 = 0
+ an – 2 4n – 2 + ... + 4a1 + a0 ....(2)
Here, x2 – x + 1 = 0 has no real roots.
and x2 + x – 1 = 0 has two real roots On subtracting eq. (1) from eq. (2), we get
3. (c) Given, Sm = n and Sn = m
( ) (
3 = an 4n - 2n + a n -1 4n -1 - 2n -1 )
m
S m = [2a + (m - 1)d ] = n ...(i) +... + 2a1
2
Clearly, LHS is odd number and RHS is even
n
Sn = [2a + (n - 1)d ] = m ...(ii) number.
2
\ no polynomials exists.
On subtracting Eq. (ii) from Eq. (i), we get 6. (b) Four digit numbers which are divisible by 7
(m - n ) d are 1001, 1008, 1015,...,9996.
2a + [ m( m - 1) - n(n - 1)] = n - m Hence, total number of such numbers = 1286
2 2
EBD_7839
10 KVPY SOLVED PAPER 2018 STREAM : SA

Median When height of cylinder is doubled, then


2
æNö
th
æN ö
th volume of solid, V2 = 2pr 2h + pr 3
ç ÷ observation + ç + 1÷ observation 3
=è ø è2 ø
2
2 2
2 pr 2 h + p r 3
V2 3 3
[\ N is even] \ = =
V1 2 pr 2 h + 2 pr 3
Median
3
th th
æ 1286 ö æ 1286 ö 2
çè ÷ observation + ç + 1÷ observation
2 ø è 2 ø 2h +
r
= Þ 3 =3Þh=r
2 2 2 2 3
h+ r
3
643th + 644th
= When the radius is doubled, then volume of
2
solid,

=
(1001 + (642)7) + (1001 + (643)7) 16p r 3
2 V3 = 4p r 2 h +
3
2 (1001) + 7 ( 642 + 643) 16
= 4h + r
2
\
V3
= 3 = 4h + 8h
= 1001 + 4497.5 = 5498.5 V1 2 h+h
h+ r
3
7. (c) Let the height and radius of cylinder are h
are r, respectively.
é r hù
\ volume of cylinder = pr2h =6 êëQ 3 = 2 úû

Hence, volume is increased by 500%.


r 8. (c) Let DPQR
r
Given, QR2 + PR2 = 5PQ2
Median PM and QN intersect at G.

1
Þ PN = NR = PR &
h h 2
1
QM = MR = QR
r 2

2 3 N
and volume of hemisphere = pr
3 G

2
\volumeof solid,V1 = pr 2h + pr 3 Q R
3 M
KVPY SOLVED PAPER 2018 STREAM : SA 11

2 1
and median of DABC :
QG = QN , GM = PM AD = l, BE = m, CF = n
3 3
AD is median,
2 2
æ2 ö æ1 ö AB + AC c+b
Þ QG 2 + GM 2 = ç QN ÷ + ç PM ÷ \ AD < Þ l<
è3 ø è3 ø 2 2

4 1 a+b a+c
= QN 2 + PM 2 Similarly, n < and m <
9 9 2 2
\l + m + n < a + b + c
4 æ 2 PQ 2 + 2QR2 - PR 2 ö l +m+n
= Þ <1
9 çè 4 ÷ø
a+b+c
...(1)

Also, in DBGC, BG + GC > BC


1 æ 2 PQ 2 + 2 PR 2 - QR 2 ö
+ ç ÷ø 2
9è 4 \ (m + n) > a
3
2 2
é 8PQ 2 + 8QR 2 - 4 PR 2 ù Similarly, ( n + l ) > b and ( m + l ) > c
ê ú 3 3
1 +2 PQ 2 + 2 PR 2 - QR 2 ú
= ê 4
9ê 4 ú Q ( l + m + n) > a + b + c
ê ú 3
ëê ûú
l + m+ n 3
Þ > ...(2)
a+b+c 4
1 é10 PQ 2 + 7QR 2 - 2PR 2 ù
= ê ú \ From eqs. (1) and (2), we get
9ë 4 û
l + m+ n æ 3 ö
Þ Î ç ,1÷
= ê
(
1 é 2 5 PQ - PR + 7QR
2 2
) 2
ù
ú
a+b+c è4 ø
9ê 4 úû 10. (a) Let A(x0, y0)
ë
Given x2 + y2 < 1
Let any arbitrary point B(x, y)
1 é 2QR2 + 7QR 2 ù 1
= ê ú = QR = QM
2 2

9ë 4 û 4 Y(0,1)
A(X0,Y0)
Þ QG 2 + GM 2 = QM 2
B
(X,Y)
\ ÐQGM = 90o
9. (c) Let in DABC
BC = a, AC = b, AB = c 1
X' X
O (1,0)
A

F E

G
Y'
B D C AB2 = (x – x0)2 + (y – y0)2
EBD_7839
12 KVPY SOLVED PAPER 2018 STREAM : SA

Also AB2 = (OA – OB)2 Taking log base 10 both side

( ) log10 N = log10 9 + log10 520


2
Þ AB2 = x02 + y02 –1 [Q OB = 1]
= 2log10 3 + 20 log10 5
\ Minimum value of (x – x0)2 + (y – y0) is
= 2 × 0.4771 + 20 × (1 – 0.3010)
( )
2
x02 + y02 - 1 = 14 characters value
11. (c) Given, in DPQR, Hence, the number has 15 digits
PQ = 3
S = Sum of digits of the number
Altitude RS = 3 Þ PS = QR Now, the last digit of N is 5.
R \ minimum value of S = 1 + 5 = 6
and maximum value of S = 9 × 14 + 5
= 126 + 5 = 131
\ 6 < S < 140
P
S Q 14. (a) Given, PQR is an acute angle triangle.
In DSQR, QR2 = SR2 + SQ2
Q
( 3)
2
+ ( QP - PS )
2
Þ PS = 2

[Q SQ = PQ – PS]
r p
PS = 3 + ( 3 - PS )
2 2

PS 2 = 3 + 9 - 6 PS + PS 2 Þ PS = 2
In DPRS, P R
Q1Q2 Q3
( 3)
2
PR = PS + RS = ( 2) +
2
2 2 2
= 4+ 3 q

Þ PR = 7 \ PQ < QR
12. (d) Total number of students = 50
Þ ÐQRP < QPR ...(1)
Average marks of student = 47.5
\ Total marks of students = 50 × 47.5 = 2375
1
Now, the student get integer marks Again, PQ3 = PR
2
Hence, for maximum number of students who
could get marks more than 47.5, Also, PQ2 : Q2 R = r : p (angle bisector
we will divide total marks by 48. theorem).
2375 æ r ö
\ » 49 Þ PQ2 = ç
48 ÷ PR
èr+ pø
13. (b) Given number,
N = 152 × 518 But, r < p
N = 32 × 52 × 518
1
N = 9 × 520 Þ PQ2 < PR < PQ3 ...(2)
2
KVPY SOLVED PAPER 2018 STREAM : SA 13

Now, PHYSICS
16. (a) Here, temperature of water is increased from
Ð QPQ 2 + ÐPQ2Q + ÐPQQ2 = ÐRQQ2 0°C to 10°C and we know, density of water
+ ÐQQ2 R + ÐQRQ2 initially increases upto a maximum at 4°C and
then it decreases.
Þ ÐQPQ2 + ÐPQ2Q = QQ2 R + < QRQ2
So, upthrust on block of wood also increases
[Since, QQ2 is angle bisector of Ð Q] till temperature reaches 4°C and then
decreases from 4°C to 10°C.
Þ ÐQQ2 P <ÐQQ2 R from (1)
Hence, volume of block above water also
But ÐQQ2 P +ÐQQ2 R =180° increases upto 4°C and then decreases from
4°C to 10°C. At 4°C volume is maximum.
Hence, ÐQQ2 P < 90o and ÐQQ2 R > 90o \ Graph (a) correctly depicts variation of
Here, < QQ1P = 90° volume versus temperature.
17. (c) Smallest length of rope required
\ Q1 lies inside DPQQ2
= thickness of ice × fraction of ice block out
Þ PQ1 < PQ2 ...(3) of water.
from (2) and (3), Fraction of thickness of ice block out of wa-
Þ PQ1 < PQ2 < PQ3 ter

15. (a) Given, (a – 8)2 – (b – 7)2 = 5 æ r ö 0.9


x = 1– ç ice ÷ = 1– Þ x = 0.1
è rwater ø 1
Þ ( a - 8 + b - 7)( a - 8 - b + 7 ) = 5

Þ ( a + b - 15)( a - b - 1) = 5 Man

There are four case 0.8 m

a + b - 15 = 5; a - b - 1 = 1 ...(1) 8m

a + b - 15 = 1; a - b - 1 = 5 ...(2)
a + n – 15 = –5; a – b – 1 = –1 ...(3) \ Shortest length of rope required
a + b - 15 = –1; a - b - 1 = -5 ...(4) = 8 × 0.1 = 0.8 m.
Hence, nearest option is (c) 0.9 m.
Solving (1), we get: a = 11, b = 9.
18. (d) No water comes out of hole in free fall on box
Solving (2), we get: a = 11, b = 5 it is because in free fall, both water and box
Solving (3); we get: a = 5 , b = 5. cover equal distance downwards in equal
Solving (4), we get: a = 5, b = 9 time.
19. (c) Mass of water evaporated in two hours
D(11,5) 4 C(11,9) m = 2h × 20 g/h
= 40 g = 40 × 10–3 kg
6 6 During evaporation heat absorbed by water
Q = mL ...(i)
If this heat is taken entirely from water in
A(5,5) 4 B(5,9) earthen pot, and DT is decrease of tempera-
ture of pot then,
\ perimeter = 2(4 + 6) = 20
Q = MsDT ...(ii)
EBD_7839
14 KVPY SOLVED PAPER 2018 STREAM : SA

where, M = mass of water in pot 23. (d) As we know, volume charge density,
and s = specific heat of water. Q Q/t I
r= = =
From Eqs. (i) and (ii) L´ A L/t ´ A v´ A
mL = MsDT
500 ´ 10 –6
m L 40 ´10 –3 =
Þ DT = ´ = ´ 540 = 5.4°C 3 ´ 107 ´ 1.50 ´10 –6
M s 4
20. (d) Two mirrors are inclined at an angle, q = ? 5
= ´10 –5 =1.1´10–5 Cm –3
According to question, emergent ray is 3´1.5
parallel to incident ray 24. (a) A lunar eclipse occurs only on a full moon
\ deviation angle d = 180° day not on a new moon day.
But d = 360° – 2q 25. (d) Intensity of radiation
or, 360° – 2q = 180° I µ A or I = kA
or, 2q = 180° q
where, k = constant and A = surface area
\ q = 90° Now, if I0 = intensity of parent star.
21. (a) According to principle of thermometry, Then, I0 = kp (100 R)2 = k p R2 × 10000
When exoplanet is in front of star, observed
T – TLFP
= constant for every intensity will be minimum. Let intensity
TUFP – TLFP
minimum is Imin
thermometric scale.
i.e., I min = k [p (100 R )2 – p R 2 ]

æ L – TLFP ö æ C – TLFP ö
çè T ÷ø Liquid = çè T ÷ Star radius
UFP – TLFP based UFP – TLFP ø Centigrade 100 R
scale scale R
Exoplanet
radius R
L – (– 50) C –0
Þ 150 – – 50 =
( ) 100 –0
Þ I min = k pR 2 (10000 –1)
L + 50 C
Þ = = k pR 2 ´ 9999
150 + 50 100
Þ L + 50 = 2C I min k pR 2 ´ 9999
\ =
Therefore, at 0°L, centigrade scale reading I0 k pR 2 ´10000
or, I min = I 0 ´ 0.9999
50
0 + 50 = 2 C or C = = 25° L
2 26. (b) When current flows through a conductor of
and at 100° L, centigrade scale reading tapered cross-section, current flow through
every section remains constant i.e., I1 = I2
150
100 + 50 = 2 C or C = = 75° L
2 I A1 A2 I
22. (b) 1 a-V = q (DV)
= 2e × 1 V = 2eV [Q q = 2e]
KVPY SOLVED PAPER 2018 STREAM : SA 15

j1A1 = j2A2 and d2 = diameter of area on earth.


j1 A Area observed on earth A0
Þ = 2 <1 =
j2 A1 Area of screen A
æ p × d2 ö
2
or, j1 < j2 ç ÷
i.e., Current density (j) increases in the narrow è 4 ø d 22
= = ...(ii)
region. æ p × d12 ö d12
ç ÷
Also, j = nend Þ nend1 < nend 2 è 4 ø
From equation (i) and (ii)
or, n d1 < nd 2 2 2
A0 æ h ö æ 500 ´10+3 ö
i.e., Drift velocity (nd) increases in the narrow =ç ÷ = ç ÷
region. A è f1 ø çè 50´10–2 ÷ø
And, Electric field, E = jr = (10× 103 × 102)2 = (106)2 = 1012
where, r = resistivity of material. 29. (d) A cylindrical lens produces erect and laterally
Clearly, E1 < E2 inverted image of the object.
27. (None) 30. (a) Given,
Rainbow is circular because locous of u = 45 ms–1, g = – 10 ms–2
reflected rays reaching eye of observer is a Now using, n2 – u2 = 2gh
circle not due to roundness of earth. n2 = (45)2 – 20h
There is no rainbow on moon as there is no
Þ n = 2025 – 20h
atmosphere.
In case of a primary rainbow, violet colour is 2025
If n = 0, h = » 101 m
on inside and red colour is on outside of arc. 20
In case of a secondary rainbow, red colour is And if h = 0, n = 45 ms –1
on inside and violet colour is on outside of
Clearly with increasing height, velocity
arc.
decreases, hence n versus h graph must be
In late afternoon rainbow is visible in east negative at all points.
side when light of sun in west side is reflected
Hence graph (a) correctly depicts velocity
and refracted by a layer of water droplets.
profile of ball thrown upward, as function of
28. (c) Consider the given diagram, height.
From figure,
CHEMISTRY
d1 d2 d h
q1 =q2 Þ = Þ 2= ...(i) 31. (a) Given: Density of water = 1.0 g mL–1
f h d1 f
Volume of water = 250 mL
where, d1 = diameter of camera screen
\ Mass of water = Density × Volume
Screen = 1.0 × 250 = 250 g
18 g of water contains
f = 50 cm q1 Camera
= 6.023 × 1023 molecules
\ 250 g of water contains

q2 6.023 ´ 1023
h = 500 km = ´ 250
18
= 83.65 × 1023 molecules
Earth
32. (a) Dil.aqueous solutions of NH3 is NH4OH.
ˆˆ† NH 4+ + OH -
NH 4OH ‡ˆˆ
EBD_7839
16 KVPY SOLVED PAPER 2018 STREAM : SA

On adding solid ammonium chloride,


CH2 CH2 CH3
the reaction moves backward due to 37. (c) H3C
*
CH2
common ion effect. The concentration of
OH – decreases and, hence, the pH H
decreases. Br
X
33. (d) Given, Ksp = 1 × 10–10
Here, the marked carbon (*) is chiral as it
ˆˆ† Ba 2+ + SO 24 -
BaSO 4 ‡ˆˆ has 4 different groups attached to it.
s s
CH2 CH2
H3C CH3
Let the solubity of Ba2+ and SO 24 - be s
\ Ksp = s2 H
Br
1 × 10–10 = s2 Y
s = 10–5 mol/L Here, no chiral carbon is present, as two
Thus, solubility of BaSO4 in pure water similar groups are attached to carbon.
= Solubility × M.W
= 10–5 × 233 = 2.3 × 10–3 38. (b)
34. (b) The maximum number of electrons in the O
shell with principle quantum number (n) is b c
2n2. N CH3 N
35. (d) From ideal gas equation a H H
PV = nRT N
Maximum number of moles in container, H
In the given compound most acidic proton
PV 2 ´ 2.4 will be Nb — H. This is because its conjugate
n= = = 0.18 moles base will be most stable and the most
RT 0.0821 ´ 298
nucleophilic nitrogen will be Nc, because the
Maximum weight of N2 in container
lone pair of electrons present on this N is
= 0.183 × 28 = 5.127 g
localised.
Hence, at 5.127 g exploding can occur. Thus,
39. (a) The reaction between haloalkane and
the maximum amount of nitrogen that can
AgNO3 gives carbocation intermediate. So,
be safely put in this container at 298 K
more easily it will be formed, more readily it
temperature and exert pressure less than 2
will react to give precipitate.
atm will be closest to 4.2 g.
In the given compound
c a
36. (a) Cl Cl
d b d b
Cl Cl Cl Cl
¾¾¾®
O a O +
Cl a
Cl
d b
Cl Cl

¾¾¾®
O
+
O being electronegative, will attract electron
density and cleave C–Cl a bond density
easily.
This reaction is Friedal-Craft acylation.
KVPY SOLVED PAPER 2018 STREAM : SA 17

40. (d) The species which follows Huckel’s rule 44. (c) It will reduce HNO3 to NO2 (brown gas),
(4n + 2)p will be most stable species. when exposted to HNO3 vapour.
+ – Since, KI is the strongest reducing agent.
2 KI + 4 HNO3 ¾¾® I2 + 2NO2 + 2KNO3
+ 2H2O
– –
It has 8p e s, doesn't It has 4p e s, doesn't 45. (b) The role of haemoglobin is to transport
follow Huckel's rule follow Huckel's rule oxygen from lungs or gills to different parts
+ of the body to perform metabolism.

BIOLOGY

It has 10 p e s, follows

It has 2 pe s, follows 46. (c) Penicillin is a secondary metabolite.
Huckel's rule Huckel's rule Secondary metabolites are organic
As both the species in option (d) follow compounds produced by bacteria, fungi or
Huckel’s rule. Thus, it is correct option. plants which are not directly involved in the
41. (a) As we know that, normal growth, development or
reproduction of the organism, e.g.,
-Z 2 antibiotics like penicillin, streptomycin, etc.
En µ
n2 Other molecules like ethanol, lactate and
citric acid are primary metabolites which are
-Z formed during growth phase as a result of
or E2 s µ
(2) 2 energy metabolism.
Therefore, as the atomic number increases, 47. (b) Lecithin is a phospholipid that plays a
the energy of orbital decreases. The atomic structural role in cell membranes. Lecithins
number of H, Li, Na and K respectively, are are composed of phosphoric acid, cholines,
1, 3, 11 and 19. esters of glycerol and two fatty acids. The
Thus, the correct order of energy of chain length, position and degree of
2s-orbitals is K < Na < Li > H. unsaturation of these fatty acids vary and
this variation results in different lecithins
1
42. (c) Hybridisation = [Valene electrons + with different biological functions.
2
48. (a) Water potential is the measure of the
Number of monoatomic m Anion/cations]
potential energy in water. It is denoted by
1 the Greek letter y(Psi) and is expressed in
\ For XeF4 = ( 8 + 4 - 0) = 6 units of pressure (pressure is a form of energy
2
\ The hybridisation is sp3d2. called Mega Pascals (MPa). Pure water at
standard temperature and atmospheric
43. (c) In Cr2 O37- , let the oxidation state of Cr be x. pressure has a water potential of zero. As
\ 2(x) + 7 (–2) = –2 solute is added, its value becomes more
2x – 14 = –2 negative. This causes water potential to
2x = 12 decrease.
x = +6 49. (d) An action potential is generated by the rapid
influx of Na+ ions followed by a slightly
In ClO3- , let the oxidation state of Cl be x.
slower efflux of K+ ions. The action potential
\ 1(x) + 3 (–2) = –1 is the mechanism by which nerve cells
x–6 = –1 communicate and conduct information and
x = +5 muscle cells are induced to contract.
EBD_7839
18 KVPY SOLVED PAPER 2018 STREAM : SA

50. (b) Erythropoietin (EPO) is a hormone produced enterokinase present in intestinal mucosa.
by the kidneys. It promotes the formation of This results in the subsequent activation of
red blood cells by the bone marrow. pancreatic digestive enzymes.
Chemically, erythropoietin is a protein with 56. (b) Salamanders are a group of amphibians
an attached sugar (a glycoprotein). It is typically characterised by a lizard-like
produced by kidney cells when the body appearance with slender bodies, blunt
experie nces low oxygen level. The resulting snouts, short limbs and a tail. Salamanders
rise in red blood cells increases the oxygen breath through their skin and the thin
carrying capacity of the blood. membranes in the mouth and throat.
51. (a) Tendrils are the modifications of stem or leaf. 57. (c) Mature human erythrocytes (Red blood
The tendril is a thread-like twisting, clinging cells) lack a nucleus. The absence of a
growth on the vines of the plant that enables nucleus is an adaptation of the red blood
it to attach itself to another object or another cell for its role. It allows the RBC to contain
plant for support. Plants such as grapes, more haemoglobin and therefore carry more
peas and cucumbers have tendrils. oxygen molecules. It also allows the cell to
52. (c) Ribosomes consist of two biomolecules (i.e., have its distinctive biconcave disc shape
RNA and proteins). In E. coli, they contain which aids to maneuver through tiny blood
34% protein and 66% RNA, while in vessels.
eukaryotes, they contain 40% protein and 58. (d) The first enzyme that the food encounters
60% RNA. Chemical composition of large in the digestive system is amylase.
subunit is different from that of small subunit. Digestion begins in the mouth with the
53. (d) Microtubule does not play a role in skeletal secretion of saliva an d its digestive
muscle. They are major components of enzymes. Saliva contains the digestive
cytoskeleton. Microtubules are hollow enzyme amylase, which works on
fibrous shafts whose main function is to help carbohydrate, starch present in breads,
support and give shape to the cell. They potatoes or pasta to help break them down
also serve a transportation function as they into simple sugars.
are the routes upon which organelles move 59. (c) Glycoproteins are formed in the Golgi
through the cell. The form spindle fibres apparatus of the cell. Glycoproteins are
during cell division. proteins that contain covalently attached
54. (b) In the stratosphere, ozone is created sugar residues.
primarily by ultraviolet radiation. When high Glycoproteins are present at the surface of
energy ultraviolet rays strike ordinary cells where they function as membrane
oxygen molecules (O 2), they split the proteins and play a role in cell to cell
molecule into two single oxygen atoms, interactions.
known as atomic oxygen. 60. (a) An example of nastic movement in plants is
O2 ® O + O folding up of the leaves of Mimosa pudica.
A freed oxygen then combines with another Nastic movements in plants are reversible
oxygen molecule to form a molecule of ozone and repeatable movements in response to a
(O3). stimulus whose direction is determined by
55. (a) Trypsinogen is an inactive enzyme the anatomy of the plant. The leaves of the
(zymogen) secreted by the pancreas, from Mimosa pudica fold up when touched and
which the digestive enzyme trypsin is formed returns to full leaf in a few minutes. The
in the duodenum. Trysinogen is converted leaves of the Mimosa achieve this rapid
into its active form trypsin by an enzyme folding by a change in turgor pressure.
KVPY SOLVED PAPER 2018 STREAM : SA 19

PART-II (b) even odd even odd even


MATHEMATICS = 2 × 3 × 2 × 3 × 2 = 72
61. (b) Given, n2 – 10n – 36 m = 108 + 72 = 180
n is a natural number. Case II: When the digits are not repeated.
\ product of its digits is ³ 0 The possibility of arrangement is odd even
\ n2–10n – 36 > 0 odd even odd

10 ± 100 + 144 = 3 ´ 2 ´ 1´ 2 ´ 1 = 12
n=
2 Þ n = 12
n = 5 ± 61 m 180
\ = = 15
( ) (
\ n Î - ¥,5 - 61 È 5 + 61, ¥ ) n 12
63. (b) Let height and radius of cone is h and r
But n is positive integer.
respectively such that h, r Î I
\ n > 13
Q volume of cone = surface area of cone
When n is a two digit number then
maximum product = 9 × 9 = 81 1
\ pr 2 h = prl + pr 2
\ n2 - 10n - 36 £ 81 3

n2 - 10n - 117 £ 0 1
Þ pr 2 h = pr h 2 + r 2 + pr 2
3
\ n Î [5 – 142,5 + 142]
Here n is taken as two digit number 1
Þ rh = h2 + r 2 + r [ r ¹ 0]
3
\ n Î[13,17 ] = 13,14,15,16
\ product of digits = 3, 4, 5, 6 (respectively) Þ rh - 3r = 3 h2 + r 2
When put n = 13 in the given product,
Þ r 2 h2 + 9r 2 - 6hr 2 = 9h2 + 9r 2
13 - 10 ´ 13 - 36 = 169 - 166 = 3
2

6r 2
\ n = 13 satisfies ( )
Þ h 2 r 2 - 9 = 6 hr 2 Þ h =
r2 - 9
62. (c) Here, m is 5-digits number using digits 1, 2,
3, 4, 5 with repetition such that sum of two
adjacent digits is odd and n is 5-digits num- æ r2 ö 54
Þ h = 6ç 2 Þ h = 6+
ber using digits 1, 2, 3, 4, 5 without repetition è r - 9 ÷ø r2 - 9
such that sum of any two adjacents digits is
odd. h and r are integers
Sum of two digits are odd if one is even and Here (r2 – 9) should be factor of 54.
other is odd. \ r2 – 9 = 1, 2, 3, 6, 9, 18, 27, 54.
Even = 2, 4 Þ r2 = 10, 11, 12, 15, 18, 27, 36, 63.
Odd = 1, 3, 5 \ r = 6 (only possible value)
Case I : When the digits are repeated.
54
Two possibilities \ h = 6+ = 6 + 2 =8
36 – 9
(a) odd even odd even odd
= 3 ×2 × 3 × 2 × 3 = 108 \ r = 6, h = 8
EBD_7839
20 KVPY SOLVED PAPER 2018 STREAM : SA

64. (d) Given, ABCD is a square. 65. (b) We have,


ABCD is a trapezium.
D C AB is parallel to CD.
R Area of trapezium = 12

N D b C

S Q

h
M

A a
P B A b E B

ÐCAP = ÐMAP = 45o 1


Þ ´ h ( AB + CD ) = 12
2
ÐAMP = ÐMPQ = 90(alternate angles)
24
Þ ÐMPA = 45o Þ AB + CD =
h
\ AM = MP = QN Sides and height of trapezium are integer.
PQRS is a square, \ h is a factor of 24
Q MN = PQ = PS h = 1, 2, 3, 4, 6, 8, 12, 24
AB + CD = 24, 12, 8, 6, 4, 3, 2, 1
Also PS = 2 PM = 2 AM
But AB + CD > h
Þ AN = AM + MN = 3 AM
AB + CD = 24, 12, 8, 6
In DANQ, In DBEC
AQ2 = AN2 + QN2 BEC is a right angled triangle

Þ AB 2 = ( 3 AM ) + AM 2 [Q AQ = AB ]
2 \ h must be 3 or 4
When h = 3, BE = 4, CE =5
Þ AB 2 = 10 AM 2 Þ AB + CD = 8
Now, PS2 = (2AM)2 = 4AM2
Þ AE + BE + AE = 8
area of square PQRS Þ 2 AE = 8 - BE = 8 - 4
\
area of square ABCD
Þ AE = 2

PS 2 4 AM 2 2 \ AB = 4 + 2 = 6 Þ and CD = 2
= 2
= 2
=
AB 10 AM 5
\ | AB - CD |=| 6 - 2 |= 4
KVPY SOLVED PAPER 2018 STREAM : SA 21

PHYSICS
2( x - 90) 60
66. (b) According to principle of calorimetry, Case II, = ... (ii)
v 60
Heat lost by steam = Heat gained by mixture
Substituting value of x from eq. (i) in eq. (ii)
i.e., Heat of condensation of steam + Heat
2x – 180 = v
given by water formed = Heat gained by mix-
ture 3
Þ v - 180 = v
or, m.L + mswDT = M.smDT 2
Þ mL + msw(100 – 70) = 500 × sw × (70 – 25) i.e., speed of sound, v = 360 ms–1
500 ´ sw ´ 45 500 ´ 45 68. (b) Given: d1 = 5 cm, m1 = 1.33
Þ m= =
L + 30sw æ L ö d2 = 2 cm, m2 = 1.5
ç + 30 ÷
s
è w ø d1 and d2 are the thickness of slabs of
500 ´ 45 medium with refractive index m1 and m2,
or, m = » 40g
(540 + 30) respectively.
i.e., 40 g steam is condensed in the process
d1 d2
of heating mixture from 25°C to 70°C. using formula, d = + + .....
μ1 μ2
As per question, in 1 min, 50 g of steam is
condensed.
5 2
\ Time to condensed 40 g of ice, Apparent depth, d = +
1.33 1.5
40 ´ 60 = 5.088 cm = 5.1 cm
t0 = s = 48s
50 69. (b) According to energy conservation principle,
Very close to answer option (b) 45 s. Initial kinetic energy = Potential energy at
67. (c) According to question, drummer does not minimum separation r
hear any echo i.e., time between two succes-
1 2 1 2e 2
sive wavefronts = time taken by a wavefront mv = ×
2 4pe 0 r
to return back to drummer.
where, m = reduced mass of system
x
m × 4m
= .
Drummer

Mountain

m + 4m

1 æ m × 4m ö 2 1 2e2
or, ç ÷ v = ×
2 è m + 4m ø 4pe0 r

2x or, minimum separation,


= time interval between two successive
v
wavefronts. 5e2
r=
v = speed of sound, then 4pe0 mv 2

2 x 60 70. (b) Given, potential function for the oscillating


Case I, = ... (i)
v 40 particle
EBD_7839
22 KVPY SOLVED PAPER 2018 STREAM : SA

The electronic configuration of O 22 - (18) is


ì k ( x + a )2
ï , x<0
ï 2 s1s 2s *1s 2 s 2s 2s *2s 2 s2pz2 p 2px2 = p 2 py2
V (x) = í 2
ï k ( x - a)
ïî , x>0 p*2p2x = p *2 py2
2
1
So, potential energy of the particle of mass m. B.O = (10 - 8) = 1
2

ì km ( x + a)2 The electronic configuration of O -2 (17) is


ï , x<0
ï 2
U ( x) = í s1s 2s *1s 2 s 2s 2s *2s 2 s2pz2 p 2px2 = p 2 py2
2
ï km ( x - a)
ïî , x>0
2 p*2p2x = p *2 p1y

1 3
dU ìkm ( x + a), x < 0 B.O = (10 - 7) = = 1.5
=í 2 2
dx îkm ( x - a), x > 0
The electronic configuration of B2 (10) is

dU s1s 2s *1s 2 s 2s 2s *2s 2 p 2 p1x = p 2 py1


If = 0, when x = ± a.
dx
1 2
B.O = [6 - 4] = = 1
2 2
d 2U
Now, = km > 0 Electronic configuration of N +2 (13) is
dx 2
So, at x = ± a particle is in unstable s1s 2s *1s 2 s2 s 2s *2s 2 p2 p2x = p2 py2 s2p1z
equilibrium.
1 5
Hence, for – a > x and x > a particle is B.O = [9 - 4] = = 2.5
unbounded. 2 2
Thus, option (c) is correct.
In region, – a £ x £ a, time period of particle
72. (d) Given, mass of ethanol = 1kg = 1000 g
reduces from a maximum. Latent heat of vaporisation of ethanol
Hence, graph (b) correctly depicts variation = 855 Jg–1
of oscillating period T as a function of en- Specific heat capacity of ethanol
ergy E. = 2.44 J/gK–1
Heat, q = mcDT + heat of vaporisation
CHEMISTRY = 1000 × 2.44 (351. 45 – 293.45 + 855 × 1000 J)
= 9.97 × 105 J
1
71. (c) B.O. = ( Nb - Na ) 73. (b)
2
where, Nb = electrons in bonding orbitals
Na = electrons in antibonding orbitals.
The electronic configuration of CO (14) is
s1s s*1s 2 s 2s 2 s *2 s 2 2 pz2 p 2 px2 = p 2p2y Moles of 1, 2-dibromo propane

1 6 20.2
\ B.O = (10 - 4) = = 3 = = 0.01 mole
2 2 202
KVPY SOLVED PAPER 2018 STREAM : SA 23

BIOLOGY
358
Moles of prop-1-ene = = 0.085 mole
42 76. (a) The genotypes of offspring of parents
having IAIO and IAIB blood groups are:
0.085
% yield = ´ 100 = 85% A O
I I ×
A B
I I
0.1 Parents

74. (a) The lower stability of ethyl anion (CH3CH2–)


compared to methyl anion (CH3–) is because
Genotype A A A B A O B O
of + I-effect of methyl group of ethyl anion. I I I I I I I I
Phenotype Blood Group A Blood Group AB Blood Group A Blood Group B
The higher stability of ethyl radical Offspring
compared to methyl radical is due to
s and p-orbital conjugation is ethyl radical. From the above cross, it is shown that none
of the offspring will be of blood group O.
H \ The probability of their first child having
H H
¬¾® H type O blood is zero.
C C C CH2
H 77. (c) In the given question, both parents are
H heterozygous for two pairs of genes. This
H
Hyper conjugation means the cross is a dihybrid cross.
in ethyl radical
Let us assume a dihybrid cross,

Pure breeding – Yellow round × Wrinkled green


traits seeds seeds
75. (d) (YYRR) (yyrr)
F1 – Yellow round seeds
(YyRr)

Gametes

YR Yr yR yr

Due to the presence of lone pair at axial


position in BrF5, bond angles will not remain YR Yr yR yr
identical as in regular octahedron (i.e., 90°).

Cl YYRR YYRr YyRR YyRr


YR yellow yellow yellow yellow
Cl round round round round
Cl P
Cl YYRr Yyrr YyRr Yyrr
Cl Yr yellow yellow yellow yellow
round wrinkled round wrinkled
Trigonal bipyamidal
YyRR YyRr yyRR yyRr
Trigonal bipyramidal geometry does not yR yellow yellow green green
round round round round
have all identical angles, as angle between
equatorial bonds is 120° and axial-equatorial YyRr Yyrr yyRr yyrr
yr yellow yellow green green
bond is 90°. wrinkled
round wrinkled round
EBD_7839
24 KVPY SOLVED PAPER 2018 STREAM : SA

The genotype ratio is 1 : 2 : 1 : 2 : 4 : 2 : 1 : 2 : 1 Myopia or near sightedness is a condition


The phenotypic ratio is 9 : 3 : 3 : 1 in which close objects apear clearly but far
\ The number of different genotypes and ones do not. It is corrected by using concave
phenotypes obtained would be 9 and 4, lens.
respectively. 80. (c) The tendrils are sensitive to touch. When
78. (d) The H+ ion concentration of an aqueous they come in contact with any support, the
solution is 0.001 M or 1 × 10–3 M part of the tendril in contact with the object
Since we know that pH = – log[H+] does not grow rapidly as the part of the
Using this equation, by putting in the values tendril away from the object. This causes
pH = – log 10–3 = – (–3) log 10 = 3 the tendril to circle around the object and
pH = 3 thus cling to it. This process is known as
We know that positive thigmotropism. Thigmotropism is
pOH = 14 – pH = 14 – 3 = 11 the directional response of a plant organ to
touch or physical contact with a solid object.
\ pOH = 11
This differential response is generally
79. (b) Hypermetropia or far sightedness is a vision
caused by the induction of some differential
condition in which nearby objects appear
growth.
blur. It is corrected by using convex lens.
Telegram @unacademyplusdiscounts

Join Us on Telegram for More Such Books

https://telegram.me/unacademyplusdiscounts

Join us from the above link or search ''unacademyplusdiscounts'' in Telegram


1
Time : 3 Hours Stream-SA Maximum Marks : 100
INSTRUCTIONS
1. There are 80 questions in this paper.
2. The question paper contains two parts; Part–I (1 Mark Questions) and Part–II (2 Marks
Questions). There are four sections; Mathematics, Physics, Chemistry and Biology in each part.
3. There are four options given with each question, only one of them is correct.
4. For each incorrect answer 0.25 Mark in Part–I and 0.5 Mark in Part–II will be deducted.

PART-I (1 MARK QUESTIONS)

MATHEMATICS
1. The number of points (x, y) having integral
2
coordinates satisfying the condition x + y 2 < 25
is
B
(a) 69 (b) 80 (c) 81 (d) 77
2. Find the sum of all the real roots of the equation
P (x) = 2 x 98 + 3 x 97 + 2 x 96 +... + 2 x + 3 = 0.
(a) 7 (b) 3 A
(c) 2 (d) None of these
3. Suppose P is an interior point of a triangle ABC
such that the ratios (a) 30 3 (b) 21 (c) 20 3 (d) 30
d ( A, BC ) d ( B, CA) d (C , AB) 6. Let S(M) denote the sum of the digits of a
, , are all equal. positive integer M written in base 10. Let N be
d ( P, BC ) d ( P, CA) d ( P, AB )
Here d(X, Y Z) denotes the perpendicular the smallest positive integer such that S(N) = 2013.
distance from a point X to the line Y Z. Then the value of S(5N + 2013)
Then the common value of these ratios is. (a) 15 (b) 17
2 (c) 25 (d) None of these
(a) 2 (b) 3 (c) 4 (d) 7. If three positive real numbers a, b, c are in A. P.
3
4. The mean of fifteen different natural numbers is such that abc = 4, then the minimum possible
13. The maximum value for the second largest of value of b is
these numbers is
(a) 46 (b) 51 (c) 52 (d) 53 (a) 23/2 (b) 22/3 (c) 21/3 (d) 25/2
5. A sphere with centre A and radius 7 cm and a 8. If the polynomial of the lowest degree with
larger sphere with centre B touches each other,
integer coefficients one of whose roots is
two common planks to both the spheres have
an angle of 60º in between them (refer the figure). 2 + 3 3 is f(x). The degree of f(x) is:
Then find the radius of larger sphere.
(a) 6 (b) 7 (c) 5 (d) 4
EBD_7839
2 KVPY-SA

9. In a triangle ABC, D is a point on BC such that PHYSICS


AD is the internal bisector of ÐA. Suppose 16. A U-tube contains water and methylated spirit
ÐB = 2ÐC and CD = AB. Then ÐA = separated by mercury. The mercury columns in
the two arms are in level with 10.0 cm of water in
(a) 60° (b) 70° (c) 45° (d) 72°
one arm and 12.5 cm of spirit in the other. If 15.0
10. Number of pairs of positive integers a, b such cm of water and spirit each are further poured
into the respective arms of the tube, what is the
a2 + b b2 + a
that 2
and 2 are both integers is: difference in the levels of mercury in the two
b -a a -b arms ? (Specific gravity of mercury = 13.6)
(a) 5 (b) 7 (c) 10 (d) 6 (a) 2.20 cm (b) 0.22 cm
11. The diagonals AC an d BD of a cyclic (c) 22 cm (d) 0.02 cm
17. Water flows through a tunnel in a reservoir of
quadrilateral ABCD meet at right angles in E. dam towards the turbine installed in the power
Let R is radius of circumscribing circle and plant. This is situated h m below the reservoir. If
EA2 + EB2 + EC2 + ED2 = n·R2. Then n is: the ratio of cross-sectional areas of the tunnel at
the reservoir and power station is h, find the
(a) 4 (b) 3 (c) 11 (d) 9 speed of the water entering into the turbine.
12. How many pairs of positive integers m, n
2gh 2gh
1 4 1 (a) h 2 (b)
satisfy + = where n is an odd integer h +1 h2 - 1
m n 12
less than 60? 2gh h2 - 1
(c) h (d) h
h2 - 1 2gh
(a) 4 (b) 7 (c) 5 (d) 3
18. A body cools in 7 minute from 60°C to 40°C.
13. Positive integers from 1 to 21 are arranged in 3 What will be its temperature after the next 7
groups of 7 integers each, in some particular minute? The temperature of the surroundings is
order. Then the highest possible mean of the 10°C. Assume that Newton’s law of cooling holds
good throughout the process.
medians of these 3 groups is (a) 8°C (b) 24°C
(a) 16 (b) 12.5 (c) 11 (d) 14 (c) 28°C (d) 32°C
14. ABCD is a square and E is the intersection point 19. A child running a temperature of 101°F is given
an antipyrin which causes an increase in the rate
of the diagonals. If N is any point on AE. of evaporation of sweat from his body. If the
Consider the following statements: fever is brought down to 98°F in 20 min, what is
(I) AB2 – BN2 = AN · NC the average rate of extra evaporation by the drug.
Assume the evaporation mechanism to be the
(II) AN2 + NC2 = 2 BN2 only way to which heat is lost. The mass of the
Which of the following is correct child is 30 kg. The specific heat of human body
(a) Only I is true (b) Only II is true is approximately the same as that of water and
latent heat of evaporation of water at that tem-
(c) I & II are true (d) I & II are false
perature is about 580 cal g–1.
15. The perimeter of rhombus is 2p cm, the sum of (a) 4.31 g/min (b) 4.31 g/sec
its diagonals is m cm. Then area of rhombus is- (c) 4.41 g/sec (d) 3.41 g/min
20. A person speaking normally produces a sound
m2 - p 2 m2 - p 2 intensity of 40 dB at a distance of 1 m. If the
(a) (b) threshold intensity for reasonable audibility is
4 2
20 dB, the maximum distance at which he can be
1 heard clearly is
(c) (m. p) (d) m2 – p2 (a) 4 m (b) 5 m
2
(c) 10 m (d) 20 m
MOCK TEST-1 3
21. In a lamp and scale arrangement to measure small
deflection, the arrangement is shown in the figure
SS¢ is the glass scale placed at a distance of 1 m (a) (b)
from the plane mirror MM and I is the position
of the light spot formed after reflection from the
undeflected mirror MM. The mirror is deflected
by 10° and comes to the deflected position M
‘M’. The distance moved by the spot on the (c) (d)
scale (IR) is :
26. A piece of wire is bent in the shape of a parabola
y = kx2 (y-axis vertical) with a bead of mass m on
it. The bead can slide on the wire without friction.
It stays at the lowest point of the parabola when
the wire is at rest. The wire is now accelerated
parallel to the x-axis with a constant acceleration
a. The distance of the new equilibrium position
of the bead, where the bead can stay at rest with
(a) 24.6 cm (b) 36.4 cm respect to the wire, from the y-axis is
(c) 46.4 cm (d) 34.6 cm
22. A steady current flows in a metallic conductor a a
(a) (b)
of non-uniform cross-section. The quantity/ gk 2 gk
quantities constant along the length of the
2a a
conductor is/are (c) (d)
(a) current, electric field and drift speed gk 4 gk
(b) drift speed only 27. A ballet dancer, dancing on a smooth floor is
(c) current and drift speed spinning about a vertical axis with her arms folded
(d) current only with an angular velocity of 20 rad/s. When she
23. When white light passes through a hollow prism stretches her arms fully, the spinning speed
then, decreases by 10 rad/s. If I is the initial moment of
(a) There is no dispersion and no angular inertia of the dancer, the new moment of inertia is
deviation. (a) 2I (b) 3I
(b) There is dispersion but no deviation.
(c) There is angular deviation but no (c) I/2 (d) I/3
dispersion. 28. The nearest star to our solar system is 4.29 light
(d) There is dispersion as well as deviation. years away. How much is this distance in terms
24. A certain planet completes one rotation about of parsecs? How much parallax would this star
its axis in time T. The weight of an object placed (named Alpha Centauri) show when viewed from
at the equator on the planet's surface is a fraction two locations of the Earth six months apart in its
f (f is close to unity) of its weight recorded at a orbit around the Sun ?
latitude of 60°. The density of the planet (a) 1.319 parsec, 2.638 second of arc
(assumed to be uniform) is given by : (b) 3.119 parsec, 6.238 second of arc
(c) 1.913 parsec, 2.863 second of arc
æ 4 - f ö 3p æ 4 - f ö 3p (d) 13.19 parsec, 26.38 second of arc
(a) ç
è 1 - f ø÷ 4GT 2
(b) çè + ø÷ 29. The intensity of radiation emitted by the sun
1 f 4GT 2
has its maximum value at a wavelength of 510
æ 4 - 3 f ö 3p æ 4 - 2 f ö 3p nm and that emitted by the north star has the
(c) çè 1 - f ø÷ 2 (d) çè - f ø÷ maximum value at 350 nm. If these stars behave
4GT 1 3GT 2 like black bodies, then the ratio of the surface
25. A particle of mass m is at rest at the origin at time temperature of the sun and north star is
t = 0. It is subjected to a force F(t) = F0e–bt in the (a) 1.46 (b) 0.69
x direction. Its speed v(t) is depicted by which (c) 1.21 (d) 0.83
of the following curves?
EBD_7839
4 KVPY-SA

30. When a block of iron floats in mercury at 0°C, a 36. Which has the highest e/m ratio ?
fraction k1 of its volume is submerged , with at a (a) He2+ (b) H+ (c) He+ (d) D+
temperature 60°C, a fraction k2 is seen to be 37. Which of the following compound is planar and
submerged. If the coefficient of volume non-polar?
expansion of iron is gFe and that of mercury is (a) XeO4 (b) SF4 (c) XeF4 (d) CF4
38. Which of the following will show aromatic
gHg, then the ratio k1 / k2 can be expressed as :
behaviour ?
1 + 60 g Fe 1 – 60 g Fe
(a) 1 + 60 g (b) 1 + 60 g
Hg Hg (a) (b)
1 – 60 g Fe 1 + 60 g Hg
(c) 1 – 60 g (d)
Hg 1 + 60 g Fe
CHEMISTRY (c) (d)
31. Na+, Mg2+, Al3+ and Si4+ ions are isoelectronic.
The value of ionic radii of these ions would be in 39. The oxidant which cannot act as reducing agent
the order : is:
(a) Na + > Mg 2+ > Al3+ > Si 4+ (a) SO2 (b) NO2 (c) CO2 (d) C1O2
(b) Na + < Mg 2+ < Al3+ < Si 4+ 40. Number of moles of NaOH present in 2 litre of
(c) Na + > Mg 2+ > Al3+ < Si 4+ 0.5 M NaOH is :
(a) 1.5 (b) 2.0 (c) 1.0 (d) 2.5
(d) Na + < Mg 2+ > Al3+ > Si 4+
41. The value of Planck's constant is 6.63 × 10–34 Js.
32. The heat of combustion of C, S and CS2 are –393.3 kJ,
–293.7 kJ, and –1108.76 kJ. What will be the heat The velocity of light is 3.0 × 108 m s–1. Which
of formation of CS2? value is closest to the wavelength in nanometers of
(a) –128.06 kJ (b) +970 kJ a quantum of light with frequency of 8 × 1015 s–1 ?
(c) +1108.7 kJ (d) +12 kJ (a) 3 × 107 (b) 2 × 10–25
(c) 5 × 10 –18 (d) 4 × 101
33. The IUPAC name of
O 42. The closed containers of the same capacity and
at the same temperature are filled with 44 g of H2
CH3 C CH CH CH3 is: in one and 44 g of CO2 in the other. If the pressure
of carbon dioxide in the second container is 1atm,
C2H5 CH3 then pressure of hydrogen in the first container
(a) 3-(Methylethyl) pentan-2-one would be:
(b) 3-(Methylethyl) pentan-4-one (a) 1 atm (b) 10 atm (c) 22 atm (d) 44 atm
(c) 3-Ethyl-4-methylpentan-2-one 43. Hyperconjugation involves overlap of the
(d) 3-Ethyl-2-methylpentan-4-one following orbitals:
34. The correct order of increasing pH values of the (a) s-s (b) s - p (c) p- p (d) p-p
aqueous solutions of baking soda, rock salt, 44. Which of the following metals burn in air at high
washing soda and slaked lime is: temperature with the evolution of heat ?
(a) Baking Soda < Rock Salt < Washing Soda (a) Cu (b) Hg (c) Pb (d) Al
< Slaked lime
45. The bond order in N +2 is:
(b) Rock Salt < Baking Soda < Washing Soda
< Slaked lime (a) 1.5 (b) 3.0 (c) 2.5 (d) 2.0
(c) Slaked lime < Washing Soda < Rock Salt BIOLOGY
< Baking Soda
(d) Washing Soda < Baking Soda < Rock Salt 46. Hormone which stimulates pancreas to secrete
< Slaked lime enzymes is:
35. The weight of oxalic acid required to neutralise (a) Enterocrinin (b) CCK-PZ
100 mL of normal NaOH is : (c) Gastrin (d) Secretin
(a) 6.3 g (b) 126 g (c) 530 g (d) 63 g
MOCK TEST-1 5

47. Which of the following accelerates extrinsic or 54. The acidic condition within the lysosome is
tissue thromboplastin formation? maintained by:
(a) Factor VII (proconvertin) (a) Digestive enzymes synthesised on RER
(b) Factor XI (plasma thromboplastin (b) Pumping Cl– ion out of lysosome
antecedent) (c) Pumping protons (H+) into the lysosome
(c) Heparin (d) All of the above
(d) Calcium 55. What is the role of a calcium ion (Ca2+) in muscle
48. Two pigment system theory of photosynthesis contraction?
was proposed by or concept of evidence for (a) It opens up a binding site for a myosin head
existence of two photosystems in on a thin filament.
photosynthesis was given by: (b) It opens up a binding site for an actin
(a) Hill (b) Blackmann molecule on a thick filament.
(c) Emerson (d) Arnon
(c) It binds to a myosin head.
49. A person suffers a permanent occlusion of a
(d) It release from a myosin head initiates a
blood vessel to the right side of the cerebrum.
power stroke.
The left side of the brain is unaffected. Which of
56. Person who is habitual of drinking (alcoholics)
the following could be a direct permanent result
is always short of which vitamin?
of this stroke?
1. Right arm weakness. (a) K (b) B (c) C (d) D
2. Loss of left knee-jerk reflex. 57. Take the odd one out.
3. Blindness in the left visual field without (a) Rabies, Influenza, AIDS
complete blindness in either eye. (b) Amoebiasis, Giardiasis, Trypanosomiasis
(a) Only 1 (b) Only 2 (c) Taeniasis, Ascariasis, Elephantiasis
(c) Only 3 (d) Both 1 and 2 (d) Cancer, Tuberculosis, Tetanus
50. Aquatic photodiffraction produces zones: 58. The process of migration of chloride ions from
(a) Euphotic, disphotic and aphotic plasma to RBC and carbonate ions from RBC to
(b) Aphotic, euphotic and disphotic plasma is:
(c) Euphotic, aphotic and disphotic (a) Chloride shift (b) Ionic shift
(d) Disphotic, aphotic and euphotic (c) Atomic shift (d) Na+ pump
51. You experience a rapid increase in heart rate, your 59. Which of the following food chain produces a
muscles tense, and your breathing becomes greater availability of food to next trophic level?
shallow. These changes in your physiology are (a) Diatoms ® Fish ® Large fish ® Birds
most likely mediated by: ® Man
(a) Neurotransmitters (b) Hormones (b) Green plant ® Rabbit ® Man
(c) Target cells (d) Receptors (c) Green plant ® Grasshopper ® Frog
52. Which of the following plant tissue culture ® Snake ® Peacock
shows totipotency? (d) Green plant ® Rabbit ® Wolf ® Man
(a) Meristem (b) Collenchyma 60. The major lymphatic vessel in the body is the
(c) Sieve tube (d) Xylem vessel thoracic duct. It empties into:
53. The controlling centre of autonomic nervous (a) liver
system is: (b) kidney
(a) Cerebrum (b) Spinal cord
(c) spleen
(c) Cerebellum (d) Medulla oblongata
(d) a vein other than the portal vein
EBD_7839
6 KVPY-SA
PART-II (2 MARKS QUESTIONS)

MATHEMATICS one fourth the circumference of the tube.


61. The smallest positive integer n, so that 999999 · Find the angle q that the radius to the interface
n = 111 …. 11 is: makes with the vertical in equilibrium position.
If the whole is given a small displacement from
1054 + 1 1054 - 1 its equilibrium position, and the resulting
(a) (b) oscillations are simple harmonic. Find the time
9(106 - 1) 9(106 - 1)
period of these oscillations.
1054 - 1 æ 1ö 1.803R
(c) (d) None of these (a) tan -1 ç ÷ , 2p
9(106 + 1) è 5ø g
62. There are four bus routes between A and B;
æ 1 ö 2p 1.803 R
and three bus routes between B and C. A man (b) cot -1 ç ÷ ,
can travel round-trip in number of ways by è 5ø g
bus from A to C via B. If he does not want to
use a bus route more than once, in how many 2p 1.803R 3
(c) cot ( 5) ,
-1
ways can he make round trip? g
(a) 72 (b) 144 (c) 14 (d) 19
63. A piece of paper is in the form of a sector, making 2 p 1.803 R3
an angle a. The paper is rolled to form a right (d) tan -1 ( 5) ,
g
circular cone of radius 5 cm and height 12 cm.
Then the value of angle a is 67. A particle is moving in a circle of radius r under
the action of a force F = ar2 which is directed
10p 9p 5p 6p towards centre of the circle. Total mechanical
(a) (b) (c) (d) energy (kinetic energy + potential energy) of the
13 13 13 13
particle is (take potential energy = 0 for r = 0):
64. Area of the quadrilaterals formed by joining the
mid points of the adjacent sides of a quadrilateral 1 3 5 3
is ____ the area of given quadrilateral. (a) ar (b) ar
2 6
(a) 1/8th (b) 1/2th (c) 1/4th (d) 3/4th
4 3
65. If APB and CQD are two parallel lines, then the (c) ar (d) ar3
bisectors of the angles APQ, BPQ, CQP and 3
PQD form 68. A plane mirror is held at a height h above the
(a) a square bottom of an empty beaker. The beaker is now
filled with water up to depth d. The general
(b) a rhombus
expression for the distance from a scratch at the
(c) a rectangle bottom of the beaker to its image in terms of h
(d) any other parallelogram and the depth d of water in the beaker is :
PHYSICS
66. Two non-viscous,
incompressible and
immiscible liquids of
densities r and 1.5 r
are poured into the R
two limbs of a circular O æ m ö d æ m - 1ö
(a) 2h – d ç (b) 2h –
tube of radius R and
q è m - 1÷ø 2 çè m ÷ø
small cross section
kept fixed in a vertical æ m - 1ö æ 2m - 1ö
plane as shown in fig. (c) 2h – d ç (d) 2h – d ç
Each liquid occupies è m ÷ø è m ÷ø
MOCK TEST-1 7
69. The xy - plane separates two media A and B of
refractive indices m1 = 1.5 and m2 = 2. A ray of 3
(i) 2B(s) + O 2 (g) ¾¾ ® B2O3 (s);
light travels from A to B. Its directions in the 2
two media are given by unit vectors û1 = aiˆ + bjˆ DH = –1273 kJ per mol
and û2 = ciˆ + djˆ . Then : 1
(ii) H 2 (g) + O 2 (g) ¾¾ ® H 2O(l);
(a) (a / c) = (4 / 3) (b) (a / c) = (3 / 4) 2
(c) (b / d) = (4 / 3) (d) (b / d) = (3 / 4) DH = –286 kJ per mol
70. A cylinder of mass Mc and sphere of mass Ms (iii) H 2 O(l) ¾¾ ® H 2 O(g);
are placed at points A and B of two inclines, DH = 44 kJ per mol
respectively (See Figure). If they roll on the (iv) 2B (s) + 3H2 (g) ¾¾ ® B2H6 (g);
incline without slipping such that their DH = 36 kJ per mol
sin qc (a) + 2035 kJ per mol (b) – 2035 kJ per mol
accelerations are the same, then the ratio
sin q s (c) + 2167 kJ per mol (d) – 2167 kJ per mol
is: 73. The Statue of Liberty is made of 2.0 × 105 lbs of
8 copper sheets bolted to a framework (1lb = 454
(a) g). How many atoms of copper are on the statue?
7 (Atomic weight: Cu = 63.5g/mole).
MC
A
15 (a) 2.1 × 1027 (b) 8.6 × 1029
(c) 4.3 × 10 26 (d) 8.6 × 1026
(b)
14 74. Select the incorrect graph for velocity of e– in
M
8 B S 1
(c) an orbit vs. Z, and n :
7 n
qC
15 qS
(d) C
D
14
v v
CHEMISTRY (a) (b)
+ n 1/n
71. In which case the N O2 will attack at the meta
position
CCl3 NO2
v v
(c) (d)

Z n
(i) (ii)
+
D /H 2O
+ ¾¾¾¾
® A;
NH 3 O– 75.
A (major product) is–
(a)

(iii) (iv)
(b)
(a) i, ii, iii (b) ii, iv
(c) ii and iii only (d) ii only
72. Diborane is a potential rocket fuel which (c)
under goes combustion according to th e
equation
B2 H 6 (g) + 3O 2 (g) ¾¾
® B 2O 3 (s) + 3H 2O(g) (d)
Calculate the enthalpy change for th e
combustion of diborane. Given
EBD_7839
8 KVPY-SA

BIOLOGY 79. On a warm summer’s day, the transpiration pull


is the main force that drives water from root
76. In an experiment, three bottles were filled with
parenchyma into the root xylem. The table shows
water from an aquatic ecosystem. This water
values of Yp (pressure potential) and Ys
contained tiny plants and animals of the
(solute potential) in root xylem and root
ecosystem. The following experiments were
parenchyma, in kPa. In which of the alternatives
done with the bottles.
(a – d) would transpiration pull cause water to
move from root parenchyma into the root xylem?
Bottle Condition Oxygen Oxygen
Number measurement (mg/L) Root Parenchyma Root Xylem
1 Control Done 9 yp ys yp yys
immediately (a) 200 –190 –200 5
2 Light for Done after 10 (b) –200 220 65 –5
one hour one hour (c) 200 –220 65 –5
3 Dark for Done after 4 (d) 200 –220 –65 –5
one hour one hour
80. In grasshopper, rosy body colour is caused by
a recessive mutation. The wild-type body colour
The gross primary productivity for this
is green. If the gene for body colour is on the X
ecosystem is:
chromosome, what kind of progeny would be
(a) 1 mg/L/h (b) 5 mg/L/h
(c) 6 mg/L/h (d) 14 mg/L/h obtained from a mating between a rosy female
77. Assume that the average amino acid residue and a wild-type male?
has a molecular weight of 110. The DNA strand (a) All the daughters will be green and all the
coding for a polypeptide chain of molecular sons will be rosy.
weight 11,000 has a length of: (b) 50% daughters will be green and 50% sons
(a) 300 nucleotides (b) 250 nucleotides will be rosy.
(c) 500 nucleotides (d) 800 nucleotides (c) All offspring will be green irrespective of
78. Turgor pressure is required to maintain the
sex.
shape of all cells except:
(a) Meristematic cells (b) Root cells (d) All offspring will be rosy irrespective of
(c) Collenchyma cells (d) Lignified cells sex.

ANS W ER KEYS
Part-I Part-II
1 (a) 11 (a) 21 (b) 31 (a) 41 (d ) 51 (a) 61 (b ) 71 (a)
2 (d ) 12 (d ) 22 (d) 32 (a) 42 (c) 52 (a) 62 (a) 72 (b)
3 (b ) 13 (d ) 23 (a) 33 (c) 43 (b ) 53 (d ) 63 (a) 73 (b)
4 (b ) 14 (c) 24 (a) 34 (b) 44 (d ) 54 (c) 64 (b ) 74 (d)
5 (b ) 15 (a) 25 (c) 35 (a) 45 (c) 55 (a) 65 (c) 75 (d)
6 (d ) 16 (b ) 26 (b) 36 (b ) 46 (b ) 56 (b ) 66 (a) 76 (c)
7 (b ) 17 (c) 27 (a) 37 (c) 47 (a) 57 (d ) 67 (b ) 77 (a)
8 (a) 18 (c) 28 (a) 38 (b) 48 (c) 58 (a) 68 (c) 78 (d)
9 (d ) 19 (a) 29 (b) 39 (c) 49 (c) 59 (b ) 69 (a) 79 (a)
10 (d ) 20 (c) 30 (a) 40 (c) 50 (a) 60 (d ) 70 (d ) 80 (a)
Telegram @unacademyplusdiscounts

Join Us on Telegram for More Such Books

https://telegram.me/unacademyplusdiscounts

Join us from the above link or search ''unacademyplusdiscounts'' in Telegram


21
Time : 3 Hours Stream-SA Maximum Marks : 100
INSTRUCTIONS
1. There are 80 questions in this paper.
2. The question paper contains two parts; Part–I (1 Mark Questions) and Part–II (2 Marks
Questions). There are four sections; Mathematics, Physics, Chemistry and Biology in each part.
3. There are four options given with each question, only one of them is correct.
4. For each incorrect answer 0.25 Mark in Part–I and 0.5 Mark in Part–II will be deducted.

PART-I (1 MARK QUESTIONS)

MATHEMATICS 5. If a, b, c, d are positive real numbers such that


a + b + c + d = 2, then M = (a + b) (c + d) satisfies
1. The vertex C is a right angle in the triangle ABC.
the relation
If the points D an d E are taken on the
hypotenuse, so that BC = BD and AC = AE, (a) 0 £ M £1 (b) 1 £ M £ 2
then, the sum of the perpendicular from D and E (c) 2 £ M £ 3 (d) 3 £ M £ 4
on AC and BC respectively is 2 2
6. If x + y = 25, xy = 12, then x =
(a) DE (b) DA (c) D B (d) CA (a) {3, 4} (b) {3, –3}
z (2 x + y ) (c) {3, 4, –3, –4} (d) {–3, –3}
2. If 3x = 5y = 75z, then the value of is
xy
are A. P., then æç + - ö÷
1 1 1 1 1 1
7. If , ,
(a) 0 (b) 1 (c) 2 (d) 3 a b c èa b cø
3. Find the values of k1 × k2, if p(x) = x + k1.x2011 + k2
2010
æ 1 1 1 ö is equal to
leaves remainder 0, when divided by (x – 1) and ç + - ÷
leaves remainder 4 when divided by (x + 1). èb c aø

(a) 3 (b) –3 (c) –2 (d) 2 4 3 b 2 – ac


4. The mean of a group of eleven consecutive (a) – (b)
ac b2 a 2b 2c2
natural numbers is m. What will be the
percentage change in the mean when next six 4 1
consecutive natural numbers are included in the (c) – (d) None of these
ac b2
group?
8. If a, b, c are distinct +ve real numbers and
m a2 + b2 + c2 = 1 then ab + bc + ca is
(a) m % (b) %
3 (a) less than 1 (b) equal to 1
m 300 (c) greater than 1 (d) any real no.
(c) % (d) % 9. In rectangle ABCD, AB = 5 and BC = 3. Points F
300 m and G are on line segment CD so that
EBD_7839
10 KVPY-SA

DF = 1 and GC = 2. Lines AG and FB intersect at (i) AD = DE


a
E. The area of DAEB is equal to . Find a. 1
14 (ii) ÐDAE = (ÐC – ÐB).
(a) 7 5 (b) 7 2 (c) 6 2 (d) 6 8 2
10. If n = 1 + x, where x is a product of four Then, which of the following is/are true?
consecutive positive integers, then which of the (a) Only (i) (b) Only (ii)
following not true ? (c) Both (i) and (ii) (d) None
(a) n is odd 15. The mean of three positive numbers is 10 more
(b) n is prime than the smallest of the numbers and 15 less
(c) n is a perfect square than the largest of the three. If the median of the
(d) None of these three numbers is 5, then the means of squares
11. A circular metallic sheet is divided into two parts of the numbers is
in such a way that each part can be folded into 2 2
a cone. If the ratio of their curved surface areas (a) 108 (b) 116
3 3
is 1 : 2, then ratio of their volumes is:
(a) 1 : 8 (b) 1: 16 1 2
(c) 208 (d) 216
3 3
(c) 1: 10 (d) 2 : 3
12. In a parallelogram ABCD, a point P is taken on PHYSICS
AD, such that AP : AD is 1 : n. If BP intersects 16. Two identical glass bulbs are interconnected by
AC at Q then AQ : AC is a thin glass tube. A gas is filled in these bulbs at
NTP. If one bulb is placed on ice and another
2 1 bulb is placed on hot bath, then the pressure of
(a) (b) the gas become 1.5 times. The temperature of
n +1 n -1
hot bath will be
1 2
(c) (d)
n +1 n -1
13. In the figure, OABCD is a sector of a circle. If
» » = CD
AB = BC » , then x =
Ice Hot bath
A
B (a) 100º C (b) 182º C
(c) 256º C (d) 546º C
17. A uniform cylinder of length L and mass M
x having cross-sectional area A is suspended, with
C
its length vertical, from a fixed point by a massless
spring such that it is half submerged in a liquid
of density s at equilibrium position. The
D O extension x 0 of the spring when it is in
equilibrium is:
(a) 105° (b) 120°
(c) 150° (d) 144° Mg Mg æ LAs ö
1–
k çè M ÷ø
14. ABC is a right angled triangle at A and AB > AC. (a) (b)
k
D bisects BC. DE is perpendicular to
hypotenuse BC and meets the bisector of the Mg æ LAs ö Mg æ LAs ö
1– 1+
k çè 2 M ÷ø k çè M ÷ø
(c) (d)
right angle A in E. Consider the following
statements.
MOCK TEST-2 11

18. A student uses a simple pendulum of exactly a(m/s2 ) a(m/s 2)


1 m length to determine g, the acceleration due
0.64 0.64
to gravity. He uses a stop watch with the least (a) (b)
count of 1 second for this and records 40 second x(m) x(m)
100 200 100 200
for 20 oscillations. For this observation, which
of the following statement (s) is (are) true? a(m/s 2) a(m/s 2)
(I) Error DT in measuring T, the time period, is 0.64 0.64
0.05 second 200 x(m)
(II) Error DT in measuring T, the time period, is (c) 100
(d) x(m)
100 200
1 second. –0.64 –0.64
(III) Percentage error in the determination of g
22. Circular part in the centre of retina of human eye
is 5%
is called
(IV) Percentage error in the determination of g
(a) Blind spot (b) Yellow spot
is 2.5%
(c) Red spot (d) None of these
(a) I and II (b) II and III
23. A student performs on experiment to determine
(c) I and III (d) IV and II
the Young’s modulus of a wire, exactly 2 m long,
19. The ice storm in the province of Jammu strained by Searle’s method. In a particular reading, the
many wires to the breaking point. In a particular student measures the extension in the length of
situation, the transmission pylons are separated the wire to be 0.8 mm with on uncertainty of ±
by 500 m of wire. The top grounding wire is 15° 0.05 mm at a load of exactly 1.0 kg. The student
from the horizontal at the pylons, and has a also measures the diameter of the wire to be 0.4
diameter of 1.5 cm. The steel wire has a density mm with an uncertainty of ± 0.01 mm. Take g =
of 7860 kg/m3. When ice (density 900 kg/m3) built 9.8 m s–2(exact). The Young’s modulus obtained
up on the wire to a total diameter of 10.0 cm, the from the reading is close to
wire snapped. What was the breaking stress (a) (2.0 ± 0.3) × 1011 Nm–2
(force/unit area) in N/m2 in the wire at the (b) (2.0 ± 0.2) × 1011 Nm–2
breaking point? You may assume the ice has no (c) (2.0 ± 0.1) × 1011 Nm–2
strength. (d) (2.0 ± 0.05) × 1011 Nm–2
(a) 7.4 × 107 N/m2 (b) 4.5 × 108 N/m2 24. Three elephants A, B and C are moving along a
6
(c) 2.6 × 10 N/m 2 (d) 1.15 × 107 N/m2 straight line with constant speed in same
20. The dimensions of Stefan’s Boltzmann constant direction as shown in figure. Speed of A is 5 m/s
s can be written in terms of Plank’s constant h, and speed of C is 10 m/s. Initially separation
Boltzmann constant kB and speed of light c as between A and B is ‘d’ and between B and C is
s = h a k Bb c g , then also d. When ‘B’ catches ‘C’ separation between
(a) a = 3, b = 4, g = –3 A and C becomes 3d. Then the speed of B will
(b) a = 3, a = –4, a = 2 be –
(c) a = –3, b = 4, g = –2 5 m/s u 10 m/s
(d) a = 2, a = –3, a = –1
21. The v - x graph for a car in a race on a straight A B C
road is given.
v(m/s)
d d

8 (a) 7.5 m/s (b) 15 m/s


(c) 20 m/s (d) 5 m/s
25. An air bubble in a water tank rises from the
x (m)
100 200 bottom to the top. Which of the following
Identify the correct a – x graph statements are true :
EBD_7839
12 KVPY-SA

(I) Bubble rises upwards because pressure at 30. A pulley of radius 2 m is rotated about its axis by
the bottom is less than that at the top. a force F = (20t – 5t2) newton (where t is mea-
(II) Bubble rises upwards because pressure at sured in seconds) applied tangentially. If the
the bottom is greater than that at the top. moment of inertia of the pulley about its axis of
(III) As the bubble rises, its size increases. rotation is 10 kg-m2 the number of rotations made
(IV) As the bubble rises, its size decreases. by the pulley before its direction of motion is
(a) II and III (b) II and IV reversed, is:
(c) I and IV (d) III and IV (a) more than 3 but less than 6
26. In a certain region of space, the potential field (b) more than 6 but less than 9
depends on x and y-coordinates as (c) more than 9
V = (–x2 + y2). The corresponding electric field (d) less than 3
lines in xy-plane are correctly represented by
CHEMISTRY
y y
31. Consider a real gas placed in a container. If the
(a) x (d) x inter-molecular attractions are supposed to
disappear suddenly which of the following
would happen?
y (a) Pressure decreases
y (b) Pressure increases
(c) Pressure remains unchanged
(c) x (d) x (d) Gas collapse
32. Conjugate acid of HF2- is
(a) H+ (b) HF
(c) F2 (d) H2F2
27. The splitting of white light into several colours 33. Two elements A and B on burning in air give
on passing through a glass prism is due to corresponding oxides. Oxides of both A and B
(a) Refraction (b) Reflection
are soluble in water. The aqueous solution of
(c) Interference (d) Diffraction
oxide of A is alkaline and reacts with aqueous
28. Two different masses m and 3m of an ideal gas
solution of oxide of B to give another compound.
are heated separately in a vessel of constant
Identify A and B
volume, the pressure P and absolute temperature
(a) A and B both are metals
T, graphs for these two cases are shown in the
figure as A and B. The ratio of slopes of curves B (b) A and B are non-metals
to A is (c) A is metal and B is non-metal
P B
(a) 3 : 1 (d) A is non-metal and B is metal
(b) 1 : 3 A 34. Which of the following is strongest nucleophile?
3m
(c) 9 : 1 m (a) Br– (b) : OH–
-
(d) 1 : 9 T (c) : CN (d) C6 H5O :
29. The resistance of a wire is R. It is bent at the 35. Which of the following metal sulphides has
middle by 180° and both the ends are twisted maximum solubility in water?
together to make a shorter wire. The resistance (a) CdS (Ksp = 36 × 10–30)
of the new wire is (b) FeS (Ksp = 11 × 10–20)
(a) 2 R (b) R/2 (c) HgS (Ksp = 32 × 10–54)
(c) R/4 (d) R/8 (d) ZnS (Ksp = 11 × 10–22)
MOCK TEST-2 13

36. In the Kjeldahl’s method for estimation of (a) Absorbs CO2 and increases O2 content.
nitrogen present in a soil sample, ammonia (b) Eliminates moisture.
evolved from 0.75 g of sample neutralized 10 mL (c) Absorbs CO2.
of 1 M H2SO4. The percentage of nitrogen in the (d) Produces ozone.
soil is : 42. What is the IUPAC name of the compound ?
(a) 37.33 (b) 45.33
(c) 35.33 (d) 43.33
37. Which of the following describes the best H
relationship between the methyl groups in the CH3 Cl
chair conformation of the substance shown
below? (a) 2 - Chloro - 2 - butene
(b) 3 - Chloro - 1 - butene
CH3 (c) 3 - Methyl - 3 - chloropropene - 1
CH3 (d) 3 - Chloro - 3 - methyl - 1 - propene
(a) Trans (b) Anti 43. Which molecular geometry is least likely to result
(c) Gauche (d) Eclipsed from a trigonal bipyramidal electron geometry?
38. Which graph shows how the energy (E) of a (a) Trigonal planar (b) See-saw
photon of light is related to its wavelengths (l) ? (c) Linear (d) T-shaped
44. Cyclohexene on reaction with cold alkaline
KMnO4 forms,
­ ­ (a) trans-hexanediol (b) hexadiketone
E E
(a) (b) (c) cis-hexanediol (d) pentadiketone
l ® l® 45. Which of the following has unpaired electron(s)?

(a) N2 (b) O-2

­ ­ (c) N 22+ (d) O 22 -


E E
(c) (d)
BIOLOGY
l® l®
46. Which is th e most abun dant gas in our
39. Which of the underlined atoms in the molecules atmosphere that cannot be utilised by plants
shown below have sp-hybridization? directly in its atmospheric form and is, therefore,
(u) CH2CHCH3 (v) CH2CCHCl captured by certain bacteria that live
(w) CH 3CH +2 (x) H — C ºº C — H symbiotically in the nodules of roots?
(y) CH3CN (z) (CH3)2CNNH2 (a) Oxygen (b) Nitrogen
(a) x and z (b) x, y and z (c) Neon (d) Hydrogen
(c) u, w and x (d) v, x and y 47. Phospholipids are important cell membrane
40. The following quantum numbers are possible constituents, because they:
for how many orbital(s) n = 3, l = 2, m = +2 ? (a) contain glycerol.
(a) 1 (b) 3 (c) 2 (d) 4 (b) can form bilayers in water.
(c) combine covalently with proteins.
41. KO2 is used in oxygen cylinders in space and
(d) contain polar and non-polar portions.
submarines because it:
EBD_7839
14 KVPY-SA

48. Allergens are non-infectious foreign substances 55. The number of essential amino acids is:
that: (a) 9 (b) 8 (c) 10 (d) 14
(a) Increases the secretion of IgA 56. All are the products of anaerobic respiration
(b) Increases the secretion of IgE
except:
(c) Increases the secretion of IgG
(d) Increases the secretion of IgM (a) Ethyl alcohol (b) Lactic acid
49. Which of the following is not found in birds? (c) Fumaric acid (d) Butyric acid
(a) Hind limb (b) Fore limb 57. Cell mediated immunity, also involved in the
(c) Pelvic girdle (d) Pectoral girdle killing of the cancerous cells, is the responsibility
50. A drop of each of the following, is placed of which of the following group of cells?
separately on four slides. Which of them will (a) B- cells (b) CT- cells
not coagulate?
(c) HT- cells (d) Malignant cells
(a) Whole blood from pulmonary vein
(b) Blood plasma 58. A patient is generally advised to specially
(c) Blood serum consume more meat, lentils, milk and eggs in diet
(d) Sample from the thoracic duct of lymphatic only when he suffers from:
system (a) Scurvy (b) Kwashiorkar
51. Cancer cells are :
(c) Rickets (d) Anaemia
(a) HeLa cells (b) CD4 cell
59. The tumour inducing capacity of Agrobacterium
(c) Memory cell (d) Plasma cell
52. Which of the following options does not hold tumefaciens is located in large extrachromosomal
good regarding anaerobic respiration or plasmid called:
fermentation? (a) Ri plasmid (b) Lambda phage
(a) Occurs inside the mitochondrion (c) pBR322 (d) Ti plasmid
(b) Partial breakdown of glucose occurs
60. The process by which a lake ecosystem is altered
(c) Net gain of only 2 ATP molecules
(d) None of these by eutrophication involves several stages, each
53. Phloem parenchyma is absent in: causing the next. Which of the following stages
(a) Dicots (b) Monocots would occur second in the chain of causation?
(c) Gymnosperms (d) All of these (a) Algal blooms occur.
54. Which among the following has highest water (b) Oxygen levels drop in deeper water.
potential?
(c) Phosphorus input from sewage and
(a) 1 M salt solution
(b) 1 M glucose solution agricultural runoff increases.
(c) Distilled water (d) Respiratory demand from decomposers
(d) None of the above increases.

PART-II (2 MARKS QUESTIONS)

MATHEMATICS 62. The number of 3 letters words, with or without


meaning which can be formed out of the letters
61. Suppose n is an integer such that the sum of
of the word ‘NUMBER’.
digits of n is 2, and 1010 < n < 1011. Find the
Statement I : When repetition of letters is not
number of different values of n.
allowed is 120.
(a) 9 (b) 10
Statement II : When repetition of letters is
(c) 11 (d) 12
allowed is 216.
MOCK TEST-2 15

Choose the correct option. (II) The net elongation of the spring is
(a) Only Statement I is correct
8pR3rg
(b) Only Statement II is correct
3K
(c) Both I and II are correct
(d) Both I and II are false (III) The light sphere is partially submerged
63. In a parallelogram ABCD, A point ‘K’ is taken on (IV) The light sphere is completely submerged
diagonal BD. When AK is extended, it intersects Select correct statement/(s)
CD and BC at M and L respectively. If AK = 6 (a) I and IV (b) II only
units, MK = 4 units, then LM is
(c) II and IV (d) I and II
(a) 9 (b) 6
67. The phase space diagram for a ball thrown
(c) 5 (d) 24 vertically up from ground is

1 Momentum Momentum
64. A conical cup contains water equivalent to
27
of its volume, then ratio of depth of water to the (a) (b)
Position Position
height of cone is
(a) 1 : 6 (b) 1 : 4
(c) 1 : 2 (d) 1 : 3
Momentum
65. If a rectangle and parallelogram have equal area
and have same base and are situated on the same
side, then the quotient of (c) (d)
Position Position

Perimeter of rectangle
Perimeter of parallelogram is- 68. A massless piston divides a closed, thermally
(a) 1 insulated cylinder into two equal parts. One part
(b) >1 contains M = 28 g of nitrogen. At this temperature,
(c) <1 one third of molecules are dissociated into atoms
and the other part is evacuated. The piston is
(d) Cannot be determined
released and the gas fills the whole volume of the
PHYSICS cylinder at temperature T0. Then, the piston is
66. A solid sphere of radius R and density r is slowly displaced back to its initial position.
attached to one end of a mass-less spring of Calculate the increase in internal energy of the
force constant K. The other end of the spring is gas. Neglect further dissociation of molecules
connected to another solid sphere of radius R during, the motion of the piston.
and density 3r. The complete arrangement is
placed in a liquid of density 2r and is allowed to (a)
3
8
RT0 ( 2 –1 ) (b)
8
3
RT0 ( 2 –1 )
reach equilibrium. The correct statement is
(I) The net elongation of the spring is

4pR3rg (c)
1
3
RT0 ( 2 –1 ) (d)
1
8
RT0 ( 2 –1 )
3K
EBD_7839
16 KVPY-SA

69. A target is made of two plates, one of wood and (c) In NH3 the atomic dipole and bond dipole
the other of iron. The thickness of the wooden are in the opposite directions, whereas in
plate is 4 cm and that of iron plate is 2 cm. A NF3 these are in the same direction.
bullet fired goes through the wood first and then
(d) In NH3 as well as in NF3, the atomic dipole
penetrates 1 cm into iron. A similar bullet fired
with the same velocity from opposite direction and bond dipole are in the same direction.
goes through iron first and then penetrates 2 cm 72. Arrange the following substances in order of
into wood. If a1 and a2 be the retardations offered decreasing heat of combustion (maximum ®
to the bullet by wood and iron plates minimum).
respectively, then
(a) a1 = 2a2 (b) a2 = 2a1
(c) a1 = a2 (d) data insufficient
70. A cylindrical rod of length l = 64 cm and cross-

( )
sectional radius r = 2 / p cm is placed at a
distance 50 r from a infrared point source S of
power 1.25 kW as shown in Fig.

(a) I > II > IV > III (b) III > IV > II > I
(c) II > IV > I > III (d) I > III > II > IV
73. A mixture of O2 and gas "Y" (mol. mass 80) in
The lateral surface of the rod is perfectly
the mole ratio a : b has a mean molecular mass
insulated from the surroundings. The cross
40. What would be mean molecular mass, if the
section A absorbs 80% of the incident energy,
gases are mixed in the ratio b : a under identical
and has temperature TA in steady state. The
conditions? (Assume that gases are
surface B is radiating energy into space and the
non-reacting)
wavelength emitted by it with maximum energy
(a) 40 (b) 48 (c) 62 (d) 72
density is 100,000 Å. Determine the temperature
of end B and find the value of TA if conductivity 74. A gaseous mixture contains three gases A, B
and C with a total number of moles of 10 and
T total pressure of 10 atm. The partial pressure of
varies with temperature as K = . Assume that
TA A and B are 3 atm and 1 atm respectively. If C
the rate of flow of heat through the rod is steady. has molecular weight of 2 g/mol then, the weight
(Wein’s constant = 0. 003 mK) of C present in the mixture will be :
(a) 8 g (b) 12 g (c) 3 g (d) 6 g
(a) 500 K and 300 K (b) 50 K and 30 K
75. Arrange in the order of increasing acidity.
(c) 300 K and 500 K (d) 50° C and 30° C
CHEMISTRY
OH NH2
71. The electronegativity difference between N and
F is greater than that between N and H, yet the (I) (II)
dipole moment of NH3 (1.5 D) is larger than that
of NF3 (0.2D). This is because OH
(a) In NH3, the atomic dipole and bond dipole
(III)
are in the same direction, whereas in NF3
these are in opposite directions.
(a) III < I < II (b) I < III < II
(b) In NH3 as well as NF3, the atomic dipole and
(c) III < II < I (d) II < I < III
bond dipole are in opposite directions.
MOCK TEST-2 17

BIOLOGY (c) (d)


76. The nucleus of a human liver cell contains 6 (c) (d)
picograms (pg) of DNA. What amount of DNA
is likely to found in the nucleus of an actively
dividing human skin cell at the end of Hydrophilic end
interphase? Hydrophobic end
(a) 12 pg (b) 6 pg (c) 24 pg (d) 18 pg Phospholipid
77. Three uniformly watered plants A, B and C were 80. Photosynthetic features of four plants P, Q, R
kept in 45% relative humidity, 45% relative and S are depicted in the graphs below.
humidity with blowing wind and 95% relative
humidity respectively. Arrange these plants in

Uptake of CO2
the order (fastest to slowest) in which they dry P
up.
(a) A= B, C (b) B, A, C Q
(c) C, B, A (d) C, A = B
78. A scientist introduced bacterial plasmids into Leaf temperature
baker ’s yeast. However, the cells lost these
plasmids over a period of time. Which of the
Uptake of CO2

following needs to be inserted into the bacterial R


plasmid to overcome this problem?
(a) Centromere S
(b) Yeast origin of replication
(c) Telomere Concentration of CO2 within
(d) Bacterial origin of replication the intercellular space
79. A red blood corpuscle (RBC) was kept in a These plant species belong to:
solution and treated so that it became inside - (a) P: C3 plant Q: C4 plant R: CAM plant
out. What will be the polarity of the phospholipid S: shade plant
(b) P: C4 plant Q: C3 plant R: C4 plant
bilayer in this cell? S: C3 plant
(a) (b) (c) P: C4 plant Q: sun plant R: C3 plant
S: CAM plant
(a) (b) (d) P: C3 plant Q: C4 plant R: C3 plant
S: C4 plant
EBD_7839
18 KVPY-SA

ANS W ER KEYS
Part-I Part-II
1 (a) 11 (c) 21 (c) 31 (b) 41 (a) 51 (a) 61 (c) 71 (a)
2 (b ) 12 (c) 22 (b) 32 (d) 42 (b ) 52 (a) 62 (c) 72 (c)
3 (c) 13 (c) 23 (b) 33 (c) 43 (a) 53 (b ) 63 (c) 73 (d)
4 (d ) 14 (c) 24 (b) 34 (c) 44 (c) 54 (c) 64 (d ) 74 (b)
5 (a) 15 (d ) 25 (a) 35 (b) 45 (b ) 55 (a) 65 (c) 75 (d)
6 (c) 16 (d ) 26 (b) 36 (a) 46 (b ) 56 (c) 66 (a) 76 (a)
7 (a) 17 (c) 27 (a) 37 (c) 47 (d ) 57 (b ) 67 (d ) 77 (b)
8 (a) 18 (c) 28 (a) 38 (d) 48 (b ) 58 (b ) 68 (b ) 78 (b)
9 (a) 19 (b ) 29 (c) 39 (d) 49 (b ) 59 (d ) 69 (b ) 79 (a)
10 (b ) 20 (c) 30 (a) 40 (a) 50 (c) 60 (a) 70 (c) 80 (b)
Telegram @unacademyplusdiscounts

Join Us on Telegram for More Such Books

https://telegram.me/unacademyplusdiscounts

Join us from the above link or search ''unacademyplusdiscounts'' in Telegram


31
Time : 3 Hours Stream-SA Maximum Marks : 100
INSTRUCTIONS
1. There are 80 questions in this paper.
2. The question paper contains two parts; Part–I (1 Mark Questions) and Part–II (2 Marks
Questions). There are four sections; Mathematics, Physics, Chemistry and Biology in each part.
3. There are four options given with each question, only one of them is correct.
4. For each incorrect answer 0.25 Mark in Part–I and 0.5 Mark in Part–II will be deducted.

PART-I (1 MARK QUESTIONS)

MATHEMATICS 5. If the bisectors of two adjacent angles A and B of


a quadrilateral ABCD intersect at a point O such
1. In an international convention, 5 participants from
that ÐC + ÐD = k ÐAOB, then find the value of
each of USA, China and Russia were arranged
k – 1 is
around a circle. In how many ways this
arrangement can be made if participants from the 5 6
(a) 1 (b) (c) 2 (d)
same country are always together? 3 2
6. A number N is divisible by 3 and 4 but not by 9 then
(a) (3!)(5!)(5!)(5!) (b) (2!)(5!)(5!)(5!)
which one of the following cannot be an integer?
(c) (3!)(5!)(5!)(5!) (d) None of these
(a) N/6 (b) N/42
2. Two numbers A and B are such that their HCF is 72
(c) N/18 (d) N/21
and when written in prime factorization form
A = 2a3b7c while B = 2d3e5f where all a, b, c, d, e and 7. Consider the following situation : –
f are distinct natural numbers. Find the minimum (i) A set S = { 1, 2, 3, ….., 19, 20} i.e 1st 20 positive
value of LCM of two numbers A and B. integers.
(a) 25345271 (b) 26345271 (ii) Another set P is a subset of set “S” and
(c) 26335171 (d) 26355471 minimum number of elements in set P is 2.
1 7 (iii) HCF of any two elements of set P is not more
3. If, a, b, c are real numbers such that a + = ;
b 3 than 2.
1
b + = 4; then value of abc is: What is the maximum number of elements that
c
set P can have?
(a) 0‘ (b) 4 (c) 1 (d) 2
(a) 9 (b) 10 (c) 11 (d) 13
4. Let ABC be a triangle in which AB = AC and
ÐCAB = 90°. Suppose M and N are points on 8. Let T be the set of integers {3, 11, 19, 27... 451,
hypotenuse BC such that BM2 + CN2 = MN2. 459, 467} and S be a subset of T such that the
Then ÐMAN = sum of no two elements of S is 470. The maximum
(a) 45° (b) 60° possible number of elements in S is
(c) 30° (d) None of these (a) 32 (b) 28 (c) 29 (d) 30
EBD_7839
20 KVPY-SA

9. In a Annual general meeting of XYZ company, 15. If x, y are real numbers such that
x x
participants are ‘2a’ number of executives, ‘b’ +1 -1
y y
number of managers, ‘c’ number of senior 3 -3 = 24 then the value of (x + y)/(x – y)
managers and director of the company. The is
seating arrangement is made in such a way that (a) 0 (b) 1 (c) 2 (d) 3
all the executives are seated at the either end (‘a’
number of executives on one end), All the PHYSICS
managers will be together while Director don’t 16. A stone weighs (10.0 ± 0.1) kg in air. The weight
want to sit next to an executive or a manager, in of the stone in water is (5.0 ± 0.1) kg. Find the
how many ways they can be arranged in a straight maximum percentage error in the measurement
line? of specific gravity.
(a) {(a)!}{(a)!}(b!){(c+1)!}{cP2}
(a) 5% (b) 0.5%
(b) {(2a)!}(b!){(c+1)!}{cP2}
(c) 0.25% (d) 0.2%
(c) {(2a)!}(b!){(c)!}{cP2}
17. Which of the following graph cannot possibly
(d) None of these
represent one dimensional motion of a particle?
10. When you reverse the digits of the number 13, x x
the number increases by 18. How many other two
digit numbers increase by 18 when their digits (a) t (b) t
are reversed?
(a) 5 (b) 6 (c) 7 (d) 8 speed
11. ABC is a triangle and AD is the median. If the
coordinates of A are ( 4, 7, –8) and the coordinates (c) (d) All of these
of centroid of the triangle ABC are (1, 1, 1), what t
are the coordinates of D?
æ 1 ö æ 1 11ö 18. While an aquarium is being filled with water, a
(a) ç – , 2,11÷ (b) ç – , –2, ÷ motionless fish looks up vertically through the
è 2 ø è 2 2ø
surface of the water at a monochromatic plane
(c) (–1, 2, 11) (d) (–5, –11, 19) wave source of frequency f. If the index of
12. x and y are two non-negative numbers such that refraction of water is m and water level rises at a
2x + y = 10. The sum of the maximum and minimum rate of dh/dt, the shift in the frequency df/f, that
values of (x + y) is the fish observes is (velocity of light is c) :
(a) 6 (b) 9 (c) 10 (d) 15 ( m - 1) dh / dt m dh
13. Three circles each of radius 1 touch one another (a) (b)
c c dt
externally and they lie between two parallel lines.
c dh / dt c dh
The minimum possible distance between the lines (c) (d)
is. ( m - 1) m dt
(a) 2 + 3 (b) 3 + 3 19. Which of the following is/are correct.
I. An object moving the earth under the
1
(c) 4 (d) 2 + influence of Earth’s gravitational force is in
3 a state of “free-fall”.
14. In a small village, there are 87 families, of which II. The dominant effect causing a slowdown in
52 families have at most 2 children. In a rural the rotation of the earth is the gravitational pull
development programme 20 families are to be of other planets in the solar system.
chosen for assistance, of which at least 18 III. Generally the path of a projectile from the
families must have at most 2 children. In how earth is parabolic but it is elliptical for
many ways can the choice be made? projectiles going to a very large height.
(a) 52C18 × 35C2 IV. The path of a projectile is independent of
(b) 52C18 × 35C2 + 52C19 × 35C1 + 52C20 the gravitational force of earth.
(c) 52C18 + 35C2 + 52C19 (a) I and II (b) II and III
(d) 52C18 × 35C2 + 35C1 × 52C19 (c) I and III (d) I and IV
MOCK TEST-3 21

20. A calorimeter (of water equivalent 50 g) contains 24. An a particle is moving along a circle of radius R
250 g of water and 50 g of ice at 0°C. 30 g of water with a constant angular velocity w. Point A lies
at 80°C is added to it. The final condition of the in the same plane at a distance 2R from the centre.
system will be : Point A records magnetic field produced by a
(a) the temperature of the system will be 4.2°C. particle. If the minimum time interval between
(b) the temperature of the system will still be two successive times at which A records zero
0°C and the entire ice will melt. magnetic field is t, the angular speed w, in terms
(c) the temperature will be 0°C and half of the of t is
ice will melt. (a) 2p/t (b) 2p/3t
(d) the temperature will be 0°C and 20 g of ice (c) p/3t (d) p/t
will left. 25. An ideal monoatomic gas is confined in a
21. Two identical balls A and B each of mass 0.1 kg horizontal cylinder by a spring loaded piston
are attached to two identical massless springs. (as shown in the figure). Initially the gas is at
The spring mass system is constrained to move temperature T1, pressure P1 and volume V1 and
inside a rigid smooth pipe bent in the form of a the spring is in its relaxed state. The gas is then
circle as shown in the figure. The pipe is fixed in heated very slowly to temperature T2, pressure
a horizontal plane. The centres of the balls can P2 and volume V2. During this process the piston
move in a circle of radius 0.06 m. Each spring has moves out by a distance x. Ignoring the friction
a natural length of 0.06p m and force constant between the piston and the cylinder, the incorrect
0.1 N/m. Initially both the balls are displaced by statement is
an angle q = p/6 radian with respect to the
diameter PQ of the circle and released from rest.
The frequency of oscillation of the ball B is
(a) p Hz
1 (a) If V2 = 2V1 and T2 = 3T1, then the energy
(b) Hz 1
p stored in the spring is P1V1
4
(c) 2p Hz (b) If V2 = 2V1 and T2 = 3T1, then the change in
1 internal energy is 3P1V1
(d) Hz
2p (c) If V2 = 3V1 and T2 = 4T1, then the work done
bT 2 by the gas is P1V1
22. The emf is given by E = aT + and a = (d) If V2 = 3V1 and T2 = 4T1, then the heat sup-
2
-1 17
10µV/ºC2 , b = µV/ºC2. If temperature of cold plied to the gas is PV
20 6 1 1
junction is zero find neutral temperature and
temperature of inversion ? 26. Mark the correct statements.
(a) 200°C and 400°C (b) 100°C and 200°C I. Gravitational potential at the centre of
(c) 20°C and 40°C (d) 400°C and 20°C curvature of a thin hemispherical shell of
23. Which of the following graphs correctly radius R and mass M is equal to GM/R
represents the relation between (ln E) and II. Gravitation field strength at a point lying
(ln T) where E is the amount of radiation emitted on the axis at a distance of ‘a’ on a thin,
per unit time from unit area of a body and T is
the absolute temperature uniform circular ring of radius R and mass
ln E ln E GMR
M is equal 2
(a) (b) ( a + R 2 )3 2
III. Newton's law of gravitation for gravitational
ln T ln T
force between two bodies is applicable only
ln E
ln E when bodies have sphrical symmetric
distribution of mass
(c) (d)
ln T (a) I and II (b) II and III
ln T
(c) I and III (d) None of these
EBD_7839
22 KVPY-SA

27. A solid homogeneous sphere of mass M and (b) The high boiling point of water.
radius R is moving on a rough horizontal surface, (c) Solubility of polar compounds in water.
partly rolling and partly sliding. During this kind (d) Ability of water to form coordinate covalent
of motion of the sphere bond.
(a) total kinetic energy is conserved 32. 1, 2-Dibromocyclohexane on dehydrobro-
(b) the angular momentum of the sphere about mination gives :
the point of contact with the plane is
conserved (a) (b)
(c) only the rotational kinetic energy about the
centre of mass is conserved
(d) angular momentum about the centre of mass (c) (d)
is conserved.
28. A large number of droplets, each of radius a,
coalesce to form a bigger drop of radius b. Br
Assume that the energy released in the process 33. Electronic configuration of an atom is 2, 8, 1.
is converted into the kinetic energy of the drop. Which of the following elements is similar with
The velocity of the drop is (S = surface tension it in chemical reactivity?
and r = density of drop) (a) K (b) Cl (c) N (d) Ar
1/2 1/2 34. The number of secondary hydrogens in
é3 S æ 1 1ö ù é 6 S æ 1 1ö ù 2, 2-dimethylbutane is :
(a) ê ç - ÷ú (b) ê r çè a - b ÷ø ú
ë r è a bø û ë û (a) 8 (b) 6 (c) 4 (d) 2
1/2 1/2 35. Which of the following sets of quantum numbers
é 2 S æ 1 1ö ù é S æ 1 1ö ù
(c) ê çè - ÷ø ú (d) ê çè - ÷ø ú is correct for an electron in 4f orbital ?
ë r a b û ër a b û (a) n = 4, l = 3, m = + 1, s = + ½
29. A cyclist is riding with a speed of 27 km/h. As he
approaches a circular turn on the road of radius (b) n = 4, l = 4, m = – 4, s = – ½
80 m, he applies brakes and reduces his speed at (c) n = 4, l = 3, m = + 4, s = + ½
the constant rate of 0.50 m/s every second. What (d) n = 3, l = 2, m = – 2, s = + ½
is the magnitude and direction of the net 36. Which solution has pH equal to 10 ?
acceleration of the cyclist on the circular turn ? (a) 10–4 M KOH (b) 10–10 M KOH
(a) 8.6 m/sec2, 45°28¢ (b) 6.8 m/sec2, 54°28¢
(c) 10–10 M HCl (d) 10–4 M HCl
(c) 0.86 m/sec2, 54°28¢ (d) 0.68 m/sec2, 45°28¢
30. A diver looking up through the water sees the 37. Bleaching powder is soluble in cold water giving
outside world contained in a circular horizon. a milky solution due to –
(a) available chlorine
4
The refractive index of water is and the diver’ss (b) lime present in it
3
eyes are 15 cm below the surface of water. Then (c) calcium carbonate formation
the radius of the circle is: (d) the absorption of carbon dioxide from
(a) 15 ´ 3 ´ 5 cm (b) 15 ´ 3 7 cm atmosphere
38. What is the weight of oxygen required for the
15 ´ 7 15 ´ 3
(c) cm (d) cm complete combustion of 2.8 kg of ethylene ?
3 7
(a) 2.8 kg (b) 6.4 kg (c) 9.6 kg (d) 96 kg
CHEMISTRY 39. The structure of the noble gas compound XeF4
is
31. If one assumes linear structure instead of bent
structure for water, then which one of the (a) square planar
following properties cannot be explained? (b) distorted tetrahedral
(a) The formation of intermolecular hydrogen (c) tetrahedral
bond in water. (d) octahedral
MOCK TEST-3 23

40. Fire extinguishers contain H2SO4 and which one 48. In terms of being open or closed, what is the
of the following? state of the mitral and tricuspid valves (left and
right atrioventricular valves, respectively) at the
(a) NaHCO3 and Na2CO3 both
end of the first heart sound?
(b) Na2CO3 only (a) Both are closed
(c) NaHCO3 only (b) Both are open
(c) Mitral is closed, tricuspid is open
(d) CaCO3 only
(d) Mitral is open, tricuspid is closed
41. What happens when Al is added to KOH 49. Complete failure of the anterior lobe of pituitary
solution? causes:
(a) No reaction takes place (a) Conn’s disease
(b) Oxygen is evolved (b) Acromegaly
(c) Cushing’s disease
(c) Water is produced
(d) Simmond’s disease
(d) Hydrogen is evolved 50. During translation, proteins are synthesised by:
42. Which pair of atoms form strongest ionic bond? (a) ribosomes using the information on DNA.
(a) Al and As (b) Al and N (b) lysosome using the information on DNA.
(c) ribosome using the information on mRNA.
(c) Al and Se (d) Al and O
(d) lysosome using the information on mRNA.
43. In which of the following ways does the hydride 51. Hydrolysis of maltose gives rise to
ion tend to function ? (a) two molecules of glucose.
(a) An electrophile (b) A nucleophile (b) two molecules of galactose.
(c) A free radical (d) An acid (c) one molecule of glucose and one molecule
of galactose.
44. Le-Chatelier principle is not applicable to : (d) one molecule of glucose and one molecule
ˆˆ† 2HI (g)
(a) H 2 (g) + I2 (g) ‡ˆˆ of fructose.
ˆˆ† FeS(s)
(b) Fe (s) + S(s) ‡ˆˆ 52. Chargaff’s rules of base pairing states that:
(a) the ratio of purines to pyrimidines is
ˆˆ† 2NH3 (g)
(c) N 2 (g) + 3H 2 (g) ‡ˆˆ
roughly equal in all tested organisms.
ˆˆ† 2NO (g)
(d) N 2 (g) + O 2 (g) ‡ˆˆ (b) the ratio of A to T is roughly equal in all
45. Which of the following would not rearrange to tested organisms.
a more stable form? (c) the ratio of A + T and G + C is roughly equal
in all tested organisms.
+ (d) Both (a) and (b)
(a) (b) H 53. In the Hardy-Weinberg equation, the term 2 pq
represents the
+ (a) overall gene frequency of the population.
(c) + (d) (b) frequency of both homozygous genotypes.
(c) frequency of the heterozygous genotype.
BIOLOGY (d) allele frequencies of the population.
54. The glomerular filtrate consists of:
46. In the genetic code dictionary, how many codons (a) urea, sodium chloride, fibrinogen and water
are used to code for all the 20 essential amino (b) glucose, amino acids, urea, oxytocin and
acids? calcitonin
(a) 60 (b) 20 (c) 64 (d) 61 (c) Both (a) and (b)
47. Hemichordates have now been placed with the (d) urea, glucose, salts and water
non-chordates, close to echinoderms, because 55. Choose the mismatch pair.
(a) notochord is absent. (a) Glutamic acid – Acidic
(b) pharyngeal gill-slits are lacking. (b) Lysine – Basic
(c) dorsal nerve cord is absent. (c) Valine – Charged
(d) heart is lacking. (d) Phenylalanine – Aromatic
EBD_7839
24 KVPY-SA

56. The distance between A and B genes is too long (b) Killing the bacteria ingested with food and
on a chromosome, the strength of linkage drinks.
between them is: (c) Promoting the formation of pepsin.
(a) More (d) Softening fibrous food elements.
(b) Less 59. Which of the following is a method for birth control?
(c) No relation to distance (a) IUDs (b) GIFT
(d) Same irrespective of close or distant location (c) HTF (d) IVF-ET
57. A triple antigen (vaccine) is:
60. Affinity of CO for haemoglobin as compared to
(a) BCG (b) DPT (c) ATS (d) TAB
O2 is:
58. Which of the following is not the function of
(a) 2 times (b) 20 times
HCl in stomach?
(a) Breaking down proteins into peptones. (c) 100 times (d) 200 times

PART-II (2 MARKS QUESTIONS)


MATHEMATICS 65. The value of
61. ABCD is a parallelogram. A straight line is drawn
through A and meets CB, CD produced in E, F. 21 + 3 59 + 16 + 3 722 + 49 is
Then CB · CE + CD · CF =
(a) 4 (b) 5
(a) AC2 + AE.AF (b) AB2 + AE.AF
2
(c) 6 (d) 8
(c) AC + AB.AF (d) AB2 + AB.AF
62. HCF of A & B is h1, a & C is h2 and that of B & C PHYSICS
is h3. Find HCF of h1, h2 and h3. 66. A proton, a deuteron and an a-particle having
(a) 1 (b) h1 h2 h 3 the same kinetic energy are moving in circular
(c) h1 + h2 + h3 (d) (h1 h2 h3 )2 trajectories in a constant magnetic field. If rp, rd
63. In the given figure, a circle with centre B overlaps and ra denote respectively the radii of the
another circle with centre A and a square. The trajectories of these particles, then
ratio of areas of P and Q is 5 : 4 and the area of Q (a) ra = rp < rd (b) ra > rd > rp
1 (c) ra = rd > rp (d) rp = rd = ra
is the area of circle B. The radii of circle A and 67. A block of mass m is on an inclined plane of
8
angle q. The coefficient of friction between the
circle B are 10 cm and 8 cm respectively. block and the plane is m and tan q > µ. The block
is held stationary by applying a force P parallel
to the plane. The direction of force pointing up
A P B Q 7 cm
the plane is taken to be positive. As P is varied
from P1 = mg(sin q – m cos q ) to P2 = mg(sin q +
Find the area of the unshaded part of the figure. m cos q), the frictional force f versus P graph will
(Take p = 3.14) look like
(a) 449.75 cm2 (b) 520.60 cm2 f f
(c) 563.72 cm2 (d) 507.44 cm2
64. From the middle point C of an arc AB of a circle, a (a) (b)
diameter CD is drawn and also a chord CE which
meets the straight line AB in F. If a circle, drawn
with centre C to bisect FE, meets BD in G, then f f
EF =
(a) BG (b) 2 BG
3 (c) (d)
(c) BG (d) None of these
2
MOCK TEST-3 25
68. A balloon of volume V, contains a gas whose CHEMISTRY
density is to that of the air at the earth’s surface
as 1 : 15. If the envelope of the balloon be of 71. In forming (i) N 2 ® N +2 and (ii) O2 ® O2+ ; the
weight w but of negligible volume, find the electrons respectively are removed from :
acceleration with which it will begin to ascend. (a) (p*2pz) and (p*2py or p*2px)
æ 7Vg s - w ö æ 2Vg s - w ö (b) (p 2pz) and (p2py or p2px)
(a) ç ÷ ´ g (b) ç ´g
è Vg s + w ø è Vg s + w ø÷ (c) (p2pz) and (p*2py or p*2px)
æ 14Vg s - w ö æ 14Vg s + w ö (d) (p*2pz) and (p2py or p2px)
(c) ç ´ g (d) ç ´g 72. Arrange the following in decreasing order of
è Vg s + w ø÷ è Vg s - w ø÷
stability.
69. In the diagram below (not to scale), each of the
loudspeakers emits a continuous sound of the
same frequency.
A microphone moved along the line PQ detects
a series of maximum and minimum sound
intensities. Which one of the following actions
on its own, will lead to an increase in the distance I II III
between the maxima of sound intensity ? (a) I > II > III (b) I > III > II
Q (c) II > I > III (d) II > III > I
Loudspeaker 73. If degree of dissociation of pure water at 100 °C
is 1.8 × 10–8, then the dissociation constant of
water will be : (Density of H2O = 1 g/cc)
Loudspeaker (a) 1 × 10–12 (b) 1 × 10–14
(c) 1.8 × 10–12 (d) 1.8 × 10–14

P 74. E Values of some redox couples are given
(a) Decreasing the frequency of the sound below. On the basis of these values choose the
emitted by the loudspeakers. correct option.

(b) Increasing the frequency of the sound E values: Br 2/Br – = + 1.90;
emitted by the loudspeakers. Ag+ /Ag(s) = + 0.80;
(c) Increasing the separation of the
loudspeakers Cu2+ /Cu(s) = + 0.34;
(d) Decreasing the distance of the loudspeakers I2(s) /I– = + 0.54
from the line PQ. (a) Cu will reduce Br – (b) Cu will reduce Ag
70. A 0.5 kg block slides from the point A on
horizontal track with an initial speed of 3 m/s (c) Cu will reduce I – (d) Cu will reduce Br2
towards a weightless horizontal spring of length 75. Identify the unknown compounds.
1 m and force constant 2 Nm–1. The part AB of Sn
HNO Br
the track is frictionless and the part BC has the ¾¾¾¾

(A) ¾¾¾
H 2SO 4
2 ®
(B) ¾¾¾
HCl
® (C)
FeBr3
coefficients of static and kinetic friction as 0.22 (50 ° C)
and 0.2 respectively. If the distances AB and BD (a) A ® Nitrobenzene, B ® Dinitrobenzene,
are 2 m and 2.14 m respectively, find the total
distance through which the block moves before C ® p-Bromoaniline
it comes to rest completely. (b) A ® C6H5SO3H2,
[Take g = 10 m /s2] R B ® m-Benzenesulphonic acid,
(a) 2.42 m C ® m-Benzenesulphonate
(c) A ® C6H5NO2, B ® m-Bromonitrobenzene,
(b) 1.42 m
mk R
C ® m-Bromoaniline
(c) 4.24 m
(d) A ® p-Nitrobenzene,
(d) 2.14 m A B C E C
2m 2.14m
B ® m-Trinitrobenzene,
mg C ® m-Bromoaniline
EBD_7839
26 KVPY-SA

BIOLOGY (a) A – I; B – IV; C – III; D – II


76. There are several types of enzyme catalysed (b) A – I; B – IV; C – II; D – III
reactions. In one type of enzyme catalysed
reaction, in addition to the catalytic site to which (c) A – IV; B – I; C – II; D – III
the substrate (X) binds, the enzyme also has a (d) A – IV; B – I; C – III; D – II
site to which some other substance (Y) can bind. 78. If alveolar ventilation is 4200 mL/min, respiratory
When Y binds to such an enzyme, the enzyme
can still bind to the substrate but cannot convert frequency is 12 breaths per minute and tidal
it to the product. Which of the following will volume is 500 mL, what is anatomical dead space
occur in such a case? ventilation?
(a) The affinity of the enzyme for the substrate (a) 1800 mL/min. (b) 6000 mL/min.
will reduce.
(b) Vmax of the reaction will decrease. (c) 350 mL/min. (d) 1200 mL/min.
(c) Y will alter the conformation of X. 79. Total population of 800 individuals which
(d) The effect of Y can be overcome by formed F2 population (9 : 3 : 3 : 1) of a cross
increasing the concentration of X. between yellow round and green wrinkled. Find
77. Match column-I (function) with column-II (types the number of plants with yellow and wrinkled
of enzymes) and select the correct option.
seeds.
Column-I Column-II
(Function) (Types of enzymes) (a) 150 (b) 400
A. Enzyme catalyses I. Isomerases (c) 800 (d) 300
breakdown without 80. Choose the correct option with appropriate
addition of water. medium of circulation and transport against
B. Enzyme catalyses II. Oxidoreductase each animal.
the conversion of
an aldose sugar to Column-I Column-II
a ketose sugar. A. Hydra (i) Water
C. Enzyme catalyses III. Ligases surrounding
transfer of electrons
the body
from one molecule
to another. B. Octopus (ii) Haemolymph
D. Enzyme catalyses IV. Lyases C. Prawn (iii) Blood
bonding of two
components with the (a) A – (iii) (b) B – (iii)
help of ATP. (c) B – (ii) (d) C – (i)

ANS W ER KEYS
Part-I Part-II
1 (b ) 11 (b ) 21 (b) 31 (c) 41 (d ) 51 (a) 61 (a) 71 (c)
2 (d ) 12 (d ) 22 (a) 32 (b) 42 (d ) 52 (d ) 62 (a) 72 (c)
3 (c) 13 (a) 23 (d) 33 (a) 43 (b ) 53 (c) 63 (d ) 73 (d)
4 (a) 14 (b ) 24 (b) 34 (d) 44 (b ) 54 (d ) 64 (b ) 74 (d)
5 (a) 15 (d ) 25 (d) 35 (a) 45 (c) 55 (c) 65 (b ) 75 (c)
6 (c) 16 (a) 26 (b) 36 (a) 46 (d ) 56 (b ) 66 (a) 76 (b)
7 (b ) 17 (d ) 27 (b) 37 (b) 47 (a) 57 (b ) 67 (a) 77 (c)
8 (d ) 18 (a) 28 (b) 38 (c) 48 (a) 58 (a) 68 (c) 78 (a)
9 (c) 19 (c) 29 (c) 39 (a) 49 (d ) 59 (a) 69 (a) 79 (a)
10 (b ) 20 (d ) 30 (d) 40 (a) 50 (c) 60 (d ) 70 (c) 80 (b)
41
Time : 3 Hours Stream-SA Maximum Marks : 100
INSTRUCTIONS
1. There are 80 questions in this paper.
2. The question paper contains two parts; Part–I (1 Mark Questions) and Part–II (2 Marks
Questions). There are four sections; Mathematics, Physics, Chemistry and Biology in each part.
3. There are four options given with each question, only one of them is correct.
4. For each incorrect answer 0.25 Mark in Part–I and 0.5 Mark in Part–II will be deducted.

PART-I (1 MARK QUESTIONS)

MATHEMATICS 4. ABC is a triangle. BF, CG are any two lines drawn


from the extremities of the base BC to meet AC
1. In an international convention, 5 participants from
and AB in F and G respectively and intersect in H.
each of USA, China and Russia were arranged Then,
around a circle. In how many ways this (a) AF + AG = HF + HG
arrangement can be made if no two participants (b) AF + AG < HF + HG
from USA are together? (c) AF + AG > HF + HG
(a) (9!)(10P5) (b) (9!)(10C5) (d) None of these
(c) (3!)(5!)(5!)(5!) (d) None of these 5. If ABCD is rhombus, then find the correct
statement:
2. Find the smallest number that is divisible by all (a) AC2 + BD2 = 6AB2
the numbers from 1 to 15 except one number. (b) AC2 + BD2 = 4AB2
(a) 180180 (b) 120120 (c) AC2 + BD2 = 3AB2
(c) 32760 (d) 27720 (d) AC2 + BD2 = 2AB2
x+3 6. How many two digit numbers ‘ab’ exist such
3. Given real numbers and b, if a = ,
4 ab
that has quotient 6 & remainder 7.
2x + 1 7 (a + b)
b= , b < < 2a , find the range of
3 3 (a) 3 (b) 1 (c) 0 (d) 2
the value of x. 7. If HCF of three numbers is 14 such that the
numbers are in the form of 14x, 14y and 14z, and
5 5 their LCM is L then which of the following best
(a) < x<3 (b) < x<5
3 3 describes L?
(a) L = 14xyz (b) 14 < L < 14xyz
1
(c) 1 < x<3 (d) None of these (c) 14 £ L £ 14xyz (d) L = 14(x + y + 2)
3
EBD_7839
28 KVPY-SA

8. If u, v, w and m are natural numbers such that 14. Seven people leave their bags outside temple
um + vm = wm, then one of the following is true. and while returning after worshiping the deity,
(a) m ³ min (u, v, w) (b) m ³ max (u, v, w) picked one bag each at random. In how many
(c) m < min (u, v, w) (d) None of these ways at least one and at most three of them get
9. In an annual general meeting of a company 25 their correct bags?
executives, 2 managers and a director are 7
sitting around a circular table such that director (a) C3 × 9 + 7 C5 × 44 + 7 C1 × 265
has no manager next to him. In how many ways 7
this arrangement can be made? (b) C6 × 265 + 7 C5 × 9 + 7 C7 × 44
(a) (2!)(26!) (b) (2!)(25!) 7
(c) (2!)(26!) (d) None of these
(c) C5 × 9 + 7 C2 × 44 + 7 C1 × 265
10. In a 4-digit number, the sum of the first two digits (d) None of these
is equal to that of the last two digits. The sum of 15. If the digits of a three digit number are reserved,
the first and the last digit is equal to the third then the number so obtained is less than the
digit. Finally, the sum of the second and fourth original number by 297. If the sum of the digits
digits is twice the sum of the other two digits. of the number is 8 and its hundred’s digit has
What is the third digit of the number? the largest possible value, then the ten’s digit
(a) 5 (b) 8 (c) 1 (d) 4 of the number is
11. The mid-points of the sides of a triangle are (a) 3 (b) 2 (c) 1 (d) 0
(5, 7, 11), (0, 8, 5) and (2, 3, – 1), then the vertices
are PHYSICS
(a) (7, 2, 5), (3, 12, 17), (– 3, 4, – 7) 16. A block of mass M is tied to one end on a
(b) (7, 2, 5), (3, 12, 17), (3, 4, 7) massless rope. The other end of the rope is in
(c) (7, 2, 5), (– 3, 11, 15), (3, 4, 8) the hands of a man of mass 2 M as shown in
(d) None of the above
figure. The block and the man are resting on a
12. A man sold a chair and a table together for
` 1520 thereby making a profit of 25% on the rough wedge of mass M as shown in figure. The
chair and 10% on table. By selling them together whole system is resting of a smooth horizontal
for ` 1535 he would have made a profit of 10% surface. The man pulls the rope. Pulley is
on the chair and 25% on the table. Find the cost massless and frictionless. What is the
price of each. displacement of the wedge. When the block
(a) ` 600, ` 700 (b) ` 6,000, ` 7,000
meets the pulley. (Man does not leave his
(c) ` 760, ` 768 (d) ` 900, ` 1000
13. Suppose Q is a point on the circle with centre P position during the pull).
and radius 1, as shown in the figure; R is a point (a) 0.5 m
outside thr circle such that QR = 1 and M
(b) 1 m
ÐQRP = 2°. Let S be the point where the segment 2m
RP intersects the given circle. Then measure of 2
(c) m M
ÐRQS equals. 3
(d) zero
Q 17. A research satellite of mass 200 kg circles the
1 1 earth in an orbit of average radius 3R/2 where R
R P is the radius of the earth. Assuming the
S 1
gravitational pull on a mass of 1 kg on the earth’s
2° surface to be 10 N, the pull on the satellite will be
(a) 880 N (b) 889 N
(a) 86° (b) 87° (c) 88° (d) 89° (c) 890 N (d) 892 N
MOCK TEST-4 29

18. A charged ball B hangs from a silk thread S, 21. Consider the figure shown. Reflected ray AB
which makes an angle q with a large charged and refracted ray AC are perpendicular.
conducting sheet P, as shown in the figure. The
Refractive index of the material of the sphere is
surface charge density s of the sheet is
proportional to A
60°
(a) sin q C
incident
(b) tan q ray B
(c) cos q Glass sphere
(d) cot q
(a) 1.5 (b) 2
19. Two communicating vessels contain mercury.
The diameter of one vessel is n times larger than (c) 3 (d) 2.5
the diameter of the other. A column of water of 22. A potential difference applied to the ends of a
height h is poured into the left vessel. The wire made up of an alloy drives a current through
mercury level will rise in the right-hand vessel it such that J = a + br, where r is the distance of
the point from the axis. If R be the radius of the
(s = relative density of mercury and r = density wire, then find the total current through any cross
of water) by section of the wire.
pR 2 2pR2
2
n h (a) (3 a + 2bR) (b) ( a + bR)
(a) 6 6
( n + 1) 2 s 2pR2 2pR2
(c) (3 a – 2bR) (d) (3 a + 2bR)
6 6
h 23. A bob of mass m attached to an inextensible
(b)
( n + 1) s
2
string of length l is suspended from a vertical
support. The bob rotates in a horizontal circle
h with an angular speed w rad/s about the vertical.
(c) About the point of suspension:
( n + 1) 2 s (a) angular momentum is conserved.
(b) angular momentum changes in magnitude
h
(d) but not in direction.
n2 s (c) angular momentum changes in direction but
20. In the Rutherford experiment, a-particles are not in magnitude.
(d) angular momentum changes both in
scattered from a nucleus as shown. Out of the
direction and magnitude.
four paths, which path is not possible?
24. A thick-walled hollow sphere has outside radius
A R0. It rolls down an incline without slipping and
(a) D
its speed at the bottom is V0. Now the incline is
B waxed, so that it is practically frictionless and
(b) B
the sphere is observed to slide down (without
C any rolling). Its speed at the bottom is observed
(c) C D to be 5V0/4. The radius of gyration of the hollow
sphere about an axis through its centre is
(d) A (a) 3R0/2 (b) 3R0/4
(c) 9R0/16 (d) 3R0
EBD_7839
30 KVPY-SA

25. Material A has critical angle iA, and material B


has critical angle iB (iB > iA), then which of the
following is true
(i) light can be totally internally reflected
when it passes from B to A
(ii) light can be totally internally reflected
when it passes from A to B
(iii) critical angle for total internal reflection is
(a) 586 kmh–1 (b) 400 2 kmh–1
iB – iA
(iv) critical angle between A and B is sin –1 (c) 717 kmh–1 (d) 400 kmh–1
æ sin i A ö 29. Two rings each of radius ‘a’ are coaxial and the
çè sin i ÷ø distance between their centres is a. The masses
B
of the rings are M1 and M2. The work done in
(a) (i) and (iii) (b) (i) and (iv) transporting a particle of a small mass m from
(c) (ii) and (iii) (d) (ii) and (iv) centre C1 to C2 is :
M1 M2
26. If x, y and z in figure are identical lamps which of
the following change in brightness will occur,
a a
when switch S is closed?
a
C1 C2
S z

x
y Gm( M 2 - M1 )
(a)
e a
(a) x stays the same
Gm( M 2 - M1)
(b) x increases y decreases (b) ( 2 + 1)
(c) x increases y stays the same a 2
(d) x decreases y increases
Gm( M 2 - M1)
27. A student measured the diameter of a wire using (c) ( 2 - 1)
a 2
a screw gauge with the least count 0.001 cm and
listed the measurements. The measured value Gm( M 2 - M1 )
should be recorded as (d) a
(a) 5.3200 cm (b) 5.3 cm 2
(c) 5.32 cm (d) 5.320 cm 30. A small spherical solid ball is dropped from a
28. Passengers in the jet transport A flying east at a great height in a viscous liquid. Its journey in
speed of 800 kmh –1 observe a second jet plane the liquid is best described in the diagram given
B that passes under the transport in horizontal below by the
flight. Although the nose of B is pointed in the
Velocity (v)

(a) Curve A A
B
45° north east direction, plane B appears to the
(b) Curve B
passengers in A to be moving away from the C
transport at the 60° angle as shown. The true (c) Curve C D

velocity of B is (d) Curve D Time (t)


MOCK TEST-4 31

CHEMISTRY
31. 100 mL O2 and H2 kept at same temperature and (c) (d)
pressure. What is true about their number of

molecules? 36. A substance on treatment with dil.H2 SO 4
(a) NO > NH (b) NO < NH liberates a colourless gas which turns acidified
2 2 2 2
(c) NO = NH (d) NO + NH = 1 mole dichromate solution green. The reaction indicates
2 2 2 2 the presence of
32. The combination of plots which does not
represent isothermal expansion of an ideal gas (a) CO 32- (b) S2 - (c) SO 32 - (d) NO 3-
is:
37. What is the correct order of electronegativity?
(a) M– < M2– < M3– < M4–
(b) M– > M2– > M3– > M4–
(c) M– < M2– > M3– < M4–
(A) (B) (d) M4– < M2– < M3– < M–
38. The difference in number of water molecules in
Gypsum and Plaster of Paris is
1
(a) 5 (b) 2 (c) 1/2 (d) 1
2
(C) (D)
OH O

OH
39. The IUPAC name of is:
(a) (B) and (D) (b) (A) and (C)
(c) (B) and (C) (d) (A) and (D)
33. When steam reacts with red hot coke to form (a) 2-Carboxyphenol
CO2 and hydrogen : (b) 2-Hydroxybenzoic acid
(a) Water acts as an oxidising agent. (c) 1-Carboxy-2-hydroxybenzene
(b) Water acts as a reducing agent. (d) 2-Carboxy-1-hydroxybenzene
(c) Carbon acts as an oxidising agent. 40. The bond having the highest bond energy is :
(d) There is no oxidation or reduction. (a) C = C (b) C = S
34. Which of the following is not correct for (c) C = O (d) P = N
electronic distribution in the ground state ? 41. A buffer solution is prepared by mixing 0.1 M
ammonia and 1.0 M ammonium chloride.
(a) Co : [Ar] At 298 K, the pKb of NH4OH is 5.0. The pH of
the buffer is
(b) Ni : [Ar] (a) 10.0 (b) 9.0 (c) 6.0 (d) 8.0
42. The correct order of increasing chemical
(c) Cu : [Ar] reactivity is –
(d) All of the above (a) Zn < Fe < Mg < K (b) Fe < Mg < Zn < K
35. The non aromatic compound among the (c) Fe < Mg < K < Zn (d) Fe < Zn < Mg < K
following is : 43. The shape of methyl carbanion is similar to that
of –
(a) (b) (a) BF3 (b) NH3
S (c) Methyl free radical (d) Methyl carbocation
EBD_7839
32 KVPY-SA

44. Calculate the standard enthalpy change (in kJ 51. From the experiments carried out by Avery,
mol–1) for the reaction H2(g) + O2(g) ® H2O2(g), MacLeod and McCarty by using various
given that bond enthalpy of H–H, O=O, O–H enzymes, which of the following results
and O–O (in kJ mol–1) are respectively 438, 498, prominently proved that DNA is the
464 and 138. transforming material?
(a) – 130 (b) – 65 (c) + 130 (d) – 334 (a) DNA of heat killed ‘S’ + R type + DNAase ®
45. In pyrophosphoric acid, H4P2O7, number of s non-virulent strain
and dp – pp bonds are respectively
(a) 8 and 2 (b) 6 and 2 (b) DNA of heat killed ‘S’ + R type + RNAase
(c) 12 and zero (d) 12 and 2 ® virulent strain
(c) DNA of heat killed ‘S’ + R type ® virulent
BIOLOGY
strain
46. Smoking destroys the cilia in the respiratory
(d) DNA of heat killed ‘S’ + R type + Protease
passageways. This
(a) makes it harder to move air in and out of the ® virulent strain
lungs. 52. Match the column-I with column-II and choose
(b) decreases the surface area for respiration. the correct option.
(c) slows blood flow through lung blood Column-I Column-II
vessels.
(d) makes it harder to keep the lungs clean. A. Human embryos I. Chemical evolution
47. Cholesterol is required for the synthesis of: have gill
(a) Relaxin (b) Vitamin E B. Oparin and II. Stimulation
(c) Oestradiol (d) All of these Haldane experiment
48. What pathway is taken by water and solutes C. Miller and Urey III. Wings of bird and
when they travel through a nephron?
butterfly
(a) Glomerulus ® Bowman’s capsule ®
proximal tubule ® loop of Henle ® distal D. Analogous IV. Ontogeny repeats
tubule ® collecting ducts organs phylogeny
(b) Bowman’s capsule ® glomerulus ® distal (a) A – III; B – IV; C – II; D – I
tubule ® loop of Henle ® proximal tubule (b) A – II; B – I; C – IV; D – III
® collecting ducts
(c) A – IV; B – I; C – II; D – III
(c) Glomerulus ® Bowman’s capsule ® distal
tubule ® loop of Henle ® proximal tubule (d) A – IV; B – I; C – III; D – II
® collecting ducts 53. Baculoviruses are excellent candidates for
(d) Glomerulus ® Bowman’s capsule ® (a) species-specific, narrow spectrum pesticidal
proximal tubule® collecting ducts ® distal applications.
tubule ® loop of Henle (b) species-specific, broad spectrum pesticidal
49. Gram-negative cells stain pink because
(a) they have specialised lipids in their cell applications.
walls. (c) species-specific, narrow spectrum insecticidal
(b) their peptidoglycan layer is thin. applications.
(c) their peptidoglycan layer is thick. (d) species-specific, broad spectrum insecticidal
(d) they are receptive to antibiotics. applications.
50. Which one of the following is common to
54. The most abundant structural polysaccharide is
glycolysis as well as Krebs cycle in eukaryotes?
(a) Substrate level phosphorylation cellulose and the second largest structural
(b) Photophosphorylation polysaccharide is:
(c) Localisation in mitochondria (a) Chitin (b) Mannan
(d) Production of FADH2 (c) Glycogen (d) Hyaluronic acid
MOCK TEST-4 33
55. “A” cells start division and enter in “B” stage of 58. Hormones that are secreted by one endocrine
meiotic division and get temporarily “C” at this gland and control the activities of another
stage, called “D”. Identify A, B, C and D. endocrine glands are called:
(a) A: Oogonia; B: Metaphase I; C: Arrested; (a) Growth hormones (b) Steroid hormones
D: Primary oocyte. (c) Tropic hormones (d) Peptide hormones
(b) A: Oogonia; B: Anaphase I; C: Released; 59. In terms of DNA and RNA structure, what is a
D: Secondary oocyte. nucleotide?
(c) A: Oogonia; B: Prophase I; C: Arrested; D: (a) A nucleotide is a heterocyclic base.
Primary oocyte. (b) A nucleotide is a sugar molecule covalently
(d) A: Oogonia; B: Telophase I; C: Released; bonded to a heterocyclic base.
D: Secondary oocyte. (c) A nucleotide is a sugar molecule bonded
56. Select the correct order of geological time scale to phosphate group and a heterocyclic base.
of earth. (d) A nucleotide is a heterocyclic base bonded
(a) Palaeozoic ® Archaeozoic ® Coenozoic to phosphate group.
(b) Archaeozoic ® Palaeozoic ® Proterozoic 60. A human bone marrow cell, in prophase of
(c) Palaeozoic ® Mesozoic ® Coenozoic mitosis, contains 46 chromosomes. How many
(d) Mesozoic ® Archaeozoic ® Proterozoic chromatids does it contain altogether?
57. PS I occurs in: (a) 46
(a) Appressed part of granal thylakoids (b) 92
(b) Appressed and non-appressed part of (c) 23
granal thylakoids (d) 23 or 46, depending when you look during
(c) Stroma prophase
(d) Stroma thylakoids and non-appressed part
of granal thylakoids
PART-II (2 MARKS QUESTIONS)
MATHEMATICS
61. PQRS is a rectangle in which PQ = 2 PS, T and p 3 3 p
(a) - (b) -
U are mid points of PS and PQ respectively. QT 6 4 4 24
and US intersect at V. The area of QRSV divided
by area of PQT is- 3 p 3 p
(c) - (d) -
9 7 6 8 4 12 4 8
(a) (b) (c) (d)
7 4 5 3 64. Let ABC be a triangle and D be the mid-point of
62. Let us define a set S such that any element of S
can be expressed as sum of a two digit number side BC. Suppose ÐDAB = ÐBCA and
and number formed by reversing that 2 digit ÐDAC = 15°. Show that ÐADC is obtuse.
number. What is the HCF of all the elements of S ?
(a) 3 (b) 9 (c) 7 (d) 11 Further, if O is the circumcentre of ADC, then
63. A semi-circle of diameter 1 unit sits at the top of DAOD is :
a semi-circle of diameter 2 units. The shaded (a) Equilateral (b) Right angled
region inside the smaller semi-circle but outside (c) Isosceles (d) None of these
the larger semi-circle is called a lune. The area
of the lune is 65. 7+4 3 - 7-4 3 =

(a) 4 (b) 2 (c) 4 3 (d) 2 3


EBD_7839
34 KVPY-SA

PHYSICS é æT ö2ù A
66. Figure shows a smooth spherical ball of mass m (a) ê1 - ç M ÷ ú
striking two identical equilateral triangular ë è T ø û Mg
wedges, each of mass m. The velocity of ball at
the instant of impact is v0. If e is the coefficient é æ T ö2ù A
(b) ê1 - ç ÷ ú
of restitution, then velocity of either wedge after êë è TM ø úû Mg
impact is
éæ T ö 2 ù A
m v0 m (c) ê ç M ÷ - 1ú
ëè T ø û Mg
3 5
(a) (1 + e)v0 (b) (1 + e)v0 é æ T ö 2 ù Mg
5 3 (d) ê ç M ÷ - 1ú
ëè T ø û A
ev0
(c) 3(1 + e)v0 (d) 69. Two spheres P and Q of equal radii have densities
3
67. An upright U-tube manometer with its limbs 0.6m r1 and r2 , respectively. The spheres are
high and spaced 0.3m apart contains a liquid to connected by a massless string and placed in
a height of 0.4m in each limb. If the U-tube is
liquids L1 and L2 of densities s1 and s2 and
rotated at 10 radians/second about a vertical
axis at 0.1m from one limb. Choose the correct viscosities h1 and h2, respectively. They float in
statement/s equilibrium with the sphere P in L1 and sphere Q
in L2 and the string being taut (see figure). If
ur
10rad/s sphere P alone in L2 has terminal velocity V P ,
ur
and Q alone in L1 has terminal velocity V Q , then
z2
0.4m ur
z1 VP h
zmin (a) ur = 1
VQ h2 L1

0.1m 0.2m ur
I. The value of z1 = 0.324 VP h L2
(b) ur = 2
II. The value of z2 = 0.477 VQ h1
III. The value of zmin = 0.273 ur ur
(c) V P .V Q > 0
IV. The value of z1 + z2 = 0.8
ur ur
(a) I only (b) I and II only (d) V P .V Q = 0
(c) III only (d) I, II, III and IV 70. Students I, II and III perform an experiment for
68. A pendulum made of a uniform wire of cross measuring the acceleration due to gravity (g)
sectional area A has time period T. When an
using a simple pendulum. They use different
additional mass M is added to its bob, the time
period changes to TM. If the Young’s modulus of lengths of the pendulum and / or record time for
different number of oscillations.
1
the material of the wire is Y then is equal to: The observations are shown in the table.
Y
Least count for length = 0.1 cm
(g = gravitational acceleration)
least count for time = 0.1 s
MOCK TEST-4 35

74. Bond order normally gives idea of stability of a


Numbe r of
Total time
Time
molecular species. All the molecules viz. H2, Li2
Student
Le ngth of the
oscillations
for (n)
pe riod and B2 have the same bond order yet they are
pen dulum(cm) oscil lations
(n)
(s)
(s) not equally stable. Their stability order is
I 64.0 8 128.0 16.0 (a) H2 > B2 > Li2 (b) Li2 > H2 > B2
II 64.0 4 64.0 16.0 (c) Li2 > B2 > H2 (d) H2 > Li2 > B2
III 20.0 4 36.0 9.0 75. The stability of 2, 3 - dimethyl but - 2- ene is
more than 2- butene. This can be explained in
If EI, EII and EIII are the percentage errors in g, terms of
(a) resonance
æ Dg ö (b) hyperconjugation
i.e., ç ´ 100 ÷ for students I, II and III,
è g ø (c) electromeric effect
respectively, then (d) inductive effect
(a) EI = 0 BIOLOGY
(b) EI is minimum 76. Analysis of a protein sequence from a recently
(c) EI = EII mutated gene showed that the newly expressed
(d) EII is minimum protein had a completely different sequence from
the wild-type protein. This was due to a:
(a) Missense mutation
CHEMISTRY
(b) Nonsense mutation
71. Which of the following reagents convert (c) Silent mutation
propene to 1-propanol? (d) Frameshift mutation
(a) H2O, H2SO4 77. A student while studying the anatomy of leaves
(b) Aqueous KOH of four specimens (A-D), observed the following
(c) MgSO4, NaBH4/H2O characters:
(d) B2H6, H2O2, OH– A. Reticulate venation and no bundle sheath
B. Parallel venation and bundle sheath
72. If S + O2 ¾¾ ® SO2 ; DH = -298.2 kJ ... (i)
1 containing chloroplasts
SO 2 + O2 ¾¾ ® SO3 ; DH = – 98.7 kJ ... (ii) C. Reticulate venation and bundle sheath
2
SO3 + H 2 O ¾¾® H 2SO 4 ; DH = -130.2 kJ ... (iii) containing chloroplasts
1 D. Parallel venation and bundle sheath without
H 2 + O 2 ¾¾ ® H 2O; DH = -287.3 kJ ... (iv) chloroplasts
2
Then the enthalpy of formation of H2SO4 at Identify the type of plants.
298 K is (a) A and C are C4 dicots, while B and D are C4
(a) –814.4 kJ (b) –650.3 kJ monocots.
(c) –320.5 kJ (d) –233.5 kJ (b) A and C are C3 dicots, while B and D are C3
73. A metal oxide has the formula Z2O3. It can be monocots.
reduced by hydrogen to give free metal and (c) A and D are C 3 dicot and monocot,
water. 0.1596 g of the metal oxide requires 6 mg respectively, while B and C are C4 monocot
of hydrogen for complete reduction. The atomic and dicot, respectively.
(d) A and D are C 4 dicot and monocot,
weight of the metal is
respectively, while B and C are C3 monocot
(a) 27.9 (b) 159.6 (c) 79.8 (d) 55.8
and dicot, respectively.
EBD_7839
36 KVPY-SA

78. Because of the relatively high altitude of 80. Match the items given in column-I with those in
Antonito, Colorado, the town has a normal column-II and select the correct option given
barometric pressure of about 600 mm Hg rather below.
than 760 mm Hg as at sea level. The partial Column-I Column-II
pressure of oxygen in Antonito’s air is A. Tidal volume I. 2500 – 3000 mL
approximately B. Inspiratory II. 1100 – 1200 mL
(a) 75 mm Hg (b) 126 mm Hg Reserve volume
(c) 160 mm Hg (d) 760 mm Hg C. Expiratory III. 500 – 550 mL
79. The following structures are ‘analogous’ from Reserve volume
the evolutionary point of view except: D. Residual volume IV. 1000 – 1100 mL
(a) Sting of scorpion and honeybee
(a) A – III; B – II; C – I; D – IV
(b) Wings of an owl and wings of a butterfly
(b) A – III; B – I; C –IV; D – II
(c) Spines of a cactus and quills of a porcupine
(c) A – IV; B – III; C – II; D – I
(d) Tendrils of pea and spines of barberry
(d) A – I; B – IV; C – II; D – III

ANS W ER KEYS
Part-I Part-II
1 (a) 11 (a) 21 (c) 31 (c) 41 (d) 51 (a) 61 (d ) 71 (d)
2 (d ) 12 (a) 22 (d ) 32 (a) 42 (d) 52 (c) 62 (d ) 72 (a)
3 (a) 13 (b) 23 (c) 33 (a) 43 (b) 53 (c) 63 (b ) 73 (d)
4 (c) 14 (c) 24 (b ) 34 (d ) 44 (a) 54 (a) 64 (a) 74 (d)
5 (b ) 15 (c) 25 (d ) 35 (d ) 45 (d ) 55 (c) 65 (d ) 75 (b)
6 (b ) 16 (a) 26 (b ) 36 (c) 46 (d) 56 (c) 66 (a) 76 (d)
7 (c) 17 (b) 27 (d ) 37 (b ) 47 (c) 57 (d ) 67 (d ) 77 (c)
8 (c) 18 (b) 28 (c) 38 (d ) 48 (a) 58 (c) 68 (c) 78 (b)
9 (d ) 19 (b) 29 (c) 39 (b ) 49 (b) 59 (c) 69 (a) 79 (d)
10 (a) 20 (c) 30 (c) 40 (c) 50 (a) 60 (b ) 70 (b ) 80 (b)
51
Time : 3 Hours Stream-SA Maximum Marks : 100
INSTRUCTIONS
1. There are 80 questions in this paper.
2. The question paper contains two parts; Part–I (1 Mark Questions) and Part–II (2 Marks
Questions). There are four sections; Mathematics, Physics, Chemistry and Biology in each part.
3. There are four options given with each question, only one of them is correct.
4. For each incorrect answer 0.25 Mark in Part–I and 0.5 Mark in Part–II will be deducted.

PART-I (1 MARK QUESTIONS)

MATHEMATICS 5. A trapezium ABCD in which P and Q are the


1. Let A1, B1, C1, D1 be the midpoints of the sides of mid-points of diagonals AC and BD respectively.
a convex quadrilateral ABCD and let A2, B2, C2, Then PQ =
D 2 be the midpoints of the sides of the
quadrilateral A 1B 1C 1D 1. If A 2B 2C 2D 2 is a 1 1
(a) (AB + CD) (b) (AB – CD)
rectangle with sides 4 and 6, and the product of 3 2
the lengths of the diagonals of ABCD is denoted
1 1
é n ù (c) (AB – CD) (d) (AB + CD)
by n then find ê 3 2
ú , where [·] denotes the
ë 100 û 6. Let X be a four-digit positive integer such that
greatest integer function. the unit digit of X is prime and the product of all
(a) 1 (b) 2 (c) 3 (d) 4 digits of X is also prime. How many such
integers are possible?
2. If unit digit of a! + b! + c! is 9 then find the
value of {(a!)(b!)(c!)} (a) 4 (b) 8 (c) 12 (d) 24
(a) 8 (b) 5040 (c) 0 (d) 12 7. If number 368A37982B is divisible by 36, here
A and B are digits then what could be the sum
3. If x + xy + x = 14, y + xy + y = 28 and z = x + y,
2 2
of all the values of A?
then the sum of all possible values of z is:
(a) 12 (b) 21 (c) 24 (d) 16
(a) 7 (b) 6 (c) –1 (d) –6
8. Consider the set S = {2, 3, 4, ...., 2n + 1), where n
4. N is a natural number such that 6000 < N < 8000.
The sum of number N and a new number formed is a positive integer larger than 2007. Define, X
by reversing its digits is S. if S is P times sum of as the average of the odd integers in S and, Y as
digits of number N then find the minimum value the average of the even integers in S. What is
of P. the value of X – Y?
(a) 330.25 (b) 335.50 (a) 1 (b) n/2
(c) 332.75 (d) None of these
(c) (n + 1)/2n (d) 2008
EBD_7839
38 KVPY-SA

9. In figure, ABCD is a rectangle of 20 cm × 10 cm. 14. A bucket is raised from a well by means of a
A semi-circle is drawn with centre at O and radius rope which is wound round a wheel of diameter
10 2 cm and it passes through A and B. Find 77 cm. Given that the bucket ascends in 1 min.
the area of the shaded region in the figure. 28 seconds with a uniform speed of 1.1 m/sec,
20 cm calculate the number of compolete revolutions
D C
the wheel makes in raising the bucket.
P 10 cm (a) 38 (b) 40
A B (c) 45 (d) None of these
15. If x1, x2, x3,....., xn are the roots of x n + ax + b = 0,
then the value of (x1 – x2) (x1 – x3) (x1 – x4) ...
10 2 cm 10 2 cm
(x1 – xn) is equal to
L O M
(b) n ( x1 )
n -1
20 2 cm (a) nx1 + b
æ pö
(a) (p – 10) × 50 (b) ç 2 - ÷ ´ 100 (c) n ( x1 )
n -1
+a (d) n ( x1 )
n -1
+b
è 2ø

æ pö æ pö PHYSICS
(c) ç 3 - ÷ ´ 100 (d) ç 5 - ÷ ´ 100
è 2ø è 2ø 16. A particle which is simultaneously subjected to
10. Find the remainder when x2018 + x2017 + x2016 +...+ x2 two perpendicular simple harmonic motions
+ x + 1 is divided by x + 1. represented by; x = a1 cos wt and y = a2 cos 2 wt
traces a curve given by:
(a) 1 (b) 0 (c) –1 (d) 2 y y
11. N is a 99 digit number comprises of only 4’s and
8’s. If N is divisible by 72 then find the minimum a2 a2
value of sum of digits of the number N. a1
(a) x (b) O a1
x
(a) 360 (b) 369 O

(c) 387 (d) None of these


12. If XY is a line parallel to side BC of DABC. BE || y y
AC and CF || AB meets XY in E and F respectively.
Then ar DABE = ? a2 a2
a1 a1
(a) ar DACY (b) ar DAYF (c) x (d) x
O O
(c) ar DACF (d) ar (Quad. BCFX)
13. Let a, b are the roots of the equation
l (x2 – x) + x + 5 = 0. 17. For a particle executing simple harmonic motion,
If l1 and l2 are two values of l for which the roots which of the following statement is not correct ?
a b 4 (a) The total energy of the particle always
a, b are related by + = , then find the
b a 5 remains the same.
(b) The restoring force of always directed
é l1 l2 ù towards a fixed point.
êl + l ú
value of ê 2 1
ú where [n] denotes greatest (c) The restoring force is maximum at the
ë 10 û extreme positions.
integer function of n. (d) The acceleration of the particle is maximum
(a) 24 (b) 25 (c) 30 (d) 32 at the equilibrium position.
MOCK TEST-5 39

18. There are two processes ABC and DEF. In which 21. What do you conclude from the graph about
of the process is the amount of work done by the frequency of KE, PE and SHM?
the gas greater?
Energy
Total energy

A B

PE KE

0 t
T/4 2T/4 3T/4 4T/4

(a) Frequency of KE and PE is double the


frequency of SHM
(b) Frequency of KE and PE is four times the
frequency SHM.
(a) ABC (c) Frequency of PE is double the frequency of
(b) DEF K.E.
(c) Equal in both processes (d) Frequency of KE and PE is equal to the
frequency of SHM.
(d) It cannot be predicted
22. Two wooden blocks A and B float in a liquid of
19. A person sitting at the rear end of the density rL as shown. The distance L and H are
compartment throws a ball towards the front end. shown. After some time, block B falls into the
The ball follows a parabolic path. The train is liquid, so that L decreases and H increases. If
moving with uniform velocity of 20 ms–1. A person density of block B is rB, find the correct option.
standing outside on the ground also observes the
ball. How will the maximum height (hm) attained (a) rL = rB B
and the ranges (R) seen by the thrower and the
outside observer compare each other? (b) rL > rB A L
(a) same hm different R (b) same h m and R (c) rL < rB H

(c) different h m same R (d) different h m and R


(d) Unpridictable
20. A river flow with a speed more than the maximum
23. Trajectories are shown in figure are for three
speed with which a person can swim in the still
kicked footballs, ignoring the effect of the air on
water. He intends to cross the river by shortest
the footballs. If T1, T2 and T3 are their respective
possible path (i.e., he wants to reach the point
time of flights then:
on the opposite bank which directly opposite to
the starting point). Which of the following is y
correct statement?
(I) He should start normal to the river bank
1
(II) He should start in such a way that, he moves 2 3
normal to the bank, relative to the bank.
(III) He should start in a particular (calculated) x
R
direction making an obtuse angle with the 1.5 R
2R
direction of water current
(IV) The man cannot cross the river, in that way (a) T1 > T3 (b) T1 < T3
(a) I only (b) II only T3
(c) III only (d) IV only (c) T2 = (d) T1 = T2 = T3
2
EBD_7839
40 KVPY-SA

24. A ray of light travelling inside a rectangular glass æ


-1 8GM ö
block of refractive index 2 is incident on the (c) sin ç 5 1 + 2 ÷
ç 5v 0 R ÷ø
glass-air surface at an angle of incidence of 45º. è
The refractive index of air is one. Under these æ 8GM ö
conditions the ray will (d) cos -1 ç 5 1 + 2 ÷
ç 5v 0 R ÷ø
(a) emerge into the air without any deviation è
(b) be reflected back into the glass 28. An artificial satellite of mass m of a planet of
(c) be absorbed mass M, revolves in a circular orbit whose radius
(d) emerge into the air with an angle of is n times the radius R of the planet. In the process
refraction equal to 90º of motion, the satellite experiences a slight
25. A light ray falls on a square glass slab as shown resistance due to cosmic dust. Assuming
in the diagram. The index of refraction of the resistance force on satellite depends on velocity
glass, if total internal reflection is to occur at as F = av2 where ‘a’ is constant. Calculate how
the vertical face, is equal to : long the satellite will stay in orbit before it falls
( 2 + 1) onto the planet’s surface?
(a)
2
45° Incident ray

(a)
m R ( n -1 ) (b)
mR ( n -1 )
5 a GM a GM
(b)

3
2
(c)
mR ( n +1 ) (d)
m R ( n -1 )
(c) a GM a GM
2
3 29. Two satelites A and B of equal mass move in the
(d) equatorial plane of the earth, close to earth’s
2
26. The period of oscillation of a simple pendulum surface. Satellite A moves in the same direction
is 2p L / g . Measured value of L is 20.0 cm as that of the rotation of the earth while satellite
known to 1 mm accuracy and time for 100 B moves in the opposite direction. Calculate the
oscillations of the pendulum is found to be 90 s ratio of the kinetic energy of B to that of A in the
using a wrist watch of 1 s resolution. What is reference frame fixed to the earth.
the percentage error in the determination of g ? (g = 9.8 m s–2 and radius of the earth = 6.37 × 106 km)
(a) 4% (b) 1% (c) 6% (d) 3% (a) 2.17 (b) 1.27 (c) 7.12 (d) 1.72
27. A spaceship is sent to investigate a planet of 30. Two soap bubbles of different size are formed at
mass M and radius R. While hanging motionless
two ends of a tube as shown in fig. If the stop
in space at a distance 5R from the centre of the
planet, the spaceship fires an instrument package cock is opened then which bulb shrinks ?
with speed v 0 as shown in the figure. The T
package has mass m, which is much smaller than
the mass of the spaceship. For what angle q will
the package just graze the surface of the planet?
v0 r
R
m (I) Smaller bubble shrinks and the bigger bubble
R expands.
M
5R (II) Bigger bubble shrinks and the smaller bubble
æ1 8GM ö expands.
(a) s in -1 ç 1 + ÷ (III) Both bubble shrinks.
ç5 5v 20 R ÷ø
è (IV) Both bubble expands.
æ1 8GM ö Choose the correct statement
(b) cos -1 ç 1 + 2 ÷
ç5 5v 0 R ÷ø (a) I only (b) II only (c) III (d) IV
è
MOCK TEST-5 41

CHEMISTRY 37. Tautomerism will be exhibited by


(a) (CH3)2NH (b) (CH3)3CNO
31. Which of the following represents the correct
order of stability of the given carbocations ? (c) R3CNO2 (d) RCH2NO2
38. The beans are cooked earlier in pressure cooker,
+ | because,
(a) F3C > F3C - C+ > CH3
| (a) boiling point increases with increasing
pressure.
+ | +
(b) H3C > F3C - C + > F3C (b) boiling point decreases with increasing
| pressure.
| (c) internal energy is not lost while cooking in
+ +
(c) F3C - C+ > F3C > H3C pressure cooker.
| (d) extra pressure of pressure cooker softens
| + + the beans.
(d) F3C - C+ > H3C > F3C 39. The correct order for acid strength of compounds
|
CHºCH, CH3–CºCH and CH2=CH2 is as follows:
32. Caffiene has a molecular mass of 194. If it (a) CHºCH > CH2=CH2 > CH3–CºCH
contains 28.9% by mass of nitrogen, number of (b) CH3–CºCH > CHºCH > CH2=CH2
atoms of nitrogen in one molecule of caffeine is:
(c) CH3–CºCH > CH2=CH2 > HCºCH
(a) 4 (b) 6 (c) 2 (d) 3
(d) HCºCH > CH3–CºCH > CH2=CH2
33. Global warming is mainly due to increase of:
(a) Methane and nitrous oxide in atmosphere. 40. A solution contains 10 mL 0.1 N NaOH and
(b) Methane and CO2 in atmosphere. 10 mL 0.05 N H2SO4, pH of this solution is :
(c) Methane and O3 in atmosphere. (a) less than 7 (b) 7
(d) Methane and CO in atmosphere. (c) zero (d) greater than 7
34. Which compound (s) out of the following is/are 41. The maximum number of 90º angles between
not aromatic? bond pair-bond pair of electrons is observed in
(a) dsp2 hybridization (b) sp3d hybridization
(c) dsp3 hybridization (d) sp3d2 hybridization
42. The reaction A + B ‡ˆˆ ˆˆ† C + D + heat, has
+ + – reached equilibrium. The reaction may be made
(A) (B) (C) (D)
to proceed forward by
(a) (B), (C) and (D) (b) (C) and (D) (a) adding more C
(c) (B) (d) (A) and (C) (b) adding more D
35. What is the maximum number of electron in a (c) decreasing the temperature
subshell that can have the quantum numbers (d) increasing the temperature
n = 3 and l = 2? 43. Which of the following compounds contains 1°,
(a) 2 (b) 5 (c) 6 (d) 10 2°, 3° as well as 4° carbon atoms ?
36. Given (a) Neopentane
Fe (aq) + e ® Fe (aq); E = + 0.77 V
3+ – 2+ o (b) 2-Methylpentane
Al (aq) + 3e ® Al(s); E = – 1.66 V
3+ – o (c) 2,3-Dimethylbutane

Br2(aq) + 2e ® 2Br (aq); E = + 1.09 V
– o (d) 2,2,3-Trimethylpentane
Considering the electrode potentials, which of 44. H2O is polar, whereas BeF2 is not. It is because
the following represents the correct order of (a) EN of F > EN of O.
reducing power? (b) H2O involves H-bonding, whereas BeF2 is
2+ - - 2+ a discrete molecule.
(a) Fe < Al < Br (aq)(b) Br (aq) < Fe < Al
(c) H 2O is linear and BeF2 is angular.
(c) Al < Br - < Fe2+ (d) Al < Fe2+ < Br - (aq) (d) H2O is angular and BeF2 is linear.
EBD_7839
42 KVPY-SA

45. Unlike the other elements of its group carbon (b) A complex consisting of eight negatively
and silicon does not form MX2 type molecules charged histone proteins (two of each H2A,
because : H2B, H3 and H4) that aid in the packaging
(a) Energetically this is not possible. of DNA.
(b) They undergoes catenation. (c) A complex consisting of nine positively
(c) They are non-metallic. charged histone proteins (H1 and two of
(d) They do not contain d-orbital. each H2A, H2B, H3 and H4) that aid in the
packaging of DNA.
BIOLOGY
(d) A complex consisting of nine negatively
46. A segment of DNA molecule contains 200 charged histone proteins (H1 and two of
guanine and 200 thymine bases. What will be each H2A, H2B, H3 and H4) that aid in the
the total number of nucleotides in this segment packaging of DNA.
of DNA?
51. Rate of respiration is directly proportional to
(a) 400 (b) 200 (c) 800 (d) 100
(a) Concentration of oxygen in blood
47. If a grasshopper cell contained 200 units of DNA
during G2, what would be the quantity of DNA (b) Concentration of carbon dioxide in blood
in one of the grasshopper cells at the end of (c) Oxygen in trachea
telophase II of meiosis? (d) Diaphragm expansion
(a) 50 units 52. Haem protein complexes that act as oxidising
(b) 100 units agents during plant respiration are known as:
(c) Between 50 and 100 units (a) Chlorophyll (b) Haemoglobin
(d) 200 units (c) Myoglobin (d) Cytochrome
48. Which of the following is the correct group of 53. Which of the following pair is correctly matched?
vestigial organs in man?
(a) Anaphase I: Homologous chromosomes are
(a) Nictitating membrane, ear muscles, eyelids
separated.
and coccyx.
(b) Appendix, coccyx, ear muscles and elbow (b) Metaphase I: Pairing of maternal and
joint. paternal chromosomes.
(c) Wisdom tooth, coccyx, body hair and (c) Interphase: A nuclear envelope encloses
eyelids . each haploid set of chromosomes.
(d) Wisdom tooth, body hairs, nictitating (d) Prophase I: Non-homologous chromosomes
membrane and vermiform appendix. are separated.
49. Oxidative phosphorylation refers to: 54. STD/VD/UTI are:
(a) anaerobic production of ATP (a) devices which are used to delay the
(b) the citric acid cycle production of ATP pregnancy.
(c) production of ATP by chemiosmosis (b) infections which are caused by food
(d) alcoholic fermentation contamination.
50. In tertiary structure of DNA, what is a histone (c) diseases which are transmitted through
octamer? sexual intercourse.
(a) A complex consisting of eight positively (d) action plans and programmes to create
charged histone proteins (two of each H2A,
awareness about various reproductive
H2B, H3 and H4) that aid in the packaging
health related problems.
of DNA.
MOCK TEST-5 43

55. B-lymphocytes are primarily involved in: 58. Sucrose is composed of:
(a) Humoral immunity (a) Glucose and fructose
(b) Autoimmune disorders (b) Glucose and glycogen
(c) Graft rejection (c) Two molecules of glucose
(d) Cell-mediated immunity (d) Glycogen and fructose

56. The differences in D and L forms of sugars are 59. The Miller-Urey abiotic synthesis experiment
(and other subsequent, similar experiments)
based on
showed that
(a) Spatial arrangement of carbon atom to
(a) simple organic molecules can form
which the functional group is attached.
spontaneously under conditions like those
(b) Configuration of H and OH groups around thought to prevail early in the earth’s history.
the penultimate carbon.
(b) the earliest life forms introduced large
(c) If the plane of polarised light is rotated to amounts of oxygen to the atmosphere.
left (levo), it is L form; and if shifted to right (c) life can be created in a test tube.
(dextro), it is D form.
(d) long chains of DNA can form under abiotic
(d) Special arrangement of H and OH groups conditions.
at the last carbon atom. 60. Life on earth is carbon based. Similar molecules
57. Splitting of water in photosynthesis is called: could be formed with:
(a) Dark reaction (b) Photolysis (a) Potassium (b) Aluminum
(c) Electron transfer (d) Phototropism (c) Iron (d) Silicon

PART-II (2 MARKS QUESTIONS)

MATHEMATICS
61. In the figure, the two circles touch each other at A
C. The diameter AB. of the bigger circle is
tangent to the smaller circle at D. If DE bisects
ÐADC, find q P

Q
E
H

C
B D R

(a) 30º
(a) 24° (b) 38° (c) 45° (d) 52°
(b) 60º
62. In the figure BE ^ AC. AD is any line from A to BC
(c) 90º
intersecting BE in H. Also P, Q and R are mid
points of AH, AB and BC. Then find ÐPQR. (d) cannot be determined
EBD_7839
44 KVPY-SA

63. Let A = {q : sin(q) = tan(q)} and 67. A plane is in level flight at constant speed and
B = {q : cos(q) = 1} be two sets. Then : each of its two wings has an area of 25 m2. If the
(a) A = B speed of the air is 180 km/h over the lower wing
and 234 km/h over the upper wing surface,
(b) AË B
determine the plane’s mass. (Take air density to
(c) BË A be 1 kg m–3).
(d) A Ì B and B - A ¹ f (a) 440 kg (b) 400 kg
64. In the given figure ABCDE is a regular pentagon. (c) 4400 kg (d) 4044 kg
BP is drawn parallel to AC and it meets DC 68. Two positive charges of magnitude ‘q’ are
extended at P. EQ is drawn parallel to AD and placed, at the ends of a side (side 1) of a square
meets CD extended at Q. Then choose the of side ‘2a’. Two negative charges of the same
correct option. magnitude are kept at the other corners. Starting
from rest, if a charge Q moves from the middle
of side 1 to the centre of square, its kinetic
energy at the centre of square is
(a) zero
1 2 qQ æ 1 ö
1+
4 pe 0 a çè ÷
(b)

1 2qQ æ 2 ö
(c) 4pe a çè1 - ÷
0 5ø

(a) ar (ABCDE) = ar DAPQ 1 2qQ æ 1 ö


(d) 4pe a çè1 - ÷
(b) ar DEDQ = ar DBAC 0 5ø
(c) ar (ABCDE) = ar (PBEQ) 69. A long rectangular slab of transparent medium
of thickness d is placed on a table with length
(d) ar DBPC = ar DADE parallel to x-axis and width parallel to y-axis. A
65. If highest power of 7 in N! is k and that in (N + 1)! ray of light is traveling along y-axis at origin.
is K + 2 then how many values of N exist if The refractive index µ of the medium varies
N < 100. x/d
as, m = 1 + e .
(a) 0 (b) 1 (c) 2 (d) 3
The refractive index of air is 1. The value of x,
PHYSICS where the ray intersects the upper surface of
66. A car having a mass of 200 kg is rolling at a the slab-air boundary is
speed of 1 m/s towards a spring - stop system.
If the spring is non - linear such that it develops Glassy shaddy
d
300 x2 N force for a deflection of x m. The
maximum deceleration that the car A undergoes;
q
x

(a) 1 m/s2 (b) 1.5 m/s2 (a) d ln 2 (b) d ln 4


(c) 2 m/s 2
(d) 2.5 m/s 2 (c) 3d ln 2 (d) none of these
MOCK TEST-5 45

70. A thin tube of uniform cross-section is sealed at V(L)


(28.6 L, 373K)
both ends. It lies horizontally, the middle 5 cm
containing mercury and the two equal end (b) (22.4L,
containing air at the same pressure P. When the 273K)
tube is held at an angle of 60° with the vertical T(K)
direction, the length of the air column above and
V(L)
below the mercury column are 46 cm and 44.5 cm (30.6 L, 373K)
respectively. Calculate the pressure P in
centimeters of mercury. (The temperature of the (c) (22.4L,
273K)
system is kept at 30°C).
T(K)
(a) 75.4 cm (b) 57.4 cm
V(L)
(c) 70.4 cm (d) 44.5 cm

CHEMISTRY (d) (22.4L,


273K)
(14.2 L, 373K)
71. The geometry of ammonia molecule can be best T(K)
described as:
(a) Nitrogen at one vertex of a regular
tetrahedron, the other three vertices being 74.
occupied by three hydrogens.
(A) (B) (C) (D)
(b) Nitrogen at the centre of the tetrahedron,
three of the vertices being occupied by Decreasing order of rate of electrophilic aromatic
three hydrogens. substitution is:
(c) Nitrogen at the centre of an equilateral (a) A > B > C > D (b) A > C > B > D
triangle, three corners being occupied by (c) B > A > C > D (d) B > C > A > D
three hydrogens. 75. The density (in g mL–1) of a 3.60 M sulphuric
(d) Nitrogen at the junction of a T, three open acid solution that is 29% H2SO4 (molar mass =
ends being occupied by three hydrogens. 98 g mol–1) by mass will be
72. The oxidation states of sulphur in the anions (a) 1.45 (b) 1.64 (c) 1.88 (d) 1.22

SO 32 -, S2O24-, and S 2 O 62 - follow the order.. BIOLOGY

(a) S2O42– < SO32– < S2O62– 76. Maltose can be broken down into glucose
(b) SO32– < S2O42– < S2O62– molecules by the enzyme maltase. Which of the
following would slow the reaction rate?
(c) S2O42– < S2O62– < SO32–
(a) Adding maltase
(d) S2O62– < S2O42– < SO32–
(b) Adding maltose
73. Which of the following volume (V) - temperature
(T ) plots represents the behaviour of one mole (c) Removing glucose
of an ideal gas at one atmospheric pressure ? (d) Diluting with water
77. From the cross AABBCC × aabbcc, how many
V(L) (38.8 L, 373K) different kinds of (i) F1 gametes, (ii) F2 genotypes
(a) (22.4L, and (iii) F2 phenotypes would be expected?
273K)
(a) (i) 16, (ii) 24, (iii) 16 (b) (i) 8, (ii) 27, (iii) 8
T(K) (c) (i) 8, (ii) 64, (iii) 16 (d) (i) 8, (ii) 32, (iii) 16
EBD_7839
46 KVPY-SA

78. Tidal volume and Expiratory reserve volume of 80. Tay-Sachs disease, which is lethal, results from
an athlete is 500 mL and 1000 mL respectively. having the homozygous recessive condition of
What will be his Expiratory capacity if the re- the responsible gene. Which one of the
sidual volume is 1200 mL? following statements is true?
(a) 1500 mL (b) 1700 mL (a) Because homozygous recessive
(c) 2200 mL (d) 2700 mL individuals die, the recessive allele will
eventually be lost from the population.
79. If in a test tube, mRNA of human, tRNA of Rhizo-
bium and ribosomes of sunflower are added along (b) Only homozygous dominant individuals
with sufficient amino acids and ATP, the will be able to survive and reproduce.
polypeptide synthesised will be of the nature (c) Heterozygous individuals will survive and
of: may pass the recessive allele on to their
(a) body cells of human offspring.
(b) Rhizobium (d) In the heterozygous condition, the
dominant allele will overcome the recessive
(c) Sunflower
allele and only the dominant allele will be
(d) Both (a) and (b) passed on to offspring.

ANS W ER KEYS
Part-I Part-II
1 (b ) 11 (d ) 21 (a) 31 (b) 41 (d ) 51 (b ) 61 (a) 71 (b)
2 (d ) 12 (c) 22 (b) 32 (a) 42 (c) 52 (d ) 62 (c) 72 (a)
3 (c) 13 (b ) 23 (d) 33 (b) 43 (d ) 53 (a) 63 (b ) 73 (c)
4 (c) 14 (b ) 24 (d) 34 (a) 44 (d ) 54 (c) 64 (a) 74 (b)
5 (b ) 15 (c) 25 (d) 35 (d) 45 (a) 55 (a) 65 (c) 75 (d)
6 (a) 16 (a) 26 (d) 36 (d) 46 (c) 56 (b ) 66 (b ) 76 (d )
7 (b ) 17 (d ) 27 (a) 37 (d) 47 (a) 57 (b ) 67 (c) 77 (b)
8 (a) 18 (b ) 28 (a) 38 (a) 48 (d ) 58 (a) 68 (d ) 78 (a)
9 (d ) 19 (a) 29 (b) 39 (d) 49 (c) 59 (a) 69 (b ) 79 (a)
10 (a) 20 (d ) 30 (a) 40 (b) 50 (a) 60 (d ) 70 (a) 80 (c)
Telegram @unacademyplusdiscounts

Join Us on Telegram for More Such Books

https://telegram.me/unacademyplusdiscounts

Join us from the above link or search ''unacademyplusdiscounts'' in Telegram


61
Time : 3 Hours Stream-SA Maximum Marks : 100
INSTRUCTIONS
1. There are 80 questions in this paper.
2. The question paper contains two parts; Part–I (1 Mark Questions) and Part–II (2 Marks
Questions). There are four sections; Mathematics, Physics, Chemistry and Biology in each part.
3. There are four options given with each question, only one of them is correct.
4. For each incorrect answer 0.25 Mark in Part–I and 0.5 Mark in Part–II will be deducted.

PART-I (1 MARK QUESTIONS)

MATHEMATICS 4. Consider a set S = { 1, “2, 2, 2”2, ……. 1024},


consider another set P that is subset of S and
1. Let ABCD be a quadrilateral; X and Y be the has two distinct elements. If product of these
midpoints of AC and BD respectively and the two elements is more than or equal to 1024 the
lines through X and Y respectively parallel to how many values of set P exist?
BD, AC meet in O. Let P, Q, R, S be the midpoints (a) 55 (b) 110
of AB, BC, CD, DA respectively. Consider the
(c) 140 (d) None of these
statements.
5. A square ABCD is constructed inside a DPQR,
(i) quadrilaterals APOS and APXS have the having sides 10, 17, 21 as shown in figure. Find
same area. the approximation value of perimeter of square
(ii) the areas of the quadrilateral APOS, BQOP, ABCD.
CROQ, DSOR are all equal.
Then, which of the following is/are true?
(a) Only (i) (b) Only (ii)
(c) Only (i) and (ii) (d) None
2. If a(b!) is completely divisible by 511 where a is a
single digit no then find the minimum value of b.
(a) 40 (b) 45 (c) 50 (d) 46
3. If p = (k + 10)(k + 11)(k + 12)(k + 13)(k + 14) and
q = p + 2 then which one of these is false for q.
(a) q is even
(b) when divided by 2 it leaves remainder 0
(c) q is divisible by 3
(a) 28 (b) 23.2 (c) 25.4 (d) 28.8
(d) q is not prime
EBD_7839
48 KVPY-SA

6. Let S be the set of integers x such that: 11. N is a 75 digit number that is formed by writing
(i) 100 d” x d” 200 all the two digit numbers in which ten’s digit is
(ii) x is odd greater than unit’s digit, in increasing order one
after the other. What is remainder when n
(iii) x is divisible by 3 but not by 7
divided by 16?
How many elements does S contain?
(a) 5 (b) 7
(a) 16 (b) 12 (c) 11 (d) 13
(c) 9 (d) None of these
7. If a four digit number “abcd” is divisible by 4
12. ABCD is a parallelogram, X and Y are midpoints
and another four digit number “acbd” is
of BC and CD respectively. Then choose the
divisible by 8 then what is the minimum value
correct option.
of a + b + c + d ?
(a) 33 (b) 34 5
(a) ar DAXY = ar (ABCD)
(c) 36 (d) None of these 8
8. Suppose, the seed of any positive integer n is 1
(b) ar DAXY = ar (ABCD)
defined as follows: 2
Seed(n) = n, if n < 10 = seed(s(n)), otherwise, 3
where s(n) indicates the sum of digits of n. For (c) ar DAXY = ar (ABCD)
8
example, seed(7) = 7, seed(248) = seed(2 + 4 + 8) =
3
seed(14) = seed(1 + 4) = seed(5) = 5 etc. How (d) ar DAXY = ar (ABCD)
many positive integers n, such that n < 500, will 4
have seed(n) = 9? 13. Find all real numbers a for which the equation
(a) 39 (b) 72 (c) 81 (d) 55 x2 + (a “ 2)x + 1 = 3|x| has exactly three distinct
real solutions in x.
9. AB is a line segment of 4 cm length. P is the mid-
point of AB. Circles are drawn with A, P and B as (a) 1, 2 (b) 2, 3 (c) 1, 3 (d) 2, 4
centre and radii AP = PB. The area of shaded 14. There are two concentric circles such that the
portion (in cm2) is area of the outer circle is four times the area of
the inner circle . Let A, B and C be three distinct
D C points on the perimeter of the outer circle such
that AB and AC are tangents to the inner circle.
If the area of the outer circle is 12 square
A B centimeters then find the area (in square
P
centimeters) of the triangle ABC .

9 3 8 3
(a) (b)
p p
(a) 6 3 (b) 2p - 6 3 6 3
(c) (d) None of these
(c) (d) 3 3 p
2p - 3 3
10. If f(x) is a polynomial of degree 5 with leading 15. If x, y Î[ 0,10] , then number of solutions (x, y)
coefficient 2009. Also f(1) = 1, f(2) = 3, f(3) = 5, 2

f(4) = 7, f(5) = 9. What is the value of f(6). of the inequation 3sec x -1


9 y 2 - 6 y + 2 £ 1 is
(a) 322191 (b) 11 (a) 2 (b) 1
(c) 312331 (d) 241091 (c) 4 (d) infinite
MOCK TEST-6 49

PHYSICS find the value of k. ///////////////////////////

16. 5 moles of nitrogen gas are enclosed in an M = mass of disc shaped


O
adiabatic cylindrical vessel. The piston itself is pulley and R = radius of
a rigid light cylindrical container containing 3 the pulley.
moles of Helium gas. There is a heater which (a) 1 (b) 2
M1 M2 v
gives out a power 100 cal to the nitrogen gas. A (c) 4 (d) 5
power of 30 cal is transferred to Helium through
20. A ball is dropped from a certain height on a
the bottom surface of the piston. The rate of
horizontal floor. The coefficient of restitution
increment of temperature of the nitrogen gas
assuming that the piston moves slowly : 1
between the ball and the floor is . The
2
(a) 2K/sec displacement time graph of the ball will be
(b) 4K/sec He
N2
(c) 6K/sec

(d) 8K/sec (a) (b)


17. A force (F) acting on a body is dependent on its
displacement s as F µ s -1/3 . Therefore, the
power delivered by the force varies with its
displacement as:
(c) (d)
(a) s 2/3 (b) s1/2
(c) s -5/3 (d) s0
21. Consider the following statements-
18. A horizontally oriented copper rod of length l is
rotated about a vertical axis passing through its I. Isotopes are atoms having same number of
middle. Calculate the rotated frequency at which protons but different number of neutrons.
the rod ruptures. Breaking or rupture strength II. Isotopes are atoms having same number of
of copper takes as s and density of copper r. neutrons but different number of protons.
III. Isotopes are atoms having same number
1 8s 1 3s
(a) (b) of protons and neutrons.
2p rl 2 4p rl 2
Choose the correct statement.
1 2s (a) I and II (b) II only
(c) 3p rl 2 (d) None of these (c) I only (d) I, II and III
19. A pulley is in the form of a disc of mass M and 22. Express which of the following set ups can be
radius R. In following figure two masses M1 and used to verify Ohm’s law?
M2 are connected by a light inextensible string A
which passes over the pulley. Assuming that (a) V (b) A V
the string does not slip over the pulley, the
angular momentum of system at the instant
shown, about axis of rotation of pulley is A V

é 1 ù (c) (d)
êë M 2 + M1 + k M úû vR then V A
EBD_7839
50 KVPY-SA

23. A satisfactory photographic print is obtained (a) absolute error same for both, relative error
when the exposure time is 10 sec at a distance of greater for MA and lesser for MB.
2 m from a 60 cd lamp. The time of exposure (b) absolute error same for both, relative error
required for the same quality print at a distance
greater for MB and lesser for MA.
of 4m from a 120 cd lamp is
(c) relative error same for both, absolute error
(a) 5 sec (b) 10 sec
greater for MA and lesser for MB.
(c) 15 sec (d) 20 sec
24. Two astronauts are floating in gravitational free (d) relative error same for both, absolute error
space after having lost contact with their greater for MB and lesser for MA.
spaceship. The two will 28. Suppose the drift velocity nd in a material varied
I. Keep floating at the same distance between
with the applied electric field E as nd µ E .
them
II. Move towards each other Then v – I graph for a wire made of such a material
III. Move away from each other is best given by :
IV. Will becomes stationary v v
Choose the correct statement.
(a) I only (b) II only (a) (b)
(c) III and II (d) IV and I I I
25. A defective eye cannot see close objects clearly v v
because their image is formed
(a) On the eye lens (c) (d)
(b) Between eye lens and retina I I
(c) on the retina
29. A jet plane flying at a constant velocity v at a
(d) Beyond retina
height h = 8 km, is being tracked by a radar R
26. A particle of mass m is fixed to one end of a light
located at O directly below the line of flight, if
spring of force constant k and unstretched
the angle q is decreasing at the rate of 0.025 rad/
length l. The system is rotated about the other
end of the spring with an angular velocity w, in s, the velocity of the plane when q = 60° is :
gravity free space. The increase in length of the
spring is
mw 2 l
(a)
k
mw 2 l
(b)
( k - mw ) 2

mw 2 l
(c)
( k + mw ) 2 (a) 1440 km/h (b) 960 km/h
(c) 1920 km/h (d) 480 km/h
(d) None of these
27. Two masses MA and MB (MA < MB) are weighed 30. What is the total kinetic energy of 2g of nitrogen at
using same weighing machine. Absolute error 300 K? Given : molecular weight of nitrogen = 28.
and relative error in two measurement are (a) 267.1 × 107 erg (b) 672.1 × 107 erg
(Assume only systematic errors are involved) (c) 762.1 × 104 erg (d) 267.1 × 104 erg
MOCK TEST-6 51

CHEMISTRY 37. The orbital having m = – 2 should not be present


in the following sub-shell:
31. The spectrum of He is expected to be similar to
that of : (a) d (b) f (c) g (d) p
(a) H (b) Li+ (c) Na (d) He+ 38. The screening effect of d-electron is
(a) equal to p-electron.
32. Which of the following compounds is not
aromatic? (b) much more than p-electron.
(c) same as ¦-electrons.
(a) (b) (d) less than p-electrons.
N
H 39. Which of the following does not have a
++
tetrahedral structure?

(a) BH–4 (b) BH3 (c) NH +4 (d) CH4


(c) (d)
N – 40. The plot which does not represent isothermal
expansion of an ideal gas is:
33. Among the following which compound will show
the highest lattice energy ?
(a) KF (b) NaF (c) CsF (d) RbF P P
34. Which of the following explanation is best for (a) (b)
not placing hydrogen with alkali metals or
halogen? 1/Vm Vm
(a) The ionization energy of hydrogen is high
for group of alkali metals or halogen.
(b) Hydrogen can form compounds. PVm
(c) Hydrogen is much lighter element than the (c) (d) U
alkali metals or halogens.
(d) Hydrogen atom does not contain any P Vm
neutron.
35. An element reacts with oxygen to give a 41. Which of the following substances consists of
only one element?
compound with a high melting point. This
compound is also soluble in water. The element (a) Marble (b) Sand
is likely to be – (c) Diamond (d) Glass
(a) Calcium (b) Carbon 42. What will be the molarity of a solution containing
(c) Silicon (d) Iron 5 g of sodium hydroxide in 250 mL solution?
36. The rate of reaction for which of the following is (a) 0.5 (b) 1.0 (c) 2.0 (d) 0.1
not affected by pressure: 43. The presence or absence of hydroxyl group on
which carbon atom of sugar differentiates RNA
ˆˆ† PCl5
(a) PCl3 + Cl2 ‡ˆˆ
and DNA?
ˆˆ† 2NH 3
(b) N 2 + 3H 2 ‡ˆˆ (a) 1st (b) 2nd (c) 3rd (d) 4 th
44. Which of the following is acidic salt?
ˆˆ† 2NO
(c) N 2 + O2 ‡ˆˆ
(a) Pb(OH)NO3 (b) Cu(OH)NO3
(d) ˆˆ† 2SO 3
2SO 2 + O 2 ‡ˆˆ (c) KHSO3 (d) None of these
EBD_7839
52 KVPY-SA

45. The oxidation number of Cr in CrO5 which has 53. The bacterium Bacillus thuringiensis can
the following structure is: withstand heating, dryness, and toxic chemicals
that would kill most other bacteria. This indicates
O
O || O that it is probably able to form:
Cr (a) Pseudopodia (b) Endotoxins
O O
(c) Endospores (d) Pili
(a) + 4 (b) + 5 (c) + 6 (d) + 3
54. The direction of the conduction of food through
BIOLOGY phloem is from
46. The normal pH of the arterial blood is: (a) bottom to upwards
(a) 6.8 (b) 7.8 (b) top to bottom.
(c) 7.4 (d) 8.8 (c) leaves to roots.
47. Which of the following factors influence the (d) None of the above
flowering in plants? 55. The structure of glucose and galactose are same
(a) Acidity of soil except with respect to:
(b) Amount of green pigment (a) First carbon atom
(c) Photoperiod (b) Second carbon atom
(d) Water in the soil
(c) Third carbon atom
48. It is said that the Taj Mahal may be destroyed
due to: (d) Fourth carbon atom
(a) Food in Yamuna river 56. Bamboo and grasses elongate by the activity of:
(b) Decomposition of marble as a result of high (a) Apical meristem
temperature (b) Lateral meristem
(c) Air pollutants released from oil refinery of (c) Secondary meristem
Mathura
(d) Intercalary meristem
(d) All of the above
57. Amylopsin acts upon
49. Glycolysis cannot occur in the absence of:
(a) polypeptide in acidic medium.
(a) Ca (b) Mg
(c) Mn (d) Co (b) polysaccharide in acidic medium.
50. Which of the following gland is not involved in (c) polysaccharide in alkaline medium.
the process of digestion? (d) polypeptide in alkaline medium.
(a) Liver (b) Pancreas 58. Which of the following is a micronutrient?
(c) Salivary gland (d) Adrenal gland (a) Copper (b) Calcium
51. Which of the following is false? (c) Phosphorus (d) Magnesium
(a) Cardiac muscle is striated.
59. Endocrine glands produce or action of endocrine
(b) Smooth muscle does not contain actin. glands is mediated through:
(c) Skeletal muscle is considered voluntary.
(a) Hormones (b) Enzymes
(d) Smooth muscle is found in the iris of the
(c) Minerals (d) Vitamins
eye and the walls of the bladder.
52. The compound acting as an oxygen store in skeletal 60. Which one of the following cell organelles is
muscle is: found only in plants?
(a) Myoglobin (b) Haemoglobin (a) Golgi complex (b) Mitochondria
(c) ATP (d) Myokinase (c) Plastids (d) Ribosomes
MOCK TEST-6 53

PART-II (2 MARKS QUESTIONS)


MATHEMATICS 64. ABCD is a trapezium with AB || CD. M and N are
points on AD and BC such that MN || CD. If area
61. Three circles each of radius r units are drawn
of ABNM is half the area of ABCD, then which
inside an equilateral triangle of side a units, such
of the relation is true.
that each circle touches the other two and two
sides of the triangle as shown in the figure, (P, Q (a) MN2 = AB2 + CD2
and R are the centres of the three circles). Then (b) 2MN2 = AB + CD
relation between r and a is (c) 2MN2 = AB2 + CD2
(d) None of the above
(a) a = 2 ( 3 +1)r (b) a = ( 3 +1)r
65. Lets define a set S = { Pi | set of all 3 digit natural
number that has odd number of factors}. Find
(c) a = ( 3 + 2)r (d) a = 2 ( 3 + 2)r the highest power of 12 in the product of all the
62. Square ABPQ and ADRS are drawn on the sides elements of set S.
AB and AD of a parallelogram ABCD. Then (a) 16 (b) 19
(c) 20 (d) 25
PHYSICS

66. Spherical particles of pollen are shaken up in


water and allowed to settle. The depth of water
is 2 × 10–2 m. What is the diameter of the largest
particles remaining in suspension one hour later?
[Density of pollen = 1.8 × 103 kg m–3. Viscosity
of water = 1 × 10 –2 poise and density of
water, 1 × 10–5 kg m–3.]
(a) 5.34 mm (b) 1.77 mm
(c) 7.11 mm (d) 3.54 mm
67. The density of a sphere is measured by
measuring its mass and diameter. If, it is known
(a) ÐSAQ = ÐABC that the maximum percentage errors in the
(b) ÐSRD = ÐADC measurement are 2% and 3%, then find the
maximum percentage error in the measurement
(c) ÐSAD = ÐABC of density?
(d) None of these (a) 15% (b) 18%
63. Let P = {q : sin q – cos q = 2 cos q} and (c) 9% (d) 11%
68. Students I, II and III perform an experiment for
Q = {q : sin q + cos q = 2 sin q} be two sets. measuring the acceleration due to gravity (g)
Then using a simple pendulum. They use different
lengths of the pendulum and /or record time for
(a) P Ì Q and Q – P ¹ Æ different number of oscillations. The
(b) Q Ë P observations are shown in the table.
(c) P Ë Q Least count for length = 0.1 cm
(d) P = Q Least count for time = 0.1 s
EBD_7839
54 KVPY-SA

Student Length of the No. of Total time Time 70. The reflecting surface is represented by the
pendulum oscillations for (n) period equation 2x = y2 as shown in the figure. A ray
(cm) (n) oscillations (s) travelling horizontal becomes vertical after
(s) reflection. The co-ordinates of the point of
I 64.0 8 128.0 16.0
incidence are :
II 64.0 4 64.0 16.0
III 20.0 4 36.0 9.0
If EI, EII and EIII are the percentage errors in g,
i.e.,
æ Dg ö
çè g ´ 100÷ø for students I, II and III,
respectively, then
(a) EI = 0 (b) EI is minimum (a) (1/2, 1) (b) (1, 1/2)
(c) EI = EII (d) EII is maximum (c) (1/2, 1/2) (d) None
69. A horizontally insulated cylindrical vessel of CHEMISTRY
length 2l is separated by a thin insulating piston
71. The hydrocarbon which can react with sodium
into two equal parts each of which contains n
in liquid ammonia is :
moles of an ideal monoatomic gas at temperature
T. The piston is connected to the end faces of (a) CH 3CH 2 CH 2C º CCH 2CH 2 CH3
the vessel by undeformed springs of force
constant k each. (b) CH 3CH 2C º CH
The left part in the contact with a thermostat (a (c) CH 3CH = CHCH3
device which maintains a constant temperature).
When an amount of heat Q is supplied to the (d) CH 3CH 2C º CCH 2CH3
gas in the right part, the piston is displaced to 72. The mass of BaCO3 produced when excess CO2
the left by a distance x = l/2. Determine the heat
is bubbled through a solution of 0.205 mol
Q’ given away at the temperature T to the
Ba(OH)2 is
thermostat by the left part of the piston.
(a) 81 g (b) 40.5 g (c) 20.25 g (d) 162 g
2L 73. From the following bond energies:
H – H bond energy: 431.37 kJ mol–1
k k C = C bond energy: 606.10 kJ mol–1
C – C bond energy: 336.49 kJ mol–1
n,T n,T
Q C – H bond energy: 410.50 kJ mol–1
Enthalpy for the reaction,
5
(a) Q ' = Q - kl 2 + 3nRT
2 H H H H
5 | | | |
(b) Q ' = Q + kl 2 - 3nRT C = C + H - H ¾¾
® H - C- C - H
2 | | | |
5 H H H H
(c) Q ' = Q - kl 2 - 3nRT
2 will be:
5 (a) – 243.6 kJ mol–1 (b) –120.0 kJ mol–1
(d) Q ' = Q + kl 2 + 3nRT
2 (c) 553.0 kJ mol–1 (d) 1523.6 kJ mol–1
MOCK TEST-6 55

74. The van der Waal’s constant ‘a’ for four gases P, (a) A – III; B – V; C – I; D – II; E – IV
Q, R and S are 4.17, 3.59, 6.71 and 3.8 atm L2 mol–2 (b) A – I; B – II; C – III; D – IV; E – V
respectively. Therefore, the ascending order of
(c) A – II; B – III; C – IV; D – V; E – I
their liquefaction is:
(d) A – IV; B – I; C – V; D – II; E – III
(a) R < P < S < Q (b) Q < S < R < P
(c) Q < S < P < R (d) R < P < Q < S 77. Which of the following statements are incorrect?
75. Which of the following structures correspond (i) A dominant allele determines the phenotype
to the product expected, when excess of C6H6 when paired with a recessive allele.
reacts with CH2Cl2 in presence of anhydrous (ii) A recesive allele is weaker than a dominant
AlCl3? allele.
(iii) A recessive allele shows its effects when
(a) CH paired with a dominant allele.
Cl (iv) A dominant allele is always better for an
organism.
(b) CHCl2 (a) (i), (ii) and (iv)
(b) (ii), (iii) and (iv)
Cl (c) (i), (ii) and (iii)
(c) C (d) (i), (iii) and (iv)
Cl 78. Mark the odd pair out.
(a) Totipotency: Parenchyma
(d) CH2 (b) Secondary growth: Cambium
(c) Dermal cells: Trichomes
BIOLOGY (d) Periderm: Collenchyma
76. Match the terms given in column-I with their 79. Which parental phenotypes would produce
definition given in column-II and choose the offspring with blood group phenotypes in the
correct option. expected ratio of 1 type A : 1 type B?
Column-I Column-II Blood group of Blood group of
(Terms) (Definition) mother father
A. Semi-circular canal I. Spiral organ of Corti (a) A B
B. Vestibule II. Fluid found in the (b) AB AB
scala vestibuli and (c) AB B
scala tympani
(d) AB O
C. Cochlea III. Evaluates rotational
80. Bacteriorhodopsin absorbs a quantum of light
motion
by P 570 which is transformed into P 412
D. Perilymph IV. Fluid found within the accompanied by release of H+. It is also known
organ of Corti that F0-F1 system generates ATP through the
E. Endolymph V. Responds to gravity movement of H + . What should be the
and movements of the appropriate diagram for a photosynthetic
head Halobacterium halobium?
EBD_7839
56 KVPY-SA

(a) (b)

(c) (d)

ANS W ER KEYS
Part-I Part-II
1 (c) 11 (a) 21 (c) 31 (b) 41 (c) 51 (b ) 61 (a) 71 (b)
2 (b ) 12 (c) 22 (a) 32 (a) 42 (a) 52 (a) 62 (a) 72 (b)
3 (a) 13 (c) 23 (d) 33 (b) 43 (b ) 53 (c) 63 (d ) 73 (b)
4 (b ) 14 (a) 24 (b) 34 (c) 44 (c) 54 (c) 64 (c) 74 (c)
5 (b ) 15 (c) 25 (d) 35 (a) 45 (c) 55 (d ) 65 (b ) 75 (d)
6 (d ) 16 (a) 26 (b) 36 (c) 46 (c) 56 (d ) 66 (d ) 76 (a)
7 (a) 17 (d ) 27 (a) 37 (d) 47 (c) 57 (c) 67 (d ) 77 (b)
8 (d ) 18 (a) 28 (c) 38 (d) 48 (c) 58 (a) 68 (b ) 78 (d)
9 (c) 19 (b ) 29 (b) 39 (b) 49 (b ) 59 (a) 69 (c) 79 (d)
10 (d ) 20 (c) 30 (a) 40 (d) 50 (d ) 60 (c) 70 (a) 80 (a)
Telegram @unacademyplusdiscounts

Join Us on Telegram for More Such Books

https://telegram.me/unacademyplusdiscounts

Join us from the above link or search ''unacademyplusdiscounts'' in Telegram


71
Time : 3 Hours Stream-SA Maximum Marks : 100
INSTRUCTIONS
1. There are 80 questions in this paper.
2. The question paper contains two parts; Part I (1 Mark Questions) and Part II (2 Marks Questions).
There are four sections; Mathematics, Physics, Chemistry and Biology in each part.
3. There are four options given with each question, only one of them is correct.
4. For each incorrect answer 0.25 Mark in Part–I and 0.5 Mark in Part–II will be deducted.

PART-I (1 MARK QUESTIONS)

MATHEMATICS (a) n is even


1. If p, q , r are + ve and are in A. P., the roots of (b) n is odd
quadratic equation px2 + qx + r = 0 are all real for (c) n is an odd multiple of 3
(d) n is prime
r p
(a) -7 ³ 4 3 (b) r - 7 ³ 4 3
p 6. In the given figure, ABCD is a quadrilateral in
which diagonals AC and BD intersect at a point
(c) all p and r (d) no p and r E Then
2. ab is a two digit number less than 50 such that
(ab)2 = c0cb where a,b,c are distinct digits not D
equal to zero. How many values of “ab” exist. A
(a) 2 (b) 1 (c) 4 (d) 5 N
3. What should be the value of n such that E
A = 31 + 32 + …. + 3n is a perfect square?
(a) 112 (b) 144 M
(c) 225 (d) None of these
4. x and y are two non-negative numbers such that B C
2x + y = 10. The sum of the maximum and
minimum values of (x + y) is (a) Area DAED × Area DBEC
(a) 6 (b) 9 = Area DABE × Area DCDE
(c) 10 (d) 15 (b) Area DABE × Area DBEC
5. Each of the numbers x1, x2. . . xn, n > 4, is equal = Area DAED × Area DCDE
to 1 or –1. Suppose, x1 x2 x3 x 4 + x2 x3 x 4x 5 + (c) Area DBEC × Area DCDE
x3x4x5x6 + .... + xn-3xn-2xn-1xn + xn-2xn-1xnx1
= Area DAED × Area DABE
+ xn-1xnx1x2 + xnx1x2x3 = 0, then,
(d) None of these
EBD_7839
58 KVPY-SA

7. In the diagram O is the center of a circle. 12. The diameter of one of the bases of a truncated
AE + EB = CE + ED. OP ^ B and OQ ^ CD then cone is 100 mm. If the diameter of this base is
true relation between OP and OQ is : increased by 21% such that it still remains a
truncated cone with the height and the other
base unchanged, the volume also increases by
21%. The radius of other base is
(a) 65 (b) 55 (c) 45 (d) 35
Q
O B 13. Suppose A1, A2, ......, A30 are thirty sets each
E having 5 elements and B1, B2, ......, Bn are n sets
P
A 30 n
C
each with 3 elements. Let U Aj = U Bj = S
(a) OP > OQ (b) OP < OQ i =1 j =1
1 and each element of S belongs to exactly 10 of
(c) OP = OQ (d) OP = OQ the Ai's and exactly 9 of the Bj's. Then n is equal
2
8. In the given figure, DABC is a right angled to
triangle semicircles are drawn on AB, AC and (a) 15 (b) 3 (c) 45 (d) 35
BC as diameters. It is given that AB = 3 cm and 14. Consider n is a two digit number such that the
AC = 4 cm. Find the area of shaded region. ratio of the number to the sum of its digit is
maximum. How many such numbers exist?
(a) 4 (b) 9
(c) 12 (d) 11
15. Let X and Y be two non-empty sets. Let
f : X ® Y be a function. For A Ì X and
B Ì Y, define f ( A)= { f ( x) : x Î A};
(a) 12 cm2 (b) 6 cm2
2 (d) 15 cm2
(c) 9 cm f -1 ( B)= {x Î X : f ( x) Î B}, then
9. In DABC, AB = AC, P and Q are points on AC and
AB respectively such that BC = BP = PQ = AQ. (a) f ( f -1 ( B )) = B (b) f ( f -1 ( B)) Ì B
Then, ÐAQP is equal to (use p =180º)
2p 3p
(c) f -1 ( f ( A)) = A (d) f -1 ( f ( A)) Ì A
(a) (b)
7 7 PHYSICS
4p 5p
(c) (d) 16. Time graph between the displacement x and time
7 7
10. A drain cover is made from a square metal plate t for a particle moving in a straight line is shown
of side 40 cm having 441 holes of diameter 1 cm in fig. During the interval OA, AB, BC and CD,
each drilled in it. Find the area of the remaining the acceleration of the particle is
square plate. x
(a) 1250.5 cm2 (b) 1253.5 cm2 A B C
D
(c) 1240.2 cm2 (d) 1260.2 cm2
11. A and B have some amount with them. B gave
some amount to A hence due to this change t
O
now A and B both have as much rupee as much
OA AB BC CD
they had paisa and as much paisa as much they
(a) + 0 + +
had rupee. What can be the maximum total sum
(b) – 0 + 0
of money with them?
(c) + 0 – +
(a) 199.98 (b) 188.98
(c) 198.97 (d) 198.98 (d) – 0 – 0
MOCK TEST-7 59

17. In the determination of Young’s modulus 19. A circular coil ABCD carrying a current i is placed
in a uniform magnetic field. If the magnetic force
æ 4 MLg ö r
çè Y = ÷ by using Searle’s method, a wire on the segment AB is F , the force on the
pld 2 ø
remaining segment BCDA is
of length L = 2 m and diameter d = 0.5 mm is r A
used. For a load M = 2.5 kg, an extension l = 0.25 (a) F i
mm in the length of the wire is observed. r
(b) -F D B
Quantities d and l are measured using a screw r
gauge and a micrometer, respectively. They (c) 3F
have the same pitch of 0.5 mm. The number of r C
(d) -3F
divisions on their circular scale is 100. The
contributions to the maximum probable error of 20. A catapult consists of two parallel rubber strings,
the Y measurement each of lengths 10 cm and cross sectional area
(a) due to the errors in the measurements of d 10 mm2. When stretched by 5 cm, it can throw a
and l are the same. stone of mass 100 g to a vertical height of 25 m.
(b) due to the error in the measurement of d is Determine Young’s modulus of elasticity of
twice that due to the error in the rubber.
measurement of l. (a) 9.8 × 107 N/m2 (b) 8.9 × 107 N/m2
(c) due to the error in the measurement of l is (c) 25 × 10–1 N/m2 (d) 2.5 × 10–1 N/m2
twice that due to the error in the 21. An exhausted chamber with nonconducting
measurement of d. walls is connected through a valve to the
(d) due to the error in the measurement of d is atmosphere where the pressure is p0 and the
four times that due to the error in the temperature is T0 . The valve is opened slightly
measurement of l. and air flows into the chamber until the pressure
18. A rod of length l is in motion such that its ends within the chamber is p0. Assuming the air to
A and B are moving along x-axis and y-axis behave like an ideal gas with constant heat
dq capacities, the final temperature of the air in the
respectively. It is given that = 2 rad/sec chamber is
dt
always. P is a fixed point on the rod. (a) g T0 (b) (T0 / g)
y (c) (1 / g – 1)T0 (d) (g / 1 – g) T0
B 22. A hollow sphere is held suspended. Sand is now
P
poured into it in stages. The centre of gravity of
q
the sphere with the sand

A
x
M

Let M be the projection of P on x-axis. For the


SAND
p
time interval in which q changes from 0 to ,
2
choose the correct statement. (a) rises continuously
(b) remains unchanged in the process
(a) The speed of M is always directed towards
right (c) first rises and then falls to the original
(b) M executes S.H.M. position
(c) M moves with constant speed (d) first falls and then rises to the original
(d) M moves with constant velocity position
EBD_7839
60 KVPY-SA

23. Adding detergents to water helps in removing 27. Read the following statements
dirty greasy stains. This is because S1 : An object shall weigh more at pole than at
I. It increases the oil-water surface tension equator when weighed by using a physical
II. It decreases the oil-water surface tension balance
III. Viscosity of oil is high S2 : It shall weigh the same at pole and equator
IV. Dirt is held suspended surrounded by when weighed by using a physical balance
detergent molecules S3 : It shall weigh the same at pole and equator
(a) II and IV (b) I only when weighed by using a spring balance
(c) III and IV (d) IV only S4 : It shall weigh more at the pole than at
24. The pair of physical quantities that has the equator when weighed using a spring
different dimensions is : balance
(a) Reynolds number and coefficient of friction Which of the above statements is/are correct
(b) Curie and frequency of a light wave (a) S1 and S2 (b) S1 and S4
(c) Latent heat and gravitational potential (c) S2 and S3 (d) S2 and S4
(d) Planck’s constant and torque 28. A point object is kept in front of a plane mirror.
The plane mirror is doing SHM of amplitude 2
25. The graph shown illustrates velocity versus
cm. The plane mirror moves along the x - axis
time for two cars A and B constrained to move in
which is normal to the mirror. The amplitude of
a straight line. Both cars were at the same the mirror is such that the object is always in
position at t = 0s. Consider the following front of the mirror. The amplitude of SHM of the
statements. image is
(1) Car A is travelling west and Car B is (a) 0 (b) 2 cm
travelling east. (c) 4 cm (d) 1 cm
(2) Car A overtakes Car B at t = 5 s.
R
(3) Car A overtakes Car B at t = 10 s. 29. A circular disc of radius R and thickness has
14.0
6
12.0 moment of inertia I about an axis passing through
Velocity (m/s) [west]

Car A
10.0 its centre perpendicular to its plane. It is melted
8.0
and recasted into a solid sphere. The moment of
6.0
4.0 inertia of the sphere about its diameter is
Car B
2.0 2I
0.0 (a) I (b)
0.0 2.0 4.0 6.0 8.0 10.0 12.0 14.0 8
Time (s)
I I
Which of the following is correct? (c) (d)
(a) Only statement 1 (b) Only statement 2 5 10
(c) Only statement 3 (d) Statements 1 and 2 30. A man is standing on an international space
26. In the given figure, a smooth parabolic wire track station, which is orbiting earth at an altitude
lies in the xy-plane (vertical). The shape of track 520 km with a constant speed 7.6 km/s. If the
is defined by the equation y = x2. A ring of mass man’s weight is 50 kg, his acceleration is
m which can slide freely on the wire track, is (a) 7.6 km/s2 (b) 7.6 m/s2
placed at the position A (1,1). The track is rotated (c) 8.4 m/s2 (d) 10 m/s2
with constant angular speed w such there is no CHEMISTRY
relative slipping between the ring and the track.
y 31. From a heated mixture of nitrogen, oxygen and
The value of w is
carbon, two compounds (out of the many
(a) g /2 w obtained) are isolated. The rates of diffusion of
(b) g the two isolated compounds are almost identical.
The two compounds are
(c) 2g x (a) N2O and CO2 (b) CO and NO
O (0, 0)
(d) 2 g (c) CO2 and NO2 (d) N2O and CO
MOCK TEST-7 61

32. HSO-4 + OH - ® SO 42- + H 2O


39. The size of species I, I+ and I– decreases in the
order:
Which statement is correct about conjugate acid
(a) I+ > I– > I (b) I– > I > I+
base pair? – +
(c) I > I > I (d) I > I+ > I–
(a) HSO-4 is conjugate acid of base SO24- . 40. Which of the following pairs of species have
identical shapes?
(b) HSO-4 is conjugate base of acid SO24- . (a) NO+2 and NO2- (b) PCl5 and BrF5
-
(c) SO24- is conjugate acid of base HSO-4 . (c) XeF4 and ICl4 (d) TeCl4 and XeO4
41. A metal M readily forms its sulphate MSO4 which
(d) Conjugate acid of SO24- is not possible. is water-soluble. It forms its oxide MO which
33. Equivalent mass of H 3 PO 2 when it becomes inert on heating. It forms an insoluble
disproportionate into PH3 and H3PO3 is: hyroxide M(OH)2 which is soluble in NaOH
(a) M (b) M/2 (c) M/4 (d) 3M/4 solution. Then M is
34. Which of the following electronic configuration (a) Mg (b) Ba (c) Ca (d) Be
of an atom has the lowest first ionisation 42. In the isochoric process, DH for a system is equal
enthalpy? to:
(a) 1s2 2s2 2p3 (b) 1s2 2s2 2p6 3s1 (a) P. DV (b) P V
2
(c) 1s 2s 2p2 6 (d) 1s2 2s2 2p5 (c) E + P.DV (d) D E
35. In the following reaction 43. The IUPAC name of the following compound is
H SO
CH3 – CH2 – CH2 – CH3 ¾¾¾¾
2 4
475 K
®
(a)
CH3CH = CHCH3 predominates.
(b)
CH2 = CHCH2CH3 predominates.
(c)
Both are formed in equal amounts. (a) trans-2-chloro-3-iodo-2-pentene
(d)
The amount of production depends on the (b) cis-3-iodo-4-chloro-3-pentene
nature of catalyst. (c) trans-3-iodo-4-chloro-3-pentene
36. The conjugate base of HBr is : (d) cis-2-chloro-3-iodo-2-pentene
44. In the reaction
(a) H + (b) H 2 Br +
Cr2 O 72- + 14 H + + 6I - ¾
¾® 2Cr 3+ + 7H 2 O + 3I 2
- +
(c) Br (d) Br Which element is reduced?
37. Hydrogen bonding is maximum in : (a) I (b) O (c) H (d) Cr
(a) C2H5OH (b) CH3OCH3 45. According to Einstein's photoelectric equation,
(c) (CH3)2 C = O (d) CH3CHO the graph between kinetic energy of
38. Which of the following will have larger dipole photoelectrons ejected and the frequency of the
moment? incident radiation is :

(a) (b) (a) (b)

(c) (d) (c) (d)


EBD_7839
62 KVPY-SA

BIOLOGY 53. In micturition:


46. When dominant and recessive alleles express (a) Urethra relaxes (b) Ureter contracts
together, it is called: (c) Ureter relaxes (d) Urethra contracts
(a) Co-dominance 54. Thirty percent of the bases in a sample of DNA
(b) Dominance extracted from eukaryotic cells are adenine. What
percentage of cytosine is present in this DNA?
(c) Amphidominance
(a) 10% (b) 20% (c) 30% (d) 40%
(d) Pseudodominance
55. Which of the following is incorrectly matched?
47. Stool of a person is whitish grey coloured due
(a) Ribozyme: Proteinaceous in nature
to malfunction of which of the following organ?
(b) Apoenzyme: The protein part of enzyme
(a) Pancreas (b) Spleen
(c) Co-enzyme: Loosely attached organic
(c) Kidney (d) Liver
cofactor of holoenzyme
48. Secondary air pollutant is: (d) Co-factor : Non-protein part of holoenzyme
(a) PAN (b) CO 56. Hepatitis B is transmitted through:
(c) SO2 (d) Aerosols (a) Sneezing
49. Which digestive processes take place in the (b) Female Anopheles
mouth? (c) Coughing
(a) Both chemical and physical digestion. (d) Blood transfusion
(b) Chemical digestion only. 57. At the instant following the second heart sound,
(c) Physical digestion only. which heart valves are open?
(d) Neither chemical nor physical digestion. (a) Atrioventricular valves only.
50. Which of the following characteristic distinguish (b) Both atrioventricular valves and semilunar
arthropods from annelids and molluscs? valves.
(a) An external skeleton made of chitin (c) All valves are closed.
(a polysaccharide) and protein rather than (d) Semilunar valves only.
a shell made chiefly of mineral salts. 58. Hugo de Vries gave his mutation theory on
(b) Subdivision of the legs into movable organic evolution while working on:
segments. (a) Pisum sativum
(c) Distinct group of muscles, derived from (b) Drosophila melanogaster
many body segments, that move the (c) Oenothera lamarckiana
separate parts of the exoskeleton. (d) Althea rosea
(d) All of the above 59. Source of somatostatin is the same as that of:
51. Copper-T is a device that prevents: (a) Insulin and glucagon
(a) Implantation of blastocyst (b) Vasopressin and oxytocin
(b) Ovulation (c) Thyroxine and calcitonin
(c) Fertilisation (d) Somatotropin and prolactin
(d) Egg maturation 60. DNA replication is
52. A functional piece of mRNA has 66 codons. What (a) conservative and discontinuous.
is the maximum number of amino acids that could (b) semiconservative and semi-discontinuous.
be present in the protein, coded for this mRNA? (c) semiconservative and discontinuous.
(a) 22 (b) 64 (c) 65 (d) 66 (d) conservative.
MOCK TEST-7 63

PART-II (2 MARKS QUESTIONS)

MATHEMATICS 65. Consider four digit numbers for which the first
two digits are equal and the last two digits are
61. If N is 100 consecutive even multiples of 3.
If P is the smallest prime number divisible by also equal. How many such numbers are perfect
N + 1 then which one of the following is true squares?
about P? (a) 2 (b) 4
(a) P is a single digit prime number
(c) 0 (d) 1
(b) 10 < P < 25
(c) 25 < P < 50 PHYSICS
(d) P > 100
66. A double-pane window used for insulating a
62. If a, b are the roots of x 2 + px + q = 0 and
room thermally from outside consists of two
a b glass sheets each of area 1m2 and thickness 0.01
also of x 2 n + p n x n + q n = 0 and if
, are root
b a m separated by a 0.05 m thick stagnant air space.
of xn + 1 + (x + 1)n = 0, then n can be i + p ¹ 0 In the steady state, the room glass inter-face
(a) 13 (b) 14 and the glass -outdoor interface are at constant
(c) 15 (d) None of these temperatures of 27°C and 0°C respectively.
63. What is the area of the shaded region shown Calculate the rate of heat flow through the
below, if the radius of each circle is equal to the
side of the hexagon, which in turn is equal to 6 window pane. Given thermal conductivities of
cm, and A and B are the centres of the circles ? glass and air as 0.8 and 0.08 Wm –1 K –1
respectively.
(a) 41.5 J/s (b) 80.3 J/s
A B
(c) 90 J/s (d) 90.6 J/s
67. In the arrangement shown in fig., the ends P
(a) 6 ( 9 3 - 4p ) (b) 4 ( 9 3 - 4p ) and Q of an in extensible string move
downwards with uniform speed u and v
(c) 6 ( 9 2 - 4p ) (d) None of these
respectively. Pulley A and B are fixed. Find the
64. In given figure ABCDE is any pentagon. BP velocity of mass M at the instant shown in the
drawn parallel to AC meets DC produced at P figure is
and EQ drawn parallel to AD meets CD produced
at Q. Then
(a) Area (ABCDE) = Area (DAPQ)
(b) Area (ABCDE) = Area (DACQ)
(c) Area (ABCDE) = 2 Area (DADQ)
(d) None of these
A

(u + v ) cos q
(a)
E 2
B
2(u + v )
(b)
cos q
(c) (u + v) / 2 cos q
P C D Q (d) u / cosq
EBD_7839
64 KVPY-SA

68. A rod PQ of length l is pivoted at an end P and time interval. Which of the following graphs
freely rotated in a horizontal plane at an angular correctly describes (x1 – x2) as a function of
speed w about a vertical axis passing through time ‘t’?
P. If coefficient of linear expansion of material of (x1 – x2) (x1 – x2)
rod is a, find the percentage change in its
angular velocity if temperature of system is
increased by DT is (a) t (b) t
O O
(a) (a D T ´ 100)%
(x1 – x2) (x1 – x2)
(b) (2a D T ´100)%

(c) (3a D T ´ 100) ´100%


(c) O
t (d) t
(d) (4a D T ´100)% O

69. The angle of incidence and refraction of a


monochromatic ray of light of wavelength l at CHEMISTRY
an air-glass interface are i and r, respectively. A
71. When 3, 3-dimethyl-2-butanol is heated with
parallel beam of light with a small spread dl in
H2SO4 the major product obtained is
wavelength about a mean wavelength l is
refracted at the same air-glass interface. The (a) cis and trans isomers of 2, 3-dimethyl-2-
refractive index µ of glass depends on the butene
(b) 3, 3-dimethyl-1-butene
b
wavelength l as m(l) = a + , where a and b (c) 2, 3-dimethyl-2-butene
l2 (d) 2, 3-dimethyl-1-butene
are constants. Then the angular spread in the 72. Among the following carbocations
angle of refraction of the beam is :
+
æ sin i ö (I) Ph2 CCH 2 Me
(a) ç 3 ÷ dl
è l cos r ø +
(II) PhCH 2 CH 2 CHPh
æ 2b ö +
(b) ç 3 ÷ dl (III) Ph2CH CHMe
èl ø
+
(IV) Ph2C(Me) CH 2
æ 2b tan r ö
(c) ç 3 ÷ dl The order of stability is
è al + bl ø
(a) IV > II > I > III (b) I > II > III > IV
æ (2b ( a + b / l 2 ) sin i ö (c) II > I > IV > III (d) I> IV > III > II
(d) ç ÷ dl
ç l3 ÷ 73. Four diatomic species are listed below in different
è ø
sequences. Which of these presents the correct
70. A body is at rest at x = 0. At t = 0, it starts order of their increasing bond order ?
moving in the positive x-direction with a constant (a) O-2 < NO < C22- < He+2
acceleration. At the same instant another body
passes through x = 0 moving in the positive x- (b) NO < C22 - < O 2- < He 2+
direction with a constant speed. The position of (c) C 22 - < He +2 < NO < O -2
the first body is given by x1(t) after time ‘t’; and
that of the second body by x2(t) after the same (d) He+2 < O 2- < NO < C 22 -
MOCK TEST-7 65

74. For the following three reactions (i), (ii) and (iii), 78. The steps in catalytic cycle of an enzyme action
equilibrium constants are given: are given in random order.
(i) CO(g) + H 2O(g) ƒ CO2 (g) + H 2 (g); K1 (i) The enzyme releases the products. Now
(ii) CH 4 (g) + H 2O(g) ƒ CO(g) + 3H 2 (g); K2 enzyme is free to bind another substrate.
(iii) CH 4 (g) + 2H 2O(g) ƒ CO2 (g) + 4H 2 (g); K3 (ii) The active sites, now in close proximity of
substrate breaks the chemical bonds of
(a) K1 K 2 = K3 (b) K 2 K 3 = K1 substrate and forms E-P complex.
(iii) Binding of substrate induces the enzyme
(c) K3 = K1 K2 (d) K3 .K23 = K12
to alter its shape fitting more tightly around
75. The wave number of the spectral line in the
the substrate.
emission spectrum of hydrogen will be equal to
(iv) The substrate binds to the active site of
8
times the Rydberg’s constant if the electron enzyme (i.e., fitting into the active site).
9
(a) (i), (ii), (iii), (iv)
jumps from ..........
(a) n = 3 to n = 1 (b) n = 10 to n = 1 (b) (iv), (iii), (ii), (i)
(c) n = 9 to n = 1 (d) n = 2 to n = 2 (c) (i), (iii), (ii), (iv)
(d) (i), (ii), (iv), (iii)
BIOLOGY
79. Which one of th e following groups of
76. At rest, most people require an alveolar structures/organs have similar function?
ventilation for about 4 lt./minute. Suppose an (a) Typhlosole in earthworm, intestinal villi in
emphysema victim with a tidal volume of 0.5 lt. rat and contractile vacuole in Amoeba.
has suffered alveolar damage that has produced (b) Nephridia in earthworm, Malpighian
a respiratory dead space of 0.3 lt. What will be tubules in cockroach and urinary tubules
that person’s frequency of breathing at rest? in rat.
(a) 5 breaths/min (b) 8 breaths/min (c) Antennae of cockroach, tympanum of frog
(c) 20 breaths/min (d) 50 breaths/min and clitellum of earthworm.
77. In a plant, genes A, B and C control the pod (d) Incisors of rat, gizzard (proventriculus) of
colour, height of plant and pod shape, cockroach and tube feet of starfish.
respectively. Study the following genotypes and 80. Match column-I with column-II and select the
corresponding phenotypes: correct answer using the codes given below.
AA ® Green pod Column-I Column-II
aa ® Yellow pod A. Triglyceride I. Animal hormones
Aa ® Purple pod B. Membrane lipid II. Coating on leaves
B_ ® Tall plant and stems
bb ® Dwarf plant C. Steroid III. Phospholipids
C_ ®Full pod shape D. Wax IV. Fat stored in the
cc ® Constricted pod shape form of droplets
If AabbCC and aaBbCc are crossed, what
(a) A – IV; B – III; C – I; D – II
proportion of plants would be tall, have purple
and full pod in the progeny? (b) A – II; B – III; C – IV; D – I
(a) 25% (b) 50% (c) A – III; B – IV; C – I; D – II
(c) 75% (d) 100% (d) A – IV; B – I; C – II; D – III
EBD_7839
66 KVPY-SA

ANS W ER KEYS
Part-I Part-II
1 (b ) 11 (c) 21 (a) 31 (a) 41 (d ) 51 (a) 61 (d ) 71 (c)
2 (b ) 12 (b ) 22 (d) 32 (a) 42 (d ) 52 (c) 62 (b ) 72 (b )
3 (d ) 13 (c) 23 (d) 33 (d) 43 (a) 53 (a) 63 (a) 73 (d)
4 (d ) 14 (b ) 24 (d) 34 (b) 44 (d ) 54 (b ) 64 (a) 74 (c)
5 (c) 15 (b ) 25 (c) 35 (a) 45 (c) 55 (a) 65 (d ) 75 (a)
6 (a) 16 (b ) 26 (c) 36 (c) 46 (a) 56 (d ) 66 (a) 76 (c)
7 (d ) 17 (a) 27 (d) 37 (a) 47 (d ) 57 (c) 67 (c) 77 (a)
8 (b ) 18 (b ) 28 (c) 38 (c) 48 (a) 58 (c) 68 (b ) 78 (b)
9 (d ) 19 (b ) 29 (c) 39 (b) 49 (a) 59 (a) 69 (c) 79 (b)
10 (b ) 20 (a) 30 (c) 40 (c) 50 (d ) 60 (c) 70 (b ) 80 (a)
Telegram @unacademyplusdiscounts

Join Us on Telegram for More Such Books

https://telegram.me/unacademyplusdiscounts

Join us from the above link or search ''unacademyplusdiscounts'' in Telegram


81
Time : 3 Hours Stream-SA Maximum Marks : 100
INSTRUCTIONS
1. There are 80 questions in this paper.
2. The question paper contains two parts; Part–I (1 Mark Questions) and Part–II (2 Marks
Questions). There are four sections; Mathematics, Physics, Chemistry and Biology in each part.
3. There are four options given with each question, only one of them is correct.
4. For each incorrect answer 0.25 Mark in Part–I and 0.5 Mark in Part–II will be deducted.

PART-I (1 MARK QUESTIONS)

MATHEMATICS (a) 2r = x + t
(b) 2t = r + s
p æ Pö
1. In a DPQR , ÐR = . If tan çè ÷ø and (c) 2s = r + t
2 2
(d) r + s + t = 0
æ Qö 5. Let a, b, c be distinct digits. Consider a two
tan ç ÷ are roots of the equation
è 2ø digit number ‘ab’ and three digits ‘ccb’ both
defined under the usual decimal number
ax 2 + bx + c = 0 system. If (ab)² = ccb and ccb > 300, then the
(a ¹ 0), then value of b is:
(a) a + b = c (b) b + c = 0 (a) 1 (b) 0
(c) a + c = b (d) b = c (c) 5 (d) 6
2. If N is a two digit number such that last two 6. A point E is taken on the side BC of a
digit of N3 is 44 then what is the summation of parallelogram ABCD. AE and DC are produced
all the possible values of N? to meet at F. Then
(a) 78 (b) 80 (a) Area (DABCD) = Area (DACF)
(c) 72 (d) 86 (b) Area (DADF) = Area (ABFC)
3. How many values of natural number N exist such (c) Area (DABF) = Area (DACF)
that both N and N + 3293 are perfect squares? (d) None of these
(a) 0 (b) 1
(c) 9 (d) None of these A B
4. If (k, k) is the set of solution of system of equations
ax + by + (t – s) = 0 an d bx + ay
E
+ (s – r) = 0, a ¹ b. Then which of the following
must be true.
D C F
EBD_7839
68 KVPY-SA

7. In the figure, O is the centre of the circle BCD. AB = 4 units and CD = 2 units, what is the area,
ABC and EDC are straight lines, BC= DC and in sq. units of the largest circle?
ÐAED = 70°, Find ÐBOD (a) 128p (b) 64À
(c) 48À (d) 16p
11. If cost price of 40 pens is more than cost of 15
copies but less than cost of 16 copies. What
can be the minimum total cost of a pen and a
copy if price of both pen and copy is an integer
(a) 16 (b) 18
(a) 40° (b) 70° (c) 20 (d) 21
(c) 80° (d) 90°
8. In the figure shown, two identical circles of 12. A hollow cube is inscribed in a hollow sphere,
radii r are drawn. If the centre of the circles lie such that all the corners of cube touches
on each other circumference, what is the area sphere. Then the ratio of air between sphere
of shaded region? and cube to the air in cube is

3p - 2 2
(a) (b)
2 3p - 2

3p - 8 8
(c) (d)
8 3p - 8

13. Let S = {x Î R : x ³ 0 and


2 2 3 2 3 2
(a) pr - r (b) 2pr 2 - r 2 | x - 3 | + x ( x - 6) + 6 = 0 . Then S :
3 2 2
(a) contains exactly one element.
1 2 2 (b) contains exactly two elements.
pr
(c) pr (d)
3 2 (c) contains exactly four elements.
(d) is an empty set.
9. If ABC is a triangle right angled at B and M, N
14. How many two digit numbers exist such that the
are the mid-points of AB and BC, then
product of two digit number and two digit number
4(AN2 + CM2) is equal to : formed by reversing its digits end with 5.
(a) 4 AC2 (b) 5 AC2 (a) 2 (b) 3
5 (c) 4 (d) 5
(c) AC2 (d) 6AC2 15. If two roots of the equation
4
10. AD is a diameter of a circle. Two more circles ( p - 1)( x 2 + x + 1)2 - ( p + 1)( x 4 + x 2 + 1) = 0
1- x
pass through A and intersect AD in B and C are real and distinct and f ( x ) = , then
1+ x
respectively, such that AB and AC are diameters æ æ 1 öö
f ( f ( x)) + f ç f ç ÷ ÷ is equal to
of these circles and AD > AC > AB. If the è è x øø
(a) p (b) –p
circumference of the middle circle is average of
(c) 2p (d) –2p
the circumference of the other two, then given
MOCK TEST-8 69

PHYSICS
2 3 Gm 3 Gm
16. One end of uniform wire of length L and of weight (a) (b)
a a
W is attached rigidly to a point in the roof and a
weight W1 is suspended from its lower end. If s
6 3 Gm 3 3 Gm
is the area of cross section of the wire, the stress (c) (d)
æ 3L ö a a
in the wire at a height ç ÷ from its lower end is 19. Consider a Vernier callipers in which each 1 cm
è 4ø
on the main scale is divided into 8 equal divisions
W1 é Wù
(a)
s
(b) êëW1 + 4 úû s and a screw gauge with 100 divisions on its
circular scale. In the Vernier callipers, 5 divisions
é 3W ù W1 + W of the Vernier scale coincide with 4 divisions on
(c)êëW1 + 4 úû / s (d)
s the main scale and in the screw gauge, one
complete rotation of the circular scale moves it
17. Two plates identical in size, one of black and
by two divisions on the linear scale.
rough surface (B1) and the other smooth and
Select the correct statement :
polished (A2 ) are interconnected by a thin
I. If the pitch of the screw gauge is twice the
horizontal pipe with a mercury pellet at the centre.
least count of the Vernier callipers, the least
Two more plates A1 (identical to A2) and B2
count of the screw gauge is 0.01 mm
(identical to B1 ) are heated to the same
II. If the pitch of the screw gauge is twice the
temperature and placed closed to the plates B1,
least count of the Vernier callipers, the least
and A2 as shown in the fig. The mercury pellet
count of the screw gauge is 0.005 mm
III. If the least count of the linear scale of the
screw gauge is twice the least count of the
Vernier callipers, the least count of the screw
P
gauge is 0.01 mm
IV. If the least count of the linear scale of the
screw gauge is twice the least count of the
Vernier callipers, the least count of the screw
A1 B1 A2 B2
gauge is 0.005 mm
(a) I only
(a) moves to the right
(b) II and III
(b) moves to the left
(c) III and IV
(c) remains stationary (d) I and IV
(d) starts oscillating left and right 20. In an ideal transformer the number of turns of
18. Three equal masses (each m) are placed at the primary and secondary coil is given as 100 and
corners of an equilateral triangle of side ‘a’. Then 300 respectively. If the power input is 60 W, the
the escape velocity of an object from the power output is
circumcentre P of triangle is : (a) 100 W (b) 300 W
(c) 180 W (d) 60 W
21. A cylindrical solid of length L and radius a is
having varying resistivity given by r = r0x,
where r0 is a positive constant and x is
measured from left end of solid. The cell shown
in the figure is having emf V and negligible
internal resistance. The magnitude of electric field
as a function of x is best described by
EBD_7839
70 KVPY-SA
r = r 0x 24. An electron moves along the line PQ as shown
which lies in the same plane as a circular loop of
x
conducting wire as shown in figure. What will be
the direction of the induced current initially in the
V loop when electron comes closer to loop?
2V 2V
(a) x (b) x loop
L 2 r0 L2

V P Q
(c) x (d) None of these
L2 (a) Anticlockwise
22. Consider a moving charged particle in a region (b) Clockwise
of magnetic field. Which of the following (c) Direction can not be predicted
statements are correct ? (d) No current will be induced
(a) If v is parallel to B, then path of particle 25. Choose the incorrect statement:
is spiral.
(a) Torque depends on transverse component
(b) If v is perpendicular to B, then path of
of force.
particle is a circle.
(b) Angular momentum depends on transverse
(c) If v has a component along B, then path
of particle is zig-zag. component of linear momentum.
(d) If v is along B, then path of particle is a (c) Radial component of linear momentum does
circle. not contribute to the angular momentum.
23. Two masses m1 and m2 are connected by a (d) None of these
massless spring of spring constant k and 26. A set of n identical cubical blocks lies at rest
unstretched length l. The masses are placed on parallel to each other along a line on a smooth
a frictionless straight channel, which are horizontal surface. The separation between the
consider our x-axis. They are initially at x = 0 and near surfaces of any two adjacent blocks is L.
x = l respectively. At t = 0, a velocity v0 is The block of one end is given a speed v towards
suddenly imparted to the first particle. At a later the next one at time t = 0. All collisions are
completely inelastic, then ;
time t, the centre of mass of the two masses is at:
(n - 1) L
(a) the last block starts moving at t =
v
(b) the last block starts moving at
n ( n + 1) L
t=
2v
(c) the centre of mass of the system will have a
m2l final speed v
(a) x =
m1 + m2 (d) the centre of mass of the system will have a

m1l mvt v
(b) x = + 2 0 final speed
m1 + m2 m1 + m2 n
27. The figure shows a spherical hollow inside a
m2l m vt lead sphere of radius R, the surface of the hollow
(c) x = + 2 0
m1 + m1 m1 + m2 passes through the centre of the sphere and
"touches" the right side of the sphere. The mass
m2l m1v0t of the sphere before hollowing was M. With
(d) x = m + m + m + m
1 2 1 2 what gravitational force does the hollowed out
MOCK TEST-8 71

lead sphere attract a small sphere of mass m that If equal amount of heat is supplied to the two
lies at a distance 'd' from the centre of the lead spheres, then
sphere, on the straight line connecting the (a) temperature of A will be greater than B
centres of the sphere and of the hollow : (b) temperature of B will be greater than A
(c) their temperature will be equal
(d) can’t be predicted
30. An aeroplane flying at a constant speed releases
m a bomb. As the bomb moves away from the
R aeroplane, it will
d (a) always be vertically below the aeroplane
on ly i f th e aer opl an e was flyi n g
GMm é 1 ù horizontally
(a) 2 ê
1- 2ú (b) always be vertically below the aeroplane
d ê æ R ö ú
8 ç1 - ÷ only if the aeroplane was flying at an
êë è 2 d ø úû angle of 45° to the horizontal
(c) always be vertically below the aeroplane
GMm é 1 ù (d) gradually fall behind the aeroplane if the
(b) 2 ê
1+ 2ú
d ê æ R ö ú aeroplane was flying horizontally.
4 ç1 - ÷
êë è 2 d ø úû CHEMISTRY
31. The compound which contains both ionic and
GMm é 1 ù
(c) 2 ê
1- 2ú
covalent bond is:
d ê æ R ö ú (a) KCl (b) KCN (c) CH4 (d) H2
4 ç1 - ÷
êë è 2d ø úû 32. Geometrical isomerism is possible in :
(a) CH3CH(CH3 )CH 2CH 2CH3
GMm é 1 ù
(d) 2 ê
1+ 2ú (b) CH3CH=CHCH3
d ê æ R ö ú
8 ç1 - ÷
êë è 2 d ø úû (c) CH3CH=CH 2
(d) ClH2C - CH2Cl
28. In the given pressure-temperature diagram, for
water, which point indicates triple point? 33. The number of hydrogen atoms present in 25.6 g
D B of sucrose (C12H22O11) which has a molar mass
(a) A water of 342.3 g is:
(b) C steam (a) 22 × 1023 (b) 9.91 × 1023
Pressure (P)

ice C A
P (c) 11 × 1023 (d) 44 × 1023
(c) P
F E 34. For the reaction
(d) E ˆˆ† 2NH (g), DH = ?
Temp. (T) N 2 (g) + 3H 2 (g) ‡ˆˆ 3
29. Consider two identical iron spheres, (a) DE + 2RT (b) DE –2RT
one of which lies on a (c) DE – RT (d) None of these
thermally insulating plate, 35. The number of moles of KMnO4 needed to react
while the other hangs from completely with one mole of ferrous oxalate in
thermally insulating thread. acidic solution are :
The spheres are arranged so that their centres
3 2 4
of mass are at the same height level (see figure). (a) (b) (c) (d) 1
5 5 5
EBD_7839
72 KVPY-SA

36. Which of the following has the maximum number 43. At 550K, the Kc for the following reaction is 104
of unpaired electrons? mol–1 L
(a) Mn (b) Ti (c) V (d) Al ˆˆ† Z (g) . At equilibrium, it was
X (g) + Y (g) ‡ˆˆ
37. Consider the following reaction :
1 1
observed that [X] = [Y] = [Z] . What is
Water
H2SO4 + NH3 ¾¾¾® +
NH4 + HSO4– 2 2
value of [Z] (in mol L–1) at equilibrium?
Which of the following is correct for above (a) 2 × 10–4 (b) 10–4
equation? (c) 2 × 10 4 (d) 104
(a) HSO–4 is a weak conjugate acid of H2SO4. 44. Molecular weight of a gas that diffuses twice as
(b) NH3 is a weak base. rapidly as the gas with molecular weight 64 is:
+
(c) NH4 is a weak conjugate acid of NH3. (a) 16 (b) 8 (c) 64 (d) 6.4
(d) H2SO4 is a weak acid. 45. Which of the following oxides is not expected to
38. In which of the following reactions, the react with sodium hydroxide?
underlined substance has been reduced? (a) CaO (b) SiO2
(a) CO + CuO ¾® CO2 + Cu (c) BeO (d) B2O3
(b) CuO + 2HCl ¾® CuCl2 + H2O BIOLOGY
(c) 4H2O + 3Fe ¾® 4H2 + Fe3O4 46. Maltose consists of which one of the following?
(d) C + 4HNO3 ¾® CO2 + 2H2O + 4NO2 (a) b- glucose and b-galactose
(b) a- glucose and a-fructose
39. Which step is chain propagation step in the fol-
(c) a-sucrose and b-glucose
lowing mechanism?
(d) Glucose and glucose
hv
(a) Cl 2 ¾¾ ® Clg + Clg 47. Which one of the following shortens when a
(b) Clg + CH 4 ¾¾ ® g CH3 + HCl muscle fibre contracts?
(c) Clg + Clg ¾¾ ® Cl 2 (a) Thick filament (b) Sarcomere
g g (c) Troponin (d) Thin filament
(d) CH3 + Cl ¾¾ ® CH3 Cl
40. Aluminium is usually found in +3 oxidation 48. Bacteria having single flagellum on both sides:
(a) Amphitrichous (b) Cephalotrichous
state. In contrast, thallium exists in +1 and + 3
(c) Peritrichous (d) Lophotrichous
oxidation states. This is due to:
49. Meals which are rich in fat are not digested in
(a) Inert pair effect the intestine in absence of:
(b) Diagonal relationship (a) Pepsin (b) Enterokinase
(c) Lattice effect (c) Insulin (d) Steapsin
(d) Lanthanoid contraction 50. Sinks are related to:
41. Which of the following represents the correct (a) Transport of minerals
order of increasing electron gain enthalpy with (b) Stomata
negative sign for the elements O, S, F and Cl ? (c) Enzymes
(a) Cl < F < O < S (b) O < S < F < Cl (d) Phytochrome
51. Pectin is:
(c) F < S < O < Cl (d) S < O < Cl < F (a) Monosaccharides
42. When a metal is added to dilute HCl solution, (b) Homopolysaccharides
there is no evolution of gas. Metal is – (c) Mucopolysaccharides
(d) Heteropolysaccharides
(a) K (b) Na (c) Ag (d) Zn
MOCK TEST-8 73

52. The most common indicator organism of 56. Which of the following is not a steroid hormone?
polluted water is: (a) Androgen (b) Aldosterone
(a) E. coli (b) P. typhi (c) Estrogen (d) Relaxin
(c) Vibrio (d) Entamoeba 57. Which of the following hormones can replace
53. Pentoses and hexoses are the most common: vernalisation?
(a) Disaccharides (a) Ethylene (b) Gibberellins
(b) Monosaccharides (c) Cytokinins (d) Auxins
58. Which of the following conversions takes place
(c) Oilgosaccharides
in the blood clotting pathways?
(d) Polysaccharides
(a) Conversion of vitamin K to prothrombin.
54. Which of the following elements are involved in
(b) Conversion of fibrin to fibrinogen.
muscle contraction?
(c) Conversion of thrombin to prothrombin.
(a) Ca2+ and Mg2+ (b) Ca2+ and Na+
(d) None of the above
(c) Na+ and K+ (d) Mg2+ and K+ 59. Contraction of gall bladder is induced by:
55. In dicotyledonous stem, fascicular cambium is a (a) Gastrin (b) Cholecystokinin
meristematic tissue. It is an example of which of (c) Secretin (d) Enterogastrone
the following meristem? 60. Smallest cell of human body is:
(a) Lateral (b) Secondary (a) Erythrocyte (b) Monocyte
(c) Apical (d) Intercalary (c) Neuron (d) Blood platelets

PART-II (2 MARKS QUESTIONS)

MATHEMATICS 64. Through the vertex A of a parallelogram ABCD,


61. If a8 and 8a is completely divisible by 50! Then line AEF is drawn to meet BC at E and DC
which one of the following is true about
produced at F. Then which of the following is
‘highest value of a’?
(a) 10 < a < 14 (b) 14 < a < 16 true.
(c) 16 < a < 18 (d) 18 < a < 20
(a) The triangles BEF and DCE are equal in
p p
62. Let - < q < – . Suppose a1 and b1 are the area
6 12
1
roots of the equation x2 – 2x sec a + 1 = 0 and (b) Area (DBEF) = Area (DABD)
2
a2 and b2 are the roots of the equation x2 + 2x
tan q – 1 = 0. If a1 > b1 and a2 > b2, then a1 + b2 1
(c) Area (DDCE) = Area (DABF)
equals 2
(a) 2 (sec q – tan q) (b) 2 sec q (d) Area (DBEF) = 2 Area (DDCE)
(c) –2 tan q (d) 0
65. For a positive integer n, let pn denote the
63. The sum of the area of two circles, which touch
product of the digits of n, and sn denote the
each other externally is 153p. If the sum of their
sum of the digits of n. the number of integers
radii is 15, then ratio of the areas of smaller to
between 10 and 1000 for which pn + sn = n is
the larger circle is
(a) 1 : 2 (b) 1 : 4 (a) 81 (b) 16

(c) 1 : 6 (d) 1 : 5 (c) 18 (d) 9


EBD_7839
74 KVPY-SA

PHYSICS 70. Two identical uniform rectangular blocks (with


longest side l) and a solid sphere of radius R are
66. A vertical cylinder contains a movable frictionless
to be balanced at the edge of a heavy table such
piston separating the former into two portions.
that the centre of the sphere remains at the
The two portions are filled with ideal gases, and
maximum possible horizontal distance from the
at equilibrium, the ratio of the volumes of upper
vertical edge of the table without toppling as
to the lower portions is 5 : 3. When the
indicated in the figure. If the mass of each block
temperature of the system is doubled, the ratio
M
becomes 3 : 2. Then the ratio of number of is M and of the sphere is , the maximum
2
molecules in the upper to the lower portions is
distance x that can be achieved is:
(a) 35 : 26 (b) 26 : 35
(c) 5 : 6 (d) 2 : 3
67. Steel ruptures when a shear of 3.5 × 108 N m–2 is
applied. The force needed to punch a 1 cm
diameter hole in a steel sheet 0.3 cm thick is
nearly:
(a) 1.4 × 104 N (b) 2.7 × 104 N
(c) 3.3 × 104 N (d) 1.1 × 104 N 8l 5l
(a) (b)
68. A uniformly tapering conical wire is made from a 15 6
material of Young's modulus Y and has a normal, 3l 7l
unextended length L. The radii, at the upper and (c) +R (d) +R
4 15
lower ends of this conical wire, have values R
and 3R, respectively. The upper end of the wire
is fixed to a rigid support and a mass M is CHEMISTRY
suspended from its lower end. The equilibrium 71. Find the product of the given reaction
extended length, of this wire, would equal :
CH3
æ 2 Mg ö æ 1 Mg ö CH3 H+
L ç1 + L ç1 + ¾
¾¾®
(a) è 9 pYR 2 ÷ø (b) è 9 pYR 2 ÷ø OH
D

æ 1 Mg ö æ 2 Mg ö CH3
L ç1 + L ç1 +
(c) è 3 pYR 2 ÷ø (d) è 3 pYR 2 ÷ø (a) CH3

69. Two 20 kg cannon balls are chained together


and fired horizontally with a velocity of 200 m/s
from the top of a 30 m wall. The chain breaks (b)
during the flight of the cannon balls and one of
them strikes the ground at t = 1.5s, at a distance CH3
of 250 m from the foot of the wall, and 5m to the
(c)
right of the line of fire. Determine the position of CH3
the other cannon ball at that instant. Neglect the
resistance of air. H3C
(a) (20,550) (b) (550,20)
(d)
(c) (2,55) (d) (55,2) H3C
MOCK TEST-8 75

72. 1 c.c. N2O at NTP contains : 77. Sex determination in grasshoppers, humans, and
Drosophila is similar because
1.8
(a) ´ 10 22 atoms (a) females are hemizygous.
224
(b) males have one X chromosome and females
6.02
(b) ´ 10 23 molecules have two X chromosomes.
22400
(c) all males always have one Y chromosome
in all three species.
(c) 1.32 ´ 10 23 electrons
224 (d) the ratio of autosomes to sex chromosomes
(d) All of the above is the same in all three organisms.
73. 3-Methyl-pent-2-ene on reaction with HBr in 78. Which of the following pair is not correctly
presence of peroxide forms an addition product. matched?
The number of possible stereoisomers for the (a) Rods – Twilight vision
product is : (b) Ciliary body – Iris
(a) Six (b) Zero (c) Retina – Optic chiasma
(c) Two (d) Four (d) Vitreous humour – Posterior compartment
74. Which of the following compounds of elements 79. Match the genetic phenomena given in column-I
in group IV would you expect to be most ionic in with their respective ratios given in column-II
character? and select the correct option.
(a) CCl4 (b) SiCl4
Column-I Column-II
(c) PbCl 2 (d) PbCl 4
(Genetic phenomena) (Ratios)
75. The boiling point of p-nitrophenol is higher than A. Inhibitory gene ratio I. 9:3:4
that of o-nitrophenol because
B. Complementary gene II. 1:1:1:1
(a) NO2 group at p-position behaves in a
ratio
different way from that at o-position.
(b) intramolecular hydrogen bonding exists in C. Recessive epistasis III. 12 : 3 : 1
p-nitrophenol. ratio
(c) there is intermolecular hydrogen bonding D. Dihybrid test cross IV. 13 : 3
in p-nitrophenol. ratio
(d) p-nitrophenol has a higher molecular weight E. Dominant epistasis V. 9: 7
than o-nitrophenol. ratio
BIOLOGY (a) A – V; B – IV; C – III; D – II; E – I
76. Which of the following are not used in the (b) A – IV; B – V; C – I; D – II; E – III
conversion by pyruvate to acetyl CoA? (c) A – I; B – II; C – IV; D – III; E – V
(i) Oxidative dehydrogenation (d) A – II; B – I; C – IV; D – V; E – III
(ii) Oxidative dehydration 80. The concentration of OH– ions in a solution
(iii) Oxidative phosphorylation with the H+ ions concentration of 1.3 ´ 10–4 M
(iv) Oxidative decarboxylation is:
(a) (i), (ii) and (iii) (b) (i) and (ii) (a) 7.7 ´ 10–4 M (b) 1.3 ´ 10–4 M
(c) (ii) and (iv) (d) (i) and (iii) (c) 2.6 ´ 10–8 M (d) 7.7 ´ 10–11 M
EBD_7839
76 KVPY-SA

ANS W ER KEYS
Part-I Part-II
1 (a) 11 (b ) 21 (a) 31 (b ) 41 (b ) 51 (d ) 61 (b ) 71 (d)
2 (a) 12 (a) 22 (b) 32 (b) 42 (c) 52 (a) 62 (c) 72 (d)
3 (d ) 13 (b ) 23 (d) 33 (b) 43 (a) 53 (b ) 63 (b ) 73 (d)
4 (c) 14 (b ) 24 (a) 34 (b) 44 (a) 54 (a) 64 (a) 74 (c)
5 (a) 15 (a) 25 (a) 35 (a) 45 (a) 55 (a) 65 (d ) 75 (c)
6 (b ) 16 (c) 26 (d) 36 (a) 46 (d ) 56 (d ) 66 (a) 76 (a)
7 (c) 17 (c) 27 (a) 37 (c) 47 (b ) 57 (b ) 67 (c) 77 (b)
8 (a) 18 (c) 28 (c) 38 (c) 48 (a) 58 (d ) 68 (c) 78 (c)
9 (b ) 19 (b ) 29 (b) 39 (b) 49 (d ) 59 (b ) 69 (b ) 79 (b)
10 (d ) 20 (d ) 30 (c) 40 (a) 50 (a) 60 (a) 70 (a) 80 (d)
Telegram @unacademyplusdiscounts

Join Us on Telegram for More Such Books

https://telegram.me/unacademyplusdiscounts

Join us from the above link or search ''unacademyplusdiscounts'' in Telegram


91
Time : 3 Hours Stream-SA Maximum Marks : 100
INSTRUCTIONS
1. There are 80 questions in this paper.
2. The question paper contains two parts; Part–I (1 Mark Questions) and Part–II (2 Marks
Questions). There are four sections; Mathematics, Physics, Chemistry and Biology in each part.
3. There are four options given with each question, only one of them is correct.
4. For each incorrect answer 0.25 Mark in Part–I and 0.5 Mark in Part–II will be deducted.

PART-I (1 MARK QUESTIONS)

MATHEMATICS (a) 122 (b) 47


(c) 99 (d) None of these
1. If p(x) = x3 – 3x2 + 2x + 5 and p(a) = p(b) = p(c) = 0,
6. A triangle has side lengths 4, 6, 8. A tangent is
then the value of (2 – a) (2 – b) (2 – c) is
(a) 5 (b) 3 (c) 9 (d) 2 drawn to the incircle parallel to side 4 cutting
2. How many 2 digit numbers exist such that sum other two sides at M and N, then the length of
of the squares of the numbers and the number MN is
formed by reversing its digits ends with 3. 10 20 5 4
(a) 6 (b) 1 (c) 12 (d) 8 (a) (b) (c) (d)
9 9 3 3
3. If p and q are distinct integers such that
2005 + p = q2 and 2005 + q = p2 then the product 7. In the given fig., OB is the perpendicular bisector
p.q is: of the line segment DE, FA ^ OB and FE
(a) 2004 (b) 2009 (c) 2005 (d) – 2005 intersects OB at the point C.
4. In figure DABC is equilateral triangle. Find D
height of DABC if the distances from the
point O to the sides are 4, 5 and 6 respectively. F

C
B
O A

E
(a) 15 (b) 14 1 1 1 1 1
(c) 11 (d) None of these (a) + = (b) + = OC2
OA OC OB OA OB
If 100! = K ( 2!) ( 5!) ( 7!) Then find the
x y 2
5. 1 1 1 1 1 2
(c) = 2+ 2 (d) + =
maxi-mum possible value of x + y + z OA OC OB OA OB OC
EBD_7839
78 KVPY-SA

8. In the diagram, the circle and the square have If the remaining 9 are drinking water from a pond,
the same centre O and equal areas. The circle then the difference between the number of deer
has radius 1 and intersects one side of the who are grazing and those who are playing is a
square at P and Q. The length of PQ is multiple of
(a) 4 (b) 6 (c) 8 (d) 9

12. Find square root of 5 + 10 + 15 + 6

æ æ 3ö æ5öö
(a) ç 1 + ç ÷ + ç ÷ ÷
ç è 2ø è 2 ø ÷ø
è

æ æ 3ö æ5öö
(a) 4 - p (b) 1 (b) ç 1 + ç ÷ - ç ÷ ÷
ç è 2ø è 2 ø ÷ø
è
(c) 2 (d) 4-p

9. In the figure, the area of square ABCD is 4 cm2 æ æ1ö æ3ö æ5öö
(c) ç ç ÷ + ç ÷ + ç ÷ ÷
and E is mid point of AB, F, G, H and K are the ç è2ø è2ø è 2 ø ÷ø
è
mid points of DE, CF, DG and CH respectively.
The area of DKDC is æ æ 3ö æ5öö
(d) ç 1 + ç ÷ + ç ÷ ÷
ç è 4ø è 2 ø ÷ø
è

13. If a number N = x. yz where x, y and z are single


digits with not more than one digit is zero. Then
which of the following will be always be an
integer?
(a) 37 N (b) 355 N (c) 396 N (d) 497 N
1 2 1 2 14. If highest power of 7 in N! is k and that in
(a) cm (b) cm (N + 3)! is K + 2 then how many values of N
4 8
exist if N < 100.
1 1 (a) 2 (b) 4 (c) 5 (d) 6
(c) cm2 (d) cm2
16 32 15. In a particular batch of Pioneer career Kolkata
10. PQ is a chord of length 8 cm of a circle of radius there are 4 boys and certain number of girls. In
5cm. The tangents at P and Q intersects at a every mock test only 5 students including at
point T. Then the length of TP is least 3 boys can appear. If different group of
20 students write the Mock exam every time, if num-
(a) cm (b) 24.5 cm ber of times test conducted is 66 then find the
3
total number of students in the class.
20
(c) cm (d) 12 cm (a) 5 (b) 6
3
(c) 9 (d) None of these
11. Half of a herd of deer are grazing in the field and
three fourth of the remaining are playing nearby.
MOCK TEST-9 79

PHYSICS
v0 wind
16. A cylindrical drum, pushed along by a board
rolls forward on the ground. There is no slip-
q
ping at any contact. Find the distance moved
by the man who is pushing the board, when axis
of the cylinder covers a distance L.

L
(a) tan–1(1.2) (b) tan –1(0.2)
–1
(c) tan (2) (d) tan –1(0.4)
20. Carbon monoxide is carried around a closed cycle
abc in which bc is an isothermal process as
(a) 2 L (b) L shown in the figure . The gas absorbs 7000 J of
(c) L / 2 (d) 4 L heat as its temperture increases from 300 K to
17. An experiment is performed to obtain the value 1000 K in going from a to b.
of acceleration due to gravity g by using a simple
pendulum of length L. In this experiment time P
for 100 oscillations is measured by using a P2 b
watch of 1 second least count and the value is
90.0 seconds. The length L is measured by using P1 a c
a meter scale of least count 1 mm and the value V
O V1 V2
is 20.0 cm. The error in the determination of g
would be: The quantity of heat rejected by the gas during
(a) 1.7% (b) 2.7% the process ca is
(c) 4.4% (d) 2.27% (a) 4200 J (b) 5000 J
18. An experiment on the take-off performance of (c) 9000 J (d) 9800 J
an aeroplane shown that the acceleration varies 21. A biconvex lens of focal length f forms a circular
as shown in the figure, and that 12 s to take-off image of sun of radius r in focal plane. Then :
from a rest position. The distance along the
runway covered by the aeroplane is

(a) 210 m (b) 2100 m (a) p r2 µ f


(c) 21000 m (d) None of these
19. A boy throws a ball upward with velocity v0 = (b) p r2 µ f 2
20 m/s making an angle q with the vertical. The (c) If lower half part is covered by black sheet,
wind imparts a horizontal acceleration of 4 m/s2
p r2
to the left. The angle at which the ball must be then area of the image is equal to
2
thrown so that the ball returns to the boy’s hand
(d) If f is doubled, intensity will increase
is (g = 10 m/s2)
EBD_7839
80 KVPY-SA

22. A mass is at the centre of a square, with four


masses at the corners as shown. v(cm)
v(cm)
5M 3M 2M 3M
(c) (d)
(A) M (B) M O u(cm)
O u(cm)
M 5M M 2M
26. A body travels uniformly a distance of (13.8 ±
5M 3M 2M 3M
0.2) m in a time (4.0 ± 0.3) s. Calculate its velocity
(C) 2M (D) 2M with error limits. What is percentage error in
M 5M M 2M velocity?
Rank the choices according to the magnitude of (a) (3.5 + 3.1) m/sec, + 10%
the gravitational force on the centre mass. (b) (3.5 + 1.3) m/sec, + 9%
(a) FA = FB < FC = FD (b) FA > FB < FC < FD
(c) (3.5 + 0.31) m/sec, + 9%
(c) FA = FB > FC = FD (d) None of these (d) (5.3 + 0.31) m/sec, + 9%
23. A ball B is suspended from a string of length l 27. For a particle executing S.H.M. the displacement
attached to a cart A, which may roll on a
x is given by x = A cos wt. Identify the graph
frictionless surface. Initially the cart is at rest
and the ball is given a horizontal velocity v0. which represents the variation of potential
Determine the velocity of B as it reaches the energy (P.E.) as a function of time t and
maximum height. displacement x.
n0 Cart A mA
(a)
m A + mB
mB n 0
(b)
m A + mB
B v0
mB n 0
(c) mB (a) I, III (b) II, IV
m A - mB
(c) II, III (d) I, IV
(d) n0 ( m A + mB ) 28. A 50 kg woman is on a large swing (generally
24. The radius of the earth is reduced by 4%. The seen in fairs) of radius 9 m that rotates in a vertical
mass of the earth remains unchanged. What will
circle 6 rev/min. What is the magnitude of her
be the change in escape velocity?
(a) Increased by 2% weight when she has moved halfway up ?
(b) Decreased by 4% (a) 521 N (b) 512 N
(c) Increased by 6% (c) 251 N (d) 215 N
(d) Decreased by 8% 29. Let the x-z plane be the boundary between two
25. A student measures the focal length of a convex transparent media. Medium 1 in z ³ 0 has a
lens by putting an object pin at a distance ‘u’
from the lens and measuring the distance ‘v’ of refractive index of 2 and medium 2 with z < 0
the image pin. The graph between ‘u’ and ‘v’
has a refractive index of 3. A ray of light
plotted by the student should look like
in medium 1 given by the vector
v(cm) v(cm) r
A = 6 3iˆ + 8 3 ˆj - 10kˆ is incident on the plane of
(a) (b) separation. The angle of refraction in medium 2 is:
O u(cm) O u(cm) (a) 45° (b) 60° (c) 75° (d) 30°
MOCK TEST-9 81
30. Determine the minimum coefficient of friction
between a thin rod and a floor at which a person OH
can slowly lift the rod from the floor without OH
slipping, to the vertical position, applying to its
end a force always perpendicular to its length. (c)

1 1
(a) (b) H
2 2 2 H H H
(c) 2 (d) 2 2
CHEMISTRY OH
H
31. Which of the following pairs of compounds
illustrate the law of multiple proportions ? (d)
(a) H2O and Na2O (b) MgO and Na2O
(c) Na2O and BaO (d) SnCl2 and SnCl4
HO
32. Which bonds are formed by a carbon atom with H H H
sp2-hybridisation?
(a) 4p-bonds 36. Which of the following behaves as both
(b) 2p-bonds and 2s-bonds oxidising and reducing agents ?
(c) 1p-bond and 3s-bonds
(a) H2SO4 (b) SO2 (c) H2O (d) HNO3
(d) 4s-bonds
37. The molecule which does not exhibit dipole
33. Solubility of MX2 type electrolyte is 0.5 × 10–4 moment is :
mol/litre. The value of Ksp of the electrolyte is (a) NH3 (b) CHCl3 (c) H2O (d) CCl4
(a) 5 × 10–13 (b) 25 × 10–10 38. Among the following four structures I to IV,
(c) 1.25 × 10 –13 (d) 5 × 1012
34. Which of the following sub-shells will be filled CH3
by the electron after complete filling up of the
C2H5 –– CH –– C3H7,
orbitals of the third principal shell? (I)
(a) 4 s (b) 4 f (c) 4 d (d) 4 p
35. Which of the following conformers for ethylene O CH3
glycol is most stable?
CH3 –– C –– CH –– C2H5,
OH (II)
H OH H CH3
+
(a) H –– C , C2H5 –– CH –– C2H5

H
H H (III) (IV)
H It is true that,
(a) only I and II are chiral compounds.
OH
(b) only III is a chiral compound.
H H (c) only II and IV are chiral compounds.
(d) all four are chiral compounds.
(b) 39. In the reaction, CO32– + H2O ® HCO3– + OH–
water is a
H H (a) Bronsted acid (b) Bronsted base
OH (c) Conjugate acid (d) Conjugate base
EBD_7839
82 KVPY-SA

40. The correct order of increasing thermal stability 47. Match column-I with column-II and select the
of K2CO3, MgCO3, CaCO3 and BeCO3 is correct option from the codes given below.
(a) BeCO3< MgCO3 < CaCO3 < K2CO3 Column-I Column-II
(b) MgCO3 < BeCO3 < CaCO3 < K2CO3 A. Stele I. Innermost
(c) K2CO3 < MgCO3 < CaCO3 < BeCO3 layer of cortex
(d) BeCO3 < MgCO3 < K2CO3 < CaCO3 B. Endodermis II. Suberin
41. The reactivities of iron, magnesium, sodium and C. Casparian strip III. All the tissues
zinc towards water are in the following order – exterior to
(a) Fe > Mg > Na > Zn vascular
cambium
(b) Zn > Na > Mg > Fe
D. Bark IV. All the tissues
(c) Na > Mg > Zn > Fe inner to
(d) Mg > Na > Fe > Zn endodermis
42. The IUPAC name of the compound (a) A – IV; B – I; C – II; D – III
CH3CH = CHC º CH is (b) A – III; B – II; C – I; D – IV
(a) Pent-l-yn-3-ene (b) Pent-4-yn-2-ene (c) A – I; B – II; C – III; D – IV
(c) Pent-3-en-1-yne (d) Pent-2-en-4-yne (d) A – IV; B – II; C – I; D – III
43. The two functional groups present in a typical 48. Which of the following will not make minerals
carbohydrate are: more available to plants?
(a) – CHO and – COOH (a) Increasing the rainfall in a wet forested area.
(b) and – OH (b) Raising the pH of a very acidic soil.
(c) – OH and – CHO (c) Tillering a packed-down or water-logged
(d) – OH and – COOH soil.
44. A sample of Na2CO3.H2O weighing 0.62 g is (d) Introducing fungi that can form mycorrhiza
added to 100 mL of 0.1 N H2SO4 solution. The into a soil that lacks them.
resulting solution would be: 49. Which of the following is an amino acid, that is
(a) Acidic (b) Neutral metabolised by the brain?
(c) Alkaline (d) None of the above (a) Alanine (b) Histidine
45. Which one of the following conformations of (c) Glycine (d) Glutamic acid
cyclohexane is chiral?
50. Which of the following hormone is a derivative
(a) Boat (b) Twist boat of amino acid?
(c) Rigid (d) Chair (a) Oestrogen (b) Epinephrine
BIOLOGY (c) Progesterone (d) Prostaglandin
51. Under which of the following circumstances will
46. Sugars are not as good as fats as a source of insulin be secreted?
energy for cellular respiration, because sugars (a) The blood sugar level in the liver is low.
(a) produce toxic amino groups when broken (b) The blood sugar level in the hepatic portal
down. vein is low.
(b) contain more hydrogen. (c) The blood sugar level in the islets of
(c) usually bypass glycolysis and the Krebs Langerhans is high.
cycle. (d) The glycogen level in the skeletal muscle is
(d) contain fewer hydrogen atoms and electrons. high.
MOCK TEST-9 83

52. A student brought home a strange animal which (c) (ii) and (iii) (d) (i), (ii) and (iii)
he found outside under a rock. It had moist skin, 56. The cell in the human body invaded by the
a complete digestive tract, a ventral nerve cord, human immuno-deficiency virus (HIV) is:
and had gone through torsion. Identify the (a) T-helper cell (b) Erythrocyte
phylum of the animal. (c) B-cell (d) Macrophage
(a) Porifera (b) Annelida 57. Protein is used as respiratory substrate only
(c) Mollusca (d) Echinodermata when:
53. What is the most efficient region of water (a) Carbohydrates are absent
absorption in roots? (b) Fats are absent
(a) Root cap (c) Both carbohydrates and fats are exhausted
(b) Growing point (d) Fats and carbohydrates are abundant
(c) Zone of elongation 58. Disintegration of the platelets gives rise to which
(d) Zone of differentiation factor, required for normal blood coagulation?
54. ELISA is used to detect viruses, where (a) Thromboplastin (b) Hageman factor
(a) DNA probes are required. (c) Fibrinogen (d) None of these
(b) Southern blotting is done. 59. Small proteins produced by vertebrate cells in
(c) alkaline phosphatase is the key reagent. response to viral infections inhibiting viral
(d) catalase is the key reagent. multiplication are known as:
55. Which of the following roles, given in below (a) Lipoproteins (b) Immunoglobulins
statements, are played by pioneer species on a (c) Interferons (d) Antitoxins
60. Which of the following enzyme is used in case
barren rock?
of fungus to release DNA along with other
(i) It helps in dissolving rocks.
macromolecules from the cells?
(ii) It helps in weathering.
(a) Lysozyme (b) Cellulase
(iii) It helps in soil formation.
(c) Chitinase (d) Amylase
(a) (i) and (ii) (b) (i) and (iii)

PART-II (2 MARKS QUESTIONS)

MATHEMATICS 62. For positive real numbers x and y, let


61. In given figure, CD || AE and CY || BA. Then log y
f (x, y) = x 2 . If the sum of the solutions of the
(a) Area (DAXY) = Area (DCXY) equation 4096 f ( f (x, x), x) = x13 can be expressed
(b) Area (ABCY) = 3 Area (DAXY)
m
(c) Area (DCBX) = Area (DAXY) as ( where m, n are coprime numbers), then
n
(d) None of these
(m – 10n) is
(a) 1 (b) 2
(c) –1 (d) None of these
EBD_7839
84 KVPY-SA

63. The medians BE and CF of a DABC intersect at G. (a) The wave C is ahead by a phase angle of
Then choose the correct option. p
and the wave B lags behind by a phase
(a) ar (DGBC) = ar of quad. AFGE 2
p
(b) ar (DGBC) = ar angle
2
(c) ar (DGBC) = ar
(b) The wave C is lag behind by a phase angle
(d) ar (DBGC) = ar (BACG) p
64. A right circular cylinder has its height equal to of and the wave B is ahead by a phase
2
two times its radius. It is inscribed in a right p
circular cone having its diameter equal to 10 cm angle
2
and height 12 cm, and the axes of both the cylinder (c) The wave C is ahead by a phase angle of p
and the cone coincide. Then, the volume (in cm 3) and the wave B lags behind by a phase
of the cylinder is approximately angle p
(d) The wave C lags behind by a phase angle
(a) 107.5 (b) 118.6 of p and the wave B is ahead by a phase
(c) 127.5 (d) 128.7 angle p
65. The mean of the data set comprising of 16 68. Following are expressions for four plane simple
observations is 16. If one of the observation valued harmonic waves
16 is deleted and three new observations valued 3, æ xö
4 and 5 are added to the data, then the mean of the (i) y1 = A cos 2p ç n1t + ÷
resultant data, is: è l1 ø
(a) 15.8 (b) 14.0
æ x ö
(c) 16.8 (d) 16.0 (ii) y2 = A cos 2p ç n1t + + p÷
è l1 ø
PHYSICS
æ xö
66. The distance between object and its real image (iii) y3 = A cos 2p ç n2 t + ÷
è l2 ø
in convex lens is D, and magnification is m. The
focal length of the lens is:
æ xö
D D (iv) y4 = A cos 2p ç n2t - ÷
(a) (b) 1 - m è l2 ø
1+ m
mD mD The pairs of waves which will produce
(c) (d) destructive interference and stationary waves
(1 + m) 2
(1 - m) 2
respectively in a medium, are
67. The figure shows three progressive waves A, B (a) (iii, iv), (i, ii) (b) (i, iii), (ii, iv)
and C with their phases expressed with respect (c) (i, iv), (ii, iii) (d) (i, ii), (iii, iv)
to the wave A. What can be concluded from the 69. A rectangular glass slab ABCD, of refractive
figure? index m1, is immersed in water of refractive index
µ2(µ1 >µ2). A ray of light is incident at the surface
y
AB of the slab as shown. The maximum value of
B A C the angle of incidence amax such that the ray
comes out only from the other surface CD is
given by
wt

wt = p/2 wt = p wt = 3p/2
MOCK TEST-9 85

(a) (b)

(c) (d)

ém æ m öù 73. Calculate the total pressure in a 10.0 L cylinder


(a) sin -1 ê 1 cos ç sin -1 2 ÷ú
m
ëê 2 è m1 ø ûú which contains 0.4 g helium, 1.6 g oxygen and
é 1.4 g nitrogen at 27 °C.
æ 1 öù
(b) sin -1 êm1 cos ç sin -1 ÷ú (a) 0.492 atm (b) 49.2 atm
êë è m 2 ø úû
(c) 4.52 atm (d) 0.0492 atm
-1 æ
m ö
(c) sin ç 1 ÷ 74. The enthalpy changes for the following
è m2 ø
processes are listed below :
æm ö Cl2(g) ® 2Cl(g), 242.3 kJ mol–1
(d) sin -1 ç 2 ÷
è m1 ø I2(g) ® 2I(g), 151.0 kJ mol–1
70. A body initially at 80°C cools to 64°C in 5 minutes
ICl(g) ® I(g) + Cl(g), 211.3 kJ mol–1
and to 52°C in 5 minutes. What will be the
temperature after 15 minutes and what is the I2(s) ® I2(g), 62.76 kJ mol–1
temperature of surroundings? Given that the standard states for iodine and
(a) 16°C, 43°C (b) 6°C, 3°C chlorine are I2(s) and Cl2(g), the standard enthalpy
(c) 43°C, 16°C (d) 3°C, 6°C of formation for ICl(g) is :

CHEMISTRY (a) +16.8 kJ mol–1 (b) +244.8 kJ mol–1


(c) –14.6 kJ mol–1 (d) –16.8 kJ mol–1
71. In which of the following arrangements, the order
is NOT according to the property indicated 75. An unknown compound A has a molecular
against it? formula C4H6. When A is treated with an excess
(a) Li < Na < K < Rb : of Br2 a new substance B with formula C4H6Br4
Increasing metallic radius is formed. A forms a white precipitate with
(b) I < Br < F < Cl : ammonical silver nitrate solution. A may be
Increasing electron gain enthalpy (a) 1-Butyne (b) 2-Butyne
(with negative sign) (c) 1-Butene (d) 2-Butene
(c) B < C < N < O : BIOLOGY
Increasing first ionisation enthalpy
76. The sequence given below represents:
3+ 2+ + - Blue green algae ® Crustose lichen ® Foliose
(d) Al < Mg < Na < F :
Increasing ionic size lichen ® Mosses ® Shrubs ® Dicot trees
(a) Food chain
72. 2, 2, 6, 6-tetramethyl cyclohexanol is treated with
an acid. An alkene is formed. The structure of (b) Ecological succession
the alkene is (c) Ecological trend
(d) Food pyramid
EBD_7839
86 KVPY-SA

77. An actively dividing bacterial culture is grown 79. After which of the following treatments (i – iii),
in a medium containing radioactive adenine would an enzyme still be expected to have
(A*). After all the adenine is labelled, the bacteria activity?
are transferred to a medium containing (i) Protease treatment.
nonradioactive adenine (A). Following one (ii) Heating almost to the point of denaturation
round of DNA replication in the nonradioactive and then cooling once only.
medium, the DNA is analysed. Which of the (iii) Freezing and then thawing once only.
following sequences could represent this DNA? (a) (i) only
(a) A*A*TTGA*TC (b) (iii) only
TTAACTAG (c) (i) and (ii) only
(b) A*ATTGA*TC (d) (ii) and (iii) only
TTA*A*CTAG 80. Calvin and colleagues determined the pathway
(c) AATTGATC of carbohydrate synthesis in plants by studying
TTAACTAG the incorporation of radioactive carbon dioxide
(d) A*A*TTGA*TC into biological compounds. Suppose that
TTA*A*CTA*G photosynthesis is proceeding at a steady pace
78. Homologous structures among organisms in a typical experiment with the lights on, and
provide evidence for evolution in that these carbon dioxide is being combined with
structures are ribulose-bisphosphate (RuBP) to produce
(a) different in different animals, but are 3-phosphoglycerate (3PG). Then suddenly the
modifications of the same basic structure. source of carbon dioxide is eliminated. What
(b) similar in function, but of different basic changes in the concentrations of 3PG and RuBP
structure. would occur?
(c) all shown in the fossil record. (a) 3PG levels rise, RuBP levels fall.
(d) all produced by the same gene. (b) 3PG levels fall, RuBP levels rise.
(c) 3PG levels rise, RuBP levels rise.
(d) 3PG levels rise, RuBP levels stay the same.
ART - 1 : BIOLOGY
ANS W ER KEYS
Part-I Part-II
1 (a) 11 (d ) 21 (b) 31 (d ) 41 (c) 51 (c) 61 (c) 71 (c)
2 (d ) 12 (a) 22 (a) 32 (c) 42 (c) 52 (c) 62 (a) 72 (a)
3 (c) 13 (c) 23 (b) 33 (a) 43 (c) 53 (d ) 63 (a) 73 (a)
4 (a) 14 (d ) 24 (a) 34 (d) 44 (b ) 54 (c) 64 (c) 74 (a)
5 (d ) 15 (d ) 25 (c) 35 (a) 45 (b ) 55 (d) 65 (b ) 75 (a)
6 (b ) 16 (a) 26 (c) 36 (b) 46 (d ) 56 (a) 66 (c) 76 (b)
7 (d ) 17 (b ) 27 (a) 37 (d) 47 (a) 57 (c) 67 (b ) 77 (a)
8 (d ) 18 (a) 28 (a) 38 (a) 48 (a) 58 (a) 68 (d ) 78 (a)
9 (b ) 19 (d ) 29 (a) 39 (a) 49 (d ) 59 (c) 69 (a) 79 (b)
10 (c) 20 (d ) 30 (a) 40 (a) 50 (b ) 60 (c) 70 (c) 80 (b)
Telegram @unacademyplusdiscounts

Join Us on Telegram for More Such Books

https://telegram.me/unacademyplusdiscounts

Join us from the above link or search ''unacademyplusdiscounts'' in Telegram


10
1
Time : 3 Hours Stream-SA Maximum Marks : 100
INSTRUCTIONS
1. There are 80 questions in this paper.
2. The question paper contains two parts; Part–I (1 Mark Questions) and Part–II (2 Marks
Questions). There are four sections; Mathematics, Physics, Chemistry and Biology in each part.
3. There are four options given with each question, only one of them is correct.
4. For each incorrect answer 0.25 Mark in Part–I and 0.5 Mark in Part–II will be deducted.

PART-I (1 MARK QUESTIONS)

MATHEMATICS ratio of ar (DDPE) to (trapezium ABCD)


(a) 1 : 8 (b) 1 : 16
2 f (n ) + 1 (c) 1 : 32 (d) 1 : 27
1. If f (n + 1) = , n = 1, 2, ...... a nd
2 5. N is the product of first 100 multiple of K. If N is
f (1) = 2, then f (101) = ____________. divisible by 10100 then find the minimum number
(a) 53 (b) 52 of zeros at the end of N
(a) 24 (b) 124
(c) 51 (d) 50
(c) 97 (d) 121
2. When 648 is added or 739 subtracted from a
6. Let X be any point on the side BC of a triangle
number N result in both the cases is a perfect
ABC. If XM, XN are drawn parallel to BA and CA
cube then which one of the following is correct
about N? meeting CA. BA in M, N respectively, MN meets
(i) N is an even number CB produced in T. Then
(ii) N is divisible by 3
(iii) N is divisible by 5
(a) Only (i) & (ii) (b) Only (i) & (iii)
(c) Only (ii) & (iii) (d) None of these
3. The numbers a, b, c are the digits of a three digit
number which satisfy 49a + 7b + c = 286. Then,
the value of 100a + 10b + c is
(a) 556 (b) 256
(c) 356 (d) 286
4. In a trapezium ABCD, P and Q are points on AD
DP CQ 1
and BC respectively such that = = . (a) TB2 = TX × TC (b) TC2 = TX × TX
AP BQ 3
(c) TX2 = TB × TC (d) TX2 = 2(TB × TC)
If the diagonal BD intersect PQ at E, find the
EBD_7839
88 KVPY-SA

7. Let ABC be an acute angled triangle and CD be time of flight increased by 30 minutes, then the
the altitude through C. If AB = 8 and CD = 6, duration of flight is
then the distance between the mid-points of AD
(a) 1 hr. (b) 2 hrs.
and BC (i.e. EF) is
(c) 3 hrs. (d) 4 hrs.
12. Find 6 root of 99 + 70 2
th

(a) 1 + 2 (b) 2 + 3
(c) 3 + 5 (d) 2 + 5
13. What is the last 3 digit number of 1223334444
…… 1000 digit.
(a) 212 (b) 414
(a) 3 (b) 5 (c) 373 (d) 323
(c) 6 (d) 4 14. Let n! = 1 × 2 × 3 × … × n for integer n.
8. A circle is inscribed in a square and the square If p = 1! + (2 × 2!) + (3 × 3!) + … + (10 × 10!), then
is circumscribed by another circle. What is the p + 2 when divided by 11! Leaves a remainder of
ratio of the areas of the inner circle to the outer (a) 10 (b) 0
circle? (c) 7 (d) 1
15. In how many ways Ram can distribute 40 apples
(a) 1 : 2 (b) 1: 2
in his six children named A, B, C, D, E and F
(c) 2 :4 (d) 1: 3 such that A gets two more than B, C gets 3 more
9. P, Q and R are respectively the mid–points of than F and D gets five less than E and every one
sides BC, CA and AB of a triangle ABC. PR and must have atleast one fruit
(a) 101 (b) 91
BQ meet at X. CR and PQ meet at Y. Then
(c) 96 (d) 136
1 2
(a) XY= BC (b) XY = BC PHYSICS
3 3
16. A river is flowing with a velocity of 1 m/s towards
1 2
(c) XY = BC (d) XY = BC east directions. When the boat runs with a
4 5
velocity of 3 m/s relative to the river is the
10. In the figure, ABCD is a semicircle. ÐCAD =
direction of the river flow, the flag on the boat
flutter in north direction. If the boat runs with
the same speed but in north direction relative to
river, the flag flutters towards north-east
direction. The actual velocity of the wind should

be : ( $i ® east direction and $j ® north direction).


(a) 25° (b) 40°
(c) 45° (d) 50° (a) 4 $i + 6 $j (b) 6 $i + 4 $j
11. In a flight of 600 km an aeroplane was slowed
down due to a bad weather. If the average speed (c) 4 $i – 6 $j (d) 6 $i – 4 $j
for the trip was reduced by 200 km/h and the
MOCK TEST-10 89

17. A U-tube is of non uniform cross-section. The


area of cross-sections of two sides of tube are
A and 2A (see fig.). It contains non-viscous
liquid of mass m. The liquid is displaced slightly
and free to oscillate. Its time period of
oscillations is
2l 2 2l
m A 2A (a) 2p (b) 2p
(a) T = 2p 3g 3g
3rgA
2l l
m (c) 2p (d) 3p
(b) T = 2p 3g 3g
m
2rgA
20. Resistance of a given wire is obtained by
measuring the current flowing in it and the
m voltage difference applied across it. If the
(c) T = 2p
rgA percentage errors in the measurement of the
current and the voltage difference are 3% each,
(d) None of these
then error in the value of resistance of the wire
18. Variation of radiant energy emitted by Sun, is :
filament of tungsten lamp and welding arc as a (a) Zero (b) 1%
function of its wavelength is shown in figure. (c) 3% (d) 6%
Which of the following option is the correct
21. Consider the following statements :
match ? A gas can be liquefied by increasing the pressure
1. above the critical pressure only
2. Only when the temperature of the enclosed
gas is below the critical temperature
3. Only when the volume of the enclosed gas
is below the critical volume
Which of the statements given above is / are
(a) Sun-T3 , tungsten filament - T1, welding
correct :
arc - T2 (a) 1 and 2 (b) 2 only
(b) Sun-T2 , tungsten filament - T1, welding (c) 3 only (d) 2 and 3
arc - T3 22. The period of oscillation of a simple pendulum
(c) Sun-T3 , tungsten filament - T2, welding
arc - T1 L
is T = 2p . Measured value of L is 20.0 cm
(d) Sun-T1 , tungsten filament - T2, welding g
arc - T3 known to 1 mm accuracy and time for 100
19. A system of two identical rods (L-shaped) of oscillations of the pendulum is found to be 90 s
mass m and length l are resting on a peg P as using a wrist watch of 1s resolution. The
shown in the figure. If the system is displaced accuracy in the determination of g is :
in its plane by a small angle q, find the period of (a) 1% (b) 5%
oscillations (c) 2% (d) 3%
EBD_7839
90 KVPY-SA

23. A wire is bent in the form of circle of radius 2 m. 26. Velocity of the centre of a small cylinder is v.
Resistance per unit length of wire is 1/p W/m. There is no slipping anywhere. The angular
Battery of 6V is connected between A and B. velocity of the centre of the larger cylinder is
ÐAOB = 90°. Find the current through the (a) 2v
battery. (b) v 2R
R
v
(a) 8 A I 3v
B
(c)
(b) 4 A 2
(d) None of these
(c) 3 A 27. The refractive index of a material of a prism of
A
(d) 9 A angles 45°, 45°, 90° is 1.5. The path of the ray of
6V
light incident normally on the hypotenuse side
24. A black coloured solid sphere of radius R and is shown in figure
mass M is inside a cavity with vacuum inside.
A A
The walls of the cavity are maintained at
90° 90°
temperature T0. The initial temperature of the (a) (b)
45° 45° 45° 45°
sphere is 3T0. If the specific heat of the material B C B C
of the sphere varies as aT3 per unit mass with
the temperature T of the sphere, where a is a A A
90° 90°
constant, then the time taken for the sphere to
(c) (d)
cool down to temperature 2T0 will be (s is Stefan B 45° 45°
C B 45° 45°
C
Boltzmann constant)
28. A graph of the square of the velocity against the
Ma æ 3ö Ma æ 16 ö square of the acceleration of a given simple
(a) ln ç ÷ (b) ln ç ÷
4pR s è 2 ø
2
4pR s è 3 ø
2 harmonic motion is

v2 v2
Ma æ 16 ö Ma æ 3ö
(c) ln ç ÷ (d) ln ç ÷
16pR s è 3 ø
2
16pR s è 2 ø
2
(a) (b)
a2 a2
25. A heater is designed to operate with a power of
1000 watts in a 100 volt line.
v2 v2

Heater
(c) (d)
10W R a2 a2

29. Which one of the following statements is true?


100V (a) Both light and sound waves in air are
It is connected in a combinations with a transverse
resistance of 10 ohms and a resistance R to a 100 (b) The sound waves in air are longitudinal
volts mains as shown in the figure. What should while the light waves are transverse
be the value of R so that the heater operates (c) Both light and sound waves in air are
with a power of 62.5 watts. longitudinal
(a) 25 W (b) 0.05 W (d) Both light and sound waves can travel in
(c) 5 W (d) 65 W vacuum
MOCK TEST-10 91

30. A box of mass 1 kg is mounted with two cylinders 34. When PbO2 reacts with conc. HNO3 the gas
each of mass 1 kg, moment of inertia 0.5 kg m2 evolved is:
and radius 1m as shown in figure. Cylinders are (a) NO2 (b) O2
mounted on their control axis of rotation and
(c) N2 (d) N2O
this system is placed on a rough horizontal
surface. The rear cylinder is connected to battery 35. Which of the following pair will give
operated motor which provides a torque of displacement reaction?
100N-m to this cylinder via a belt as shown. If (a) ZnSO4 solution and Aluminium metal.
sufficient friction is present between cylinder (b) MgCl2 solution and aluminium metal.
and horizontal surface for pure rolling, find
(c) FeSO4 solution and silver metal.
acceleration of the vehicle in m/s2. (Neglect mass
of motor, belt and other accessories of vehicle). (d) AgNO3 solution and copper metal.
Electric 36. Which of the following is not aromatic?
(a) 20 m/s2 motor
(a) Benzene
(b) 10 m/s2 m (b) Naphthalene
(c) 25 m/s2 (c) Pyridine
(d) 30 m/s2 (d) 1,3,5 Heptatriene
37. For which one of the following sets of four
CHEMISTRY quantum numbers, an electron will have the
31. The maximum number of molecules are present in highest energy?
(a) 15 L of H2 gas at STP n l m s
(b) 5 L of N2 gas at STP (a) 3 2 1 1/2
(c) 0.5 g of H2 gas
(b) 4 2 –1 1/2
(d) 10 g of O2 gas
32. Which of the following will be most easily (c) 4 1 0 –1/2
attacked by an electrophile? (d) 5 0 0 –1/2
38. I, II, and III are three isotherms, respectively, at
Cl OH
T1, T2 and T3. Temperature will be in order:

(a) (b)

CH3
P

(c) (d)
I
II
33. Which of the following statement is incorrect? III
(a) Alkali metal hydroxide are hygroscopic.
(b) Dissolution of alkali metal hydroxide is V3 V2 V1
endothermic.
(c) Aqueous solution of alkali metal hydroxides (a) T1 = T2 = T3 (b) T1 < T2 < T3
are strongly basic.
(c) T1 > T2 > T3 (d) T1 > T2 = T3
(d) Alkali metal hydroxides form ionic crystals.
EBD_7839
92 KVPY-SA

39. Standard reduction potentials of the half BIOLOGY


reactions are given below : 46. The branches of the nodal tissue, which give
F2(g) + 2e– ® 2F– (aq); E° = + 2.85 V rise to minute fibres throughout the ventricular
Cl2(g) + 2e– ® 2Cl–(aq); E° = + 1.36 V musculature of the respective sides are called:
Br2(l) + 2e– ® 2Br–(aq); E° = + 1.06 V (a) SA node (b) AV node
(c) Purkinje fibre (d) bundle of His
I2(s) + 2e– ® 2I–(aq); E° = + 0.53 V
47. Peroxisomes are rich in:
The strongest oxidising and reducing agents (a) DNA
respectively are : (b) RNA
(a) F2 and I– (b) Br2 and Cl– (c) Catalytic enzymes
(c) Cl2 and Br– (d) Cl2 and I2 (d) Oxidative enzymes
48. The threshold of a neuron is:
40. 0.45 g acid of molecular weight 90 is neutralised
(a) the amount of inhibitory neurotransmitter
by 20 mL of 0.5N caustic potash. The basicity of
required to inhibit an action potential.
acid is: (b) the membrane voltage at which an axon
(a) 1 (b) 2 (c) 3 (d) 4 potential will be suppressed.
41. Which of the following oxides is amphoteric in (c) the amount of excitatory neurotransmitter
character? required to elicit an action potential.
(d) the membrane voltage at which the
(a) SnO2 (b) SiO 2
membrane potential develops into an action
(c) CO2 (d) CaO potential.
42. The highest boiling point is expected for : 49. The lacunae in vascular bundles of monocot
stems are:
(a) iso-octane
(a) Metaxylem
(b) n-octane (b) Mucilage canal
(c) 2, 2, 3, 3-tetramethylbutane (c) Lysigenous water cavity
(d) n-butane (d) Large-sized vessel
43. Pressure of a mixture of 4 g of O2 and 2 g of H2 50. Nutrient enrichment of water body is:
(a) Eutrophication
confined in a bulb of 1 litre at 0 °C is
(b) Stratification
(a) 25.22 atm (b) 31.20 atm (c) Biomagnification
(c) 45.21 atm (d) 15.21 atm (d) None of these
44. Diacidic base is: 51. Energy pyramids are used to represent energy
transfer in an ecosystem because energy is
(a) CH2(OH)2 (b) Ca(OH)2 _______ at each trophic level.
(c) CH3CH(OH)2 (d) All of these (a) Gained
45. For which of the following process, DS is (b) Lost
(c) Conserved
negative?
(d) Either conserved or gained
(a) H2 (g) ¾¾
® 2 H(g) 52. Vaccines are
(b) 2SO3 (g) ¾¾
® 2SO2(g) + O2 (g) (a) treated bacteria or viruses or one of their
proteins.
compressed
(c) N 2 (4L) ¾¾¾¾¾® N 2 (2L) (b) MHC (major histocompatibility complex)
proteins.
(d) C(diamond) ¾¾
® C (graphite)
(c) curative medicines.
(d) monoclonal antibodies.
MOCK TEST-10 93

53. The rate of photosynthesis of a freshwater plant (a) Adrenocorticotropic hormone (ACTH)
is measured using five spectral colours. Which (b) Thyroxine
sequence of colours would give an increasing (c) Oxytocin
photosynthetic response? (d) Insulin
Smallest Largest response 57. When a piece of bread is chewed, it tastes sweet
(a) Blue Green Yellow Orange Red because
(b) Green Yellow Orange Red Blue (a) the sugar contents are drawn out.
(c) Red Orange Yellow Green Blue (b) saliva converts starch into maltose.
(d) Yellow Green Orange Blue Red (c) the taste buds are stimulated by chewing.
54. All but one of the following chemicals are (d) None of the above
neurotransmitters that function in the human 58. Resting membrane potential will be greater in:
brain. Select the exception. (a) Large size fibre (b) Small size fibre
(a) Dopamine (b) Glycine (c) Both (a) and (b) (d) None of these
(c) Atropine (d) Glutamic acid 59. Substance that accumulates in a fatigued muscle is:
55. Purkinje fibres are present in: (a) Pyruvic acid
(a) Left auricle (b) Carbon dioxide
(b) Right auricle (c) ADP
(c) Ventricular myocardium (d) Lactic acid
(d) SA node 60. Inhaled air passes through which of the
56. Injections of which hormone are sometimes given following in the last?
to strengthen contractions of the uterus during (a) Bronchiole (b) Larynx
childbirth? (c) Pharynx (d) Trachea

PART-II (2 MARKS QUESTIONS)

MATHEMATICS 63. In the figure ABC is a triangle, D is mid point of


AB. P is any point on BC. Line CQ is drawn
61. ABCD is a parallelogram. Any line through A parallel to PD to intersect AB at Q. PQ is joined.
cuts DC at a point P and BC produced at Q. Then choose the correct option.
Then,
(a) Area (DBPC) = Area (DDPQ)
1
(b) Area (DBPC) = Area (DACB)
4
1
(c) Area (DDPQ) = Area (DADP)
4
(d) None of these 1
(a) ar (DBPQ) = ar (DABC)
4
ì 2 p - 2ü
62. If A = íx : - £ x £ ý, 2
î 5 5 þ (b) ar (DBPQ) = ar (DABC)
B = {y : – 1 £ y £ 1|} and f (x) = cos (5x + 2), 3
then the mapping f : A ® B is 1
(a) one-one but not onto (c) ar (DBPQ) = ar (DABC)
3
(b) onto but not one-one
(c) both one-one and onto 1
(d) ar (DBPQ) = ar (DABC)
(d) neither one-one nor onto 2
EBD_7839
94 KVPY-SA

64. There is a right circular cone of height h and 68. Two soap bubbles A and B are kept in a closed
vertical angle 60º. A sphere when placed inside chamber where the air is maintained at pressure
the cone, it touches the curved surface and the 8 N/m2. The radii of bubbles A and B are 2 cm
base of the cone. The volume of sphere is and 4 cm, respectively. Surface tension of the
soap-water used to make bubbles is 0.04 N/m.
4 3 4
(a) ph (b) ph3 Find the ratio nB / nA where nA and nB are the
3 27
number of moles of air in bubbles A and B,
4 3 4 3 respectively. [ Neglect the effect of gravity].
(c) ph (d) ph
9 81 (a) 2 (b) 9
65. In a set of 2n distinct observations, each of the (c) 8 (d) 6
observations below the median of all the 69. A beaker contains a fluid of density r kg / m3,
observations is increased by 5 and each of the specific heat S J/kgºC and viscosity h. The
remaining observations is decreased by 3. Then beaker is filled upto height h. To estimate the
the mean of the new set of observations: rate of heat transfer per unit area (Q/A) by
(a) increases by 1 (b) decreases by 1 convection when beaker is put on a hot plate, a
(c) decreases by 2 (d) increases by 2 student proposes that it should depend on h ,
PHYSICS
æ S Dq ö and æ 1 ö when Dq (in ºC) is the
çè ÷ çè rg ø÷
66. The time taken by the earth to travel over half its h ø
orbit, remote from the sun, separated by the minor
difference in the temperature between the bottom
axis is about 2 days more than half the year, then
and top of the fluid. In that situation the correct
the eccentricity of the orbit is
option for (Q/A) is:
(a) 1/30 (b) 1/60
(c) 1/15 (d) 1/70
æ S Dq ö æ 1 ö æ S Dq ö æ 1 ö
67. A 2 m wide truck is moving with a uniform speed (a) h ç (b) ç
è h ÷ø çè rg ø÷ è hh ÷ø èç rg ø÷
v0 = 8 m/s along a straight horizontal road. A
pedestrian starts to cross the road with a uniform
speed v when the truck is 4 m away from him. S Dq S Dq
(c) (d) h
The minimum value of v so that he can cross the hh h
road safely is 70. Two particles of masses m1 and m2 moving in
coplanar parabolas round the sun, collide at right
angles and coalesce when their common distance
from sun is R. The subsequent path of the
combined particles is an ellipse of major axis

( m1 + m2 ) 2 ( m1 - m2 ) 2
(a) R (b) R
2 m1m2 2m1 m2

(a) 2.62 m/s (b) 4.6 m/s ( m1 + m2 ) 2 ( m1 + m2 ) 2


(c) R (d) R
(c) 3.57 m/s (d) 1.414 m/s m1m2 3m1 m2
MOCK TEST-10 95

CHEMISTRY CH 3
|
71. In hydrogen atom, energy of first excited state is (c) CH3 – CH2–CH–CH2Br + NH3 ¾¾®
–3.4 eV. Find out KE of the same orbit of hydrogen
atom: CH3
|
(a) + 3.4 eV (b) + 6.8 eV ¾¾® CH3 – CH2 – CH – CH2NH2
(c) – 13.6 eV (d) + 13.6 eV
72. The addition of Br2 to (E)-but-2-ene gives: (d) CH3CHO + HCN ¾¾
® CH3CH (OH) CN
(a) (R,R)-2,3-dibromobutane
(b) (S,S)-2,3-dibromobutane BIOLOGY
(c) (R,S)-2,3-dibromobutane
(d) A mixture of (R,R) and 76. Plant cell ‘A’ has an osmotic pressure of 12 atm
(S,S)-2,3-dibromobutane and is immersed in solution of 10 atm osmotic
73. Consider the following changes : pressure. Another cell ‘B’ has 10 atm osmotic
pressure and is immersed in solution of osmotic
® M (g)
(1) M ( s ) ¾¾ pressure of 8 atm. Both the cells are allowed to
come to equilibrium, then removed from their
® M 2 + ( g ) + 2e -
(2) M ( s ) ¾¾ solution and brought in intimate contact.
Assuming that there is no external influencing
force, what will be the result?
® M + ( g ) + e-
(3) M ( g ) ¾¾
(a) There will be a net flow of water from ‘A’to ‘B’.
(b) There will be net flow of water from ‘B’ to ‘A’.
+
® M 2+ ( g ) + e -
(4) M ( g ) ¾¾ (c) There will be no net flow of water.
(d) Water will freely pass from ‘A’ to ‘B’ but
® M 2+ ( g ) + 2e -
(5) M ( g ) ¾¾ not from ‘B’ to ‘A’.
77. A female with normal genotype (XX, 44) showed
The second ionization energy of M could be the presence of testis determining factor
calculated from the energy values associated (TDF) gene on the X chromosome. This is most
with : likely a result of:
(a) 1 + 3 + 4 (b) 2 – 1 + 3 (a) X chromosome inactivation
(c) 1 + 5 (d) 5 – 3 (b) Dosage compensation effect
74. The oxidation states of Cr in [Cr(H2O)6]Cl3, (c) Mutation
[Cr(C6H6)2], and K2[Cr(CN)2(O)2(O2)(NH3)] (d) Meiotic recombination
respectively are : 78. Two alleles are found in Calico cats for coat
(a) +3, +4, and +6 (b) +3, +2, and +4 colour and are located on the X chromosome.
(c) +3, 0, and +6 (d) +3, 0, and + 4 One allele is responsible for black colour while
75. Which one is a nucleophilic substitution the other for orange colour. Which of the
reaction among the following ? following is correct about the phenotype of
(a) CH3 – CH = CH2 + H2O Calico cats?
H+ (a) All male cats will have the same coat colour.
¾¾¾
® CH3 - CH - CH 3 (b) Female cats can have mosaic pattern.
|
OH (c) All females will show intermediate coat
colour.
(b) RCHO + R¢MgX ¾¾® R – CH – R¢
| (d) All female cats born to mothers with black
OH coat colour will have black coat colour.
EBD_7839
96 KVPY-SA

79. The critical day lengths for 4 plants are as 80. Which of the following pathways depicts the
follows: generalised glycolytic scheme most accurately?
Plant A – 15.5 hrs Plant B – 15.5 hrs (a) Glucose ® a ® b ® c ® d ® f ® g ® h ® Pyruvate
Plant C – 10.0 hrs Plant D – 9.5 hrs
e
Plant A flowers when it receives 8.5 or more
hours of darkness. (b) Glucose « a « b « c « d « f « g « h « Pyruvate
Plant B flowers when it receives a minimum of e
15.5 hrs of light.
(c) Glucose ® a « b ® c « d « f « g « h ® Pyruvate
Plant C flowers when it receives less than 10 hrs
e
of light.
Plant D flowers when it receives less than 9.5 (d) Pyruvate ® Glucose ® a ® b ® c
hrs of light.
Which one is a long day plant? h ¬ g ¬ f ¬ e ¬ d
(a) Plant A (b) Plant A and C
(c) Plant D (d) Plant B

ANS W ER KEYS
Part-I Part-II
1 (b ) 11 (a) 21 (d ) 31 (a) 41 (a) 51 (b ) 61 (a) 71 (a)
2 (c) 12 (a) 22 (d ) 32 (b) 42 (b) 52 (a) 62 (c) 72 (c)
3 (a) 13 (d) 23 (a) 33 (b ) 43 (a) 53 (b ) 63 (d ) 73 (d)
4 (c) 14 (d) 24 (c) 34 (b ) 44 (b) 54 (c) 64 (d ) 74 (c)
5 (b ) 15 (b) 25 (c) 35 (d ) 45 (c) 55 (c) 65 (a) 75 (c)
6 (c) 16 (a) 26 (b ) 36 (d ) 46 (c) 56 (c) 66 (b ) 76 (b)
7 (b ) 17 (a) 27 (a) 37 (b ) 47 (d) 57 (b ) 67 (c) 77 (d)
8 (a) 18 (a) 28 (d ) 38 (c) 48 (d) 58 (a) 68 (d ) 78 (b)
9 (c) 19 (b) 29 (b ) 39 (a) 49 (c) 59 (d ) 69 (d ) 79 (d)
10 (b ) 20 (d) 30 (a) 40 (b ) 50 (a) 60 (a) 70 (a) 80 (c)
Telegram @unacademyplusdiscounts

Join Us on Telegram for More Such Books

https://telegram.me/unacademyplusdiscounts

Join us from the above link or search ''unacademyplusdiscounts'' in Telegram


HINTS & SOLUTIONS

MOCK TEST-1
PART – I So, ÐABP = ÐDEP and hence, AB is parallel to
DE.
MATHEMATICS
Similarly, BC is parallel to EF and CA is parallel
1. (a) Since x2 + y2 < 25 and x and y are integers, to DF.
the possible values of x and y Î (0, ± 1, ± 2, BD AE AF DC
Using these, we obtain = = =
± 3, ± 4). Thus x and y can be chosen in 9 ways DC EC FB BD
each and (x, y) can be chosen in 9 × 9 = 81 ways. Hence BD 2 = CD 2 or which is same as
However we have to exclude cases ( ± 3, ± 4), BD = CD. Thus, D is the midpoint of BC.
( ± 4, ± 3) and ( ± 4, ± 4) (i.e.) 3 × 4 = 12 cases. Similarly, E, F are the midpoints of CA and AB
Hence the number of permissible values respectively.
= 81 – 12 = 69. We infer that AD, BE, CF are indeed the medians
2. (d) p (x) = (2 x + 3) (x 97 + x 96 +... + 1). of the triangle ABC and hence P is the centroid
= (2 x + 3) (x + 1) (x 96 + x 94 +.. + x 2 + 1) of the triangle.
Also, x 96 + x 94 +... + x 2 + 1 > 1 " x Î R. AD BE CF
So required real roots are –1 and –3/2 So, = = = 3, and consequently
PD PE PF
Sum of roots is –5/2 each of the given ratios is also equal to 3.
3. (b) Let AP, BP, CP when extended, meet the
sides BC, CA, AB in D, E, F respectively. Draw x1 + ... + x15
4. (b) Given, = 13
AK, PL perpendicular to BC with K, L on BC. 15
A x1 + x2 + x3 + ... + x15 = 15 × 13 = 195
In order to set the second largest and largest,
first thirteen natural numbers are
1, 2, 3, 4, 5, 6, 7, 8, 9, 10, 11, 12, 13
F E 13 ´ 14
P So, x14 + x15 = 195 –
2
Þ x14 + x15 = 195 – 91 = 104
Now, from the options, we have
B K LD C
x14 = 51 and x15 = 53
d ( A, BC ) AK AD Now, second largest is 51.
Now, = = .
d ( P, BC ) PL PD 5. (b)
d ( B, CA) BE d (C, AB) CF
Similarly, = and = .
d ( P, CA) PE d (P, AB) PF
AD BE CF
So, we obtain = = , and hence
PD PE PF
AP BP CP
= = . C
PD PE PF
AP BP G
From = and ÐAPB = ÐDPE, it follows B r2
PD PE F
that triangles APB and DPE are similar. 60 30º r1
A D E
EBD_7839
98 KVPY-SA
r1 ® radius of smaller sphere Substituting two expressions for 3
3 and 3
9
r2 ® radius of bigger sphere into (ii),we get:
ÐBAD = 30°, r1 = 7 cm
r1
( ) (
x3 = 2 2 + 6 x - 2 + 3 2 x2 + 2 - 2 x 2 + 3, )
\ AB = = 2r1 Or equivalently,
sin 30º
AB = AF + BF (
x3 + 6 x - 3 = 2 3x 2 + 2 . )
2r1 = AF + r1 (Q BF = r1) Squaring both sides yields
AF = r1 = 7 cm x 6 - 6 x 4 - 6 x3 + 12 x 2 - 36 x + 1 = 0,
also, AC = 2r2 which is the required polynomial of least
AG + GC = 2r2 Þ AF + FG + r2 = 2r2 degree and required degree is 6.
Þ r2 = 7 + 14 Þ r2 = 21 cm 9. (d) Draw the angle bisector BE of ÐABC to
meet AC in E. Join ED.
6. (d) The number of digits of the numbers N
cannot be less than 224 in which case the sum A
will be less than 223 × 9 = 2007.
Moreover, if the number has 224 digits, the first b b
E
digit cannot be less than 6 as the sum of digits
would be less than 6 + 223 × 9 = 2013. 2b
Now, if the first digit is 6, the rest 223 digits would a b
have to be 9 giving the sum of digits as 2013 and a a
as there is no number less than this number C
B D
whose sum of digits can be 2013, the least Since, ÐB = 2ÐC, it follows that ÐEBC = ÐECB.
number N = 6999…9, with 9 appearing 223 times. We obtain EB = EC.
Therefore, 5N + 2013 = 5 × 6999 … 9 + 2013 Consider the triangles BEA and CED.
= 5 × (7000 … 0 1) + 2013 = 35000 … 0 + 2008 We observe that BA = CD, BE = CE and
= 35000 … 2008. ÐEBA = ÐECD.
Thus, the required answer = 3 + 5 + 2 + 8 = 18. Hence, BEA @ CED giving EA = ED.
7. (b) Let d be the common difference of the A. P., If ÐDAC = b, then we obtain ÐADE = b.
Let I be the point of intersection of AD and BE.
then 4 = abc = (b – d) b(b + d) = b(b2 – d2) Now, consider the triangles AIB and DIE.
Þ b3 = 4 + bd2 ³ 4 [Q b > 0, d2 ³ 0]] They are similar since ÐBAI = b = ÐIDE and
Þ b ³ 22/3 ÐAIB = ÐDIE.
It follows that ÐDEI = ÐABI = ÐDBI.
Thus, minimum possible value of b is 22/3, that is
Thus, BDE is isosceles and DB = DE = EA.
the case when d = 0. We also observe that
8. (a) Let x = 2 + 3 3 . Then squaring both the ÐCED = ÐEAD + ÐEDA = 2b = ÐA.
This implies that ED is parallel to AB.
sides we get Since, BD = AE, we conclude that BC = AC.
...(i) In particular ÐA = 2ÐC.
x2 = 2 + 2 2 3 3 + 3 9
Thus, the total angle of ABC is 5ÐC giving
And cubing both the side we get ÐC = 36°.
...(ii) We obtain ÐA = 72°.
x3 = 2 2 + 6 3 3 + 3 2 3 9 + 3
10. (d) For these conditions to be met, we must
From (i) 3 = x - 2 now squaring both the
3
have
sides we get
a2 + b ³ b2 - a b2 + a ³ a2 - b
3
(
9 = x² - 2 - 2 3 3 = x² - 2 - 2 2 x - 2 ) ( a - b + 1)( a + b) ³ 0 ( b - a + 1)( a + b) ³ 0
= x² + 2 - 2 x 2
SOLUTIONS – MOCK TEST-1 99

a ³ b -1 b ³ a -1 = EP 2 + PA2 + 2 EP × PA + PC 2 + PE 2 - 2PC × PE
or b + 1 ³ a
= 2( PA2 + PE 2 ) (QPA = PC)
Therefore, a = b, b – 1, b + 1.
Notice that we can only account for a = b – 1 Similarly, EB + ED = 2(QD + QE2).
2 2 2

and then reverse the solutions. Therefore,


Case I : a = b EA2 + EB2 + EC2 + ED2
= 2(PA2 + PE2) + 2(QD2 + QE2)
a2 + a
We must then have 2 is an integer..
a -a = 2( PA2 + OQ 2 ) + 2(QD 2 + OP 2 )
a2 + a a + 1 2 = 2( PA2 + OP 2 ) + 2(QD 2 + OQ 2 )
Notice that = = 1+
a2 - a a - 1 a -1
= 2(OA2 + OD 2 ) = 4 R 2
This gives the solution pairs (a, b) = (2, 2), (3, 3).
Þn=4
Case II : a = b – 1
12. (d) Given equation is 1/m +4/n =1/12 or
We notice that in this case a2 + b = b2 – a,
1/m = 1/12 – 4/n or m =12n/(n – 48) since m is
therefore, we only have to consider
positive hence n > 48 but n < 60 hence possible
b2 + a ( a + 1) + a a 2 + 3a + 1
2
values of n is 49, 51, 53, 55, 57 and 59, out of
= 2 = 2
a2 - b a - a -1 a - a -1 which only 49, 51 and 57 gives integral values
4a + 2 of m.
= 1+ 2
a - a -1 13. (d) We need to maximize the median in each
Notice that for a ³ 6, however, then we have group to maximize the average of all median.
a2 – a – 1 ³ 4a + 2, contradiction. Therefore, we Highest possible median is 18 at there should
consider a Î {1, 2, 3, 4, 5}. Testing these, we see be 3 numbers higher than it in a group of 7.
that only a = 1, 2 give solutions. Se we have
We, therefore, get the solution pair (a, b) = (1, 2), 1 2 3 18 19 20 21
(2, 3) and permutations (for a = b + 1). In
Similarly, 4 5 6 14 15 16 17
conclusion, all solutions are of the form
(a, b) = (1, 2), (2, 1), (2, 3), (3, 2), (2, 2), (3, 3). 7 8 9 10 11 12 13
11. (a) Let O be the centre of the circle and P, Q Medians are, 18, 14 and 10.
the feet of perpendiculars from O to AC and BD. 18 + 14 + 10 42
Clearly, OPEQ is a rectangle. Mean = = = 14
3 3
D 14. (c) B A
N
E
Q
O C C D
E (i) The diagonals of a square are
P
A B
perpendicular and bisect one another.
Therefore, AE = EC and AE ^ BE.
Now,
AN · NC = AN (NE + EC) = AN (AN + 2 NE)
EA2 + EC 2 = ( EP + PA)2 + ( PC - PE )2 = AN2 + 2 AN · NE.
EBD_7839
100 KVPY-SA

In DABN, hw = 10 + 15 = 25 cm., hs = 12.5 + 15 = 27.5 cm.


AB2 = BN2 + AN2 + 2 AN · NE (Since, in an obtuse Pressure due to water column,
P1 = 0.25 × 103 × g Nm–2.
angled triangle, the square on the side opposite Pressure due to mercury column,
the obtuse angle is greater than the sum of the P2 = 0.275 × 0.8 × 103 × g Nm–2
squares on the sides containing the obtuse Since P1 > P2 the mercury will rise in the
angle, by twice the rectangle contained by either spirit arm. If h is the difference in the levels
of these sides and the projection, on this side of mercury in the two arms then
P1 – P2 = hrg
produced, of the other side adjacent to the 0.25 × 103 × g – 0.275 × 0.8 × 103 × g
obtuse angle). = h × 13.6 × 103 g
Hence, AB2 – BN2 = AN · NC. or (0.25 – 0.275 × 0.8) = h × 13.6
(ii) AN2 = (AE – NE)2 and 0.25 - 0.2200 0.03
or h = = m = 0.22 cm
NC = (CE + NE) = (AE + NE) .
2 2 2 31.6 13.6
Adding gives 17. (c) Applying Bernoulli’s theorem at reservoir
AN + NC = 2 (AE + NE )
2 2 2 2 and power plant for the flowing water, we
get,
= 2 (BE2 + NE2) = 2 BN2.
1 1
15. (a) Let SQ = l1 P0 + rgh1 + rv12 = P0 + rgh2 + rv22
2 2
Þ v22 = v12 + 2g (h1 – h2).
Putting (h1 – h2) = h1 we obtain
v2 = h1 + h2 ...(i)
Equation of continuity yields
A1 v1 = A2 v2 ...(ii)
Eliminating v1 from equations (i) and (ii),
PR = l2 we obtain
Given l1 + l2 = m
2
also perimeter = 2p æ A2 ö
v2 = ç v2 ÷ + 2 gh
\ PQ = p/2 è A1 ø
In DPOQ 2 gh
2 2 2
Þ v2 = 2
æ l1 ö æ l2 ö æ pö æA ö
(PO) + (OQ) = (PQ) Þ ç ÷ + ç ÷ = ç ÷
2 2 2 1- ç 2 ÷
è 2ø è 2ø è 2ø è A1 ø
Putting A1/A2 = h,we obtain
Þ l12 + l22 = p2 Þ (l1 + l2)2 – 2l1 l2 = p2
2 gh
Þ m2 – 2l1 . l2 = p2 Þ 2l1 . l2 = – p2 + m2 v2 = h
h2 - 1
m2 - p2 1 m2 - p 2 18. (c) Newton’s law of cooling can be written as:
Þ l1 . l2 = Þ l1 . l2 =
2 2 4 T1 - T2 é T1 + T2 ù
= kê - T0 ú
1 m -p 2 2 t ë 2 û
Q Area of rhombus = .l .l = In first case ; T1 = 60°C, T2 = 40°C, T0
2 1 2 4
PHYSICS = 10°C and t = 7 minute

16. (b) When we pour 15 cm. of water and spirit in 60 - 40 é 60 + 40 ù


\ = kê - 10ú
the respective arms, then 7 ë 2 û
SOLUTIONS – MOCK TEST-1 101

1 22. (d) When a steady current flows in a metallic


or k= conductor of non uniform cross section
14
then drift speed
In second case; T1 = 40°C and T2 = ?, T
= 7 minute I I
vd = and Electric field E =
1 é 40 + T2 neA sA
40 - T2 ù
\ = ê - 10ú Drift velocity (vd) and electric field (E) vary
7 14 ë 2 û because cross section of the conductor is
T2 non-uniform.
or 80 - 2T2 = 20 + - 10
Hence only current remains constant along
2
or T2 = 28°C the length of the conductor.
23. (a) Since hollow prism contains only air,
19. (a) Here, fall in the temperature of child refraction and dispersion do not occur in
= 101 – 98 the same medium.
= 3°F There will neither be angular deviation nor
5 5 dispersion.
= ´ 3 = °C 24. (a) g1 = g - w 2 R cos 2 0 = g - w 2 R
9 3
m = 30 kg, S = 1000 cal kg–1 °C–1 g2 = g - w 2 R cos 2 60°
Heat lost by child
w2 R
5 =g-
DQ = ms DT = 30 ´ ´ 1000 4
3
= 5 × 104 J g1 g - w2 R
Let m is the mass of water evaporated in 20 = f = …(i)
g2 w2 R
min. Then g-
4
DQ 5 ´ 104
DQ = m'L or m ¢ = = = 86.2 g æ 4 3ö
L 580
GM çè pR ÷ø
Also g = 3 …(ii)
\ Average rate of evaporation = Gr
R2 R2
86.2
= = 4.31 g min -1 On solving above equation, we get
20
w1 g æ 4 - f ö 3p
I1 I1 = 1 =ç
20. (c) 40 = 10log I , \ I = 104 w 2 g 2 è 1 - f ø÷ 4GT 2
0 0
25. (c) Given that F(t) = F0 e - bt
I2 I2
and 20 = 10log I , \ I = 102 dv
0 0 Þ m = F0 e - bt
dt
I1
\ I = 100 dv F0 - bt
2 = e
dt m
I1 r22 r22
v t
F0 -bt
Also I = 100 = 2 = 2
2 r1 1
ò dv = m ò e dt
0 0
or r2 = 10 m. t
é e - bt ù
( )
21. (b) IR = x q F0 F
v= m ê ú = 0 é - e - bt - e -0 ù
= 1 × 20° êë -b úû 0 mb ë û
20° ´ p
=1× ´ 100 = 36.4 cm F0 é
180° Þ v= 1 - e - bt ù
mb ë û
EBD_7839
102 KVPY-SA

26. (b) 29. (b) lsunTsun = lstar Tstar


Tsun lstar 350 35
\T = l = =
star sun 510 51
= 0.69
30. (a) mg = Fb
( rFe ) V0 g = (k1V0 )(rHg )0 g … (i)
and (rFe )60Vt g = (k2Vt )(rHg )60 g
é (r Fe )0 ù
The forces acting on the bead as seen by or ê1 + g ´ 60 ú g
the observer in the accelerated frame are : ë Fe û
(a) N ; (b) mg ; (c) ma (pseudo force). é (r Hg )0 ù
Let q is the angle which the tangent at P = k2 ê úg … (ii)
makes with the X- axis. As the bead is in ëê1 + g Hg ´ 60 ûú
equilibrium with respect to the wire, From above equations, we get
therefore
N sin q = ma and N cos q = mg k1 é 1 + 60 g Fe ù
=ê ú
a
k2 ë1 + 60 g Hg û
\ tan q = … (i)
g CHEMISTRY
But y = k x2. Therefore, 31. (a) Na+ Mg2+ Al3+ Si4+
dy Protons 11 12 13 14
= 2kx = tan q … (ii)
dx Electrons 10 10 10 10
a a Size of isoelectronic cations decreases with
From (i) & (ii) 2kx = Þ x= increase in magnitude of nuclear charge
g 2kg \ Order of decreasing size is
27. (a) Angular momentum of system remains con- Na+ > Mg2+ > Al3+ > Si4+
stant 32. (a) C + 2S ¾¾® CS2
1 I w 20 DH = SDHP – SDHR
I µ Þ 2 = 1 = Þ I 2 = 2 I1 = 2 I
w I1 w2 10 = –1108.76 – [–393.3 – 2 ´ (293.7)]
28. (a) Distance = 4.29 light years = –1108.76 + 393.3 + 587.4 = –128.06 kJ
= (4.29 × 365.25 × 86400) × 3 × 108m 33. (c) Correct IUPAC name of the given
compound is 3-ethyl- 4-methyl pentan-2-
4.29 ´ 365.25 ´ 864 ´ 3 ´ 1010 one
= parsec
3.08 ´ 1016 34. (b) Higher the basic character of a compound,
lesser will be its pH value Rock Salt (NaCl)
[Q1 parsec = 3.08 ´ 1016 m] < Baking Soda (NaHCO3) < Washing Soda
= 1318656.9 × 10–6 parsec = 1.319 parsec. (Na2CO3) < Slaked lime (CaCO3)
Required parallax = 2q, where q is the 35. (a) Number of Eq. of NaOH = Number of Eq. of
annual parallax. oxalic acid
Since parsec is the distance corresponding Then 100 ´ 1 = wt.of oxalic acid
to an annual parallax of one second of arc, 1000 63
therefore q is 1.319 second of arc. \ W= 6.3 g
\ Required parallax = 2 × 1.319 second of 36. (b) e/m for He2+ = 2/4 e/m for H+ = 1/1
arc = 2.638 second of arc. +
e/m for He = 1/4 e/m for D+ = 1/2
\ Value of e/m is highest for H+.
SOLUTIONS – MOCK TEST-1 103

hc c
41. (d) E = hn = or l =
l n
37. (c) (a) 3 ´108
Þl= = 3.75 ´10 -8 m
8 ´1015
In nanometer l = 3.75× 10 » 4 × 101
42. (c) P µ n
PH 2 nH 2 22
= =
PCO 2 n CO 2 1
PH2 = 22 × PCO2
(b)
PCO = 1 atm (given)
2
\ PH2 = 22 atm
43. (b) Alkyl groups with at least one hydrogen
atom on the a-carbon atom, attached to an
unsaturated carbon atom, are able to
(c) release electrons in the following way.
H H+
s| p _
–C–C=C –C=C–C
F I II
The delocalisation involves s and p bond
C orbitals (or p orbitals in case of free
radicals) ; thus it is also known as s – p
(d) F F
F conjugation. This type of conjugation is
Non - polar (m = 0) called hyperconjugation.
Non - planar (Tetrahedral) 44. (d) Reacting of aluminium with oxygen is an
exothermic reaction i.e evolution of heat
occurs.
38. (b)
4Al + 3O 2 ¾¾
® 2Al 2 O 3 , DH = - ve
+
45. (c) N 2 = 7 + 7 – 1 = 13 electrons
Electronic Configuration is :
s1s 2 s *1s 2 s 2s 2 s * 2 s 2
2 1
p2 px 2 = p2 p y s2 pz
1
Bond order = [No. of bonding electrons
2
– No. of antibonding electrons]
39. (c) CO2 has the maximum oxidation state of +4.
So it cannot go to a higher oxidation state. 1 1
= (9 - 4) = ´ 5 = 2.5
40. (c) Given : V = 2 L, Molarity = 0.5M, Moles = ? 2 2
BIOLOGY
Molarity = No. of moles of solute
Vol. of solution in L 46. (b) Cholecystokinin (CCK or CCK-PZ) is a
peptide hormone of the gastrointestinal
No.of Moles system responsible for stimulating the
0.5 =
2 digestion of fat and protein. Its presence
\ No. of Moles = 2 × 0.5 = 1.0 causes the release of digestive enzymes and
EBD_7839
104 KVPY-SA
bile from the pancreas and gallbladder, 53. (d) The medulla (a sub-region of the brainstem)
respectively, and also acts as a hunger is a major control centre for the autonomic
suppressant. nervous system. The hypothalamus acts
47. (a) Factor VII or proconvertin is not used up to integrate autonomic functions and
during clotting. During clotting, it is receives autonomic regulatory feedback
changed into convertin which accelerates from the limbic system to do so.
tissue or extrinsic thromboplastin 54. (c) The lysosome is filled with 40 types of acid
formation. Factor XI is activated to factor hydrolases (digestive enzymes). The acidic
XIa by factor XIIa. Calcium is used for both pH £ 5 (4.6 to 5) is due to the action of ATP
intrinsic and extrinsic thromboplastin fueled proton pump in the membrane of the
formation and also in the conversion of lysosome. All the enzymes do not occur in
prothrombin into thrombin. Heparin is the same lysosome. These digestive
present as anticoagulant in the normal blood enzymes are synthesised on RER and
circulation. packed into lysosomes.
48. (c) The discovery of Emerson effect stated 55. (a) In muscle contraction, a calcium ion
that one group of pigments absorbs light facilitates the binding of a cocked myosin
of both shorter and longer wavelengths head to a site on an actin molecule.
(more than 680 nm) and another group of 56. (b) Chronic alcoholic patients are frequently
pigment absorbs light of only shorter deficient in one or more vitamins. The
wavelengths (less than 680 nm). These two deficiencies commonly involve folate,
groups of pigments are known as pigment vitamin B6, thiamine, and vitamin A.
systems or photosystems. 57. (d) Rabies, influenza and AIDS are viral
49. (c) Right cerebrum controls the left side of the diseases. Amoebiasis, ascar iasis and
body so that left arm weakens and never trypanosomiasis are caused by protozoans.
the right arm. Taeniasis, ascariasis and elephantiasis are
Deep tendon reflexes like knee-jerk operate the diseases caused by helminths but
at the level of spinal cord (not brain) and cancer, tuberculosis and tetanus are not
hence loss of neither left nor right knee- related diseases. Tuberculosis and tetanus
jerk reflex can be affected by this stroke. are bacterial diseases while cancer is caused
Right side of the cerebrum sees the left by abnormal cell division.
visual field and vice versa. Each eye sends 58. (a) During the transport of CO2 through the
information to each side of the brain so that blood, bicarbonate ions diffuse out of RBCs
blindness in neither eye is expected as the while chloride ions from plasma, enter the
RBCs to maintain ionic equilibrium. This is
direct result of a stroke on only one side of
called chloride shift.
the brain.
59. (b) Shorter the food chain, more is the
50. (a) Euphotic zone (where sufficient light
availability of food to consumer.
penetrates) is responsible for main bulk of
60. (d) The thoracic duct, which carries lymph from
primary production. Below this zone, there most of the body, drains into the venous
is disphotic zone where no effective plant system. It ends on the left side of the body
production occurs. The lightless region behind the left clavicle, usually draining into
below the disphotic zone is termed as the left subclavian on left brachio-cephalic
aphotic zone. vein.
51. (a) Neurotransmitters cause rapid responses
of the nervous system. Changes in heart PART – II
rate are mediated by the nervous system,
MATHEMATICS
and alterations have to be made rapidly.
52. (a) Meristem is a tissue, which is primarily 61. (b) The given equation is equivalent to
concerned with protoplasmic synthesis 10 k - 1
and formation of new cells by division. (106 – 1)n =
9
Since the meristems have capacity to
synthesise new cells, that is why it is called 10 k - 1
Þn= with k = 6m.
totipotent. 9 (10 6 - 1)
SOLUTIONS – MOCK TEST-1 105

Þ 2pr = r¢a Þ 2p × 5 = 13 × a
1 + 106 + K + 10 ( )
6 m-1
Then n = . 10 p
9 Þ a=
The numerator becomes a multiple of 9 if m = 9. 13
1054 - 1 64. (b)
Thus, the smallest n is n = .
(
9 10 - 1 6
)
62. (a) In the following figure :

A B C

There are 4 bus routes from A to B and 3


by mid point theorem,
routes from B to C. Therefore, there are 4 × 3
= 12 ways to go from A to C. It is round trip PS || AC || QR and, PQ || BD || SR.
so the man will travel back from C to A via B. \ PQRS forms a parallelogram.
It is restricted that man cannot use same bus Let area of DABC = a
routes from C to B and B to A more than once. a
Thus, there are 2 × 3 = 6 routes for return then ar DBPS = (direct result by mid point
journey. Therefore, the required number of 4
ways = 12 × 6 = 72. theorem)
63. (a) l¢ = r¢a ...(1) also, let area of DACD = b.
r¢ = radius of sector b
\ ar DQRD = .
l¢ = length of sector 4
a = angle of sector now, let ar DABD = c
c
\ ar DAPQ =
4
also, let ar DBCD = d
d
\ ar DSCR =
4
a+b+c+d
now, area of ABCD =
2
\ area of region PQRS
Now, cone is made by this sector a + b + c + d æ a b c dö
= -ç + + + ÷
2 è 4 4 4 4ø
Slant height, l = radius of sector a b c d
ar (PQRS) = + + +
4 4 4 4
1 æa +b + c+ dö
ar (PQRS) = ç ÷ø
2è 2
and circumference of cone = l¢ 1
ar (PQRS) = ar (ABCD)
\ 2
l = r 2 + h2 = 52 + 122
65. (c) APB P CQD and transversal pass through
Þ l = 13 cm = r¢ = radius of sector
PQ such that PH, PO, QH and QO bisectors of
also, circumference = l¢ = r¢a
ÐAPQ, ÐBPQ, ÐCQP and ÐPQD respectively.
EBD_7839
106 KVPY-SA

A P B

O
O
H R
M o
45
C D q Q
°-q
Q 45
K
P N m2g
Since AB P CD m1g
Þ ÐBPQ + ÐDQP = 180º (sum of interior
sin (45 + q)
angles on the same side of transversal) Þ = 1.5
sin (45 - q)
1 1
Þ ÐBPQ + ÐDQP = 90º sin 45° cos q + cos 45° sin q 3
2 2 Þ =
sin 45° cos q - cos 45° sin q 2
Q PO and QO bisects ÐBPQand ÐPQD
1
Þ ÐOPQ + ÐPQO = 90º ...(i) Þ tan q =
5
In D OPQ, by angle sum properly Let us now displace the liquids in anticlockwise
Þ ÐOPQ + ÐPQO + ÐPOQ = 180º direction along the circumference of tube through
an angle a.
Þ 90º + ÐPQO = 180 o The net torque
Þ ÐPOQ = 90º (from (i)) t = m2gR sin (45° + q + a) – m1gR sin (45° – q – a)
Since AB P CD and PQ is transversal = VrgR sin (45° + q + a) – 1.5VrgR (45° – q – a)
Þ ÐAPQ = ÐPQD (alternate interior angles) = VrgR sin ( q + 45°) cos a + VrgR cos (45° + q )
1 1 sin a – 1.5 VrgR sin (45° – q) cos a + 1.5 VrgR
Þ ÐAPQ = ÐPQO Þ ÐHPQ = ÐPQO cos(45° – q) sin a
2 2
But these are alternate angles Using eq. (i) we get
Þ HP P QO t = V rgR éëcos ( 45° + q ) sin a
Similarly PO P HQ +1.5 cos ( 45° - q ) sin aùû
Now HP P QO and PO P HQ t = VrgR [cos (45° + q )
ÞPOQH is a parallelogram with one angle as + 1.5 cos (45° – q )] sin a
right angle, ÐPOQ = 90º Þ POQH when a is small (given) sin a » a
is a rectangle t = VrgR [cos (45° + q ) + 1.5 cos (45° – q )]a
Since, t and a are proportional and directed
PHYSICS towards mean position.
\ The motion is simple harmonic.
66. (a) At equilibrium, taking torque of liquids Moment of inertia about O is,
about O I = VrR2 + 1.5 VrR2
æ Torque due to ö æ Torque due to ö I
çè liquid of density r÷ø = çè liquid of density1.5r÷ø T= 2p
C
m2g × QM = m1g × PN (V r ´ 2.5R 2 )
\ m2g R sin (45° + q ) = m1gR sin (45° – q ) = 2p
[cos(45 + q ) + 1.5 cos(45 - q)]V rgR
VrgR sin (45° + q ) = 1.5VrgR sin (45° – q ) ... (i) 1.803R 1
or, T = 2p (using the value tan q = )
g 5
SOLUTIONS – MOCK TEST-1 107

67. (b) As we know, dU = F.dr 70. (d) As we know,


mg sin q
r
ar 3 Acceleration, a =
2
U = ò ar dr = I
3 ...(i) m+ 2
0 r
M c .g. sin qc
mv 2 For cylinder, a c =
As, = ar 2 1 McR
2
r Mc +
m2v2 = mar3 2 R2
1 3 M .g. sin qc 2
or, 2m(KE) = ar ...(ii) = c or, ac = g sin qc
M R 2 3
2 Mc + c 2
Total energy = Potential energy + kinetic 2R
energy For sphere,
Now, from eqn (i) and (ii) M g sin qs Ms g sin qs
Total energy = K.E. + P.E. as = s =
I 2 Ms R 2
ar 3 ar 3 5 3 M s + s2 Ms +
= + = ar r 5 R2
3 2 6 5
68. (c) The distance of bottom of the beaker from or, a s = g sin qs
7
mirror given, ac = as
æ 1ö i.e., 2 g sin qc = 5 g sin qs
= h - d ç1 - ÷ 3 7
è mø
5
æ 1ö g
sin qc 7 15
So it will be at a distance = h - d ç1 - ÷ \ = =
è mø sin qs 2 14
g
from mirror. Now distance between bottom 3
of beaker and image CHEMISTRY
+
æ 1ö æ m - 1ö 71. (a) -CCl 3 ,- NO 2 an d –NH3 are electron
= h + h - d ç1 - ÷ = 2h - d ç .
è mø è m ÷ø withdrawing group, therefore, they are
69. (a) m - directing in nature.
72. (b) For the equation
B 2 H 6 (g) + 3O 2 (g) ¾¾
® B 2 O 3 (g) + 3H 2 O (g)
Eqs. (i) + 3 (ii) + 3 (iii) – (iv)
DH = – 1273 + 3(–286) + 3(44) – 36
= – 1273 – 858 + 132 – 36
= – 2035 kJ/mol
73. (b) 1 lb = 454g (given)
Mass of copper in the statue = 2.0 × 105 ×
454g
As û1 and û2 and the unit vectors and so (in grams) = 908 × 103
No. of moles of copper in the statue
a2 + b2 = c 2 + d 2
Now using Snell's law, we have Given mass 908 ´ 105
= =
At.mass 63.5
m2 sin q
= Now, No. of atom of copper on the statue =
m1 sin q ' No. of moles × NA

2 a / a2 + b2 4 a 908 ´105
or = or = .
´ 6.022 ´1023
1.5 2 2 3 c 63.5
c/ c +d
EBD_7839
108 KVPY-SA
= 8.6 × 1029 77. (a) Molecular weight of average amino acid
Therefore, 8.6 × 1029 no. of atoms of copper = 110
are on the statue. Molecular weight of polypeptide chain
74. (d) v = 2.188 × 106 Z/n = 11,000
Following the above relation, option (d) Number of amino acid residues in
represents the incorrect graph between v
polypeptide chain = 11,000/ 110 = 100
and n.
75. (d) The number of nucleotides in DNA strand
coding polypeptide chain with 100 amino
CH3 CH3
| | acids = 100 × 3 = 300 nucleotides
D+ +
CH3 – CH – CH = CH2 ¾ ¾ ® CH3 – CH – CH – CH2 – D 78. (d) Turgor pressure is the force within the cell
[1, 2-Hydride Shift] that pushes the plasma membrane against
CH3 the cell wall. All the cells except lignified
|+ cells are living cells. So, it is not required
CH3 – C – CH2 – CH2D for maintaining the shape of lignified cell.
H 2O
So, it is not required for maintaining the
CH3 shape of lignified cells.
| 79. (a) Solute potential (ys) and pressure potential
CH3 – C – CH2 – CH2–D (yp) are the two main components that
|
OH determine water potential.
BIOLOGY y = yp + ys
76. (c) Net primary productivity is the amount of From the given options,
dissolved oxygen gained in the sample Water potential of Water potential of
exposed to light. This is calculated by root parenchyma root xylem
subtracting the amount of oxygen in the (a) 10 –195
control from the amount of oxygen in (b) 20 60
bottle kept in light. (c) –20 60
From bottle 2, (d) –20 –60
Net primary productivity (NPP) = Oxygen By convention, the water potential of pure water
in bottle 2 – Oxygen in bottle 1 at standard temperatures, which is not under
= 10 – 9 any pressure, is taken to be zero. Hence, the
= 1 mg/L/h values of water potential in options (a), (b) and
Respiration rate is determined by the (c) cannot be considered.
amount of oxygen lost in the sample kept Water will move from the system having higher
in the dark. This is calculated by water potential to the one having lower water
subtracting the amount of oxygen in the potential. So, according to the water potential
bottle kept in dark from the amount of of root parenchyma and root xylem in option
oxygen in control bottle. (d), transpiration pull will cause water to move
From bottle 3 from root parenchyma to the root xylem.
Rate of respiration (R) = Oxygen in bottle 80. (a) Allele for green body colour = XG
1 – Oxygen in bottle 3 Allele for rosy body colour = Xg
=9–4 Rosy body gcolour female × Green body colour male
= 5 mg/L/h X X
g
XG Y
Gross primary productivity is calculated by X
adding the values of net primary
productivity and the respiration rate. Gross
primary productivity = NPP + R XG Xg XG X g Xg Y Xg Y
Green body Green body Rosy body Rosy body
=1+5 colour colour colour colour (Son)
(Daughter) (Daughter) (Son)
= 6 mg/L/h
MOCK TEST-2

PART – I Þ K2 – K1 = 3 ...(2)
MATHEMATICS From (a) & (b)
K2 = 1; K1 = –2
1. (a) Draw CN ^ AB and join CD and CE. \ K1 × K2 = –2
C 4. (d) Since, the mean of a group of eleven
consecutive natural numbers is m, then
G x + x + 1 + ... + x + 10
F =m
11
A B
D N E 11x + 55 = 11 m Þ x + 5 = m Þ x = m – 5
Since, BC = BD, therefore ÐBCD = ÐBDC. Let n be the mean when next six consecutive
But ÐBCD = ÐBCN + ÐNCD and natural numbers are included in the group
ÐBDC = ÐDCA + ÐA (Since, the exterior then
angle is equal to the sum of the interior non- x + x + 1 + ..... + x + 16
adjacent angle). =n
17
Hence,
16 ´ 17
ÐBCN + ÐNCD = ÐDCA + ÐA ...(1) 17 x + = 17 n 17x + 8 × 17 = 17 n
2
and since CN ^ AB, m – 5 + 8 = n Þ n = m + 3 (\ x = m – 5)
therefore, ÐBCN = ÐA Hence, required percentage change in the
(since both complement ÐNCA) ...(2) mean
From eq. (a) and eq. (b), ÐNCD = ÐDCA
n-m m+ 3-m 300
But ÐCND = ÐCFD = right angle = ´ 100 = ´ 100 = %
m m m
Therefore, ÐCND and ÐCFD are congruent. 5. (a)
Therefore, DN = DF 6. (c) x2 + y2 = 25 and xy = 12
2
Similarly, EN = EG and therefore by adding, æ 12 ö
2
Þ x + ç ÷ = 25 Þ x4 + 144 – 25x2 = 0
DE = DF + EG. èxø
2. (b) 3x = 5y = 75z = k Þ (x2 – 16) (x2 – 9) = 0
Þ 3 = k1/x, 5 = k1/y, 75 = k1/z Þ x2 = 16 and x2 = 9
\ 75 = 52 × 3 Þ x = ± 4 and x = ± 3.
Þ k1/z = k2/y. k1/x Þ k1/z = k2/y + 1/x 1 1 1 1
7. (a) - = -
1 2 1 z (2 x + y ) a b b c
Þ = + Þ = 1. æ 1 1 1 öæ 1 1 1 ö
z y x xy \ ç + - ÷ç + - ÷
è a b c øè b c a ø
3. (c) P(x) = x2010 + K1x2011 + K2
æ 2 1 öæ 2 1 ö 4 1 æ 2 2 ö 1
Q P(a) = 0 = (a)2010 + K1(a)2011 + K2 = ç - ÷ç - ÷ = - ç + ÷+
1 + K1 + K2 = 0 ...(1) è a b ø è c b ø ac b è a c ø b 2
also P(–1) = 4 = (–1)2010 + K1 (–1)2011 + K2 4 2æ2ö 1 4 3
= - ç ÷+ = -
Þ 4 = 1 + K2 – K1 ac b è b ø b 2 ac b 2
EBD_7839
110 KVPY-SA

8. (a) Q(a – b)2 + (b – c)2 + (c – a)2 > 0


11. (c) C
Þ 2(a2 + b2 + c2 – ab – bc – ca) >0
Þ 2 > 2(ab + bc + ca) Þ ab + bc + ca < 1

9. (a) D F G 360°
C
E
r q 2
3
1
A B
A B
I H J D
1 x 2–x 2
2 q
´ pr 2
Area of sector ADB 360 °
Draw EH, FI, GJ parallel to BC. =
Area of sector ACD 360° – q
AJ = GJ = 3 ´ pr 2
360°
Therefore, ΧGAJ = 45°
Now, in DAEH, 1 q
Þ =
EH = 1 + x 2 360° – q
Now, in DBHE and DBIF, Þ q = 120°
EH BH
= ¼ = q ´ 2pr = 2pr
\ ADB
FI BI 360° 3
x +1 4 - x 8
= or x = ¼ = 4pr
Þ ACB
3 4 7 3
Therefore,
1 æ 8ö
ar ( DAEB ) = ´ 5 ´ ç1 + ÷
2 è 7ø
Þ a = 75 h1 r h2 r
1 2
10. (b) Since, x is the product of four consecutive
integers, it is always divisible by 4, i.e., it is
always even. So, 1 + x is always odd.
n=1+x r1 r2
and x = (y –1)(y)(y + 1)(y + 2)
= y (y 2 – 1)(y + 2) ¼ = circumference of base = 2pr1
ADB
= (y3 – y)(y + 2) 2pr r
= y4 +2y3 – y2 – 2y = 2 pr1 Þ r1 =
3 3
Therefore, 1 + x = y4 + 2y3 – y2 – 2y + 1
= y4 + y2 + 1 + 2y3 – 2y2 – 2y 2r
Similarly, r2 =
= (y2 + y –1)2 3
So, 1+ x is a perfect square.
r 2 2 2r
Hence, n is odd as well as a perfect square. h1 = r 2 – r12 = r 2 – =
9 3
SOLUTIONS – MOCK TEST-2 111

13. (c) Const: Join B to O, C to O


5r
Similarly, h 2 = as » » = CD
AB = BC »
3
\ Angle made by them will be equal.
1 2 \ ÐAOB = 30° = ÐBOC = ÐCOD
pr h 2
V1 3 1 1 æ r1 ö æ h1 ö 1 2 2
= =ç ÷ ç ÷ = ´
V2 1 2 è r2 ø è h 2 ø 4 5
pr h
3 2 2

1
=
10

A B
12. (c)
q
a Qa In DAOC,
P AO = OC (Radius) ÐAOC = 60°
\ ÐOAC = ÐOCA = 60°
q In DCOR,
D C ÐOCR = 60°
ÐROC = 30°
Let AP = x \ ÐCRO = 90°
AP 1 similarly, we get ÐBPO = 90°
= AD = nx In quadrilateral PQRO,
AD n
PD = nx - x ÐP + ÐR = 180°
\ PQRO is a cyclic quadrilateral
In DAQP & DCQB \ x + ÐO = 180°
ÐPAQ = ÐBCQ = q (Alternate) Þ x + 30° = 180°
ÐAQP = ÐBQC = a (V.O.A) Þ x = 150°
\ DAQP ~ DCQB (By AA) 14. (c) Draw AF ^ BC.

AQ AP A
=
CQ BC

æQ BC = ADö C B
FO D
AQ x ç = AP + PD÷
Þ = ç ÷
CQ nx è BC = nxø
E
AQ 1 CQ n (i) Since, D is the mid-point of BC. Therefore,
Þ = Þ =
CQ n AQ 1 AD = DB = DC (Since, in a right triangle, the
median from the right vertex to the
CQ CQ + AQ
Þ +1 = n +1Þ = n +1 hypotenuse is equal to half the hypotenuse.)
AQ AQ
Therefore, ÐDAB = ÐDBA
AC AQ 1 But ÐDBA or ÐFBA = ÐFAC
Þ = n +1 Þ =
AQ AC n + 1 (Q they are complementary to ÐFAB)
EBD_7839
112 KVPY-SA

Therefore, ÐDAB = ÐFAC


Mg æ LAs ö
= 1-
But, since AO bisects ÐA. k çè 2M ÷ø
Therefore ÐOAB = ÐOAC and subtraction
Hence, extension of the spring when it is in
gives ÐOAD = ÐOAF
Since, DE || AF (both perpendicular to BC), Mg æ LAs ö
equilibrium is, x0 = ç1 - ÷
Therefore, ÐOAF = ÐOED, and hence k è 2M ø
ÐOAD = ÐOED. 18. (c) As the length of the string of simple
Therefore, AD = DE. pendulum is exactly l m (given), therefore
(ii) As shown above, ÐDAB = ÐDBA or ÐB. the error in length Dl = 0.
Further the possibility of error in measuring
Since, ÐFAB = ÐC (both complement
time is 1s in 40s.
ÐFAC), therefore,
(ÐFAB – ÐDAB) = (ÐC – ÐB) = 2 ÐDAO Dt DT 1
\ = =
or ÐDAE = (ÐC – ÐB). t T 40
15. (d) a > b > c 40
The time period T = = 2 seconds
= c + 10 = a – 15 = k 20
b = 5 c = k – 10
DT 1 DT 1
a = k + 15 \ = Þ = Þ DT = 0.05sec
T 40 2 40
a + b + c = 3k
k + 15 + 5 + k – 10 = 3k l l
We know that T = 2p Þ T 2 = 4 p2
10 = k g g
a = 25
l
b= 5 \ g = 4p 2
T2
c=0
Dg Dl DT
252 + 52 + 02 650 2 \ ´ 100 = ´100 + 2 ´100
mean = = = 216 g l T
3 3 3
PHYSICS Dg æ 1 ö
\ ´100 = 0 + 2 ç ÷ ´100 = 5%
g è 40 ø
16. (d) Quantity of gas in these bulbs is constant
i.e., initial no. of moles in both the bulbs = 19. (b) The vertical component of the tension in the
final number of moles µ1 + µ2 = µ1 + µ2 two ends of the wire, 2Tsin(15°), must equal
the total loaded weight W. The volume of the
PV PV 1.5PV 1.5PV wire is pRw2 L, where Rw is the radius of the
+ = +
R ( 273 ) R ( 273 ) R ( 273) R ( t ) wire, and the total volume of ice is
2 1.5 1.5 p(Ri2 – Rw2) L, where Ri is the radius of the
Þ = + Þ T = 819 K = 546ºC
273 273 T ice-covered wir e. Multiplyin g by the
17. (c) From figure, kx0 + FB = Mg respective densities, the total mass of the
500m-long ice-covered wire is 694.5 + 3531.5
L
kx0 + s Ag = Mg = 4226 kg, having a total weight of 41415N.
2 kx0 The tension in the wire is thus 41415/
(2sin15°) = 80000 N, and dividing by the
sLAg FB
Mg - cross-sectional area of 1.77 × 10–4 m3, the
Þ x0 = 2
stress is 4.52 × 108 N/m2
k
Telegram @unacademyplusdiscounts

Join Us on Telegram for More Such Books

https://telegram.me/unacademyplusdiscounts

Join us from the above link or search ''unacademyplusdiscounts'' in Telegram


SOLUTIONS – MOCK TEST-2 113

20. (c) s = h a k Bb c g or MT–3K–4 d


24. (b) B catches C in time t then t =
= [ML2T–1]a [ML2T–2k–1]b[LT–1]g u - 10
= Ma + b L2a + 2b + g T–a – 2b – g × k–b Separation by this time has increased by ‘d’
\ b = 4, a + b = 1, 2a + 2b + g = 0 between A and C, hence
and – a –2b – g = – 3. d
after solving, we get (10 - 5) ´ = d , u =15 m/s
(u - 10)
a = – 3, b = 4 and g = – 2
25. (a) Ptop V¢ = Pbottom V¢
v 8 Pbottom = (Ptop + rgh)
21. (c) =
x 100
As Pbottom > Ptop; \ V > V¢.
2
or v= x ¶V ¶
25 26. (b) Ex = – = – (– x 2 + y 2 ) = 2 x
¶x ¶x
Acceleration,
¶V -¶
dv 2 æ dx ö 2 æ 2 ö 4 and E y = - = ( - x 2 + y 2 ) = -2 y
a= = ç ÷ = ´ç x÷ = x ¶y ¶y
dt 25 è dt ø 25 è 25 ø 625
r
x = 100 m, a = 0.64 m/s2. Now E = E x iˆ + E y ˆj = 2 xiˆ - 2 yjˆ
r
Thus, a - x graph is a straight line with maximum 27. (a) White light is broken up into several
value 0.64 m/s2. component colours due to refraction
22. (b) Circular part in the centre of retina is called phenomenon.
yellow spot.
m P
28. (a) PV = RT ; \ = Cm
L mg M T
23. (b) We know that Y = ´
2 l
D
p slope of B m 3m
4 or = B= =3
slope of A mA m
4mgL 4 ´ 1´ 9.8 ´ 2
ÞY= = l
29. (c) Resistance of wire (R) = r
( ) ´ ( 0.8 ´10 )
2 2
pD l p 0.4 ´10 -3 -3
A
If wire is bent in the middle then
= 2.0 × 1011 N/m2
l
DY 2DD Dl l¢ = , A¢ = 2 A
Now = + 2
Y D l
l
[\ the value of m, g and L are exact] r
l¢ 2
\ New resistance, R¢ = r =
0.01 0.05 A¢ 2A
= 2´ + = 2 × 0.025 + 0.0625
0.4 0.8
rl R
= 0.05 + 0.0625 = 0.1125 = = .
4A 4
Þ DY = 2 × 1011 × 0.1125
= 0.225 × 1011 30. (a) F = 20t – 5t2
= 2.0 × 1011 N/m2 FR dw
Note : We can also take the value of Y from \ a= = 4t - t 2 Þ = 4t - t 2
I dt
options given without calculating it as it is same
in all options. w t
\ Y = (2 ± 0.2) × 1011 N/m2 Þ ( 2
ò d w = ò 4t - t dt)
0 0
EBD_7839
114 KVPY-SA

\ 0.75 g of sample contains


x
w 14 ´ 20 ´ 10 - 3
= ´ 100 = 37.33%
0.75
u
37. (c) CH3
O x CH3
¾®
CH3

t3 CH 3
Þ w = 2t 2 - (as w = 0 at t = 0, 6s) (chair form)
Gauche conformation
3
38. (d) Relation between wavelength and energy
q 6æ hc
2 t3 ö (E) of a photon is, E =
ò d q = ò çç 2t - ÷ dt
3 ÷ø
l
0 0è Graph (d) correctly represents the above
relation.
36 39. (d)
Þ q = 36 rad Þ n = <6
2p
H
CHEMISTRY CH2 = C = C H – C º C – H CH3 – C º N
31. (b) If intermolecular attraction disappears then ­ Cl ­ ­
sp sp sp
pressure of real gas will increase.
(v) (x) (y)
32. (d) Conjugate acid of HF2- is H2F2.
HF2- + H + ¾¾
® H 2F2 40. (a) Quantum numbers n = 3, l = 2, m = +2
Base Conjugate acid represent an orbital with
33. (c) A is metal because the oxide of metal 1
s=± (3d xy or 3d 2 2 )
dissolves in water to form alkaline solution. 2 x -y
The aqueous solution of oxide of element which is possible only for one orbital.
A reacts with aqueous solution of oxide of 41. (a) KO2 reacts with CO2 to give oxygen,
element B, thus aqueous solution of oxide
therefore, it is used in oxygen cylinders in
of element B will be acidic because reaction
space and submarine.
between two alkalies is not possible.
Therefore element B will be non-metal as 4KO 2 + 2CO2 ® 2K 2 CO3 + 3O2
oxides of non-metals are acidic in nature. 42. (b) It has chain of 4C atoms and double bond
34. (c) The order of strength of nucleophiles is given preference over Cl atom.
follows the order :
43. (a) In trigonal bipyramidal geometry, lone pair
CN– > I– > C6H5O– > OH– > Br– > Cl–
electrons cannot occupy axial positions.
35. (b) Higher the K sp, more soluble is that
compound in H2O. Hence, trigonal planar geometry is not
possible.
\ Ksp of FeS (11 × 10–20) is highest.
So it is more soluble and has maximum 44. (c) Cyclohexene + KMnO4 (cold & alkaline)
solubility in H2O. ¾
¾® cis-Hexanediol
36. (a) 10 mL, 1 M H2SO4 = 20 mL, 1 M NH3
45. (b) O -2 (17) = KK s 2s 2s*2 s 2 s2 px2 p2 py2 = p 2 pz2
Q Wt of N in one mole NH3 = 14
\ Wt. of nitrogen in 20 × 10– 3 mol NH3 p*2 py2 = p*2 p1z
= 20 × 10– 3 × 14
O2- has one unpaired electron.
SOLUTIONS – MOCK TEST-2 115

BIOLOGY 53. (b) Phloem parenchyma are unspecialised


cells, found in the phloem. They are absent
46. (b) Despite the fact that the atmosphere has
in the monocots as they have abundant
79% nitrogen gas, plants cannot use the
companion cells.
element in that form and frequently suffer
54. (c) Distilled water has the highest water
from nitrogen deficiency. potential. The value of water potential is
47. (d) Phospholipids are composed of phosphate always negative or less than zero. Water
group and one or more fatty acids. They potential of a solution is determined by
have hydrophilic (polar) phosphate group using pure water as the standard of
and long hydrophobic (non-polar) reference, which has zero water potential at
hydrocarbon ‘tails’. The phospholipids normal temperature and pressure. The
readily form membrane like structure in presence of solute particles reduces free
water. energy of the water. Hence, it decreases the
48. (b) Allergens are non-infectious foreign water potential in negative value. So, water
substances that causes allergic reaction. potential of a solution is always less than
The common allergens are dust, pollen, zero.
mould spores, fabrics, lipsticks, nail paints, 55. (a) The number of essential amino acids in
fur, heat, bacteria, etc. Allergy involves humans is 9. They are lysine, isoleucine,
mainly the secretion of IgE antibodies and leucine, methionine, phenylalanine,
histamines. Allergy causes marked dilation tryptophan, threonine, histidine and valine.
of all the peripheral blood vessels. They must be ingested in food for the
49. (b) In birds, forelimbs are modified as wings survival, as they are not synthesised in the
for flying. Therefore, the forelimbs are not body.
found in birds. 56. (c) Fumaric acid is not a product of anaerobic
50. (c) Blood serum does not coagulate because it respiration. It is an intermediate compound
does not contain white or red blood cells or a of aerobic respiration (Kreb's cycle).
clotting factor. It is the blood plasma without 57. (b) CT-cells are cytotoxic T-cells which are also
the fibrinogens. It includes all proteins not known as killer T-cells. These cells are
used in blood clotting (coagulation) and all involved in killing of the cancerous cells.
the electrolytes, antibodies, antigens, 58. (b) Kwashiorkar is caused due to the deficiency
of proteins. Meat, lentils, milk and eggs are
hormones, and any extra substances (such
rich sources of proteins.
as drugs and microorganisms).
Scurvy is caused due to deficiency of
51. (a) Cancer cells are HeLa cells. HeLa cells have
vitamin C whose sources are citrus fruit,
been used to test the effects of radiation,
tomatoes, peppers, leafy green vegetables.
cosmetics, toxins, and other chemicals on
Deficiency of vitamin D (sources - milk, egg
human cells. They have been instrumental yolk and liver) leads to rickets. Anaemia is
in gene mapping and studying human caused due to deficiency of folic acid
diseases, especially cancer. (sources - yeast, liver, fish, green
52 (a) Fermentation is the incomplete oxidation vegetables) or vitamin B12 (sources - liver,
of glucose under anaerobic conditions by eggs, meat, fish) or iron (egg yolk, spinach).
sets of reactions where pyruvic acid is 59. (d) Ti plasmid found in Agrobacterium
converted to CO2 and ethanol (alcoholic tumefaciens, produces crown gall (tumour)
fermentation) or lactic acid (lactic acid in a large number of dicot species. A.
fermentation). In fermentation, there is a tumefaciens is a gram negative soil
net gain of only 2 ATP molecules for each bacterium that infects a wide range of
molecule of glucose degraded to pyruvic plants and causes crown galls.
acid.
EBD_7839
116 KVPY-SA

60. (a) The correct sequence is : ABCD ® || gram


1. Phosphorus input from sewage and DABK ~ DMDK (By AA)
agricultural runoff increases. DABK = DMDK = g
2. Algal blooms occur. DAKB = DMKD = b
3. Respiratory demand from decomposers
AK AB
increases. \ =
MK MD
4. Oxygen levels drop in deeper water.
6 AB AB 3
PART – II Þ = Þ =
4 MD MD 2
MATHEMATICS Let AB = 3x, MD = 2x
\ AB = CD = MD + CM
61. (c) We have
Þ CM = x
(i) 1010 < n < 1011 DAMD ~ DLMC
(ii) Sum of the digits of n = 2
Clearly, nmin = 10000000001 (1 followed by 9 AM MD 10 2x
Þ = Þ = Þ LM = 5
zeros and finally 1) LM MC LM x
Obviously, we can form 10 such numbers
by shifting 1 by one place from right to left 64. (d)
again and again.
Again, there is another possibility for n,
i.e., n = 20000000000
So, finally number of different values of
n = 10 + 1 = 11
62. (c) I. Number of 3 letter words (repetition not
allowed)
= 6 × 5 × 4 = 120
(as first place can be filled in 6 different
ways, second place can be filled in 5
different ways and third place can be filled h1 ® height of cone
in 4 different ways) h2 ® depth of water
II. Number of 3 letter words (repetition is r1 ® radius of cone
allowed) r2 ® radius of water
= 6 × 6 × 6 = 216 Q DABC ~ DADE (AA)
(as each of the place can be filled in 6
different ways) h1 r2
=
h r1
63. (c)
p 2 V
Volume of water = r 2 h2 =
3 27
p 2 1 p
r 2 h2 = ´ ´ r12 h1
3 27 3
2
æ r2 ö h1
çè r ÷ø h2 = 27
1
SOLUTIONS – MOCK TEST-2 117

2 The weight of system


æ h2 ö h1
çè h ÷ø h2 = 27 4 3 4 é4 3 ù
1
= pR (3r) g + pR 3rg = 4 ê pR rg ú
3 3 ë3 û
3
æ h2 ö 1
çè h ÷ø = 27 This is to be balanced by the buoyant force.
1 This can be possible only when the light
h2 1 sphere is completely submerged. In this
=
h1 3 way the buoyant force

65. (c) P D Q éæ 4 ö ù é4 ù
l B = êç pR3 ÷ ´ 2ú ´ (2r) ´ g = 4 ê pR 3rg ú
ëè 3 ø û ë3 û
Now considering the equilibrium of the
m heavy sphere
A B
Fs + B = W
given l || M
\ Fs = W – B
\ area of rectangle ABCD = area of
parallelogram ABQP as they both lie on same 4 4
base AB and between two parallel lines. \ Kx = pR3 (3r) g - pR 3 (2r) g
3 3
In DPDA and DQCB,
PA > DA and QB > CB
as PA and QB are hypotenuse, 4 R3rg
\ x= p
also AB = PQ = DC. 3 K
now,
67. (d) When the ball is thrown upwards, at the
Perimeter of rectangle point of throw (O) the linear momentum is
Perimeter of parallelogram in upwards direction (and has a maximum
AB + CB + CD + AD 2 AB + 2 DA value) and the position is zero. As the time
=
AB + BQ + PQ + PA = 2 AB + 2PA passes, the ball moves upwards and its
Q 2PA > 2DA momentum goes on decreasing and the
position becomes positive. The momentum
Perimeter of rectangle
\ Perimeter of parallelogram < 1 becomes zero at the topmost point (A).

Momentum
PHYSICS
66. (a) Consider the equilibrium of the system of
both spheres and the spring. 0 Position
A

As the time increases, the ball starts moving


down with an increasing linear momentum
in the downward direction (negative) and
reaches back to its original position.
These characteristics are represented by
graph (d).
EBD_7839
118 KVPY-SA

68. (b) Molar mass of N2 gas in M = 28g


a1 a2
Number of moles of N2, n0 = (m/M) = 1
B¢ C¢
One-third molecules are dissociated into atoms.
v2 0
No. of moles of diatomic N2 = 2/3
C¢ C¢ A¢
No. of moles of monatomic nitrogen = 2 × (1/3)
0 v2 u
For monatomic gas Cv1 = (3/2) R
Wood Iron
For diatomic gas Cv2 = (5/2) R 4cm 2cm
For A ® B ® C, v12 – u2 = 2a1
Average molar specific heat at constant volume
and 02 – v12 = 2a2 …(1)
n1Cv1 + n2Cv2
Cn = Adding, we get –u2 = 2(4a1 + a2)
n1 + n2
For A¢ ® B¢ ® C¢, v22 – u2 = 2a2 …(2)
æ2öæ3ö æ2öæ5ö
çè ÷ø çè ÷ø R + çè ÷ø çè ÷ø R and 02 2
– v2 = 2a1 …(3)
3 2 3 2
= = 2R Cp – Cn = R
2 2 Adding, we get, –u2 – 2(2a1 + 2a2)
+
3 3
Equating (1) and (2) and solving, we get
Þ Cp = 3R 4a1 + a2 – 2a1 + 2a2 Þ a2 = 2a1
Cp 70. (c) Intensity of the source at the cross section A
g= = 1.5
Cv
P 1.25 ´ 103
Piston is displaced back to its initial I= =
4 p (50 r ) 2 4 p (50 r ) 2
position, and during the adiabatic
compression, volume of the gas mixture is Power absorbed by the end
halved. A = 80% of {I× pr2}
For adiabatic compression, T1V1g -1 = T2V2g-1
80 æ 1.25 ´ 103 ö
= ´ç 2
÷ (pr ) = 0.1 J/sec
Where T1 = T0 ; V1 = V0; 100 è p ´ 104 r 2 ø
T2 = ?; V2 = V0/2
Using, Wien’s displacement law, to
g-1
æV ö determine the temperature of the end B
T0V0g-1 = T2 ç 0 ÷ Þ T2 = T0 2
è2ø lm TB = 0.003
Increase in internal energy, 0.003
TB = = 300 K
DU = (n1 + n2 ) Cn(T2 – T0) 100000 ´ 10-10
4 8 æT ö
= ´ 0.2 R (T0 2 - T0 ) = RT0 ( 2 - 1) æ dQ ö 2 dT
3 3 çè ÷ø = - ç ÷ (pr )
dt è TA ø dx
69. (b) Let a1 and a2 be the retardations offered to
the bullet by wood and iron respectively. TB
1
0.1TA dx
ò 4 ´ 10-4 = - ò TdT \ T = 500 K
A
0 TA
SOLUTIONS – MOCK TEST-2 119

CHEMISTRY 75. (d) Order of Acidic strength µ stability of


conjugate base.
71. (a) In NH3 the atomic dipole (orbital dipole due
to lone pair) and bond dipole are in the same
direction, whereas in NF3 these are in NH2 < OH < OH
opposite directions so in the former case, I II III
they are added up whereas in the latter case
net result is reduction of dipole moment. It – – -
NH < O < O
has been shown in the following figure :
Resonance stabilized
phenoxide ion

BIOLOGY
N N 76. (a) The amount of DNA is same in all the cells
H of an organism so non-diving liver cell and
H F F
skin cell will have the same amount of
H
F DNA. In cell cycle, DNA replication occurs
m = 1.5 D m = 0.2 D during S phase of interphase. So after
interphase, the amount of DNA in actively
72. (c) Heat of combustion dividing human skin cell will be: 6 ×2 = 12
1 pg.
µ µ 77. (b) As the relative humidity of the air
Stability of molecule
No. of ‘C’ atoms in molecule surrounding the plant rises the
73. (d) Let mole fraction of O2 is x transpiration rate falls. Increased
40 = 32 × x + 80 (1 – x) movement of the air around a plant will
or x = 5 / 6 result in a higher transpiration rate. So,
plant B kept in 45% relative humidity with
5 1 blowing wind will dry fastest followed by
a : b = x : (1 – x) = :
6 6 plant A kept in 45% relative humidity and
When ratio is reversed plant C kept in 95% relative humidity.
78. (b) Bacterial plasmids do not replicate in yeast
1 5
M mixture = 32 ´ + 80 ´ = 72 because plasmids that replicate in bacteria
6 6
have ori sequences that bind bacterial DNA
74. (b) Given : P = 10 atm, polymerase, while plasmids that replicate
Total number of moles; nA + nB + nC =10 in yeast have distinct ori sequences that
PA = 3 atm, PB = 1 atm bind yeast DNA polymerase. So, they lost
nA from the cells over a period of time. This
\ PA = xA × P(total) = ´10 problem can be overcome by inserting
n A + n B + nC
yeast origin of replication (ori) in bacterial
nA plasmids.
= ´ 10 = 3 ; nA = 3 79. (a) The cell membrane is mainly composed of
10
lipids and proteins. The major lipids are
Similarly, PB = xB × P(total)
So, nB = 1 phospholipids that are arranged in a
\ nC = 10 – (nA + nB) = 10 – 4 = 6 bilayer. Also, the lipids are arranged within
Mass of C = 6 × 2 = 12 g the membrane with the polar hydrophilic
head towards the outer sides and the
EBD_7839
120 KVPY-SA

hydrophobic tails towards the inner part. concluded that P is a C3 plant and Q is a C4
When a red blood corpuscle (RBC) was plant.
kept in a solution and treated so that it Higher carbon dioxide concentration
became inside – out, the polarity of the increases rate of photosynthesis. In C 3
phosphlipid bilayer will remain the same. plants increased carbon dioxide
80. (b) Quantum yield of photosynthetic carbon concentrations stimulate photosynthesis
fixation in C3 and C4 plant is a function of over a wide concentration range. By
leaf temperature. In C 3 plants, contrast, with their adaptations, C4 plants
photorespiration increases with temperature are not as limited by carbon dioxide, and
leading to decrease in quantum yield. On under elevated carbon dioxide levels, the
the contrary, in C4 plants, photorespiration growth of C4 plants did not increase as
is low thus, quantum yield does not show a much as C3 plants. From the graph showing
temperature dependence. Hence, from the effect of carbon oxide concentration of
graph showing effect of leaf temperature on photosynthesis of plants R and S, it can
photosynthesis of plants P and Q, it can be be concluded that R is C4 plant and S is C3
plant.
MOCK TEST-3
PART-I C
MATHEMATICS 2a
P a N
1. (b) As a 1st step tie all the participants from same
nation this can be done in (5!) for each a
ab M
country, now we have 3 units that can be
arranged in a circle in (2!) ways hence total a 2b
B
number of ways is (2!)(5!)(5!)(5!). A
2. (d) Since HCF of the two numbers is 72 = 2332. Q
Hence minimum of a and d is 3 while minimum
of b and e is 2. Since we need to find the In triangles CPA and BMA,
minimum value of LCM so c has to be ÐPCA = 45° = ÐMBA; and PC = MB,
minimum so c = 1, now next minimum value 2 CA = BA.
is already taken by either b or e so next So, DCPA @ DBMA by SAS congruence rule.
minimum value that is 3 that is taken by either Hence, ÐPAC = ÐBAM = a.
a or b, next smaller value 4 will be taken by f Consequently, ÐMAP = ÐBAC = 90°,
hence f = 4, whence PAMC is a cyclic quadrilateral.
Therefore, ÐPMC = ÐPAC = a.
Then 5 and 6 will be taken by either b or e
Again, PN2 = PC2 + CN2 = BM2 + CN2 = MN2.
and a or d respectively so one acceptable
So, PN = MN, giving ÐNPM = ÐNMP = a,
combination is a = 3, b = 2, c = 1, d = 6, e = 5,
in DPMN.
f = 4, hence required LCM = 26355471
Hence, ÐPNC = 2a.
æ 1ö æ 1ö æ 1ö 7 Likewise, ÐQMB = 2b, where b = ÐCAN.
3. (c) çè a + ÷ø çè b + ÷ø çè c + ÷ø = ´ 4 ´ 1 Also, DNCP @ DQBM, as CP = BM, NC = BQ
b c a 3
and ÐNCP = 90° = ÐQBM.
Þ abc + (a + b + c)
Therefore, ÐCPN = ÐBMQ = 2b,
æ 1 1 1ö 1 28 hence 2a+2b= 90° implies a + b = 45°.
+ çè + + ÷ø + =
Finally, ÐMAN = 90° – (a + b) = 45°
a b c abc 3
5. (a) Let AO and OB be the bisectors of ÐA and
1 æ 1ö ÐB respectively.
Þ abc + + ça + ÷
abc è bø
1
Then Ð1 = ÐA
æ 1ö æ 1 ö 28 2
+ çb + ÷ + çc + ÷ =
è cø è aø 3
1
Ð2 = ÐB
1 æ 7ö 28 2
Þ abc + + ç ÷ + (4) + (1) =
abc è 3 ø 3 D C
1
Þ abc + = 2 \ abc = 1.
abc O
4. (a) Draw CP perpendicular to CB and BQ
perpendicular to CB such that CP = BM, 1 2
BQ = CN. Join PA, PM, PN, QA, QM, QN. A B
EBD_7839
122 KVPY-SA

In DAOB, Ð1 + Ð2 + ÐAOB = 180° Tn = a + (n – 1)d


[Angle sum property of a D] Þ 467 = 3 + (n – 1)8
ÐAOB = 180° – (Ð1 + Ð2) Þ n = 59;
Now, sum of angles of quadrilateral is 360° The maximum number of terms in
ÐA + ÐB + ÐC + ÐD = 360° S = (59 + 1)/2 = 30;
9. (c) Since ‘a’ seats from both the end is reserved
1 1 1
( ÐA + ÐB) = ÐC + ÐD = 180° for executives, they can be arranged in (2a!)
2 2 2 ways. In between we will have (b + c + 1) seats.
1 1 All the managers are together can be
Þ ( ÐC + ÐD ) = 180° - ( ÐA + ÐB) arranged in (b!) ways, now out of (c+1) seats
2 2
senior managers and director can be seated
1 in such a way that director has senior
Þ ( ÐC + ÐD ) = ÐAOB manager in both the sides. i.e (SM, D, SM)
2
this can be done in (cC2)(2!) = cP2 ways.
ÐC + ÐD = 2 ÐAOB Þ k = 2
Now bunch of managers, group of director
The value of k – 1 is 2 – 1 = 1 (which has three members) and remaining (c
6. (c) As per the given condition N must be – 2) senior managers can be arranged in c!.
divisible by 12, so N must be in the form of Hence total number of ways is
12k where k is not divisible by 3 (As N is not {(2a)!}(b!){c!}{cP2}.
divisible by 9). Hence N/18 cannot be an
10. (b) Consider a two digit number ‘ab’ = 10a + b,
integer.
when digits are reversed it becomes
7. (b) Let 1st consider the prime numbers as their ‘ba’= 10b + a, and the difference between
HCF will always be 1, Elements must be these two numbers is 18
{1, 2, 3, 5, 7, 11, 13, 17, 19) in this case for any
(10b + a) – (10a + b) = 9(b – a) =18
two elements HCF must be 1. Now we will
or b – a = 2
consider each and every left over numbers.
So the values of a and b is (1, 3), (2, 4),
4 – we can include that as then HCF of 2 and
4 will become 2. (3, 5), (4, 6), (5, 7), (6, 8) and (7, 9)
6 – we can not include this as then HCF of Hence, other than 13, there are 6 more such
3 and 6 will become 3 numbers.
8 – we can not include this as then HCF of 11. (b) Let coordinates of D be (x, y, z)
4 and 8 will become 4. Co-ordinates of centroid is (1, 1, 1), and of A,
9 – we can not include this as then HCF of is (4, 7, 8)
3 and 9 will become 3 Centroid divides median is 2 : 1 ratio
Similarly we cant include 10, 12, 14, 16 and AO
18. So, = 2 :1 .
OD
Hence maximum number of elements is 10.
A(4, 7, 8)
8. (d) First we observe the given set then we will
find that the sum of the first and the last term 2
is 470, the sum of the second and the second (
O 1, 1
last term is 470, and so on. ,1
)
Hence if we take only one of those two terms 1
B C
will be in Set S. i.e., we can have maximum D(x, y, z)
half of the consecutive terms of T in S. Also,
the terms of this sequence are in A.P, 2 ´ x + 1´ 4
where a = 3, d = 8 For x : 1 = Þ x = – 1/2.
1+ 2
SOLUTIONS – MOCK TEST-3 123

2y + 1 ´ 7 14. (b) The following are the number of possible


For y : 1 = Þ y = – 2. choices:
1+ 2
52 C 35
18 × C2 (18 families having atmost
2 ´ z + 1 ´ –8 2 children and 2 selected from other type of
For z : 1 = Þ z = + 11/2.
3 families)
52 C 35
\ Co-ordinates of D are (–1/2, –2, 11/2). 19 × C1 (19 families having atmost
12. (d) Given 2x + y = 10 2 children and 1 selected from other type of
on adding y both sides, we get families)
52C (All selected 20 families having atmost
2x + y + y = 10 + y 20
2 children). Hence, the total number of
Þ 2(x + y) = 10 + y
possible choices is :
Þ x + y = 5 + y/2
= 52C18 × 35C2 + 52C19 × 35C1 + 52C20
Now, (x + y)max when y is maximum & maxi-
mum value of y will be 10. ( y = 10 – 2x) 15. (d) 3x/y = t
So (x + y)max = 5 + 5 = 10 t
3t - = 24 Þ 8t = 3 × 24
& (x + y)min when y = 0 3
\ minimum value of x + y = 5 t=9
So, sum of (x + y)max & (x + y)min = 15 So, 3x/y = t Þ 3x/y = 9
3x/y = 32 Þ x = 2y
13. (a)
x + y 3y
\ = =3
x- y y
1
PHYSICS
16. (a) Weight of stone in air = (10.0 ± 0.1) kg
Loss of weight in water = (10.0 ± 0.1) – (5.0
± 0.1) = (5 ± 0.2) kg
When two quantities are subtracted (or
added), the absolute errors are added up.
Now, specific gravity
l| |m weight in air (10.0 ± 0.1) kg
r = 1 cm = =
loss of weight in water (5 ± 0.2) kg
O1O2 = O2O3 = O3O1 = 2 cm
\ Maximum percentage error in specific
\ O1O2O3 ® equilateral triangle
gravity =
Let O3P is height of triangle O1O2O3
0.1 0.2
´ 100 + ´ 100 = 1% + 4% = 5% .
3 10.0 5.0
O3P = ´ 2 O3P = 3 cm
2 17. (d) In (a), at the same time particle has two
So, minimum distance b/w positions which is not possible. In (b),
Parallel line D = O1M + O3P + O3K particle has two velocities at the same time.
In (c), speed is negative which is not
=1+ 3 + 1= 2 + 3 possible.
EBD_7839
124 KVPY-SA

moving to a large height the gravitional


dy - dh
18. (a) Given, = forced decreases quite rapidly
dt dt
æ 1ö
S ç as F µ 2 ÷ . Under such a rapidly
è r ø
h
decreasing variable force, the path of
projectile become elliptical.
20. (d) The heat available on water
y = 30 × 1 × (80 – 0)
= 2400 cal.
To melt the ice, it need
The apparent distance of source from fish,
= mL = 50 × 80
x = µh + y
= 4000 cal
dx dh dy So if m amount of ice is melt, then
or =m +
dt dt dt m × 80 = 2400
dh dh or m = 30 g
or v= m - The ice left = 20 g
dt dt
21. (b) The equivalent system is shown in figure.
dh
= (m - 1) … (i) k
dt
m m
We know that
k
df v
= mm m
f c The reduced mass, m = =
m+m 2
df and k2 = 2k.
or v =c … (ii)
f
k 2k k
Now from equations (i) and (ii), we have \ w= = =2
m m/2 m
df (m - 1) dh
= . 0.1
f c dt = 2 = 2 rad/s
0.1
19. (c) The normal force exerted by the astronaut
on orbiting space station is zero (until the w 2 1
or f = = = Hz
astronaut exerts some muscular force). 2p 2p p
Therefore the apparent weight of astronaut
in an orbiting space station is zero. dE –a
22. (a) At T = Tn; =0 or Tn =
Astronaut is called in a state of dT b
weightlessness. This is because astronaut
as well as space -ship are freely falling –10
= = 200ºC and, Ti = 2Tn = 400 ºC
bodies. -1/ 20
Upto ordinary heights, the change in the
distance of a projectle from the centre of 23. (d) E = A sT( )
4

earth is negligible compared to the radius


of earth. Hence the projectile moved under \ lnE = ln ( As ) + 4lnT
a nearly uniform gravitational force and the
It represents straight line.
path is parabolic. But for the projection
SOLUTIONS – MOCK TEST-3 125

24. (b) Point A shall record zero magnetic field (due Also, V2 = V1 + Ax, V1 = Ax
to a-particle) when the a-particle is at posi-
tion P and Q as shown in figure. The time V1
\ x=
taken by a-particle to go from P to Q is A

1 2p w
P 1 P1 A V1 1
t= \ Energy = ´ = PV
1 1
3 w 60°
2 2 A 4
O 2R A
60° \ (a) is correct
2p
or w= Now
3t Q

æ kx ö
25. (d) Applying combined gas law W = ò PdV = ò ç P1 + ÷ dV
è Aø
PV
1 1 = P2V2
T1 T2 kx
= ò P1dV + ò A dV
If V2 = 2 V1 and T2 = 3T1 then
kx
1 1 = P2 ´ 2V1 Þ P = 2 P
PV \ W = ò P1dV + ò ´ (dx ) A
T1 3T1
1
3
2 A

Now change in internal energy kx 2


\ W = P1 (V2 – V1 ) +
f f 2
DU = [nR (T2 – T1)] = [P V – P V ]
2 2 2 2 1 1
é PV
1 1 P2V2 4 P1 ù
For monoatomic gas f = 3 ê Here on applying T = T we get P2 = 3 ú
ê 1 2 ú
3 é3 ù ê 2V1 ú
DU =
2 êë 2 P1 ´ 2V1 – PV
1 1 ú = 3P1V1
û êë and V2 = V1 + Ax Þ x = A [QV2 = 3V1 ] úû
\ (b) is the correct option.
Now assuming that the pressure on the piston 1 P1 A 2V1 7
\ W = 2P1V1 + ´ ´ = PV
1 1
on the right hand side (not considering the 2 3 A 3
affect of spring) remains the same (c) C is correct option
throughout the motion of the piston then,
Heat supplied
kx kx Q = W + DU
Pressure of gas = P1 + Þ P2 = P1 +
A A
7 PV
1 1 + 3 ( P V – PV )
where k is spring constant and A = area of = 2 2 1 1
3 2
piston
1 2 7 PV
1 1 3 é4 ù
Energy stored = kx = + ê P1 3V1 – PV
1 1ú
2 3 2 ë3 û
kx
P2 = P1 + 41
A = PV
1 1
6
3 kx 26. (b) Since ervey element of the hemispherical
P1 = P1 +
2 A shell is at a distance R from the centre of
curvature, therefore gravitational potential
P1 kx P1 A at its centre = – GM/R. It is not equal to Gm/
= \ kx =
2 A 2 R. Hence, statement I is incorrect.
EBD_7839
126 KVPY-SA

Gravitational field strength at a point, lying v


on the axis of a thin uniform circular ring of
radius R, can be calculated by considering
two equal elemental lengths of the ring such ac
p
that these are lengths lying on the ends of a
diameter and thus the expression given in a at
option (b) is obtained. Hence, statement II
is correct.
Newton's law of gravitation is applicable to
P is the point at which cyclist applies brakes. At
only those bodies which have spherically
this point, tangential acceleration a t, being
symmetric distribution of mass. For example,
if we consider two solid hemispheres of negative, will act opposite to vr .
equal radii; one made of wood and the other
Total acceleration, a = a 2c + a 2t
made of iron. These two hemispheres a
rejoined together to form a shape of complete
sphere, we cannot apply Newton's
gravitational law to this sphere. So, statement a = (0.7) 2 + (0.5)2 ms-2 = 0.86ms -2
III is correct. a c 0.7
27. (b) Angular momentum about the point of tan q = = = 1.4 \ q = 54°28 ¢
a t 0.5
contact with the surface includes the angular
momentum about the centre. Due to friction
linear momentum is conserved. 4
30. (d) Given, µ = , h = 15 cm
4 3 4 3 3
28. (b) n pa = pb
3 3

b3
\ n=
a3
W = T DA = S[n ´ 4pa 2 - 4pb2 ]

1 2 S éb ù
3
2 2
or mv = ê 3 ´ 4pa - 4pb ú
2 ëa û

1 æ4 ö é b3 ù
or r ´ ç pb3 ÷ v 2 = S ê 3 ´ 4pa 2 - 4pb 2 ú 1 R 3
2 è3 ø ëa û Þ sin C = = =
m 2
R +h 2 4
6S æ 1 1 ö Þ 16 R2 = 9 R2 + 9 h2
\ v= ç - ÷
r è a bø or, 7R2 = 9 h2
29. (c) Speed, v = 27 km/h 3 3
or, R= h= ´ 15cm
5 -1 -1 7 7
= 27 ´ ms = 7.5ms
18 CHEMISTRY
v2
centripetal acceleration, a c = 31. (c) If the structure of water is linear, then µ = 0,
r hence it will be non-polar and thus the
(7.5) 2 -2 solubility of polar compound in water
or a c = ms = 0.7ms -2
80 cannot be explained.
SOLUTIONS – MOCK TEST-3 127

Br Q 28 g of C 2 H 4 undergo complete
–HBr combustion by 96 g of O2.
32. (b) ¾¾¾®
(alc.) KOH
\ 2800 g of C 2 H4 undergo complete
Br Br combustion by 9600 g or 9.6 kg of O2.
1,2-dibromo-
cyclohexane
39. (a) Structure of XeF4,
–HBr
¾ ¾¾®
(alc.) KOH
Br
1,3-cyclo-
hexadiene 40. (a) Fire extinguishers contain a mixture of
Na2CO3, NaHCO3 and H2SO4.
33. (a) 11 Na – 2, 8, 1
41. (d) When Al is added to KOH solution,
19K – 2, 8, 8, 1 hydrogen gas is evolved.
Na an d K have same electronic 2KOH + 2Al + 2H 2 O ® 2KAlO 2 + 3H 2 ­
configuration of valence shell, so they have 42. (d) Among given non-metals, O-atom has
similar chemical reactivity. high electron affinity and, so, strong ionic
1° bond is formed between Al and O-atoms.
CH3 43. (b) Nucleophiles are either negatively charged
1° 4° 2° 1° or neutral species, hence, hydride ion (H–)
34. (d) CH3 – C – CH2 – CH3
act as a nucleophile.
CH3 44. (b) Le-Chatelier principle is not applicable to
1° solid-solid equilibrium.
2, 2-Dimethylbutane
45. (c) +
Hydrogens attached to 2° Carbon are
secondary hydrogen. The number of (methyl cyclopropyl carbocation)
secondary hydrogen atoms in the above It is stable 2° carbocation and can not
compound is two. rearrange to the more stable 3° carbocation.
35. (a) The possible quantum numbers for 4f It belongs to non-classical carbocations.
electron are; BIOLOGY
n = 4, l = 3, m = – 3, –2 –1, 0, 1 , 2 , 3 and 46. (d) Out of a total of 64 codons, 3 codons do
1 not code for any amino acid. Hence, only
s=± 61 codons are used in the formation of the
2
20 amino acids.
Among various possiblities only option (a)
47. (a) Hemichordates have now been placed with
is possible.
the non-chordates because true notochord
36. (a) pOH = – log [OH–]
is absent in them but a buccal diverticulum
pH + pOH = 14
is present in pre-oral region which is often
For 10–4MKOH, [OH–] = 10–4 called stomochord.
pOH = – log [OH–] = – log 10–4 = 4 48. (a) The first heart sound, "Lub," indicates that
pH = 14 – pOH = 10 the heart has entered systole. Systole is
37. (b) Bleaching powder is actually a mixture of typically defined as ventricular contraction.
calcium hypochlorite, CaOCl2, and the As the ventricles contract, the AV valves
basic chloride, CaCl2, with some slaked lime, close to prevent regurgitation into the atria.
Ca(OH)2. Hence, the first heart sound indicates the
38. (c) C2H4 + 3 O2 ¾ ¾® 2CO2 + 2H2O closure of both the mitral and tricuspid
valves.
28 g 96 g
EBD_7839
128 KVPY-SA

49. (d) Simmond’s disease is a chronic deficiency of 59. (a) Intrautrine device (IUDs) inhibits
function of the pituitary gland, a form of fertilisation and helps in birth control.
hypopituitarism, that leads to atrophy of many 60. (d) Carbon monoxide (CO) is mainly produced
of the viscera including the heart, liver, spleen, due to incomplete combustion of fossil fuels
kidneys, thyroid, adrenals, and gonads. This in automobiles, furnaces, stoves, factories,
disease results in emaciation and death if left power plants, etc. CO has 200 times greater
untreated. affinity with blood haemoglobin as
50. (c) Translation is the process of decoding of compared to O2.
the messages from mRNA to protein with
the help of tRNA, ribosome and enzyme. PART-II
51. (a) Maltose is a disaccharide, which gives two MATHEMATICS
molecules of glucose on hydrolysis. It is
present during the germination of starchy 61. (a) Describe circumscribing circle CFE.
seeds. It is produced commercially from the Produce CA to meet circle in G.
starch by hydrolysing the enzyme, diastase. Join GE, GF. Draw DP, BQ to make with
52. (d) Chargaff found that the relative ratio of AC, ÐCDP, ÐCB Q = ÐFGC, ÐEGC
purines to pyrimidines is equal. Adenine respectively.
and guanine are purines and cytosine and F
thymine are pyrimidines; therefore, ratio of
adenine and thymine should be equal.
Chargaff also found that there is no G
A
conserved ratio between specific pairs D Q
(e.g., A + T and G + C).
53. (c) The terms p and q are the allele frequencies P
C E
for a given gene, so 2pq represents the two B
possible ways that heterozygotes can form.
54. (d) Glomerular filtrate contain urea, glucose, Since, ÐCBQ = ÐEGC, therefore
salts and water. It is the same as plasma of quadrilateral BECQ is cyclic.
blood, the only difference being that it does Therefore, ÐBEG + ÐBQG = 2 right angles.
not contain blood cells. But ÐBEG = ÐFCG + ÐFGC
55. (c) Valine is a neutral essential amino acid
= ÐFCG + ÐCDP.
which contains equal number of amino and
carboxylic groups. Therefore, supplementary ÐCPD
56. (b) The strength of linkage between two genes = supplementary ÐBQC.
is inversely proportional to the distance Therefore, BQ is parallel DP.
between the two. This means, that greater Since, ABCD is a parallelogram, therefore
the distance, lower is the linkage strength. AP = CQ.
Two linked genes show higher frequency Since, BEGQ is cyclic, therefore
of recombination if the distance between
CB · CE = CQ · CG.
them is higher and lower frequency if the
distance is smaller. Also, DFGP is cyclic. Therefore,
57. (b) DTP vaccine is for three diseases- CD · CF = CP · CG (Since, two chords of a
Diphtheria, Pertussis and Tetanus. BCG is circle intersect at a point, either inside or
for tuberculosis, ATS for tetanus and TAB outside the circle, the rectangle contained
for typhoid. by the segments of the one is equal to that
58. (a) HCl provides the acidic pH optimal for contained by the segments of the other.)
activation of pepsin. Pepsin is a protein- Adding yields,
digesting enzyme. The activated pepsin CB · CE + CD · CF = CG (CQ + CP)
converts proteins into proteases and = CG · AC = AC2 + AC · AG
peptides.
SOLUTIONS – MOCK TEST-3 129

Therefore, CB · CE + CD · CF Since, CD is a diameter in the circle CBD,


= AC2 + AE · AF. therefore ÐDBC = right angle.
Hence, in DGBC, CG2 = BG2 + BC2.
62. (a) From the given information we can say that
A is a multiple of h1 & h2, and similarly B is a But CG = CK = radii in circle C.
multiple of h1 & h3, while C is a multiple of Therefore, CK2 = BG2 + BC2 ...(1)
h2 & h3 hence we can assume that A = h1h2a, Since, CK2 = CE2 + EK2 + 2 CE · EK
b = h1h3b and C = h2h3c, if we take A and B and CE2 + 2 CE · EK = CE (CE + 2 EK)
then since their HCF is h 1 so we can = CE · CF
conclude that h2a and h3b are co-prime to Therefore, CK2 = EK2 + CE · CF ...(2)
each other, hence we can say that h1, h2 and Since, ÐBFE is measured by half the
h3 are co-prime to each other and their HCF difference of the arcs AC, BE or BC, BE, i.e.,
is 1. CE, therefore ÐBFE = ÐCBE.
63. (d) Area of circle A = 3.14 × 10 × 10 = 314 Therefore, CB touches circle on DBFE (Since,
Area of circle B = 3.14 × 8 × 8 = 200.96 if through an extremity of a chord in a circle
a straight line is drawn and if either of the
1 angles which the straight line makes with
Area of Q = × Area of B the chord is equal to the angle in the alternate
8
segment, the straight line touches the circle.)
1 Therefore, CB2= CE · CF
= × 200.96 = 25. 12
8 Hence, eq. (b) becomes
CK2 = EK2 + CB2 ...(3)
Area of P 5 From eq. (a) and eq. (c),
Now, Area of Q =
4 Therefore, BG = EK or EF = 2BG.

5
Þ Area of P = × Area of Q 65. (b) 21 + 3 59 + 16 + 3 722 + 49
4

5 =
= × 25.12 = 31.4 21 + 3 59 + 16 + 3 729
4
Area of square = 7 × 7 = 49
= 21 + 3 59 + 25 = 21 + 3 64
Required Area = (314+200.96+ 49 –25.12– 31.4)
= 507.44 cm2 = 25 = 5
64. (b) Suppose a circle with centre C bisects FE in PHYSICS
K. Join BE, BC, GC.
P 2 mk
66. (a) Radius, r = =
D qB qB
G
F 2mk
A rP = ,
B qB
K
E 2 ´ (2 m) k
C rd = ,
qB

2 ´ (4m) k
and ra = = rp
(2 q ) B
EBD_7839
130 KVPY-SA

67. (a) As tan q > µ, the block has a tendency to


(Vg s + w)
move down the incline. Therefore a force P ´ a = force of buoyance – wt. of the
is applied upwards along the incline. Here, g
at equilibrium P + f = mg sin q balloon with gas = V 15sg – (Vgs + w)
Þ f = mg sin q – P
æ 14Vg s - w ö
or a = ç ´g
N è Vg s + w ø÷
f

P 69. (a) Q
q
mg sinq mg cosq
x1

mg d x = nl
x2
q

D P
Now as P increases, f decreases linearly with
respect to P. Dx £ d Þ nl £ d
When P = mg sin q, f = 0.
When P is increased further, the block has d ædö df
a tendency to move upwards along the n£ or n £ ç ÷ f or nmax =
l è vø v
incline.
by decreasing f, number of maxima will decrease,
N thus, their separation will increases.
70. (c) As the track AB is frictionless. The block
moves through this distance without any change
P q in K.E. when the block enters the track BC.
mg cosq having coefficient of kinetic friction 0.2, its K.E.
mg sinq f
is lost in doing work against friction
mg 1 1
q Initial K.E. of block mv2 = × 0.5 × 32 = 2.25 J
2 2
Work done against friction over the distance BD
Therefore the frictional force acts = ( frictional force ) × ( distance BD) = (µk R) s =
downwards along the incline. µk mg s = 0.2 × 0.5 × 10 × 2.14 = 2.14 J.
Here, at equilibrium P = f + mg sin q This work against friction is less than the initial
K.E. of the block, the block moves further and
\ f = P – mg sin q
compresses the spring by a distance x (say).
Now as P increases, f increases linearly w.r.t Then according to law of conservation of energy.
P. Initial K.E. of block = Work done against friction
This is represented by graph (a). + Potential energy of compressed spring
1 1
68. (c) Let s be the density of the gas, then that of i.e. mv2 = µk mg ( BD + x) + kx2
2 2
the air is 15s. Then the weight of the balloon = 1
weight of the gas + weight of the envelope = or 2.25 = 0.2 × 0.5 × 10 (2.14 + x ) + × 2x2
Vgs + w. 2
or x2 + x – 0.11 = 0.
If f be the required acceleration of the balloon Solving ; we get, x = 0.1 m or –1.1 m.
acting vertically upward and then from “mass × As x cannot be negative, therefore we have x =
acceleration = forces acting in the sense of 0.1 m
acceleration” we get
SOLUTIONS – MOCK TEST-3 131

After moving through the distance x, the block


ca 2
comes to rest. Now the compressed spring exerts Thus, K a = = ca 2
1- a
a force F = Kx = 2 × 0.1 = 0.2 N on the block while
the limiting frictional force between the block 1000
and the track = ´ (1.8 ´ 10-8 )2 = 1.8 × 10–14
18
BC = µs mg = 0.22 × 0.5 × 10 = 1.1 N
Since F < limiting friction : therefore the block 74. (d) Higher the negative value of reduction
does not move further. potential, better is the reducing ability.
The total distance moved by the block According to given information Cu will be
= AB + BD + x able to reduce Br2.
= 2 + 2.14 + 0.1= 4.24 m. 75. (c)
CHEMISTRY
71. (c) Electronic configuration of N2 and N2+;
N2(14) = (s1s)2(s*1s)2(s2s)2(s*2s)2(p2px2
¾¾® ¾¾®
= p2py2) (s2pz)2
N+2 (13) = (s1s)2(s*1s)2(s2s)2(s*2s)2
(p2px2 = p2py2)(s2pz)1
Electron is removed from l (s2pz) orbital.
Electronic configuration of O2 and O+2 ; ¾¾®
O2(16) = (s1s)2(s*1s)2(s2s)2(s*2s)2
(s2pz)2(p2px2 = p2py2)(p* 2px1 = p*2py1)
O+2(15) = (s1s)2(s*1s)2(s2s)2(s*2s)2
(s2pz)2(p2px2 = p2py2)(p*2px1 = p*2py) BIOLOGY
Electron is removed from either p*2px or 76. (b) This is an example of non-competitive
p*2py orbital. inhibition. Binding of inhibitor alters the
72. (c) enzyme conformation thus the reaction
– – –
catalysed by the enzyme is affected. Since
CH2 CH2 CH2
inhibitor binds at the site other than active
OCH3
> > site, the affinity for the substrate is not
OCH3
affected and such inhibition cannot be
OCH3 overcome by increasing the concentration
II I III of substrate. Vmax decreases and Km stays
Stabilised by –I effect Destabilised by +R’ Destabilised by +R ’
stabilised by –I stabilised by –I effect constant.
which is minimum
in para isomer 77. (c) Lyases enzyme catalyses breakdown
without addition of water.
73. (d) Molarity, Isomerase enzyme catalyses the
wt. of solute per litre of solution conversion of an aldose sugar to a ketose
= sugar.
Mol. wt. of solute
Oxidoreductase enzyme catalyses the
1000 transfer of electrons from one molecule to
Molarity of H2O = mol/L
18 another molecule.
H2O ƒ H+ + OH– Ligases enzyme catalyses the bonding of
c (1 – a) ca ca two RNA molecules.
EBD_7839
132 KVPY-SA

78. (a) Dead space represents the volume of Anatomical dead space ventilation
ventilated air that does not participate in gas = Respiratory rate × Dead space
exchange. The two types of dead space are = 150 mL ×12 = 1800 mL/min
anatomical dead space and physiological dead 79. (a) The total number of progeny = 800
space. Anatomical dead space is represented
In F2 population, the ratio (9 : 3 : 3 : 1)
by the volume of air that fills the conducting
represent
zone of respiration made up by the nose,
9 – Yellow and round
trachea, and bronchi. This volume is
considered to be 30% of normal tidal volume 3 – Yellow and wrinkled
(500 mL); therefore, the value of anatomical 3 – Green and round
dead space is 150 mL. 1 – Green and Wrinkled
OR So, Total number of yellow and wrinkled
3
Alveolar ventilation = Respiratory rate seeds = ´ 800 = 150
16
(Tidal volume – Dead space)
80. (b) Water surrounding the body in Hydra and
4200 = 12 (500 – Dead space) blood in prawn are the media of circulation
Dead space = 150 mL and transport.
MOCK TEST-4

PART-I When number is not divisible by 11 then


smallest such number is 8 × 9 × 5 × 7 × 13
MATHEMATICS
When number is not divisible by 13 then
1. (a) Since, there is no restriction with smallest such number is 8 × 9 × 5 × 7 × 11
participants from China and Russia hence And smallest of all these numbers is
8 × 9 × 5 × 7 × 11 = 27720
these 10 members can be arranged around
circle in (9!) ways, this will create 10 spaces, 7
3. (a) The inequalities b< < 2a implies
so now 5 participants from USA can be 3
arranged in these 10 places in 2x + 1 7 x + 3
< < , so 2x + 1 < 7 and
(10C5×5!) = (10P5), hence total number of 3 3 2
ways is (9!)(10P5). 5
14 < 3x + 9, hence < x < 3.
2. (d) Since that number is divisible by 14 3
numbers from 1 to 15, so let us start
eliminating the number. The number has to 5
< x < 3.
Thus the range of x is
be divisible by 2 because if it is not divisible 3
by 2 then that number can not be divisible 4. (c) Draw HD and HE parallel to AB and AC
by 4, 6, 8, 10, 12, and 14 but the given respectively.
number is not divisible by only one number
from 1 to 15 hence 2 cant be that number,
on the same logic, 3, 4, 5, 6, 7, are eliminated
(e.g if number is not divisible by 7 then
number can not be divisible by 14 also,
hence it has to be divisible by 7) the given
number may not be divisible by 8 and 9,
now consider 10, since that number is
divisible by 2 and 5 both hence that
The figure ADHE is a parallelogram
number must be divisible by 10, consider
11, that number may not be divisible by 11, (A quadrilateral is a parallelogram if both
since number is divisible by 3 and 4 hence pairs of opposite sides are equal or
it has to be divisible by 12, number may parallel).
not be divisible by 13, again since number Therefore, AD = HE and AE = DH.
is divisible by 2 and 7 hence number has to
be divisible by 14, similarly number has to In the DDHF,
be divisible by 15. DH + DF > HF (Any two sides of a
Hence number may not be divisible by 8, 9, triangle are together greater than the third)
11 and 13. Therefore, AE + DF > HF ...(1)
When number is not divisible by 8 then
smallest such number is 4 × 9 × 5 × 7 × 11 × 13 Similarly in DEGH,
When number is not divisible by 9 then EH + EG > HG or AD + EG >HG ...(2)
smallest such number is 8 × 3 × 5 × 7 × 11 × 13 Adding eq. (1) and eq. (2) gives
AE + EG + AD + DF > HF + HG.
Therefore, AF + AG > HF + HG.
EBD_7839
134 KVPY-SA

5. (b) 10. (a) Let the four digit number be ‘abcd’.


A B a+ b= c+ d ...(1)
[Since the sum of the first two digits is equal
to that of the last two digits ]
O a+d=c ...(2)
[The sum of the first and the last digit is
equal to the third digit]
D C b + d = 2 × (a + c) ...(3)
[the sum of the second and fourth digits is
Q AO2 + OB2 = AB2B ...(i) twice the sum of the other two digits]
AO + OD = AD
2 2 2
...(ii) After solving these equations together we
OC + OD = CD
2 2 2
...(iii) get.
b = 2d; d = 4a; c = 5a. Hence we can say
OC + OB = BC
2 2 2
...(iv) that c has to be a multiple of 5 as a is an
After adding (i), (ii), (iii) and (iv) we get integer. The only single digit number
(AC)2 + (BD)2 = 4AB2 multiple of 5 is 5.
Hence c, the third digit has to be 5.
6. (b) Given two digit number is ab = 10a + b 11. (a) Let the vertices of a triangle be A(x1, y1, z1),
10a + b B(x2, y2, z2), C(x3, y3, z3).
has quotient = 6 & remainder = 7
a+b A(x1, y1, z1)
Hence 10a + b = 6a + 6b + 7
(0, 8, 5) E D (5, 7, 11)
4a - 7
=b
5
Now by trial & error method we will get B(x2, y2, z2) C(x3, y3, z3)
F(2, 3, – 1)
a = 3, b = 1 or a = 8, b = 5 Since D, E and F are the mid-points of AC,
But a = 3, b = 1, a + b = 4 so it can’t give us BC and AB
remainder 7, hence only 1 two digit number
æ x + x 2 y1 + y2 z1 + z 2 ö
exist. \ ç 1 , , ÷ = ( 0,8, 5 )
7. (c) The three numbers are 14x, 14y and 14z then è 2 2 2 ø
any two numbers in pairs are co-prime to Þ x1 + x2 = 0, y1 + y2 = 16, z1 + z2 = 10 …(i)
each other. If all x, y and z are co-prime to
æ x 2 + x 3 y 2 + y3 z 2 + z 3 ö
each other then their LCM = 14xyz. But L ç , , = ( 2,3, -1)
can be less than 14xyz in all other cases è 2 2 2 ÷ø
hence option C best describes the L. Þ x2 + x3 = 4, y2 + y3 = 6, z2 + z3 = – 2 …(ii)
8. (c) um + v m = wm, this one is similar to
æ x1 + x3 y1 + y3 z1 + z3 ö
Pythagoras theorem, so we can take some and ç , , = ( 5,7,11)
Pythagoras triplets like 3, 4, 5 or, 5, 12, 13 è 2 2 2 ÷ø
i.e 32 + 42 = 52 than correct option is (c). Þ x1 + x3 = 10, y1 + y3 = 14, z1 + z3 = 22. …(iii)
9. (d) Without restriction they can be arranged On adding eqs. (i), (ii) and (iii), we get
2(x1 + x2 + x3) = 14, 2(y1 + y2 + y3) = 36.
in (27!) ways, and the number of ways in 2(z1 + z2 + z3) = 30,
which at least one manager is next to Þ x1 + x2 + x3 = 7, y1 + y2 + y3 = 18,
director is 102(25!) [from question number 4]. z1 + z2 + z3 = 15. …(iv)
On solving eqs. (i), (ii), (iii) and (iv), we get
So required number of ways is x3 = 7, x1 = 3, x2 = – 3; y3 = 2, y1 = 12, y2 = 4
(27!) – 102(25!) = 26 × 27(25!) – 102(25!) and z3 = 5, z1 = 17, z2 = – 7.
Hence, vertices of a triangle are (7, 2, 5),
= 600(25!) (3, 12, 17) and (– 3, 4, – 7).
SOLUTIONS – MOCK TEST-4 135

12. (a) Let the cost price of one chair be ` x and


that of one table be ` y.
Profit on a chair = 25%
\ selling price of one chair
25 125
= x+ x= x
100 100
Profit on a table = 10%
In DPQS
\ selling price of one table
ÐPQS = ÐPSQ = a
10y 110 2a + 2° = 180° Þ a = 89°
= y+ = y
100y 100 \ ÐRQS = ÐQ – ÐPQS Þ ÐRQS = 176° – 89°
By the given condition, we have Þ ÐRQS = 87°
125 110 14. (c) No. of ways getting one correct
x+ y = 1520
100 100 æ 1 1 1 1ö
= 7 C1 6!ç1 - + - + ... + ÷ = 7 C × 265
Þ 125x + 110y = 152000 ...(i) è 1! 2! 3! 6! ø 1
Þ 25x + 22y = 30400 No. of ways getting two correct
If profit on a chair is 10% and on a table is æ 1 1 1 1ö
= 7C2 × 5!ç1 - + - + ... - ÷ = 7 C × 44
25%, then total selling price is `1535. è 1! 2! 3! 5! ø 2
No. of ways getting three correct
æ 10 ö æ 25 ö
\ çè x + x÷ + ç y +
ø è
y÷ = 1535 æ 1 1 1 1ö
100 100 ø = 7 C3 × 4!ç1 - + - + ÷ = 7 C3 × 9
è 1! 2! 3! 4! ø
Required no. of ways
110 125
Þ x+ y = 1535 = 7 C3 × 9 + 7 C2 × 44 + 7 C1 × 265 .
100 100
Þ 110x + 125y = 153500 15. (c) Suppose the number is x + 10y + 100z
Þ 22x + 25y = 30700 ...(ii) Now, on reversing the new number
Eqn. (ii) – (i) gives = z + 10y + 100x
3x – 3y = – 300 Þ x – y = – 100 ...(iii) According to question
(i) + (ii) gives x + 10y + 100z – z – 10y – 100x = 297
47x + 47y = 61100 Þ x + y = 1300 ...(iv) Þz=x+3
(iii) + (iv) gives 2x = 1200 Þ x = 600 Also, x + y + z = 8
and (iv) – (iii) gives 2y = 1400 So, possible cases are 512, 431 but hundred
Þ y = 700 ...(iv) possible value is 5 (maximum)
Hence, the cost price of a chair is ` 600 and So, tens digit is 1.
that of a table is ` 700. PHYSICS
13. (b) PQ = PS = 1 = QR
16. (a) Man and wedge will displace as one unit of
In DPQR mass 3 M. If its displacement is Dx1, towards
PQ = QR = 1 left, then
\ ÐQRP = ÐQPR = 2° 0 = M ( 2 - Dx1 ) + 3M ( -Dx1 )
\ ÐQ + 2° + 2° = 180°
2
ÐQ = 176° \ Dx1 = = 0.5 m
4
EBD_7839
136 KVPY-SA

20. (c) As a-particles are doubly ionised helium


g
17. (b) g¢ = 2 He++ i.e. Positively charged and nucleus is
æ hö
çè 1 + ÷ø also positively charged and we know that
R
like charges repel each other.
gR 2 gR 2 4g 21. (c) From Snell’s law,
= = = 60°
A
( R + h) 2
( 3 R / 2) 2 9
sin 60°
30° 30°
30°
90°
m = = 3
sin 30°
4g
\ W¢ = mg¢ = 200×
9
4 ´ 10 22. (d) Let c be the centre of the cross-section
= 200 × = 889 N. considered at any section of the wire.
9
If r be the radius of a circular strip concentric
18. (b) The electric field due to the sheet,
with the cross-section considered and, dr
s be its thickness then (fig.) current through
E= +
the circular strip of thickness dr will be
Î0 r r
+q q
di = j.dS = (a + br) (2prdr) cos 0
Now for the T
+
equilibrium of the ball Eq or di = 2p (ar + br 2 ) dr
+
T sin q = Eq
+ mg
and T cos q = mg
+
Eq sq r
\ tan q = = .
mg Î0 mg C
Thus s µ tan q dr R

19. (b) The total current i can be obtained by


summing up the currents flowing through
individual circular strips,
R
i.e., i = ò di = ò 2p (ar + br 2 ) dr
0
pd 2 p(nd )2
x´ = y é é r2 ù
R
é r3 ù ù
R
4 4 i = 2p êa
or ê ú + b ê ú ú
ê ëê 2 ûú 0 ëê 3 úû 0 úû
\ x = n2y ë
From Pascal’s law
2pR 2
rw gh = r Hg g ( x + y) = (3a + 2bR)
6
23. (c) Torque working on the bob of mass m is,
or 1 × gh = sg (n2 y + y )
t = mg ´ l sinq . (Direction parallel to plane
h of rotation of particle)
\ y=
s(1 + n 2 )
SOLUTIONS – MOCK TEST-4 137

æ 1ö
25. (d) We know that C = sin -1 ç ÷ .
è mø
q
l l Given , iB > i A \ m B < m A .
So B is rarer and A is denser. Light will be totally
m mg reflected, when it passes from A to B. Now crit-
ur ical angle for A to B
As t is perpendicular to L , direction of L
changes but magnitude remains same. æ 1 ö
C AB = sin -1 ç = sin -1 ( A m B )
24. (b) When body rolls dawn on inclined plane è B m A ÷ø
with velocity V0 at bottom then body has -1 æ m ö -1 æ sin i A ö
= sin ç B ÷ = sin ç
both rotational and translational kinetic è mA ø è sin iB ÷ø
energy. 26. (b) If R is the resistance of each lamp, then current
Therefore, by law of conservation of e
(initially) in x and y is same, i = .
energy, 2R
So power consumed,
P.E. = K.Etrans + K.Erotational
2
æ e ö e2
1 1 Px = i 2 R = ç ÷ R = and Py also the
= mV02 + I w 2 è 2R ø 4R
2 2
same, when switch is closed,

1 1 V2 e 2e
2
mk 2 0 i= = .
= 2 mV0 + …(i) æ R ö 3R
2 R02 çè R + ÷

é 2 V ù æ 2e ö
2
4e 2
êQ I = mk , w =
2
ú Now P 'x = i ' R = ç ÷ ´R = , and
ë R0 û è 3R ø 9R
2
When body is sliding down then body has æ e ö e2
Py = ç ÷ R = .
only translatory motion. è 3R ø 9R
Clearly x-increases, y-decreases.
2
1 æ5 ö 27. (d) The least count (L.C.) of a screw guage is
\ P.E. = K.Etrans = m ç v0 ÷ ...(ii)
2 è4 ø the smallest length which can be measured
accurately with it.
Dividing (i) by (ii) we get
1
As least count is 0.001 cm = cm
é K2 ù 1000
1 Hence measured value should be recorded
mV02 ê1+ 2 ú
P.E. 2 êë R0 úû upto 3 decimal places i.e., 5.320 cm
= = 28. (c) According to law of sines or Lami’s
P.E. 1 25
´ ´ mV02
2 16 Theorem

25 K2 K2 9 75°
vB
= 16 = 1 + 2 Þ 2 = 16 v B / A = vB - vA
R0 R0
45° 60°
3
or, K = R0 . vA
4
EBD_7839
138 KVPY-SA

vA vB v
Þ = = B/ A H H
sin 75° sin 60° sin 45°
35. (d) ¾¾® sp3 Carbon
Þ vB = 717 kmh–1
29. (c) W = m (V2 – V1)

é GM1 GM 2 ù
when, V1 = - ê + , Cyclopentadiene is non aromatic, as it has
ë a 2a úû
sp3 carbon in the ring.
é GM 2 GM1 ù 36. (c)
V2 = - ê +
ë a 2a úû Na 2SO 3 + 2H 2SO 4 (dil) ¾¾
® Na 2SO 4 + H 2O
+ SO2
Gm( M 2 - M1)
\ W= ( 2 - 1) K 2 Cr2 O 7 + H 2SO 4 + 3SO 2 ¾¾ ® K 2SO 4
a 2 + Cr2 (SO4 )3 + H 2O
30. (c) When ball fall from great height, its initial Green
velocity before entering into liquid is quite The above reactions indicates the presence of
enough. So viscous force together with SO2–
3 ion in the compound.
buoyant force becomes greater than weight 37. (b) Higher the positive charge, greater is the
of the cone. So first ball retarted and there- EN and higher the negative charge lesser
after will move with constant velocity. is the EN. So, EN order is
CHEMISTRY M– > M2– > M3– > M4–
31. (c) This is Avogadro’s hypothesis. 38. (d) Gypsum is CaSO4 . 2H2O, whereas Plaster
According to this, equal volume of all gases 1
of Paris is CaSO 4 × H 2 O.
contain equal no. of molecules under similar 2
Therefore, difference in number of water
condition of temperature and pressure.
1
32. (a) For isothermal expansion, relation between molecules in them is 1 .
2
P and Vm, and, U and Vm are as: 39. (b) IUPAC name of the given compound is
2-hydroxy benzoic acid.
40. (c) More effective axial and sideways
overlapping between atomic orbitals of
carbon and oxygen atom is due to smaller
size of oxygen atom. Oxygen is more
electronegative than carbon atom, due to
Therefore (B) and (D) are not correct these factors CO has highest bond energy.
representation. 41. (d) The pOH of a buffer consisting of NH3 (i.e.
33. (a) H 2O + C ¾¾ ® CO2 + H 2 NH4OH) and salt NH4Cl (salt) is given by
Steam the equation
Water oxidises C to CO2, hence water act
as an oxidising agent, whereas, carbon act pOH = pKb + log [salt]
as reducing agent since it reduces water to [ammonia]
hydrogen gas.
34. (d) According to Hund’s rule electron pairing = 5.0 + log
[1.0]
in p, d and f orbitals cannot occur until each [ 0.1]
orbital of a given subshell contains one = 5.0 + log 10 = 5 + 1 = 6
electron each or is singly occupied. \ pH = 14 – pOH = 14 – 6 = 8
SOLUTIONS – MOCK TEST-4 139

42. (d) In the reactivity series, as we move down 50. (a) Substrate level phosphorylation means
the series the reactivity of the element synthesis of ATP by utilising energy
decrease. released directly by the substrates. In
43. (b) Methyl carbanion is sp3 hybridised, with glycolysis, there are two steps in which ATP
three bond pairs and one lone pair, same is is synthesised. In Kreb’s cycle, there is only
the case with NH3. one step where GTP is synthesised when
succinyl CoA is converted into succinic
acid.
51. (a) Heat-killed S-type bacteria caused a
transformation of the R-type bacteria into
S-type bacteria but he was not able to
understand the cause of this bacterial
transformation. The occurrence of living
S-type virulent bacteria is possible only
44. (a) H2 (g) + O2 (g) ¾¾® H2O2 (g) by their formation from R-type non-virulent
DH reaction = B.E.reactants - B.E.products bacteria, which pick up the trait of
virulence from dead bacteria. This
= [B.E.(H - H) + B.E.(O = O)] phenomenon is called Griffith effect or
transformation. Griffith proposed that a
-[2B.E.(O - H) + B.E.(O - O)]
chemical substance released by heat- killed
= [438 + 498] – [2 × (464) + 138] bacteria changed the R-bacteria into S-
= 936 – 1066 = –130 kJ mol–1 bacteria which was a permanent genetic
45. (d) Bond structure of H4P2O7 is following change.
52. (c) Human embryo have gills. This shows
O O ontogeny repeats phylogeny. Ontogeny is
s p s p
H s
O s s
P O s
Ps
O s
H the life history of an organism while
s s phylogeny is the evolutionary history of
s s
O H O H the race of that organism. Modern theory
of origin of life was propounded by Oparin
12s, 2dp – pp bonds.
and Haldane which is based on chemical
BIOLOGY evolution. Chemical evolution, also called
46. (d) Debris normally gets trapped in mucus chemogeny, involves the synthesis of
lining of the respiratory passage ways. simple organic molecules. Miller and Urey
This mucus is then swept up and away from experimentally supported Oparin and
the lungs by the action of cilia. Haldane theory with the help of stimulation
47. (c) Cholesterol is the precursor of the five major experiment. Analogous organs are those
classes of steroid hormones: progestogens, organs which are similar in shape and
glucocorticoids, mineralocorticoids, function but their origin, basic plan and
androgens, and oestrogens. These hormones development are dissimilar. Example, wings
of butterfly, bird and bat. Such similarities
are powerful signal molecules that regulate a
are because of convergent evolution for
host of organismal functions.
adaptation to a common condition.
48. (a) The route of water and solutes through the
53. (c) Baculoviruses are suitable for species-
nephron is as follows: From the glomerulus specific, narrow spectrum insecticidal
® Bowman’s capsule ® proximal tubule
applications.
® loop of Henle ® distal tubule ®
54. (a) After cellulose, chitin is the second largest
collecting ducts. structural polysaccharide and principal
49. (b) In gram negative bacteria, peptidoglycan component of exoskeleton of insects and
layer is thin and does not retain crystal crustaceans. It is a polymer of N-acetyl
violet stain like gram positive bacteria do. glucosamine.
EBD_7839
140 KVPY-SA

55. (c) Oogenesis is initiated during the embryonic Let area of DPTU = R,
development stage when some million and ar DTUV = a
oogonia (A) are formed within each foetal
ovary. No more oogonia are formed and 1
Q TU = QS (by mid point theorem)
added after birth. These cells start division 2
and enter into prophase I (B) of meiotic and also, TU || SQ.
division and get arrested (C) at a stage \ DTUV ~ DQSV
called primary oocyte (D).
56. (c) A geological time scale is a diagram that TU 2 ar DTUV
\ =
details the history of earth’s geology, noting SQ 2 ar D SQV
major events like the formation of the earth,
the first life forms and mass extinctions. The QS 2 a
Þ = Þ ar DSQV = 4a.
first geological time scale was proposed in (2 QS ) 2
ar D SQV
1913 by the British geologist Arthur Holmes Q TU is median for DPTQ,
(1890–1965). \ ar DPTU = ar DTUQ Þ ar DTUQ = R
The history of the earth has been
\ ar DUQV = R – a
subdivided into eras, eras into periods and
In D PSU, as UT is median,
periods into major divisions.
57. (d) PS II is located within the appressed granal \ ar DUTS = R \ ar DPSU = 2R,
region, whereas PS I and ATP synthase In DPSQ, as SU is median,
complex is located within the non- \ ar DSUQ = ar DPSU = 2R.
appressed thylakoid membrane regions and Q SQ is diagonal, \ ar (PQRS) = 8R
stroma lamellae. Q ar DSUQ = ar DSVQ + ar DUQV
58. (c) Tropic hormones are those hormones that Þ 2R = 4a + R – a Þ R = 3a
have other endocrine glands as their target now,
to control their function. Most of the tropic
hormones are produced and secreted by ar QRSV ar DSQR + ar DSVQ
=
anterior pituitary. ar DPQT ar DPQT
59. (c) A nucleotide is made up of a sugar
ar QRSV 4 R + 4a
molecule, phosphate group and a Þ =
heterocyclic base while a nucleoside is ar DPQT 2R
made up of a sugar molecule and a 4R
heterocyclic base. 4R +
60. (b) During prophase, each chromosome Þ ar QRSV = 3 =8
ar DPQT 2R 3
consists of two chromatids (46
chromosomes, each with 2 chromatids = 92 62. (d) Let a two digit number 10a + b if we reverse
chromatids). the digits then we have new number
10b + a. Summation of these two numbers
PART-II
are 11(a + b) hence this summation or all
MATHEMATICS the elements of the 1st must be divisible
by 11, and so the required HCF would be
61. (d) 11.
63. (b) Consider the figure,
Area of sector OACB
q 60 1 p p
= pr 2 = p ´1´ = . =
360° 360 2 3 6
SOLUTIONS – MOCK TEST-4 141

Putting b = B – 15°, we get a relation


1 + 2 cos(b + 30) = cosb
We write this in the form
(1 - 3)cos b + sin b = 1.
Since, sin b £ 1, it follows that
(1 - 3)cos b ³ 0.
p We conclude that cos b £ 0 and hence that b is
Area of shaded region = – area of DOAB
6
obtuse.
p 3 So, is angle B and hence ÐADC.
= -
6 4
We have the relation (1 - 3)cos b + sin b = 1.
Hence, area of lune = Area of semi-circle
– area of shaded region æ bö
If we set x = tan çè ÷ø , then we get, using
2
2 æ
1 æ 1ö p 3ö
= pç ÷ -ç - ÷ (1 - x 2 ) 2x
2 è 2 ø è 6 4 ø cos b = , sin b = ,
(1 + x 2 ) (1 + x 2 )
3 p p 3 p
= + - = - ( 3 , 2) x2 ∗ 2 x , 3 < 0.
4 8 6 4 24
Solving for x, we obtain x
64. (a)
= 1 or x < 3(2 ∗ 3).
If x < 3(2 ∗ 3), then

æ bö
tan ç ÷ > 2 + 3 = tan 75° giving us b > 150°.
O è 2ø
This forces that B > 165° and hence,
A
a 15 B + A > 165° + 15° = 180°, a contradiction.
°
F p
E Thus, x = 1 giving us b = .
a 2
B D C This gives B = 105° and hence, a = 30°.
Let a denote the equal angles ÐBAD = ÐDCA. Thus, ÐDAO = 60°. Since, OA = OD, the result
Using sine rule in triangles DAB and DAC, we follows.
get 65. (d) 7+4 3 - 7-4 3
AD BD CD AD
< , < .
sin B sin  sin15↓ sin  = 2 2 + ( 3) 2 + 2(2)( 3)
Eliminating a (using BD = DC and - 2 2 + ( 3) 2 - 2(2)( 3)
2a + B + 15° = p),
we obtain 1 + cos(B + 15°) = 2 sin B sin 15° = (2 + 3)2 - (2 - 3) 2
But, we know that = 2+ 3 -2+ 3= 2 3
2sin B sin 15° = cos(B – 15°) – cos(B + 15°)
EBD_7839
142 KVPY-SA

PHYSICS 0.22 ´ 102


66. (a) If v1 and v2 are the velocity of ball and the z2 = 0.273 + = 0.477m
2 ´ 9.81
wedge after collision, then
v1 l
68. (c) As we know, time period, T = 2p
g
J
60° 60°
J When additional mass M is added then
v2 v2
l + Dl
TM = 2p
60° 60° g
For ball:
2
2 J cos 60° = mv1 – (–mv0) l + Dl æT ö l + Dl
TM = or ç M ÷ =
or J = mv1 + mv0 l è T ø l
T
And J sin 60° = mv2 – 0
2
From above equations, we get æT ö Mg é Mgl ù
or, ç M ÷ = 1 +
è T ø AY êëQ Dl = AY úû
2v2
3
= v1 + v0 ...(i)
1 éæ TM ö ù A
2
[v cos60° - ( -v2 cos30°)] \ = êç ÷ - 1ú
Also e= 1 Y êè T ø úû Mg
(v0 cos 60° - 0) ë
69. (a) From the figure it is clear that
v1 + 3v2
= ...(ii) (a) s2 > s1
v0
(b) r2 > s2 [As the string is taut]
After simplifying above equations, we get
(c) r1 < s1 [As the string is taut]
3 \ r1 < s1 < s2 < r2
v2 = (1 + e)v0
5 When P alone is in L2
67. (d) Let zmin be the minimum reference level of
the dotted parabola and z1 and z2 the liquid 2pr 2 (r1 – s 2 ) g is negative as r < s
VP = 1 2
levels above the base. 9h2
r12w 2 0.12 ´ 102 Where r is radius of sphere.
z1 = + zmin = + zmin , When Q alone is in L1
2g 2 ´ 9.81

r22 w 2 0.22 ´ 102 2 pr 2 (r2 – s1 ) g is positive as r > s


z2 = + zmin = + zmin VQ = 2 1
2g 2 ´ 9.81 9 h1
ur ur
But z1 + z2 = 2 × 0.4 = 0.8m Therefore V P . V Q < O
0.22 ´ 102 + 0.1 ´ 10 2
Hence, 0.8 = + 2 zmin
2 ´ 9.81
s1
whence, zmin = 0.273m
T
Consequently, T
0.12 ´ 102 s2
z1 = 0.273 + = 0.324m
2 ´ 9.81
SOLUTIONS – MOCK TEST-4 143

CHEMISTRY
VP r1 – s 2 h1
Also V = r – s ´ h ...(i) B H
Q 2 1 2 71. (d) 6CH3 - CH = CH 2 ¾¾¾®
2 6

1-Propene ether, 0 °C
For equilibrium of Q
4 3 4 H O
T+ pr s 2 g = pr 3 r 2 g 2(CH3CH 2 CH 2 )3 B ¾¾¾®
2 2
...(ii) OH -
3 3
For equilibrium of P 6CH3CH 2 CH 2 OH+2H3BO3
1-Propanol
4 3 4
T+ pr r1g = pr 3 s1g ...(iii)
3 3 72. (a) H2SO4 can be formed by adding all the given
(iii) – (ii) gives equations. The obtained reaction is,
r1 – s2 = s1 – r2 ...(iv) H2 + S + 2O2 ¾¾® H2SO4
Now, enthalpy of formation of H2SO4 will
From (i) and (iv)
be obtained by adding DH of the given
VP h VP h1 reactions, i.e., (i) + (ii) + (iii) + (iv)
=– 1 \ =
VQ h2 h2 So,
VQ
–298.2 kJ + (–98 kJ) + (–130.2 kJ) + (–287.3 kJ)
70. (b) The time period of a simple pendulum is = –814.4 kJ
given by 73. (d) The reaction may be given as
Z2 O3 + 3H2 ¾ ¾® 2Z + 3H2O
l 2 2 l 2 l
T = 2p
g
\ T = 4p g Þ g = 4 p 2 0.1596 g of Z2O3 react with H2 = 6 mg
T
= 0.006 g
Dg Dl DT
Þ ´ 100 = ´100 + 2 ´ 100 \ 1 g of H2 react with = 0.1596
g l T 0.006
Case (i) = 26.6 g of Z2O3
Dl = 0.1 cm, l = 64cm, DT = 0.1s, T = 128s \ Eq. wt. of Z2O3 = 26.6 g
Dg (from the definition of eq. wt.)
\ ´ 100 = 0.3125
g Eq. wt. of Z + Eq. wt. of O = E + 8 = 26.6 g
Case (ii) Þ Eq. wt. of Z = 26.6 – 8 = 18.6 g
Dl = 0.1 cm, l = 64cm, DT = 0.1s, T = 64s Valency of metal in Z2O3 = 3
Atomic wt.
Dg Eq. wt.of metal <
\ ´ 100 = 0.46875 Valency
g
\ At. wt. of Z = 18.6 × 3 = 55.8 g
Case (iii)
74. (d) The molecular orbital configuration of the
Dl = 0.1 cm, l = 20cm, DT = 0.1s, T = 36s
given molecules is
Dg H2 = s1s2 (no electron anti-bonding)
\ ´ 100 = 1.055
g Li2 = s1s2 s*1s2 s2s2(two anti-bonding
electrons)
Dg
Clearly, the value of
g
´ 100 will be least
{ 1 1
B2 = s1s2 s*1s2 s2s2 s*2s2 p2p x = p2p y }
in case (i). (4 anti-bonding electrons)
EBD_7839
144 KVPY-SA

Though the bond order of all the species do not have bundle sheath. If they have
are same (B.O = 1) but stability is different. bundle sheath, its cells do not contain
This is due to difference in the presence of chloroplasts. In contrast, C4 plants have
no. of anti-bonding electron. bundle sheath cells with chloroplasts. Dicot
Higher the no. of anti-bonding electron plants have reticulate venation and
lower is the stability, hence the correct order monocots have parallel venation. Hence,
is H2 > Li2 > B2. plant A with reticulate venation but no
75. (b) The more substituted alkenes have greater bundle sheath is a C3 dicot, plant B with
parallel venation and bundle sheath
number of hyperconjugated forms. Hence
containing chloroplasts is a C4 monocot,
they are more stable.
plant C with reticulate venation and bundle
H3C H sheath containing chloroplasts is a C4 dicot
C == C and plant D with parallel venation and
H CH3 bundle sheath without chloroplasts is a C3
2-Butene monocot.
78. (b) Air at sea level with an atmospheric
H3C CH3 pressure of 700 mm Hg has a partial
C == C pressure of oxygen of 160 mm Hg (760 ×
H3C CH3 21%). Therefore, the partial pressure of O2
2,3-Dimethyl But-2-ene at Antonito, Colorado, is 600 × 21% =
126 mm Hg.
BIOLOGY 79. (d) Analogous organs have similar functions
76. (d) A frameshift mutation occurs due to but different origins. Options (a), (b), and
insertion and/or deletion of a number of (c) are the examples of analogous organs
nucleotides in a DNA sequence that is not whereas tendrils of pea and spines of
divisible by three. This changes the reading barberry are examples of homologous
frame resulting in a completely different organs.
translation from the original. A missense 80. (b) Tidal volume is the normal volume of air
mutation is a point mutation in which displaced between normal inhalation and
change in a single nucleotide results in a exhalation. It is approximately 500 mL.
codon that specifies a different amino acid. The inspiratory reserve volume (IRV) is the
A nonsense mutation is also a point additional amount of air that can be inhaled
mutation that changes a sense codon into after a normal inspiration (tidal volume). It
a nonsense codon. As a result, the altered is between 2500-3000 mL in humans.
DNA sequence prematurely stops the The expiratory reserve volume (ERV) is the
translation of mRNA leading to the amount of extra air above a normal breath,
formation of a truncated protein that may exhaled during a forceful breath out. The
be non-functional. Silent mutations are average ERV volume is about 1100 mL in
mutations in DNA that do not have an males and 800 mL in females.
observable effect on the organism's The residual volume is the amount of air
phenotype. remaining in the lungs at the end of a
77. (c) Bundle sheath is a layer of cells that forms normal exhalation. This averages 1100 mL
a sheath surrounding the vascular bundles to 1200 mL.
in plant leaves and stems. Some C3 plants
MOCK TEST-5
PART-I For P to be minimum (b + c)/(a + d) should be
maximum that means b + c should be maximum
MATHEMATICS
when b = c = 9, and a + d should minimum
1. (b) As A1B1 and C1D1 are parallel and half of when a = 6 and d = 0.
one of the diagonals of ABCD, the Then N = 6990 and then
quadrilateral A1B1C1D1 is a parallelogram.
P = 110 + 891/4 = 332.75
Similarly, A2B2C2D2 is a parallelogram.
As it is also given to be a rectangle, the 5. (b)
D C
diagonals of A1B1C1D1 are perpendicular
1
to each other with lengths 8 and 12.
Thus, it is a rhombus of side length
52 Thus, the lengths of diagonals of 3
P Q
ABCD is 2 52 each, giving the required
product as 208 i.e., n = 208. 4
é n ù é 208 ù
Therefore, ê ú =ê ú = 2.
ë 100 û ë 100 û 2
2. (d) Since unit digit of all the factorials more A R B
than 4 is 0 hence unit digit 9 is given by Join DP and produce DP to meet AB in R.
either 0! + 2! + 3! Or 1! + 2! + 3! In both
the cases the value of {(a!)(b!)(c!)} In DDPC and DRPA, we have Ð1 = alt.
= 1 × 2 × 6 = 12 Ð2 [Q AC cuts || lines AB and DC]
3. (c) Adding both equations
Ð3 =Ð4 [Vert. opp. Ðs]
x2 + xy + x + y2 + xy + y = 42,
Þ (x + y)2 + (x + y) = 42 CP = AP [Given ]
Þ (x + y) (x + y + 1) = 42 Possible \ DDPC @ DRPA, and in particular
solutions, x + y = 6, obtained by factorizing
DP = PR and DC = AR
42. Where (x + y) = 6 and (x + y + 1) = 7
or x + y + 1 = –6 and x + y = –7 In D DRB,
Sum of all possible solutions is (–7) + (6) = –1 Q PQ joins P and Q the mid-points of DR and
4. (c) Let N = abcd = 1000a + 100b + 10c + d BD respectively.
Then S = (1000a + 100b + 10c + d) + (1000d
\ PQ || RB
+ 100c + 10b + a)
= 110(a + b + c + d) + 891(a + d) 1
And, PQ = RB Þ PQ = (AB – AR)
From the given condition 2
{110 ( a + b + c + d ) + 891( a + d )} 1
= (AB – CD) [QDC = AR]
a+b+c+d 2
891( a + d ) 6. (a) Unit digit of x is prime hence it can be 2, 3,
= 110 + 5, or 7 (We have only 4 prime digits)
a+b+c+d
891 As the product of all digits of x is prime hence
= 110 + rest all digits must be 1, then possible values
ì b+c ü
í1 + ý of X are 1112, 1113, 1115 or 1117.
î a+d þ
EBD_7839
146 KVPY-SA

7. (b) If a number is divisible by 36 then it has to


ì 90 2 1 ü
be divisible by 4 and 9. Given number is =í ´ p ´ (10 2 ) - ´ 10 2 ´ 10 2 ý cm 2
368A37982B. If it is divisible by 4 then last î 360 2 þ
two digits should be divisible by 4 hence B
æ1 1 ö æp ö
must be either 0, or 4 or 8 (as 20, 24 and 28 = ç ´ p ´ 200 - ´ 200 ÷ cm 2 = ç - 1÷ ´ 100cm 2
è4 2 ø è2 ø
the last two digit is divisible by 4). Now since
the number is divisible by 9 hence sum of the The area of the shaded region
digits should be divisible by 9 or 46 + A + B = The area of the rectangle ABCD – The
should be divisible by 9. area of the segment APB
Hence 46 + A + B = 54 or 63 æp ö
= 20 ´ 10cm 2 - ç - 1 ÷ ´ 100cm 2
If 46 + A + B = 54 then A + B = 8 hence è2 ø
possible pairs of (A, B) = (8, 0), (4, 4) and (0, 8)
ì p ü
If 46 + A + B = 63 then A + B = 17 hence = í400 - ´ 100 + 100ý cm 2
î 2 þ
possible pairs of (A, B) = (9, 8)
Hence possible values of A are 0, 4, 8 and æ pö
= ç 5 - ÷ ´ 100 cm 2 .
9, and sum of all possible values of A is 21 è 2ø
8. (a) Given that S = (2, 3, ….2n + 1), hence total 10. (a) P(x) = x2018 + x2017 +....+ x + 1
numbers of element in S is 2n. remainder = P(–1)
Odd elements are (3, 5, 7, ….2n + 1) total n Þ P(–1) = (–1)2018 + (–1)2017 +....+ (–1) + 1
elements
Þ P ( -1) = 1144
- 1 + 14
-2444
1.... + 1 -
31 + 1
And even elements are (2, 4, ….2n) total n equal +ve and -ve terms
elements
\ P(–1) = 1
Hence X = [3 + 5 + 7 + ….(2n + 1)]/n
11. (d) Since sum of digits is minimum hence number
And Y = [2 + 4 + 6 + ….. + 2n]/2 of 4’s should be maximum. Let us assume
So X – Y = [(3–2) + (5–4) + ….(2n + 1–2n)]/n that number of 4’s is x and that of 8 is y then
= [1 + 1 + 1….n times]/n = n/n = 1 sum of digits is 4x + 8y. Since N is divisible
Hence X–Y = 1 (Point to note that it is by 72 hence it must be divisible by 8 and 9
independent from the value of n) both hence sum of digits should be divisible
by 9. So now we have two equations 4x + 8y
9. (d) In DOAB, OA = OB = 10 2 cm ( radius )
= multiple of 9 or 4(x + 2y) = multiple of 9 or x
and AB = 20 cm + 2y = 9k (i.e multiple of 9) and x + y = 99
(total number of digits) from these two
(Given as the side of the rectangle)
questions we will get y = 9(k – 11) that means
Þ AB2 = OA 2 + OB2 (Each = 400) y should be multiple of 9. Minimum value of
y = 0 but then we have only 4’s and last three
Þ Ð AOB = 90°
digits is 444 which is not divisible by 8 hence
We take a point P on the arc AB of the number cannot be divisible by 72.
semi-circle. Next minimum value of y is 9 then x = 90 and
Now, area of the segment APB = The area sum of digits = 4 × 90 + 9 × 8 = 432
of the sector OAPB – The area of the Here in this case last 3 digits can be 448,
DOAB 848, 488 and 888 all these are divisible by 8.
SOLUTIONS – MOCK TEST-5 147

12. (c) As per to question, figure is- Since, a, b are the roots of this equation,
A l -1 5
therefore, a + b = and ab =
l l

E F a b 4
But given + =
X Y b a 5

a 2 + b2 4
Þ =
B C ab 5
Q EB || AC and EY || BC
( a + b)
2
- 2ab 4
\ EYCB is a parallelogram, =
ab 5
also, XF || BC and CF || AB.
\ BXFC is a parallelogram. ( l - 1)
2
10
Q BXFC and EYCB lies on same base BC -
l2 l =4
and between 2 parallel lines, 5 5
\ ar (EYCB) = ar (BXFC) l

1 1 ( l - 1)
2
- 10l 4
\ ar (EYCB) = ar (BXFC) ...(1) =
2 2 5l 5
now, consider DAEB,
l2 - 12l + 1 = 4l
ar DAEB = ar DEBC,
as they both lie on same base BE and l 2 - 16l + 1 = 0
between parallel lines BE and AC. It is quadratic in l. So, let the roots be l1
Q EC is diagonal of EYCB and l2 then

1 l1 + l 2 = 16 and l1l 2 = 1
ar DEBC = ar (EYCB) ...(2)
2 l1 l 2 l12 + l 22
Therefore, + =
now consider DAFC, l 2 l1 l1l 2
ar DAFC = ar DBFC,
( l1 + l 2 )
2
as they both lie on same base FC and - 2l1l2
=
between parallel lines AB and FL. l1l2
Q BF is diagonal of BXFC,
(16)
2
-2
1 =
\ ar DAFC = ar (BXFC) ...(3) 1
2
= 256 – 2 = 254
from (1), (2) and (3)
we get é l1 l 2 ù
êl + l ú é 254 ù
ar DEBC = ar DAFC Now, ê 2 1
ú=ê = [ 25.4] = 25
ë 10 û ë 10 ûú
or ar DABE = ar DAFC.
13. (b) The given equation can be written as 14. (b) Time taken by bucket to ascend = 1 min, 28
sec. = 88 sec.
lx 2 - ( l - 1) x + 5 = 0 speed = 1.1 m/sec.
Telegram @unacademyplusdiscounts

Join Us on Telegram for More Such Books

https://telegram.me/unacademyplusdiscounts

Join us from the above link or search ''unacademyplusdiscounts'' in Telegram


EBD_7839
148 KVPY-SA

Length of the rope = Distance covered by x


bucket to ascend From eqn (1) a = cos wt
1
= (1.1 × 88) m = (1.1 × 88 × 100) cm
and from eqn (2)
= 9680 cm.
y
= cos 2 wt =[2cos2 wt –1]
77 a2
Radius of the wheel = cm.
2 2
æ xö
y = 2a 2 ç ÷ – a 2
è a1 ø
17. (d) The acceleration of the particle at
equilibrium position is zero and is maximum
at the extreme position.
pr 2 p (6) 2
18. (b) WABC = = = 18p
2 2

p ´ 32 p ´ 32
WDEF = + + (15 - 12) ´ 18
2 4
= 6.75p + 54
WDEF > WABC
19. (a) The motion of the train will affect only the
horizontal component of the velocity of the
ball. Since, vertical component is same for
Circumference of the wheel both observers, the h m will be same, but R
will be different.
æ 22 77 ö
= 2pr = ç 2 ´ ´ ÷ cm = 242cm 20. (d) We know that, to cross the river by shortest
è 7 2 ø u
path; sin a =
\ Number of revolutions v
where u = river flow speed, v = speed of
Length of the rope æ 9680 ö person.
= = = 40
Circumference of the wheel çè 242 ÷ø But as given in question u > v Þ sin a > 1,
Hence, the wheel makes 40 revolutions to which is not possible.
raise the bucket. 21. (a) KE and PE completes two vibration in a time
during which SHM completes one
15. (c) xn + ax + b = ( x - x1 )( x - x2 ) ¼( x - xn ) vibration. Thus frequency of PE or KE is
double than that of SHM.
x n + ax + b
Þ ( x - x2 )( x - x3 ) ¼( x - xn ) = 22. (b) As block B falls, L will decrease.
( x - x1 ) Block B will displace, volume of liquid (V1)
Equal to its own volume when it is in the
Þ ( x1 - x2 )( x1 - x3 )¼¼( x1 - xn ) liquid.
MB
x n + ax + b V1 = VB = ...(i)
= lim = nx1n-1 + a rB
x® x1 x - x1
When block B is on block A, it will displace
the volume of liquid (V2) whose weight is
PHYSICS
equal to the weight of the block B.
16. (a) Two perpendicular S.H.Ms are V2rLg = MBg = (V1rB)g or V2rL = V1rB.
x = a1 cos wt ....(1) Since H increases, V1 > V2
and y = a2 cos 2wt ....(2) So, rL > rB.
SOLUTIONS – MOCK TEST-5 149
23. (d) As maximum height attained by each one is From eqs (ii) and (iii)
same, so uy is also same. As
2u y 1 2m 2 - 1
T= , =
g m 2m 2
So T1 = T2 = T3.
Squaring both sides and then solving, we
1 1 get
24. (d) sin C = =
m 2
3
-1 æç 1 ö m=
\ C = sin ç ÷÷ = 45 º 2
è 2ø
sin C 1 sin 45º 1
26. (d) g = 4p 2 L / T 2 ;
Now = or =
sin r m sin r 2 t Dt
Here, T = and DT = .
sin r = 1 or r = 90º n n
25. (d) At point A by Snell’s law DT Dt
Therefore, = .
sin 45° 1 T t
m= Þ sin r = … (i)
sin r m 2 The errors in both L and t are the least count
errors. Therefore,
At point B, for total internal reflection, (Dg/g) = (DL/L) + 2 (DT/T )
1 0.1 æ 1ö
sin i1 = = + 2 ç ÷ = 0.027
m 20.0 è 90 ø
Thus, the percentage error in g is
From figure, i1 = 90° - r 100 (Dg/g) = 100 (DL/L) + 2 × 100 (DT/T )
= 3%.
27. (a) Let the speed of the instrument package is
45° v when it grazes the surface of the planet.
Air Conserving angular momentum of the
A package about the centre of the planet.
i1
90° m mv0 × 5R sin(p – q) = mvRsin 90°
i2 Þ v = 5v0sin q .....(i)
B Conserving mechanical energy
GMm 1 2
1 - + mv0
\ (sin 90° - r ) = 5R 2
m v0
GMm 1 2
1 = - + mv m q
5R
R
Þ cos r = … (ii) R 2
m

1
Þ
1
2
(
m v 2 - v02 =) 4GMm
5R
Now cos r = 1 - sin 2 r = 1 -
2m 2 2 2 8GM
Þ v - v0 = .....(ii)
5R
2m 2 - 1 Substituting the value of v from eq. (i) in equation (ii)
= … (iii) 8GM
2m 2 25v02 sin 2 q - v02 =
5R
EBD_7839
150 KVPY-SA

The angle, package just graze the surface 4T


of the planet, Therefore Pexcess = , is more for the
r
smaller bubble.
-1
æ1 8GM ö
q = sin ç 1+ ÷ Therefore, when valve T is opened, then air
ç5 5v02 R ÷
è ø rush from smaller to the bigger bubble. The
smaller bubble shrinks and the bigger
mdv
28. (a) Since, F = =av2 bubble expands.
dt
CHEMISTRY
GM é R nR ù
R t ê- + úm 31. (b) – I group destablises carbocation, and
dv ë GM GM û =t
m ò = a ò dt Þ since inductive effect decreases with
GM
v2 0 a
increasing length of carbon chain. Therefore
nR
(b) is the correct option.

or, t =
m R ( n -1 ) 32. (a) Suppose there are x-atoms of N. then
a GM x ´ 14
29. (b) Let wA and wB be the absolute angular % of N = 28.9 = ´ 100 Þ x = 4
194
speeds of A and B. Since they are in the
same orbit, their time periods must be the 33. (b) The main gas responsible for the global
same, i.e., wA = wB. Considering the warming is carbon dioxide. Other contributors
dynamics of circlar motion in the cases, include methane released from landfills and
g agriculture, nitrous oxide from fertilizers etc.
GMm
2
mwA R = Þ wA =
R 2 R
(Q GM = gR2) 34. (a) is aromatic

g +
Similarly wB =
R
2pe–
9.8
And wA = wB = (4 ´ 0 + 2) pe– in conjugation
6.37 ´106
= 124 × 10–5 rad s–1
Now wAE = wA – wE = 124 × 10–5 – 7.3
and are antiaromatic
× 10–5 = 116.7 × 10–5 rad s–1 + –
wBE = (velocity of B relative to E)
= wB – (–wE) = wB + wE 4pe– 8pe–
= 131.1 × 10–5 rad s–1
(4 × 1) pe– (4 × 2) pe–
1
mω 2ΒΕ r 2
K
Therefore, B = 2 131.32
= = 1.27 is non aromatic due to the presence
KA 1 2
mω 2ΑΕ r 2 116.7
2 of sp3 carbon.
30. (a) When the stop cock T is opened, then air
35. (d) The subshell, whose n value is 3 and l
will rush from higher pressure region. For
value is 2, is 3d. 3d-subshell contains
the smaller bubble, radius is less
maximum 10 electrons.
i.e. r < R.
SOLUTIONS – MOCK TEST-5 151

36. (d) Higher the negative value of reduction 44. (d) Dipole moment cancels each other in BeF2
potential, more will be the reducing power. because of linear shape and thus it is non-
According to the given information, polar, whereas H2O is V-shape and is polar.
reduction potential of Fe2+, Al, and Br – are F — Be — F
–0.77V, + 1.66V and –1.09V respectively.
Hence, The correct order is BeF2
Al < Fe2+ < Br–
37. (d) Tautomerism is exhibited by the oscillation
O
of hydrogen atom between two polyvalent
atoms present in the molecule. As (d) has H H
a-hydrogen atom ther efore it shows
H 2O
tautomerism whereas other structures do
not. 45. (a) The stability of dihalides (MX2) increases
down the group. Except C and Si, the other
O OH members form dihalides.
R CH2 N R CH N
Nitro form O aci-Nitro form O BIOLOGY
38. (a) The beans are cooked earlier in pressure 46. (c) In a given DNA segment, 200 guanine
cooker because boiling point increases with bases will pair with 200 cytosine bases.
increasing pressure. Similarly, 200 thymine bases will pair with
39. (d) Order of acidic strength is 200 adenine bases. Thus, total number of
CHºCH > CH3—CºCH > CH2=CH2 nucleotides in the given segment of DNA
is 800.
40. (b) Milliequivalents of NaOH = 10 × 0.1 = 1
47. (a) During meiosis I, DNA content gets
Milliequivalents of H2SO4 = 10 × 0.05 = 0.5
reduced by half and after meiosis II further
2NaOH + H 2SO 4 ¾¾ ® Na 2SO4 + H 2O
it reduced by half.
2 equ. 1 equ.
48. (d) The organs which are present in reduced
\ 1 equivalent of NaOH reacts with 0.5 eq.
form and do not perform any function in
of H2SO4 to give neutral (pH = 7) solution.
the body but correspond to the fully
41. (d) (i) dsp2 hybridisation, number of 90° angle developed functional organs of related
between bonds = 4
animals are called vestigial organs. They
(ii) sp3d or dsp3 hybridisation, number of
are believed to be remnants of organs which
90° angle between bonds = 6
(iii) sp3d2 hybridisation, number of 90° angle were complete and functional in their
between bonds = 12 ancestors. Some of these are nictitating
42. (c) The given reaction is exothermic in nature, membrane, muscles of pinna (part of external
due to release of heat. An exothermic ear), vermiform appendix, caudal vertebrae
reaction is favoured by low temperature to (also called coccyx or tail bone), third molars
proceed in forward direction. (wisdom teeth), hair on body, and nipples
in male.
49. (c) Oxidative phosphorylation refers to the
synthesis of ATP from ADP and inorganic
43. (d) phosphate by chemiosmosis. It occurs with
the help of energy obtained from oxidation
of reduced enzymes formed in cellular res-
piration.
EBD_7839
152 KVPY-SA

50. (a) A histone octamer is a complex of eight symptoms. Symptoms and signs of disease
positively charged histone proteins (two may include vaginal discharge, penile
of each H2A, H2B, H3 and H4) that aid in discharge, ulcers on or around the genitals
the packaging of DNA. Negatively and pelvic pain.
charged DNA wraps around the histone 55. (a) B-lymphocytes are involved in humoral
octamer to form the nucleosome. The DNA immunity by producing antibodies.
is held there by ionic bonds. Linker 56. (b) Depending on the shifting of plane of
histone H1 binds to each nucleosome polarised light, the sugars are denoted as
where the DNA enters and exits and this ‘L’ or ‘D’ forms. The D and L forms are mirror
draws a string of nucleosomes closer images based on the penultimate carbon
together to form the 10 nm fibre. The atom.
nucleosomes in chromatin are seen as 57. (b) Splitting of water in photosynthesis is
beads-on-string structures when viewed called photolysis.
under electron microscope. 58. (a) Sucrose is composed of glucose and
51. (b) Due to direct chemical control on respiratory fructose.
centres, CO2 stimulates respiratory centres 59. (a) In 1953, Stanley Miller and Harold Urey
in CNS. created, in the laboratory, conditions
52. (d) Cytochromes are small proteins (intrinsic comparable to those of early Earth, with
membrane proteins) that contain a cofactor, water vapour, hydrogen, methane, and
haem which holds an iron atom. The iron ammonia. The Miller-Urey apparatus
carries electrons between + 2 and + 3 produced a variety of amino acids and other
oxidation states. These form a part of organ ic compounds found in living
electron transport chain in mitochondria and organisms today.
chloroplast and act as an electron 60. (d) Silicon, like carbon, needs four electrons to
transporter or electron acceptor in fill its outer shell and could, therefore, form
respiration and photosynthesis. similar compounds.
53. (a) Anaphase I with its feature is correctly
PART-II
matched.
Metaphase I: Spindle apparatus appears MATHEMATICS
and the chromosomes are arranged on 61. (a) DABC is rt D, as AB is diameter
equatorial plate, with the centromeres
\ ÐABC = 90° – 24° = 66°
towards the pole. Spindle fibres become
attached to the centromeres of the two Also ÐADE = ÐEDC = a
homologous chromosomes. by Alternate segment theorem
Interphase: It is the time during which cell ÐADE = ÐDCE = a
is preparing for division by undergoing Q ÐACB = 90°
both cell growth and DNA replication.
\ a + q = 90° ...(1)
Prophase I: It is the lengthy phase when
also In DDBC,
compared with mitotic phase. It is
subdivided into 5 sub-phases: leptotene, ÐD + q + ÐB = 180°
zygotene, pachytene, diplotene and Þ ÐD + q = 114°
diakinesis. Þ ÐD = 114° – q
54. (c) Sexually transmitted diseases (STDs) are Q AB is a straight line
caused by infections that are passed from
\ 2a + 114° – q = 180°
one person to another during sexual
contact. Most STDs initially do not cause Þ 2a – q = 66° ...(2)
SOLUTIONS – MOCK TEST-5 153

from (1) and (2) Also, ar DADE = ar DAQD, as they both lie
3a =156° on same base. AD and between the two
parallel lines.
Þ a = 52°
\ ar (ABCDE) = ar (DAPC) + ar (DACD) + ar
Þ q = 24°
(DAQD)
62. (c)
A \ ar (ABCDE) = ar (DAPQ).
65. (c) Highest power of 10 in N! is 28 hence
120 £ N < 124
P
If N = 120, 121, 122, or 123 then (N + 1)! Will
Q have Highest power of 10 in N! is 28 and if N
E = 124 then (N + 1)! Will have highest power
H of 10 as 31.

C PHYSICS
B D R
Q Q, R are mid points of AB & BC. 66. (b) For the deflection of spring by x, the ener-
\ We can say QR || AC gy stored in the spring,
(By converse of Mid-pt. theorem) x
\ÐBHR = ÐBEC = 90º U= ò Fdx
0
Q QR is a line
\ ÐBHQ + ÐBHR = 180º x
x3
= ò 300 x 2 dx = 300 = 100 x3
Þ ÐBHQ = 90º 3
0
also ÐPQR = ÐBHQ = 90º(alternate angles)
Thus by conservation of mechanical ener-
as Q, P are mid points of AB & AH
gy, we have
63. (b) Let A = {q : sin q = tan q} and
1
B = {q : cos q = 1} 100 x3 = mv2
2
ì sin q ü
\ A = íq : sin q = ý 1
î cos q þ or 100 x3 = × 200 × (1.0)2
2
= {q : sin q (cos q – 1) = 0}
or x=1m
= {q = 0, p, 2p, 3p,.....} The maximum restoring force acts on the car
For B : cos q = 1 Þ q = p, 2p, 4p,...... F= 300 x2 = 300 (1)2 = 300 N
This shows that A is not contained in B. i.e. F 300
A Ë B. but B Ì A. \ Deceleration of the car = = = 1.5 m/s2
m 200
64. (a) ar (ABCDE) = ar DABC + ar DACD + ar
5
DADE 67. (c) Here, V1 = 234 ´ = 65 ms -1
18
ar DABC = ar DAPC, as they both lie on
same base AC and between two parallel 5
V2 = 180 ´ = 50 ms - 1 , A = 25 m2
lines. 18
EBD_7839
154 KVPY-SA

Applying Bernoulli’s theorem above and \ Using work energy theorem,


below the wings (WAB)electric= Q(VA – VB)
1 1 2kqQ é 1 ù
P1 + rv12 = P2 + rv22 1-
a êë ú
2 2 =

v1 = 234km/h æ 1 ö 2Qq é 1 ù
= ç 4pe ÷ a ê1 - ú
è 0 ø ë 5û
69. (b) Using Snell’s law (at origin),
v2 = 180 km/h
sin 90° 1
m= or sin q =
1 1 sin q m
( P2 - P1 ) = r (v12 - v22 ) = ´ (652 - 502 )
2 2
x
1 1 1 -
-2
= (65 - 50) (65 + 50) = 862.5 Nm \ tan q = = =e 2d
2 m2 -1 1 + e x/ d - 1
\ Upward thrust on the two wings
= 862.5 × (25 × 2) dy - x /2 d
or =e
This force supports the weight of the aeroplane dx
i.e., m × 9.8 = 862.5 × 50 d x
- x /2 d
or m=
862.5 ´ 50
= 4400 kg
or ò dy = ò e dx
0 0
9.8
68. (d) Potential at point A, \ x = d ln 4.
70. (a) Let A be the area of cross-section of the
2kq 2kq
VA = - tube.
a a 5
Since temperature is the same, applying
kq Boyle's law on the side AB
\ (potential due to each q = and
a P × (x × A) = P2 × (x2 × A)... (i)
Applying Boyle's law in section CD
-kq
potential due to each – q = ) P × (x × A) = P1 × (x1 × A)... (ii)
a 5
From (i) and (ii)
2a
q q P1 × (x1 × A) = P2 × (x2 × A)
A
Þ P1x1 = P2x2
where P2 = P1 + Pressure due to mercury
2a column
B
x2
m
5c P1
x x x1 D
–q –q
Potential at point B, C
P P P2
B
VB = 0 30O
A B C D
(\ Point B is equidistant from all the four 5cm A
charges) Pressure due to mercury column
SOLUTIONS – MOCK TEST-5 155

F mg sin 30° Vdg sin 30° RT 0.082 ´ 373


P= = = V= = = 30.58 L ; 30.6 L
A A A P 1

(A ´ 5) ´ dg sin 30°
= =5 sin 30° cm of Hg
A
74. (b) > > >
P2 = P1 + 5 sin 30° = P1 + 2.5
Substituting this value in (iii)
P1 × x1 = [P1 + 2.5] × x2
P1 × 46 = [P1 + 2.5] × 44.5

44.5 ´ 2.5
\ P1 = m-Xylene is more reactive than p-xylene
1.5
towards electrophilic aromatic substitution,
Substituting this value in (ii)
because of stable s-complex.
44.5 ´ 2.5 CH3 CH3
P×x = ´ 46
1.5

é 46 + 44.5 ù 44.5 ´ 2.5 + +


Þ P× ê
ë 2 úû = 1.5
´ 46 CH3 E
E CH3
x + x2 Stable (One methyl reinforces (One methyl counteracts
Þ P = 75.4 cm [ Q x = 1 ] the other) the other)
2
75. (d) Since molarity of solution is 3.60 M. It
CHEMISTRY means 3.6 moles of H2SO4 is present in its
1 litre solution.
gg
71. (b) In NH3 central atom is nitrogen which is
Mass of 3.6 moles of H2SO4
sp3 hybridized hence, it will be at the centre
of tetrahedron with H-atoms at three = Moles × Molecular mass
vertices. = 3.6 × 98 g = 352.8 g
\ 1000 mL solution has 352.8 g of H2SO4
N 29% H2SO4 by mass means 29 g of H2SO4
H H is present in 100 g of solution.
H
(Ammonia, NH3) \ 352.8 g of H2SO4 is present in

72. (a) S2O62– : 2x – 12 = –2 Þ x = 5 100


= ´ 352.8 g of solution.
SO32– : x – 6 = –2 Þ x = 4 29

S2O42– : 2x – 8 = –2 Þ x = 3 = 1216 g of solution


where x represents oxidation state. Mass 1216
Density = = = 1.216 g/mL
73. (c) Volume of 1 mole of an ideal gas at 273 K and Volume 1000
1 atm pressure is 22.4 L and that at 373 K and = 1.22 g/mL
1 atm pressure is calculated as ;
EBD_7839
156 KVPY-SA

BIOLOGY 79. (a) During protein synthesis, the sequence of


nucleotides in a messenger RNA (mRNA)
76. (d) According to Michaelis-Menten equation, molecule specifies the sequence of amino
the rate of an enzyme-catalysed reaction acids in a polypeptide. Ribosomes provide
depends on the substrate concentration. the site for the protein synthesis and tRNA
When the solution is diluted by adding reads the triplet code on mRNA and brings
water, the reaction rate is slowed down corresponding amino acids to the site of
because it becomes difficult for the enzyme protein synthesis. So, in the given situation,
and substrate to come in contact with each mRNA of human is added so it will specify
other. the sequence of amino acids so the nature
77. (b) Parents: AABBCC × aabbcc of polypeptide synthesised, will be of the
F1 genotype: AaBbCc nature of body cells of human.
If n is the number of heterozygous alleles, 80. (c) Recessive alleles are not lost from the
Number of F1 gametes = 2n = 23 = 8 population because they occur in
heterozygous individuals who are carriers.
Number of F2 genotypes = 3n = 33 = 27
Heterozygous and homozygous dominant
Number of F2 phenotypes = 2n = 23 = 8 both individuals will be able to survive and
78. (a) Tidal volume (TV) = 500 mL reproduce. In heterozygous individuals,
Expiratory residual volume (ERV) = 1000 mL both dominant and recessive alleles
Expiratory capacity (EC) = TV + ERV segregate during gamete formation and are
passed on to offsprin g with equal
= 500 + 1000
probabilities.
= 1500 mL
MOCK TEST-6
PART – I 3. (a) Since p is product of 5 consecutive number.
MATHEMATICS Hence p must be divisible by 5!
So p = 5!n = 120n
1. (c) We use the facts that the line joining the Then q = 120n + 2
midpoints of the sides of a triangle is parallel
Hence q must be even
to the third side and any median of a triangle
bisects its area and two triangles having When divided by 4 & 3 it leaves remainder 2
equal bases and bounded by same parallel So it is not divisible by 3
lines have equal area. Since it is even and it can’t be prime
(Note: [·] denotes area) 4. (b) Set is in the form of
C S = { 20, 21/2, 2, 23/2, ...... 210}
R Since P has 2 elements –
If 1st element is 210 then next element can
D X Q be any one of 1st 20 elements number of
O such sets = 20.
S If 1st element is 219/2 then next element
Y
can be any one from 21/2 to 29 number of
A P B such sets = 18
(i) Now BD is parallel to PS as well as OX. So If 1st element is 29 then next element can
OX is parallel to PS. be any one from 21 to 217/2 number of
Hence, [PXS] = [POS]. such sets = 16
Adding [PAS] to both sides we get And so on hence total number of elements
= 20 + 18 + 16 + …. + 0 = 110
[APXS] = [APOS].
(ii) Now, [APXS] = [APX] + [ASX] 5. (b) P

1 1
= [ ABX ] + [ ADX ]
2 2
1 1 B 17
= [ ABC ] + [ ADC ] 10 A
4 4 h
1
= [ ABCD ]. a a
4
1
Hence, by (i), [ APOS ] = [ ABCD]. Q R
2 c S b
Similarly, by symmetry each of the areas
[AQOP], [CROQ] and [DSOR] is equal to 21

1 Let ABCD side = a – AB = BC = CD = AD


[ ABCD ]. QD = c, CR = b
4
Thus, the four given areas are equal. DPQS ~ DAQD DPRS ~ DBRC
2. (b) Since 45! Contains 510 if a = 5 then we can
PQ PS PR PS
get 5(45!) is completely divisible by 511 Þ = ... (1) Þ = ...( 2)
(50! Contains 512) AQ AD BR BC
EBD_7839
158 KVPY-SA

10 + 17 + 21 Now we need to find the number of multiples


s= = 24 of 21 between 100 and 200, 200/21 quotient
2
is 9 and 100/21 quotients is 4, hence number
24 ´ ( 24 - 10 ) of multiples of 21 between 100 and 200 is
area ( DPQR ) = 9–4=5
( 24 - 17 )( 24 - 21)
Of these 5, 3 are odd and 2 are even. Here we
D = 84 are looking for odd multiples of 3, not
1 divisible by 7,
Þ ´ h ´ 21 = 84 Þ h = 8 And it is 16 – 3 = 13
2
From (1) 7. (a) Since abcd is divisible by 4 hence cd must
be divisible by 4 say 4k. Now consider
10 8 10 a acdb it is divisible by 8 that means cdb is
= a 2 + c2 =
2
a +c 2 aÞ 8 divisible by 8 or (10cd) + b is divisible by
8 but we know that cd = 4k hence cdb =
100a 2 40k + b it is divisible by 8 that means b = 0
Þ a2 + c2 = or 8. Since we need maximum value of
64
a + b + c + d hence b = 8, and cd = 88
36a 2 6a (8 + 8 = 16 is the maximum value). Then
Þ c2 = Þ c=
64 8 required maximum value will be
From (2) 9 + 8 + 8 + 8 = 33
8. (d) The easiest way to solve this question is
17 8 17 a
= 2
a +b = 2 that take some values of n such th at
a 2 + b2 a Þ 8 seed(n) =9 these are 18, 27, 36 etc, that means
multiples of 9, so what actually we need to
289a 2 15a find is the number of multiples of 9 less than
Þ a2 + b2 = Þ b=
64 8 500, So 500/9 and quotient is 55 hence we
\ a + b + c = 21 will have 55 such numbers.
9. (c) Q All 3 segments are congruent
6a 15a
Þ a+ + = 21 Area of 1 segment (PDC) is
8 8
Area segment (PDC)
29a 168
Þ = 21 Þ a = 1 2 æ 60º ´p ö
8 29 = ( 2) çè - sin 60º÷
ø
2 180º
4 ´ 168
Perimeter = 4a = = 23.2
29 1 æp 3ö
6. (d) It is given that x is odd, divisible by 3 but = ( 4) ç - ÷
2 è3 2 ø
not divisible by 7.
Consider the number of multiples of 3 Q 3 segments are there
between 100 and 200 (both included) is
æp 3ö
[200/3] – [100/3] = 33. \ Total area = 3 ´ ( 2) ç - ÷
è 3 2 ø
Out of these 33, 16 are odd and 17 are even
[because the starting multiple 102 and last
multiple 198 both are even] (
= 2p - 3 3 )
SOLUTIONS – MOCK TEST-6 159

10. (d) f(1) = 1; f(2) = 3; f(3) = 5; f(4) = 7; f(5) = 9 let the area of parallelogram ABCD = 16 sq.
f(1) – 1 = 0 f(2) – 3 = 0 units.
f(3) – 5 = 0 f(4) – 7 = 0 Construction 1. from Y, draw a line parallel
f(5) – 9 = 0 to AD. which cuts AB at M
Let there is a new polynomial We have YMAD as a parallelogram
p(x) = f(x) – (2x – 1) 1
p(x) has x = 1, x = 2, x = 3, x = 4, x = 5 Whose area is (ar of ABCD)
2
as its factors. \ ar YMAD parallelogram = 8 sq. units.
\ P(x) = (x – 1)(x – 2)(x – 3)(x – 4)(x – 5)
Q AY is the diagonal of parallelogram YMAD
= f(x) – (2x – 1)
\ ar DADY = 4 sq. units.
f(x) leading coefficient is 2009,
Construction 2. draw a line parallel to AB
p(x) leading coefficient will also be 2009.
from X. which meets AD at N
Þ p(x) = 2009(x – 1)(x – 2)
Similarly, we get ar DAXB = 4 sq. units.
(x – 3)(x – 4)(x – 5)
Construction 3. Join D to B,
= f(x) – (2x – 1)
as DB is diagonal, it will divide
for f(6), substituting x = 6
parallelogram in equal area.
Þ f(6) = 2009 × 5! + (12 – 1) = 241091
\ ar DBDC = 8 sq. units
11. (a) If ab is a two digit number then as per the
given condition a > b Q X and Y are mid points
When a = 1 then possible number 10 1
When a = 2 then possible numbers are 20, \ ar DXYC = (ar DBDC) = 2 sq. units
4
21 (i.e. 2 such numbers)
now ar (DAXY) = ar (ABCD) – ar (DADY) – ar
When a = 3 then possible numbers are 30,
(DXYC) – ar (DAXB)
31, 32 (i.e. 3 such number)
ar D(AXY) = 16 – (4 + 4 + 2) = 6 sq. units,
Similarly when a = 9 then possible numbers
are 90, 91,…. 98 (i.e 9 such numbers) 3
Hence total number of such two digit \ ar DAXY = ar (ABCD)
8
numbers are 1 + 2 + 3 + … + 9 = 45. And 45
13. (c) If x e – 0, then the given equation assumes
two digit numbers will take 90 digits, but we
the form,
have to consider only 75 digits.
So let us consider 1 + 2 + 3 + … + 8 = 36 and x2 + (a – 5) x + 1 = 0 …..(1)
36 two digit number will use 72 digits. If x < 0, then it takes the form,
Hence if N has 75 digits then after 72 digits x2 + (a + 1) x + 1 = 0 …..(2)
numbers that starts with 9 will appear so For these two equations to have exactly
next numbers will be 90, 91 etc, three distinct real solutions we should have
Hence N = 102021303132 …… 87909 and last (I) either (a – 5)2 > 4 and (a + 1)2 = 4;
four digit is 7909.and hence remainder when (II) or (a – 5)2 = 4 and (a + 1)2 > 4.
7909 divided by 16 remainder is 5. Case I : From (a + 1)2 = 4, we have a = 1
D Y C
or –3
12. (c)
But only a = 1 satisfies (a – 5)2 > 4.
Thus, a = 1
N
X Also, when a = 1, eq. (1) has solutions
x = 2 + ; and eq. (2) has solutions x = –1,–1.
A B As 2 ± > 0 and –1 < 0, we see that a = 1 is
M
indeed a solution.
EBD_7839
160 KVPY-SA

Case II: From (a – 5)2 = 4, we have a = 3 or 7.


3 6
Both these values of a satisfy the inequality Hence, side AB = 2 ´ =
p p
(a + 1)2 > 4.
When a = 3, eq. (1) has solutions x = 1, 1 and Area of DABC
eq. (2) has the solutions x = –2 ± 3 3 36 9 3
As 1 > 0 and –2 ± < 0, we see that a = 3 is in = ´ side2 = ´ = .
4 4 p p
fact a solution.
15. (c) The given expression can be written as
When a = 7, eq. (1) has solutions x = –1, –1,
which are negative contradicting x e–0. 2
x -1 2y 2
3sec y2 - + £1
Thus, a = 1, a = 3 are the two desired values. 3 9
14. (a) Area of outer circle = p.OA2
2
and area of inner circle = p×OD2 Þ 3sec
2 æ 1ö 1
çè y - ÷ø + £ 1
x

As given p×OA2 = 4×pOD2 3 9

where sec2 x ³ 1
OD 2 1 OD 1
Þ = Þ =
OA2 4 OA 2 2
æ 1ö 1 1
Þ 3sec 2 x ³ 3 and çè y - ÷ø + ³
OD 1 3 9 3
sin ÐOAD = = \ÐOAD = 30°
OA 2 So, we should have and
\ ÐOAD = 30° 1
sec 2 x = 1 and y - =0
3
B
1
Þ x = 0, p, 2p, 3p and y =
D 3
O \ Total number of solutions is four.

A C PHYSICS
16. (a) Net power given to N2 gas = 100 – 30 = 70
cal/sec
[Since, ÐBAC = 2ÐOAD ] The nitrogen gas expands isobarically.
Hence, DABC is equilateral. dT
\ Q = nCP or 70 = 5 ´ 7 R dT
The next step is to find side AB and OD. dt 2 dt
dT
Since pr 2 = 12, we get, \ = 2K / sec
dt
12 1 12 17. (d) Given, F = cs -1/3
r = OA = and OD =
p 2 p
F c -1/3
Using Pythagoras theorem, we get, Acceleration, a = = s
m m
12 1 12 9 dv c -1/3
DA2 = OA2 – OD2 ; = - ´ = or v = s
p 4 p p ds m
v s
3 c -1/3
Þ DA = or ò vdv = m ò s ds
p 0 0
SOLUTIONS – MOCK TEST-6 161

2 2
v 2 c s2/3 at x = 0, F = r Aw l and f = F = rw l
2 2
= 8
2 m 2/3 A 8
Rupture of rod will occur when f = s
23c 2 /3
or v = s 8s
m rw 2 l 2
\ =s Þw = or n
8 rl 2
or 1/3
v = ks
1 8s
Now power = Fv =
2p rl2
= cs -1/3 ´ ks1/3 = s0 19. (b) Angular momentum of M2 about O is
18. (a)

The stresses are zero at the free end and M2vR


maximum at the axis. Therefore the rod will
ruptures at the middle. L2 = M2vR clockwise
Let us consider an element of rod at a Angular momentum of M1 about O is
distance x from the axis, the mass of element L1 = M1vR clockwise
dm = rAdx Ldisc = Iw clockwise
\ Ltotal = L1 + L2 + Ldisc
Applying Newton’s second law
1
= M2vR + M1vR + MR2w
2
Now, v = Rw as rope does not slip
é 1 ù
\ L = ê M 2 + M1 + M ú vR
ë 2 û
Therefore, k = 2
F - ( F + dF ) = (dm)an
20. (c) The ball will stop after a long time. The
or - dF = (rAdx)w2 x final displacement of the ball will be equal
to the height. The motion is first
where w is the rotation speed. accelerated, then retarded, then accelerated
and so on. Hence the correct graph is (c).
\ ò ò
2
F = - dF = - (rAdx)w x
21. (c) For isotopes Z is same and A is different.
Therefore the number of neutrons A-Z will
x2
= -rAw2 +C also be different.
2
22. (a) In ohm's law, we check V = IR where I is the
at x = l/2, F = 0 current flowing through a resistor and V is
the potential difference across that resistor.
rAw 2l 2 Only option (a) fits the above criteria.
\ C=
8
Remember that ammeter is connected in
series with resistance and voltmeter parallel
Aw2 æ l 2 2ö
Now F = r ç -x ÷ with the net resistance.
2 è 4 ø
EBD_7839
162 KVPY-SA

A A 28. (c) i = neAvd and vd µ E (Given)


V V or, i µ E
i2 µ E i2 µ v
Hence graph (c) correctly depicts the V-I
graph for a wire made of such type of
23. (d) I1 D12 t1 = I 2 D22 t2 [D = diameter of aperture] material.
29. (b) x = h cot q
L
Here, D is constant and I =
r2

L1 L
So, 2
´ t1 = 22 ´ t2
r1 r2

60 120
Þ ´ 10 = ´ t Þ 20sec = t
(2) 2 (4) 4
24. (b) In the space, the external gravity is absent, dx
but there will be a very small gravitational \ = h d (cot q)
dt dt
force between the astronauts, due to which
both will move toward each other with a dq
= h (–cosec2q)
very small acceleration so, the best correct dt
answer should be (b). æ - dq ö
or v = h cosec2q ç
25. (d) This is the defect of hypermetropia. è dt ÷ø
26. (b) Using Newton’s second law,
= (8 × 103) cosec260° × 0.025
= 266.67 m/s
= 960 km/h
30. (a) Here, Mw = 28, T = 300 K,
R = 8.31 × 107 erg mol–1 K–1
Kinetic energy of 2g of nitrogen,

3 RT 3RT 3 ´ 8.31 ´ 10 7 ´ 300


= 2´ = =
2 Mw Mw 28

= 267.1 × 107 erg


kx = mw 2 ( l + x )
CHEMISTRY
or kx = mw 2 l + mw 2 x
31. (b) Both He and Li+ contain 2 electrons each,
mw 2 l therefore their spectrum will be similar.
\ x=
( k - mw ) 2 32. (a) Compounds (b), (c) and (d) are cyclic
planar and have 6p electron (4×1+2pe–) in
complete conjugation and thus are
DM aromatic.
27. (a) Fractional error = as DM A = DM B
M
DM A DM B : :
as M B > M A Þ M > M gg
A B N
H
SOLUTIONS – MOCK TEST-6 163

HOCH2 O OH
Compound (a) is anti-aromatic as H H
H H
it has 4p electrons in complete conjugation OH OH
and doesn’t obey Hückel rule. Ribose
5
33. (b) For compounds containing cations of same HOCH2 O OH
charge, lattice energy increases as the size
4 1
of the cation decreases. Thus, NaF has H H
highest lattice energy. The size of cations H H
3 2
is in the order : Na+ < K+ < Rb+ < Cs+ H
OH
34. (c) Hydrogen is a much lighter element than 2-Deoxy ribose
alkali metals or halogen. From the structures it is clear that 2nd carbon
35. (a) 2Ca + O2 ¾¾
® CaO (ionic compound) in DNA do not have OH group.
44. (c) KHSO3 furnish H+ ions in solution,
CaO + H 2 O ¾¾
® Ca(OH) 2 therefore, it is acidic.
45. (c) It has four O atoms as peroxide with
ˆˆ† 2NO; Dn = 0
36. (c) N 2 + O 2 ‡ˆˆ oxidation number –1 and one O atom with
37. (d) For p sub-shell, m = – 1, 0, + 1. Therefore, oxidation number –2.
m = – 2 orbital will not be present in Hence, x + 4(–1) + 1 (–2) = 0 or x = +6
p sub-shell.
BIOLOGY
38. (d) Order of screening effect is s > p > d > ¦.
39. (b) BH3 has sp2 hybridization and hence does 46. (c) The normal range of pH of arterial blood is
not have tetrahedral structure while all 7.35 – 7.45. As the pH decreases (< 7.35), it
others have tetrahedral structures. implies acidosis, while if the pH increases
(> 7.45), it implies alkalosis.
40. (d) Isothermal expansion
47. (c) Plants require a periodic exposure to light
PVm = K (graph - C) to induce flowering. This light period is
K called photoperiod. On the basis of
P = V (graph - A) photoperiod, some plants are long day
m
plants, some are short day plants and some
are day neutral plants.
48. (c) Yellowing and blackening of Taj Mahal at
P U
Agra is due to SO2 and other pollutants
released by Mathura refinery.
Vm Vm
49. (b) Magnesium is an essential element. It is
Therefore (d) is not correct representation. present in enzymes and every cell type.
These en zymes eg. hexokinase,
41. (c) Diamond is a crystalline solid which
phosphofructokinase etc., require
contains only carbon-atom.
magnesium ion for their catalytic action.
Wt ´ 1000 5 ´ 1000 50. (d) Adrenal gland, located on top of each
42. (a) M = = = 0.5
M.Wt ´ V 40 ´ 250 kidney, does not involve in the process of
digestion. It is an endocrine gland which
43. (b) RNA has D (–) – Ribose and the DNA has
secretes hormones like adrenaline and the
2–Deoxy D (–) – Ribose as the
steroids aldosterone and cortisol.
carbohydrate unit.
EBD_7839
164 KVPY-SA

Liver is involved in the process of digestion 59. (a) Hormones are chemical messengers which
by secreting bile which helps in the are secreted from endocrine glands. They
emulsification of fats. Pancreas secretes are secreted directly into the blood which
digestive juices containing many digestive carries them to organs and tissues of the
enzymes. It plays an essential role in body. These hormones stimulate the glands
to do their functions.
converting the food we eat into fuel for the
60. (c) Plants are autotrophs and synthesise their
body’s cells.
food by the process of photosynthesis with
51. (b) Although contraction of smooth muscle is the help of chloroplast (plastid). Plastids are
controlled differently from that of skeletal not found in animals.
muscle but these muscle contains actin and
PART – II
myosin. Unlike cardiac and skeletal muscles,
smooth muscles do not contain the calcium- MATHEMATICS
binding protein troponin. 61. (a) ÐA = ÐB = ÐC = 60°
52. (a) Myoglobin is the main oxygen-carrying MN = QR = 2r in quadrilateral BTQM
molecule present in skeletal muscle.
ÐB + ÐBMQ + ÐMQT + ÐQTB = 360°
53. (c) Endospores are dormant, tough, non-
reproductive structures produced by small 60° + 90° + 90° + ÐQTB = 360°
number of bacteria. The primary function Þ ÐQTB = 120°
of these is to ensure survival of bacterium DBTQ @ DBQM
through extreme environmental stress.
A
54. (c) Food (carbohydrates) is translocated from
leaves to roots along the phloem in form of
sucrose.
P a
55. (d) Glucose and galactose are stereoisomers a
r r
(have atoms bonded together in the same
T r
order, but differently arranged in space). R
Q
They differ in their stereochemistry at r r
60° r r 90°
carbon 4. x 2r N
B M C
56. (d) Bamboo and grasses elongate by the a
activity of intercalary meristems. These
Þ ÐTQB = ÐMQB = 60°
meristems are found at the base of leaves
of many monocotyledons. Intercalary In DBQM
meristems are mainly responsible for the BM x
vertical growth of plant organs. tan 60° = =
QM r
57. (c) Amylopsin is pancreatic amylase which
hydrolyses polysacchar ides (except x
cellulose) into disaccharides (mostly 3= Þx=r 3
r
maltose) in alkaline medium.
58. (a) Micronutrients are the essential elements Similarly CN = r 3
needed by organisms in small quantities. BC = M + MN + CN = a
Micronutrients include boron (B), chlorine 2x + 2r = a
(Cl), molybdenum (Mo), sodium (Na),
Þ 2(x + r) = a
manganese (Mn), copper (Cu) and Zinc
(Zn). Þ 2(r 3 + r) = a Þ a = 2r( 3 + 1)
SOLUTIONS – MOCK TEST-6 165

62. (a) ÐDAB + ÐABC = 180°


1
QConsecutive interior angles. Q ar (ABNM) = ar (ABCD)
2
ÐABC = 180 – ÐDAB ..... (1)
Q SRDA is a square 1 1 1
Þ × m (a + x) = × × (m + n) (a + b)
ÐQAB = 90° ..... (2) 2 2 2
ÐSAD = 90° ..... (3)
Q ABPQ is a square 2 ( a + x) m+n
Þ =
ÐSAD + ÐDAB + ÐQAB + ÐSAQ = 360° a+b m
90° + ÐDAB + 90° + ÐSAQ = 360°
2 ( a + x) n
ÐDAB + ÐSAQ = 180° ..... (4) Þ =1+
a+b m
ÐSAQ = 180° – ÐDAB .... (5)
By (1) & (5)
2 ( a + x) a + x
ÐSAQ = ÐABC Þ - =1 (from (1))
a+b b+ x
63. (d) P = {q : sin q - cos q = 2 cos q}
Þ 2 (a + x) (b + x) – (a + x) (a + b)
sin q = ( 2 + 1) cos q , tanq = 2 +1
= (a + b) (b + x)
Q = {q : sin q + cos q = 2 sin q} Þ 2x2 = a2 + b2
Þ 2MN2 = AB2 + CD2
cos q = ( 2 - 1)sin q or tan q = 2 +1
65. (b) We know that only perfect squares can have
\ P=Q
odd number of factors.
64. (c) Let AB = a, CD = b, MN = x
Hence set S can be defined as
= {100, 121, 144, …., 961}
Product of all the elements is
100 × 121 × 144 × …. × 961
= 102 × 112 × 122 × …. × 312
Let distance between AB and MN be “m” (10 × 11 × 12 × .. × 31)2 = [(31!)/(9!)]2
and let distance between CD and MN be Since 12 = 22 × 31
“n”.
Highest power of 2 in 31! is 26 and that in 9!
1 Is 7 hence highest power of 2 in (31!)/(9!) is
given: ar (ABNM) = ar (ABCD)
2 26 – 7 = 19 and so highest power of 22 in
Þ 2ar (ABNM) = ar (ABNM) + ar (CDMN) (31!)/(9!) is 19.
Þ ar (ABNM) = ar (CDMN) Similarly highest power of 3 in 31! Is 14 and
that in 9! Is 4 hence highest power of 3 in
1 1
Þ × m (a + x) = × n (b + x) (31!)/(9!) is 14 – 4 = 10 and so highest power
2 2
of 3 in [(31!)/(9!)]2is 20.
m b+ x Therefore, highest power of 12 is 19
Þ n =a+x ...(1)
EBD_7839
166 KVPY-SA

PHYSICS 68. (b) The time period of a simple pendulum is


given by
2
66. (d) Terminal velocity, v = 2r (r – s) g ...(i) l l
9 h T = 2p \ T 2 = 4p 2
g g
s
But we know, v = l
t Þ g = 4p 2 2
T
s 2 2 (r – s) g 9s h
\ = r Þ r2 = Dg Dl DT
t 9 h 2t (r – s) g Þ ´ 100 = ´ 100 + 2 ´ 100
g l T
Given s = 2 × 10–2 m, t = 1h = 3600 s
Case (i) Dl = 0.1 cm, l = 64cm,
\ h = 1 ´ 10 –2 poise = 1× 10–3 kg m–1s–1 DT = 0.1s, T = 128s
Substituting given values, we get Dg
\ ´ 100 = 0.3125
g
9 ´ 2 ´ 10 –2 1 ´ 10 –3
r2 = ´
2 ´ 3600 ( )
1.8 ´ 103 – 1 ´ 103 ´ 10
Case (ii) Dl = 0.1 cm, l = 64cm,
DT = 0.1s, T = 64s
9 1 Dg
= ´ ´ 10 –10 \ ´ 100 = 0.46875
36 8 g
1 Case (iii) Dl = 0.1 cm, l = 20cm,
= ´ 10 –10 DT = 0.1s, T = 36s
32
Dg
100 \ ´ 100 = 1.055
\ r= ´10 –6 m = 1.77 ´10 –6 m and g
32
Diameter D = 2r = 2 × 1.77 mm = 3.54 mm Dg
Clearly, the value of ´ 100 will be least
g
67. (d) Let m and d be the mass and diameter of the
in case (i).
sphere, then the density r of the sphere is 69. (c) Total heat given to the system is Q – Q'. So
given by from first law of thermodynamics,
Q – Q' = total work done by the gas in both
mass m 6m
r= = = the chambers (W)
volume 4 æ d ö 3 pd3
pç ÷ + change in internal energies of both the
3 è 2ø gases (DU) ... (i)
Taking log and differentiating partially we Here W = sum of potential energies stored
get in the springs

dr dm 3d (d) é 1 æ l ö 2 ù kl 2
= ´ W = 2 ê kç ÷ ú =
r m d ... (ii)
êë 2 è 2 ø úû 4
æ dr ö dm 3d (d)
for ç ÷ = + or Since the temperature of left part remains
è r ø max m d constant (piston does not conduct heat),
internal energy of left part does not change.
æ dr ö dm 3d (d)
çè r ´ 100ø÷ = ´ 100 + ´ 100 DU of right part can be given as
max
m d
3
= 2 + 3 (3) = 11% DU = nC V DT = nRDT ... (iii)
2
SOLUTIONS – MOCK TEST-6 167

DT can be found from the condition of CHEMISTRY


equilibrium at the end of the process.
71. (b) Alkynes having terminal hydrogen react
Pressure on right side = pressure on left side with Na in liquid ammonia to yield H2 gas.
nR(T + DT) nRT 2K(l / 2) CH3CH2CºCH can react with Na in liquid
or = + NH3.
A(l + l / 2) A(l - l / 2) A
Na in
CH3CH 2 C º CH ¾¾¾¾¾
®
3kl 2 liquid NH3
Simplifying this we get, DT = + 2T
2nR
1
...(iv) CH3CH 2 C º C – Na + + H 2 ( g )
2
From equations (i), (ii), (iii) and (iv), we get
72. (b) Ba(OH)2 + CO2 ® BaCO3 + H2O
kl 2
3 æ 3kl 2ö Atomic wt. of BaCO3 = 137 + 12 + 16 × 3
Q¢ = Q - - nR ç + 2T ÷
4 2 è 2nR ø = 197
Wt. of substance
5 2 No. of mole =
or Q ¢ = Q - kl - 3nRT Mol Wt.
2
Q 1 mole of Ba(OH)2 gives 1 mole of BaCO3
70. (a) \ 0.205 mole of Ba(OH)2 will give 0.205 mole
of BaCO3
\ Wt. of 0.205 mole of BaCO3 will be
0.205 × 197 = 40.385 g » 40.5 g.
73. (b) Enthalpy of reaction
= B.E(Reactants) – B.E(Products)

i + r = 90°, and Ði = Ðr = éë B.E (C= C) + 4 B.E.(C–H) + B.E.(H - H) ùû

\ i = 45° - éë B.E.(C - C) + 6 B.E.(C - H) ùû


Also i + q = 90°
= [606.1 + (4 × 410.5) + 431.37)] – [336.49 +
\ q = 90° – i = 90° – 45° = 45° (6 × 410.5)]
Given y2 = 2 x = –120.0 kJ mol–1
dy 74. (c) Easily liquefiable gases have greater
or 2 y =2 intermolecular forces which is represented
dx
by high value of 'a'. The greater the value
dy 1 of 'a' more will be liquefiability.
\ =
dx y So, the order is Q < S < P < R.
75. (d)
1
or tan 45° =
y anhydrous AlCl3
+ CH2Cl2 + ¾¾¾¾¾¾®
\ y =1

y2 12 1 CH2 + 2HCl
Now x = = =
2 2 2 Diphenylmethane
Telegram @unacademyplusdiscounts

Join Us on Telegram for More Such Books

https://telegram.me/unacademyplusdiscounts

Join us from the above link or search ''unacademyplusdiscounts'' in Telegram


EBD_7839
168 KVPY-SA

BIOLOGY 78. (d) The periderm consists of three different


layers: inner phelloderm or secondary
76. (a) Semi-circular canal is interconnected tubes cortex which is inside of cork cambium and
located inside each ear. They are lined with composed of living parenchyma cells,
cilia and filled with a fluid known as middle phellogen or cork cambium which
endolymph. With every movement of head, is a meristem that gives rise to periderm
the endolymph moves the cilia. This works and outer phellem or cork which is dead at
as a type of motion sensor, as the movements maturity and composed of suberised cells.
of the cilia are communicated to the brain. 79. (d) The mother would have genotype IAIB and
The vestibule is the central part of the the father would have genotype ii. Thus,
osseous labyrinth and responds to gravity the two genotypes possible for their
and movements of head. Cochlea is the offspring would be IAi and IBi, with equal
probability. Hence, the expected
auditory portion of the inner ear. It is a spiral-
phenotypes would be 1 type A : 1 type B.
shaped cavity in the bony labyrinth.
Perilymph is an extracellular fluid located 80. (a) Diagram given in option (a) correctly
represents the transformation of P570 into
within the cochlea (part of the inner ear) in
P412 on absorbing a quantum of light along
two of its three compartments: the scala with the release of H+ and transformation
tympani and scala vestibuli. The major cation of P412 into P570 with the gain of H+. Also
(positively charged ion) in perilymph is the formation of ATP from ADP and Pi on
sodium. Endolymph is the fluid contained in movement of H+ from outside to inside of
the membranous labyrinth of the inner ear. the cell from F0-F1 system is correctly
77. (b) A recessive allele is not weaker than the depicted. Option (b) correctly represents
the formation of ATP from F0-F1 system
dominant allele. It (recessive allele) does
but cyclical changes in bacteriorhodopsin
not show its effect (in the presence of
are not correctly shown. Option (c)
dominant allele) because of modified or correctly represents the cyclical changes
different enzymes. A recessive allele makes in bacteriorhodopsin but not the formation
its gene product even when paired with of ATP from F0-F1 system. Option (d)
the dominant allele. It is not necessary that neither correctly depicts the cyclical
dominant allele is always better (in the case changes in bacteriorhodopsin nor the
of dominant disease). formation of ATP from F0-F1 system.
MOCK TEST-7
PART-I 6. (a) Draw AM ^ BD and CN ^ BD
MATHEMATICS area DAED × area DBEC =

1. (b) For real roots q2 – 4 p r ³ 0 æ1 ö æ1 ö


ç 2 ED ´ AM ÷ ´ ç 2 BE ´ CN ÷
2
è ø è ø
æ p+rö
ç ÷ – 4 p r ³ 0 (\ p, q, r are in A. P. ) 1
è 2 ø = ED × AM × BE × CN
Þ p2 + r2 – 14 p r ³ 0 4

p2 p æ1 ö æ1 ö
Þ – 14 + 1 ³ 0 = ç BE ´ AM ÷ ´ ç DE ´ CN ÷
r2 r è2 ø è2 ø
2 = area DABE × area DCDE
æp ö p
Þ ç - 7÷ - 48 ³ 0 Þ - 7 ³ 4 3. Hence, area DAED × area DBEC
èr ø r = area DABE × area DCDE
2. (b) Since unit digit of two digit numbers is 7. (d) AE + EB = CE + ED
same as that of its square hence b = 1, 5 or Þ AB = CD
6 and square is a four digit number hence also, OP ^ AB & OQ ^ CD D
ab ³ 32 \ AB = 2AP
So we have possible options CD = 2DQ
41, 35, 45, 36 & 46 O Q
Þ 2AP = 2DQ
By checking the squares of these 5 Þ AP = DQ B
numbers we will find only 45 Satisfy the E
OD = OA = radius P
A
given condition with 452 = 2025 In DOAP
C
3( 3n - 1) In DODQ
3. (d) The value of A = since 3n – 1 is OP2 = OA2 – AP2 OD2 = OQ2 + DQ2...(2)
2 2 2 2
OA = OP + AP ...(1)
not divisible by 3 hence A has only single
OA2 = OD2
power of 3 and for any value of n it cannot
Þ OP2 + AP2 = OQ2 + DQ2
be a perfect square.
(Q AP = DQ, \AP2 = DQ2)
4. (d) 2x + y = 10
(x + y) + x = 10 Þ OP2 = OQ2 Þ OP = OQ
x + y is max when x = 0 8. (b) In right angled DABC,
\ (x + y)max = 10 AB 2 + AC 2 = BC 2 [By Pythagoras
2x + y = 10 theorem]
2(x + y) – y = 10 \BC = 5 units
\ x + y is min. y = 0 Area of shaded region
(x + y)min. = 10 = 5 = area of semicircle on side AB
2 + area of semicircle on side AC
\ (x + y)max. + (x + y)min. = 10 + 5 = 15 – area of semicircle on side BC
5. (a) Since xi will be either +1 or – 1 hence each + area of DABC
term must be either +1/–1
pr 2
Since the sum of all the terms is zero, out of Area of semicircle on side AB =
the n products that are being added, half 2
should be +1 and the other half should be 22 9 1 99
–1, so n must be an even number. = ´ ´ = sq. units
7 4 2 28
EBD_7839
170 KVPY-SA

22 1 From equation (1) & (2)


Area of semicircle on side AC = ´ 4´
7 2 b + 2(3b) = 180

44 180
= sq. units b=
7 7
Area of semicircle on side
æ 180 ö 5
22 25 1 275 ÐAQP = 180° – 2 ç ÷= p
BC = ´ ´ = sq. units è 7 ø 7
7 4 2 28
10. (b) We have,
1
Area of DABC = ´ AB ´ AC area of square metal plate = 40 × 40 cm 2
2
= 1600 cm2
1 Area of each hole
= ´ 3 ´ 4 = 6 sq. units
2
2
Now, 22 æ 1 ö 11
= pr 2 = ´ ç ÷ cm2 = cm2
Area of shaded region 7 è2ø 14

99 44 275 \Area of 441 holes


= + - +6
28 7 28
11 2
Area of shaded region = 6 sq. units = 441 ´ cm = 346.5 cm 2
14

A Hence, area of the remaining square plate


9. (d)
= (1600 – 346.5) cm2 = 1253.5 cm2
b
11. (c) Let A had a rupee and b paisa, and B had
b c rupee and d paisa
Q P
a Initial amount with A = 100a + b paisa and
2b
that with B = 100c + d paisa
2b
a After interchanging amount with A
a C
B = 100b + a paisa and that with B
In DABC = 100d + c paisa.
AB = AC Increase in amount of A = 100b+a–100a–b
Þ ÐC = ÐB = 99(b–a)
Þ ÐB = ÐC = a Decrease in amount of B = 100c + d–100d–c
By angle sum properly in DABC, = 99(c–d) these two has to be equal hence
b + a + a = 180 99(b–a) = 99(c–d), so b + d = a + c, the
Þ b + 2a = 180° ...(1) maximum value that a, b, c, and d can take
In DQPB is 99 as there is no paisa more than 99. So
Þ ÐQPB = 180 – 4b maximum value with A and B is 99.99 but in
Since ‘APC’ is a straight line that case B is giving zero amount to A and
that is not the case here so for maximum
Þ 180 – 4b + a + b = 180
sum A had 98.99 and B had 99.98 and their
Þ a = 3b ...(2)
sum is 198.97.
SOLUTIONS – MOCK TEST-7 171

12. (b) Truncated cone ® Frustum 13. (c) If elements are not repeated, then number of
elements in A1 È A2 È A3 ... È A30 is 30 × 5.
But each element is used 10 times, so
30 ´ 5
S= = 15
10
If elements in B1, B2 , ....., Bn are not
repeated, then total number of elements is
3n but each element is repeated 9 times, so
3n
S=
9

Volume, V =
3
(
ph 2 2
r1 + r2 + r1r 2 ) Þ 15 =
3n
Þ n = 45
9
21 14. (b) If n = 10a + b then the ratio (10a + b)/a + b
New diameter, d2¢ = d 2 + ´ d2
is maximum. Maximum ratio is 10 & that
100
comes when b = 0 e.g 10,20.30…….90. So 9
21 such number exists.
= 100 + ´ 100 = 121 mm
100
15. (b) f ( f -1 ( B )) = { f ( x ) : x Î f -1 ( B )}
121
r2¢ = mm = { f ( x) : f ( x ) Î B}
2
r1, h ® same X Y
A B
21 121V
New volume, V ¢ = V + ´V =
100 100 f (A)

ph 2
( )
–1
f (B)
Þ r 1 + r ¢22 + r1r2¢
3
Þ f ( f -1 ( B )) Ì B
=
121 ph 2
100 3
(
r 1 + r 22 + r1r2 ) Now if x Î B Ì Y Þ x Î Y .
It may happen that f–1(x) does not Exist in x
2 2
Þ r 1 + r ¢2 + r1r2¢ =
100
(
121 2 2
r 1 + r2 + r1r2 ) as function is not given to be subjective.
f ( f -1 ( B )) ¹ B .
Þ 100r 12 + 100r ¢22 + 100r1r2¢
Also, f -1 ( f ( A)) = {x Î X : f ( x ) Î f ( A)}
= 121r 12 + 121r 22 + 121r1r2 but

121 121 12100 f ( A) = { f ( x ) : x Î A}


Þ 100 ´ ´ + r1 From above, we can't conclude
2 2 2
-1
f ( f ( A) Ì A.
= 21r 12 + 121 ´ ( 50 ) + 121 ´ 50r1
2
If the function is non-injective, then it may
Þ (121)2 × 25 – 121 × (50)2 = 21r12
happen that x Ï A and f ( x ) Î f ( A).
Þ 63525 = 21r12
Þ r1 = 55 mm Þ f -1 ( f ( A) Ë A Þ f -1 ( f ( A)) ¹ A.
EBD_7839
172 KVPY-SA

PHYSICS r r r
19. (b) Here, FAB + FBCDA = 0
16. (b) In portion OA, slope of x – t graph is r r r r
decreasing. Þ FBCDA = - FAB = - F (Q FAB = F )
\ Vel. is decreasing, acc. is negative. In 20. (a) A stretched catapult has elastic potential
portion AB, displacement is constant v = 0, energy stored in it
a = 0. In portion BC, slope of x – t graph is æ 1 YAl 2 ö
increasing. U = ç 2 L ÷÷ ´ 2
ç
\ vel. is increasing, acc. is positive. In è ø
portion CD, slope is constant, v = const., a This energy, when imparted to the stone, it
=0 flies off a height 20 m.
17. (a) The maximum possible error in Y due to l Energy possessed by the stone = mg(h + l).
Now,
DY Dl 2Dd
and d are = +
Y l d YAl 2
U = mgh Þ = mg(h + l) ; mgh
The least count L

=
Pitch mghL 1´ 10-1 ´ 9.8 ´ 25 ´ 10-1
\ Y= =
( )
Number of divisions on circular scale Al 2 2
10 ´ 10-6 ´ 5 ´ 10-2
0.5
= mm = 0.005 mm Young's modulus, Y = 9.8 × 107 N/m2
100
21. (a) The work done of thermodynamics,
Error contribution of l
3R R V0
Dl 0.005 mm 1 = – =R
= = 2 2
=
l 0.25 mm 50
Error contribution of d Q = DU + W P0

2Δd 2 × 0.005 mm 1 or 0 = nCv(DT) – P0V0


= = =
d 0.5 mm 50 R
or 0 = n g - 1 (T – T0 ) - nRT0
dq
18. (b) = 2 \ q = 2t \ T = gT0
dt 22. (d) Initially centre of gravity is at the centre.
Let BP = a, When sand is poured it will fall and again
\ x = OM = a sin q = a sin (2t) after a limit, centre of gravity will rise.
dx d2x 23. (d) Adding detergents to water helps in
Þ =a ´ 2cos (2t) Þ 2 = – 4a sin 2t removing dirty greasy stains. This is
dt dt
because dirt is held suspended surrounded
y
by detergent molecules.
B a 24. (d) Reynold’s number = Coefficient of friction
q
P = [M0L0T0]
l
Curie is the unit of radioactivity (number of
atoms decaying per second) and frequency
A also has the unit per second.
x
O x M
Q
Latent heat = and
m
Hence M executes SHM within the given
time period and its acceleration is opposite W
Gravitation potential = .
to x that means towards left. m
SOLUTIONS – MOCK TEST-7 173

25. (c) The distance travelled by each car is equal


2 æ RDisc ö 4 3
to the area under the graph. These areas are Þ pRDisc çè ÷ = pR
6 ø 3 Sphere
equal at 10.0 s.
é RDisc ù
26. (c)
N êt = 6 , given ú
ë û
q
3 3 RDisc
Þ RDisc = 8RSphere Þ RSphere =
2
mg Moment of inertia of disc
q
1 2
I Disc = MRDisc = I (given)
2
N cos q = mg and N sin q = mw2r \ M (RDisc)2 = 2I
Moment of inertia of sphere
w 2r
\ tan q = … (i) 2 2
g ISphere = MRSphere
5
Given y = x2 2
2 æ RDisc ö M 2I I
dy = Mç ÷ = ( RDisc ) 2 = =
\ = 2x 5 è 2 ø 10 10 5
dx
30. (c) Acceleration due to gravity at are altitude
or tan q = 2 × 1 = 2 … (ii)
From above equations, we get g R 2e
h is, g h = ; where Re is the
(R e + h) 2
w= 2g (r = 1 m)
radius of the earth
27. (d) S2 is correct because whatever be the g,
the same force is acting on both the pans. 9.8 ´ (6400 ´ 103 ) 2
Using a spring balance, the value of g is gh = = 8.4 m/sec2
(6400 ´ 103 + 520 ´ 103
greater at the poll. Therefore mg at the pole
is greater S4 is correct. S2 & S4 are correct. CHEMISTRY
28. (c) 31. (a) Rate of diffusion depends upon molecular
[m = 1] weight.
+x +x
I O I r1 M2
= Þ r1 = r2 if M1 = M2
d d d +A d +A r2 M1
(i) (ii) Hence, compounds are N2O and CO2 as
both have same molar mass.
From Fig. (i) and (ii), it is clear that if the
mirror moves distance 'A', then the image 32. (a) HSO-4 + OH - ¾¾
® SO 42– + H 2 O.
moves a distance '2A'. Conjugate base
acid
29. (c) According to problem disc is melted and
recasted into a solid sphere so their volume +1 -3 +3
will be same. 33. (d) H3PO2 ¾¾
® PH3 + H3 PO3

2 4 3 n1 = | – 3 – (1) | = |– 4| = 4
VDisc = VSphere Þ pRDisc t= pRSphere
3 n2 = | 3 – (1) | = | 2 | = 2
EBD_7839
174 KVPY-SA

n1 ´ n 2 4 ´ 2 4
n-factor = n + n = 4 + 2 = 3
1 2

mol mass M ´ 3
Eq. mass = =
n - factor 4
34. (b) The first ionization enthalpy will be
39. (b) For any given element, size of its negative
minimum if an atom after losing an electron
ion, netural atom and positive ion is in the
acquires stable configuration. Hence, option
order: negative ion > neutral atom >
(b) is correct.
positive ion.
H SO 40. (c) Hybridisation Shape
35. (a) CH3 – CH2 – CH2 – CH3 ¾¾¾¾
2 4
475 K
®
NO +2 sp Linear
CH3 – CH = CH – CH 3
More symmetrical (major product)
NO -2 sp2 Angular
H 2SO4
CH3 – CH2 – CH2 – CH3 ¾¾¾¾
475 K
® PCl5 sp3 d Trigonal
bipyramidal
CH 2 = CH – CH 2 – CH3 BrF5 sp3 d 2 Square
Less symmetrical or
unsymmetrical pyramidal
(minor product) XeF4 sp3 d 2 Square planar
It is based on Saytzeff’s rule. According to ICl -4 sp3 d 2 Square planar
this more symmetrical or more alkylated
alkene predominates. TeCl4 sp3 d Trigonal
36. (c) Proton donors are acids. Thus conjugate bipyramidal
XeO4 sp 3 Tetrahedral
base of an acid has one H+ less than the
acid. 41. (d) Sulphate of alkaline earth metal are
sparingly soluble or almost not soluble in
® Br - + H +
HBr ¾¾ water, whereas, BeSO4 is soluble in water
Acid Conjugate
base due to high degree of solvation. Be(OH)2
is insoluble in water but soluble in NaOH.
37. (a) Hydrogen bonding is possible only in
42. (d) DH = DE + PDV
compounds having hydrogen attached
with F, O or N. For isochoric process, DV = 0
C2H5 - OH (H-bonding possible) \ DH = DE

CH3 - O - CH3 (H-bonding not possible) 43. (a)

O
÷÷
CH3 - C - CH 3 (H-bonding not possible)
Correct IUPAC name of above compound
O is trans-2-chloro-3-iodo-2-pentene.
÷÷
CH3 - C - H (H-bonding not possible) 44. (d) 2I- ® I2 is oxidation (loss of electrons) ;
Cr (+6) changes to Cr (+3) by gain of
38. (c) Dipole moment is maximum for p–nitro electrons. Hence Cr is reduced.
aniline, since the dipole moment of nitro 45. (c) hn = hno + KE
group and amide groups adds up because For the above relation, the correct graph is
they are in same direction. option (c).
SOLUTIONS – MOCK TEST-7 175

BIOLOGY 53. (a) Process of passing out of urine is called


micturition. During discharge of urine,
46. (a) According to principle of dominance, out
urethral sphincter relaxes and smooth
of two factors or alleles representing
muscles of bladder wall contract gradually.
different traits of a character, only one
54. (b) If 30 percent of DNA is adenine then by
expresses itself. But when both express
Chargaff’s rule, 30 percent will be thymine.
themselves, it is co-dominance.
The remaining 40 percent of the DNA will
47. (d) Bilirubin and biliverdin are the pigments
be cytosine and guanine. Since the ratio of
present in the bile juice secreted from liver.
cytosine to guanine must be equal then
They provide yellowish brown colour to
each accounts for 20 percent of the bases.
the stool. So, malfunction of liver leads to
55. (a) Ribozymes (ribonucleic acid enzymes) are
appearance of whitish grey stool.
also called catalytic RNA. They are RNA
48. (a) Secondary air pollutants are O3, PAN and
molecules capable of catalysing specific
aldehydes which are produced from oxides
biochemical reactions, similar to the act of
of nitrogen and hydrocarbon by the
protein enzymes. Ribozyme, discovered in
reaction with UV rays.
1982, demonstrated that RNA can be both
49. (a) Digestion of food begins in mouth. Both
genetic material (like DNA) and a biological
chemical and physical digestion takes place
catalyst (like protein enzymes). Examples
in mouth. The teeth cut the food into small
of ribozymes include the hammerhead
pieces, chew and grind it. So, teeth help in
ribozyme, the VS ribozyme, Leadzyme and
physical digestion.
the hairpin ribozyme.
Salivary gland produces saliva that contains 56. (d) Hepatitis B is a viral disease, transmitted
amylase enzyme. The amylase enzyme through both blood transfusion and sexual
digests the starch present in food into sugar, intercourse.
thereby helping in chemical digestion. 57. (c) The second heart sound indicates the
50. (d) An arthropod has a segmented body beginning of diastole. The second heart
covered by an exoskeleton made from chitin sound, "Dup" is due to the closure of the
and other chemicals. This exoskeleton semilunar valves. The heart is in systole
serves as protection and provides places between the first and second heart sounds.
for muscle attachment. Arthropods must During systole, the heart pumps blood from
moult because their exoskeletons do not the ventricles, and the atrioventricular
grow with them. The body feature from valves are closed to prevent regurgitation
which the phylum takes its name is the into the atria. The closure of the semilunar
join ted appendages, which in clude valves is the start of a brief period of
antennae and mouth parts as well as walking “isovolumetric relaxation”. All valves are
legs. closed during the second heart sound.
51. (a) Copper-T is an intrauterine device which 58. (c) Hugo de Vries, a Dutch botanist, one of the
prevents the fertilised egg to becoming independent rediscoverers of Mendelism,
implanted in the wall of the womb. put forward his views regarding the
52. (c) A functional strand of mRNA must have a formation of new species in 1901. According
start and a stop codon. The start codon to him, new species are not formed by
often also codes for the amino acid continuous variations but by sudden
methionine, which may or may not end up appearance of variations, which he named
being a part of the final protein. However, as mutations. Hugo de Vries stated that
the stop codon would not code for an mutations are heritable and persist in
amino acid. Thus, with 66 codons in the successive generations. He conducted his
mRNA, there could be as many as 65 amino experiments on Oenothera lamarckiana
acids in the protein product. (evening primrose).
EBD_7839
176 KVPY-SA

59. (a) Somatostatin from the hypothalamus inhibits Area of a hexagon occupied by circles
the pituitary gland's secretion of growth
hormone and thyroid stimulating hormone. 120
=2× × p × 62 = 24p sq. cm.
In addition, somatostatin is produced in the 360
pancreas and inhibits the secretion of other
Area of a shaded region = 54 3 – 24p
pancreatic hormones such as insulin and
glucagon. = 6 (9 3 – 4p) sq. cm.
60. (c) DNA replication is the process in which a
double-stranded DNA molecule is copied 64. (a) BP || AC and AD || EQ,
to produce two identical DNA molecules. Since triangles on the same base between
In DNA replication, each new DNA the same parallels are equal in area
molecule has half the original parent strand ar (DABC) = ar (DAPC) .... (1)
hence called semiconservative and one and ar (DADE) = ar (DADQ) .... (2)
strand is synthesised continuously and
adding (1) and (2), we get
other discontinuously hence called
discontinuous. ar (DABC) + ar (DADE) = ar (DAPC)
+ ar (DADQ)
PART-II
Adding ar (DACD) to both sides, we get
MATHEMATICS ar (DABC) + ar (DADE) + ar (DACD)
61. (d) Since N = 6 × 12 × 18…..600 = (6100)(100!) = ar (DAPC) + ar (DADQ) + ar (DACD)
which is divisible by all the prime numbers Hence ar (ABCDE) = ar (DAPQ)
till 100, so smallest prime number must be 65. (d) Let the four-digit number be xxyy
more than 100. Then xxyy = 1000x + 100x + 10y + y
62. (b) Here, a + b = – p, ab = q Let t = xn Þ when = 1100x + 11y = 11(100x + y)
x = a, t = an when x = b, t = bn Since the number is a perfect square i.e
\ t2 + pnt + qn = 0 \ an + bn = – pn, an bn 11(100x + y) is a perfect square, it is possible
= qn only when 100x + y be in the form of 11k2,
so that the number becomes 11(11k2) or
a b
Again and are roots of xn + 1 + (x + 1)n = 0 112 × k2 a perfect square.
b a
Now we have 11k2 =100x + y = a three digit
n n number in the form of x0y (middle digit is
æaö æa ö
Þ ç ÷ + 1 + ç + 1÷ = 0 zero)
èbø èb ø K = 3 will give us 99 a two digit number
Þ an + bn + (a + b) n = 0 Þ – pn + (– p)n = 0 hence ruled out, so we will start from
which is true only if n is even integer. k=4
63. (a) Interior angle of hexagon K = 4, 11k2 = 176, k = 5, 11k2 = 275,
if k = 6, 11k2 = 396, if k = 7 11k2 = 539,
(2 ´ 6 - 4)
= × 90 = 120º if k = 8, 11k2 = 704, if k = 9, 11k2 = 891. Out
6
of all these three digit number only 704 has
Area of hexagon middle digit 0 or is in the form of 100 x + y
1 3 hence only one number exist with
= ×6×6× × 6 = 54 3 sq. cm
2 2 x = 7, y = 4, or 7744 = 882
SOLUTIONS – MOCK TEST-7 177

PHYSICS

q1 q2
66. (a) H H
qA I II III q
K1 K2 K3 B
d1 d2 d3
q A > qB

At steady state
d 2 d
( x + y 2 ) = (l2 ).
K1 dt dt
First material : H = A(q A - q1 ) ... (i)
d1
dy dl
2y = 2l
K2 dt dt
Second material: H = A(q1 - q 2 ) ... (ii)
d2
dy l dl u
or = = ....(ii)
K dt y dt cosq
Third material : H = 3 A(q2 - q B ) ... (iii)
d3 Similar relation can be obtained from
DBCD . That is
H d1
From (i) q A - q1 = ... (iv) dy v
A K1 = ....(iii)
dt cosq
H d2 Adding equations (ii) and (iii), we get
From (ii) q1 - q 2 = ... (v)
A K2 dy (u + v )
=
dt 2 cos q
H d3
From (iii) q 2 - q B = ... (vi)
A K3 dy
Thus velocity of block, in upward di-
Adding the above three equations, we get dt

H æ d1 d 2 d3 ö (u + v )
q A - qB = + + rection is .
A çè K1 K 2 K 3 ÷ø 2 cos q
68. (b) If the temperature of surrounding increases
(q A - q B ) A by DT , the new length of rod becomes
Þ H=
d1 d 2 d 3
+ + l ¢ = l (1 + aDT )
K1 K 2 K 3
Dueto changein length, moment of inertia of
Substituting the values rod also changes and it is about an end P and
(27 - 0)1 is given as
H= = 41.54 J/s
0.01 0.05 0.01 M l ¢2
+ + l P¢ =
0.8 0.8 0.8 3
67. (c) Let at any instant the block is at a distance As no e xternal force or torque is acting on
y from line AB and length of string between rod thus its angula rmomentum remains
A and C is l. In DACD, we have constant and during heating, thus we have
x 2 + y 2 = l2 .... (i) I P w = I P¢ w '
EBD_7839
178 KVPY-SA

[If w¢ is the final angular velocity of rod v


after heating]. For t < ; the slope is negative.
a
M l2 M l 2 (1 + aDT ) 2
or w= w¢ v
3 3 For t = ; the slope is zero.
a
or w¢ = w(1 - 2aDT )
v
[using binomial expansion for small a]. For t > ; the slope is positive.
a
Thus percentage change in angular velocity
These characteristics are represented by
of rod due to heating can be given as,
graph (b).
w - w¢
Dw = ´ 100% = 2aDT ´100% CHEMISTRY
w
71. (c)
sin i
69. (c) m=
sin r
CH3 CH3 ¾ ¾ ®
or sin r µ = sin i
CH3
On differentiating partially, we have
sin r (dµ) + µcos r (dr) = 0 CH3
Methyl shift
CH3 CH3 ¾¾¾¾¾
- sin r
or dr = (dm ) … (i) CH3
m cos r
As µ = a + bl–2
CH3 CH3 CH3 CH3
-2bdl CH3 CH3 – H+
CH3 CH3
or dµ = … (ii)
l3
From above equations, we have 72. (b)
2b tan r
dr = dl .
al 3 + bl
70. (b) For the body starting from rest,
1 2
x1 = 0 + at
2 x1 – x2
1 2
Þ x1 = at
2
For the body
moving with
constant speed v/a
x2 = vt t

1 2
\ x1 - x2 = at - vt
2 Stability of carbocation depends upon
at t = 0, x1–x2 = 0 number of resonating structures and extent
of +I, –I effect.
SOLUTIONS – MOCK TEST-7 179

73. (d) Calculating the bond order of various Then n1 should be equal to 1 and n2 should
species. be equal to 3

O -2 : KK s 2s 2s*2s 2s2pz2 p 2 px2 = p2 p2y é1 1 ù


i.e., v = R ê 2 - 2 ú (1)2
ë1 3 û

p*2 px2 = p*2p1y é 9 - 1ù 8


Þ v =Rê = R.
ë 9 úû 9
1 8-5 BIOLOGY
B.O. = (Nb - N a ) = = 1.5
2 2
76. (c) Alveolar ventilation (A) = 4000 ml/min
NO : KK s 2s 2 s*2s 2 s2 pz2 p2p2x = p2p2y Tidal volume (T) = 500 ml
Respiratory dead space (D) = 300 ml
p*2 p1x = p*2p0y A
Frequency of breathing (f ) =
T– D
Nb - N a 8 - 3
B.O. = = = 2.5 4000ml/min
2 2 =
500ml - 300ml
C22 - : KK s2s 2 s* 2s 2 p2 px2 4000
= = 20 breaths/min
200
= p2 p 2y s2 p 2z 77. (a) Genotype : AabbCC × aaBbCc
Offspring :
N - Na 8 - 2
B.O. = b = =3 Gametes AbC abC
2 2
aBC AaBbCC aaBbCC
Tall, purple and Tall, yellow and
He+2 : 2 * 1
s1s s 1s
full pod full pod
aBc AaBbCc aaBbCc
N - Na 2 - 1 Tall, purple and Tall, yellow and
B. O. = b = = 0.5
2 2 full pod full pod
abC AabbCC aabbCC
From these values we conclude that the
Dwarf, purple and Dwarf, yellow and
correct order of increasing bond order is
full pod full pod
He+2 < O-2 < NO < C22- abc AabbCc aabbCc
Dwarf, purple and Dwarf, yellow and
74. (c) Reaction (iii) can be obtained by adding full pod full pod
reactions (i) and (ii), therefore, K3 = K1. K2
Proportion of progeny plants with tall, and
75. (a) According to Rydberg’s formula, purple and full pod = 2/8×100 = 25%
78. (b) The correct sequence of the steps in
é1 1ù 8
v = R ê 2 - 2 ú Z2 If v = R catalytic cycle of an enzyme actions is : (iv),
ë n1 n2 û 9
(iii), (ii), (i).
EBD_7839
180 KVPY-SA

79. (b) The structures, given in option (b) are Membrane lipids are present in form of
responsible for excretion. Nephridia, phospholipids. They form lipid bilayer
Malpighian tubules and urinary tubules are because of their amphipathic characters.
excretory organs. Steroid hormones are released by animals
80. (a) Lipid droplets are structures that store fat such as glucocorticoids (cortisol),
in the form of neutral lipids. Lipid droplets mineralocorticoids (aldosterone).
are composed of a core of triglycerides and Wax is waterproof coating on the surface
sterol esters. of leaves. It reduces transpiration.
MOCK TEST-8
PART-I From (1) and (2)
MATHEMATICS t – s = s – r Þ 2s = t + r
i.e. t, s, r are in AP
p P Q p 5. (a) (ab) 2 = ccb hence the 2 digit number
1. (a) Since ÐR = so + =
2 2 2 4 ab < 32. From observation ab = 30 or 31
doesn’t satisfy the condition hence
æ P Qö p
\ tan ç + ÷ = tan ab < 30 or a = 1 or 2.
è 2 2ø 4
(10a+b)2 = 100a2 +20ab +b2 = ccb
P Q = 100c +10c +b
tan + tan
Þ 2 2 =1 By observation, we have (21)² = 441 and
P Q hence only one value of b
1 - tan tan
2 2 6. (b) Since ABCD is a parallelogram and
diagonal AC divides it into two triangles
-b / a
Þ =1 Þ a+ b= c of equal area, we have
æ cö
1- ç ÷ ar (DADC) = ar (DABC) ... (1)
è aø
As DC || AB, so CF || AB
2. (a) If cube of a number ends in 4 means unit
Since triangles on the same base and
digit of the number should be 4
between the same parallels are equal in
(10a + 4)3 = 1000a3 + 1200a2 + 480a + 64 has area, so we have
last two digit 44
ar (DACF) = ar (DBCF) ... (2)
Unit digit is 4 but 2nd last digit is 4 then–
Adding (1) and (2), we get
Þ Unit digit of 48a is 80 hence a = 1 or 6
ar (DADC) + ar (DACF)
So required values of N is 14 and 64 and
= ar (DABC) + ar (DBCF)
their summation is 14 + 64 = 78
Þ ar (DADF) = ar (ABFC)
3. (d) Given that both N and N + 3293 are perfect
squares. We can assume that N = a2 and 7. (c) Const: Join B to D
N + 3293 = b2 then b2 – a2 = 3293 or (b + a) Q ABC is a straight line and ABDE is
(b – a) = 3293 = 1 × 3293 = 37 × 89 cyclic quad.
Case (i) when 3291 = 1 × 3293 then we will \ ÐAED = ÐDBC = 70°
get b + a = 3293 and b – a = 1 on solving we also BC = DC in DBDC
will get a = 1646 and N = 16462 = 2709316 \ ÐBDC = 70°
Case (ii) when 3291 = 37 × 89 then we will
get b + a = 89 and b – a = 37 on solving we
will get a = 26 and N = 262 = 676
Only two values of N exist.
4. (c) Q (k, k) satisfies both equations
\ a(k) + b(k) + (t – s) = 0 ...(1)
also b(k) + a(k) + (s – r) = 0 ...(2)
EBD_7839
182 KVPY-SA

In DBDC, Using Pythagoras theorem,


ÐB = 70°, AC2 = AB2 + BC2 ... (i)
ÐD = 70°, Now, AN2 = AB2 + BN2
\ ÐC = 40° (by angle sum property) Þ 4AN2 = 4AB2 + 4BN2
\ DBOD = 80° (as angle at centre is
Þ 4AN2 = 4AB2 + (2BN)2
double of the angle at the remaining arc).
Þ 4AN2 = 4AB2 + (2BN)2
P Þ 4AN2 = 4AB2 + BC2 ... (ii)
8. (a) r Also, CM = MB + BC
2 2 2

O O’ Þ 4CM2 = 4MB2 + 4BC2


Þ 4CM2 (2MB)2 + 4BC2
Þ 4CM2 = AB2 + 4BC2 ... (iii)
Q Adding (ii) and (iii), we have
For area of segment POQ 4AN2 + 4CM2 = 4AB2 + BC2 + AB2 + 4BC2
DPOO¢ is an equilateral D. Þ 4(AN2 + CM2) = 5(AB2 + BC2)
Also DQOO¢ is an equilateral D. Þ 4(AN2 + CM2) = 5AC2 (Q From (i))
\ ÐPO¢Q = 120º
\ Area of segment POQ 10. (d) C3

1 2 é120º ´p C2
ù
= r ê - sin120º ú
2 ë 180º û C1

1 2 é 2p 3ù C
= r ê - ú A
4 units B x 2 units
D
2 ë 3 2 û
Area of total shaded region
= 2 × area of POQ

é 2p 3ù
Total area = r 2 ê - ú
ë 3 2 û
C1 = 2p × 2 = 4p (\ r = 2 units)
2 2 3 2 AC = AB + BC Þ AC = 4 + x
= pr - r
3 2 4+x
Radius of C2 =
9. (b) A 2
(4 + x)
C 2 = 2p = p(4 + x)
2
M
(6 + x)
C3 = 2 p = (6 + x) p
2
[\ AD = AB + BC + CD Þ AD = 4 + x + 2
6+x
Þ AD = 6 + x; radius = ]
B N C 2
SOLUTIONS – MOCK TEST-8 183

According to question
C1 + C3 4p + p(6 + x)
C2 = Þ p(4 + x) =
2 2
Þ 2 (4 + x) = 4 + 6 + x
Þ 8 + 2x = 10 + x Þ x = 2

6+x 6+ 2 V1 = Vol. of air between cube and sphere


Radius of C3 = Þ =4
2 2
4 3 4 3 3 a3
Area of C3 = pr2 = p (4)2 Þ 16p Þ V1 = pr1 - a3 = p - a3
3 3 8
11. (b) Let cost of a pen is x and that of copy is y.
According to the condition 40x> 15y and
æ 3p ö 3
16y > 40x. Since 40x>15y hence Þ V1 = ç - 1÷ a
è 2 ø
8x > 3y or 8x > = 3y + 1 …(i)
16y > 40x or 2y > 5x or 5x < = 2y – 1 …(ii) V1
Multiply eq (i) with 5 and eq(ii) with 8 and Ratio =
Vol. of air of cube
then compare the result we will get
16y – 8> = 40x> = 15y + 5 æ 3p - 2 ö a3 3p - 2
=ç ÷ø 3 =
or 16y – 8> = 15y + 5 è 2 a 2
hence y> = 13 so minimum possible value
of y is 13. Now 8x>3y or x>3y/8 or x>39/8 13. (b) Case-I: x Î[0,9] ;
so minimum possible value of y is 5 so
minimum possible value of 2(3 - x ) + x - 6 x + 6 = 0
x + y = 13 + 5 = 18.
Þ x - 8 x + 12 = 0 Þ x = 4, 2
12. (a)
Þ x = 16, 4
Since x Î[0,9] \ x = 4

Case-II: x Î[9, ¥] ;

2( x - 3) + x - 6 x + 6 = 0

Þ x - 4 x = 0 Þ x = 16,0

Since x Î[9, ¥] \ x = 16
Let r1 be radius of hollow cylinder
A and C are opposite corners of cube Hence, x = 4 & 16
such that 14. (b) Unit digit of product of two digit number is
So AC is longest diagonal 5. Hence unit digit of these two digit must
be either odd or 5.
So, 3a = 2r1
Hence numbers are 15, 51, 35, 53,75, 57, 95,
59 & 55. So total 9 such 2 digit number
3a
r1 = exist.
2
EBD_7839
184 KVPY-SA

15. (a) Here, -G mm ' 1


4 2 4 2 2
18. (c) ´ 3 + m ' ve2 = 0
p -1 x + x +1 x + 2x + 1 - x r 2
= 2 =
p + 1 ( x + x + 1) 2
( x 2 + x + 1)2 -3Gm 1
or a + ve2 = 0
( x 2 + 1) 2 - x 2 ( x 2 + x + 1)( x 2 - x + 1) ( / cos30°) 2
= 2 2
= 2
( x + x + 1) ( x 2 + x + 1)2

p - 1 x2 - x + 1 6 3 Gm
\ = , using componendo ve = .
p + 1 x2 + x + 1 a
and dividendo. 19. (b) Vernier callipers
2 p 2( x 2 + 1) 1 1cm
Þ = Þ p = x+ ...(1) 1 MSD = = 0.125cm
2 2x x 8
1- x 5 VSD = 4MSD
As, f ( x) =
1+ x
1
æ æ 1öö 1 \ 5VSD = 4 × cm = 0.5cm
Þ f ( f ( x )) + f ç f ç ÷ ÷ = x + ...(2) 8
è è ø
x ø x
\ 1 VSD = 0.1cm
From Eqs. (1) and (2), we get
L.C = 1MSD – 1VSD
æ æ 1 öö
f ( f ( x )) + f ç f ç ÷÷ = p = 0.125cm – 0.1cm
è è x øø
= 0.025cm
PHYSICS Screw gauge
One complete revolution = 2M.S.D
3L
16. (c) Force = weight suspended + weight of If the pitch of screw gauge is twice the L.C
4
of wire of vernier callipers then pitch
= 2 × 0.025 = 0.05cm.
3W
= W1 + L.C of screw Gauge
4
force pitch
stress = = Total no. of divisions of circular scale
area
17. (c) Smooth and polished plates are poor
radiators of heat. Hence, heat coming out 0.05
from A1 is small, even though B1 being a = cm = 0.0005cm = 0.005 mm.
black and rough plate is a good absorber.
100
Effectively the heat coming to the left of (II) is a correct statement
pellet P is small. Now if the least count of the linear scale of
Black and rough plates are good radiators the screw gauge is twice the least count of
of heat. Hence, plate B2 radiates heat to a venier callipers then.
satisfactory level; however, plate A2, being L.C of linear scale of screw gauge
smooth and polisher, is a bad absorber. = 2 × 0.025 = 0.05cm.
Effectively, the heat coming to the right of Then pitch = 2 × 0.05 = 0.1cm.
P is also small.
SOLUTIONS – MOCK TEST-8 185

0.1 m2l
Then L.C of screw gauge = cm 23. (d) Initial position of cm =
100 m1 + m2
= 0.001cm = 0.01mm.
(III) is a correct statement. m1Dx1 + m2 Dx2 m1v0t + 0
Also Dcm = =
m1 + m2 m1 + m2
20. (d) Efficiency of a transformer,

Power output m2l m1v0t


h= \ final position = m + m + m + m
Power input 1 2 1 2

for an ideal transformer, h = 1 24. (a) The direction of current in the loop will be
\ power output = power input = 60 W such as to oppose the increase in the
magnetic field.
21. (a) Consider an element part of solid at a
r
distance x from left end of width dx. 25. (a) Since torque t is rotational analogue of
r r
force F and F = mass × acceleration,
x
dx r
therefore, torque t = (moment of inertia) ×
(angular acceleration) = (I a) as moment of
inertia is rotational analogue of mass.

V 26. (d)

Resistance of this elemental part is, l l 2l


t1 = , t2 = =
rdx r xdx v v/2 v
dR = = 0
2
pa pa 2 (n - 1)l
3l
t3 = , ..... , tn–1 =
L v v
r0 xdx r0 L2
R = ò dR = ò =
pa 2 2 pa 2 l 2l 3l (n - 1)l
0 So T = + + + ..... +
v v v v
Current through cylinder is,
l
V V ´ 2pa 2 = [1 + 2 + 3 + ..... + (n – 1)]
I= = v
R r0 L2
n(n - 1)l
Potential drop across element is, =
2v
2V
dV = I dR = ´ dx mv + 0 + 0 + .... + 0 v
L2 vcm = = .
nm n
dV 2V
E(x) = = x 27. (a) Mass of the removed sphere
dx L2
2
22. (b) If v ^ B, then path is circular and if v has M 4 æRö M
M '= ´ pç ÷ =
a component along B, then path will be 4 3 3 è2ø 8
pR
helical. 3
EBD_7839
186 KVPY-SA

GMm GM ' m 36. (a) Mn(25) [Ar]3d5 4s2 n=5


Thus, F = 2
- 2
d (d - R / 2) Ti(22) [Ar]3d2 4s2 n=2
28. (c) The three curves AB, CD and EF meet at V (23) [Ar]3d3 4s2 n=3
point P which is called the triple point of Al (13) [Ne]3s2 3p1 n=1
water. It is the point where all three states 37.

(c) H2SO4 is a strong acid and HSO4 is its weak
solid, liquid and gas of water co-exists. conjugate base.
29. (b) Temperature of B will be higher as due to +
NH3 is a strong base and NH 4 is its weak
expansion centre of mass of B will come
conjugate acid of NH3.
down same heat is supplied but in B,
Potential energy is decreased therefore +1 0
38. (c) H 2O ® H2
internal energy gain will be more and internal
energy is directly proportional to Oxidation number of H decreases from +1
temperature. to 0. Hence, H2O is reduced to H2.
30. (c) Since horizontal component of the velocity 39. (b) Chain propagating step is
of the bomb will be the same as the velocity
of the aeroplane, therefore horizontal Clg + CH 4 ¾¾
® g CH3 + HCl
displacements remain the same at any 40. (a) Due to the inert pair effect, thallium exists
instant of time. in more than one oxidation state. Also, for
thallium + 1 oxidation state is more stable
CHEMISTRY
than +3 oxidation state.
31. (b) In KCN, K+ and CN– have ionic bond 41. (b) O < S < F < Cl
and C º N has covalent bond. Electron gain enthalpy increases along the
32. (b) Geometrical isomerism is shown by period and decreases along the group but
compounds which have C=C and two in between III and II period elements, III
groups attached to same C atoms are period elements have greater electron gain
different. Choice (b) fulfills both conditions. enthalpy because of very high electron
density of II period elements.
42. (c) Metal + dil. HCl ® Metallic salt + Hydrogen
Ag is below hydrogen in reactivity series.
So it fail to displace hydrogen from dil.HCl.
25.6 Thus there is no product formed and no
33. (b) Moles of C12 H 22 O11 = = 0.0747
342.3 gas evolved.
Number of H-atoms
[Z]
= 0.0747 × 22 × 6.023 × 1023 = 9.91 × 1023
43. (a) Kc =
[X][Y]
34. (b) Dng = 2 – 4 = – 2, DH = DE – 2RT.
1 1
Þ [Y] = [X] = [Z] = a(say)
+2 +2 +4 2 2
35. (a) FeC2 O4 ® Fe3+ + 2CO 2 + 3e – ; In acidic
\ [Z] = 2a, [Y] = 2a, [X] = a
medium MnO 4– changes to Mn 2+ and
consumes 5e– 2a
104 = Þ a = 10–4
a.2a
\ 3e– will be consumed by 3/5 moles of
KMnO4. [Z] = 2a = 2 ×10–4
SOLUTIONS – MOCK TEST-8 187

44. (a) M1 = 64 ; r2 = 2r1 53. (b) Monosaccharides are the class of sugars
that cannot be hydrolysed to give a simpler
2
é r1 ù 1 sugar. Pentoses and hexoses are the
M2 = M1 ê ú = 64 × = 16
ë r2 û 4 examples of monosaccharides.
54. (a) Ca 2+ and Mg2+ are involved in muscle
45. (a) NaOH is a strong alkali. It combines with
acidic and amphoteric oxides to form salts. contraction. Ca 2+ ion promotes the
Since CaO is a basic oxide, hence, it does formation of actomyosin which is a
not reacts with NaOH. combination of myosin and actin. Ca2+ and
Mg2+ ions are also helpful in relasing
BIOLOGY
energy by the oxidation of food in the form
46. (d) Maltose, also known as maltobiose or malt of ATP which is broken down in ADP,
sugar, is a disaccharide formed from two phosphorous and energy. Released energy
units of glucose joined with an a–1,
is used up in the contraction of muscle
4–glycosidic bond.
fibres.
47. (b) The sarcomere is the functional unit of
55. (a) Lateral meristems are present along the
muscle. As actin myofilaments slide over
myosin, the sarcomere shortens, but not the lateral sides of stem and roots. Intrastelar
individual components that make up the cambium ring formed by intrafascicular and
sarcomere. interfascicular cambia and extrastelar cork
48. (a) Amphitrichous bacteria have a single cambium are the examples of lateral meristem.
flagellum at each of the two ends or poles. 56. (d) Relaxin is a protein hormone, secreted by
e.g., Spirillum volutans. the placenta that causes the cervix to dilate
49. (d) Pancreatic lipase (formely called steapsin) and prepares the uterus for the action of
hydrolyses fats into glycerol and fatty oxytocin during labour.
acids. 57. (b) Gibberellin overcomes vernalisation
50. (a) It is the long distance movement of organic requirement (low temperature) for
substances from the source or supply end flowering.
(region of manufacture or storage) to the
58. (d) Blood clotting involves none of the given
region of utilisation or sink end.
options. In blood clotting pathways,
51. (d) Pectin, a structural heteropolysaccharides,
prothrombin is converted into thrombin and
are abundant in fruits particularly in the
citrus fruits like oranges and lemons. They fibrinogen is converted into fibrin.
are present in the cell wall and in the 59. (b) Contraction of gall bladder is induced by
intracellular substance. They contain cholecystokinin. It is secreted by I-cells in
arabinose, galactose and galacturonic acid. the small intestine.
52. (a) E. coli resides in the large intestine of 60. (a) Blood platelets are 2–3 µm in diameter. They
human. If they are present in water supply, are considered to be the cell fragments
it represents that water supply is
instead of being cell themselves.
contaminated.
EBD_7839
188 KVPY-SA

PART-II Sum of areas of two circles = 153p


MATHEMATICS px2 + py2 = 153p
Þ x2 + y2 = 153 .... (i)
61. (b) Since highest power of 2 in 50! Is 47 hence
highest power of 8 in 50! Is [47/3] = 15 x + y = 15 .... (ii) (Given)
If we assume a = 15 then 158 or 38 and 58 (x + y)2 = x2 + y2 + 2xy
So we have to check that whether 38 and 58 Þ 152 = 153 + 2xy
are divisible by 50! Or not, Since highest 225 –153 72
Þ xy = = = 36
power of 3 in 50! Is 22 and that of 5 is 12, 2 2
hence a = 15 will satisfy the condition. Þ x (15 – x) = 36 Þ 15x – x2 – 36 = 0
62. (c) x2 – 2x sec q + 1 = 0 Þ x = sec q ± tan q or x2 – 15x + 36 = 0
and x2 + 2x tan q – 1 = 0 Þ x = –tan q ± sec q Þ (x – 12)(x – 3) = 0
p p Þ x = 12 or 3 and y = 3 or 12
Q - <q< -
6 12 Since x > y Þ x = 12 and y = 3
Ratio of circumferences smaller to larger
p p circle
Þ sec > sec q > sec
6 12
2p(3)
= = 3 : 12 = 1 : 4
p tan p 2p(12)
and - tan < tan q < -
6 12 64. (a) DABF and ||gm ABCD are on the same base
AB and between the same parallels AB and
p p DF.
also tan < - tan q < tan
12 6
1
\ ar (DABF) = ar (||gm ABCD) .... (1)
a1,b1 are roots of x2 – 2x sec q + 1 = 0 2
and a1> b1
D C F
\ a1 = sec q – tan q and b1 = sec q + tan q
a2, b2 are roots of x2 + 2x tan q – 1 = 0 and
a2 > b2 E
\ a2 = –tan q + sec q, b2 = – tan q – sec q
\ a1 + b2 = sec q – tan q – tan q – sec q
= – 2tan q
A B
63. (b) Let the radius of two circles with centres
O and O¢ be x and y respectively Q Diagonal BD of a ||gm ABCD, divides it
sum of radii = 15 into two congruent triangles.
Þ x + y = 15 \ ar (DBCD) = ar (||gm ABCD) .... (2)
ar (DABF) = ar (DBCD) .... (3) [From (1)
and (2)]
Also, DABE and DDBE are on the same
O x y O'
base BE and between the same parallels
BE and AD.
SOLUTIONS – MOCK TEST-8 189

\ ar (DABE) = ar (DDBE) ... (4)


Subtracting (4) from (3), we get 5V0
ar (DABF) – ar (DABE) P1
T0
= ar (DBCD) – ar (DDBE)
Hence, ar (DBEF) = ar (DCDE). Piston
65. (d) Since the number is between 10 to 1000, P2 3V0
that means n may be a two digit or a three
digit number, so we have two different
cases-
24
Case (i) if the n is a two digit number, 5 V0

ab = 10a +b, here none of the digit is zero. P'1


2T0
From given information a + b + ab = 10a + b Piston
or ab = 9a or b = 9, and the two digit numbers
that satisfy this condition is 19, 29, 39, 49, 16
P'2 5 V0
59, 69, 79, 89 and 99 (Total 9 numbers)
Case (ii) if the number is a 3digit number where n1 and n2 are the number of moles in
then Pn = abc and Sn = a + b + c the two portions.
a + b + c + abc = 100a + 10b + c Finally, from the F.B.D. of the piston,
or abc = 9(11a + b) P1A + W = P2A
Here LHS is always less than the RHS,
n1R(2T0 ) W n 2 R(2T0 )
because c £ 9 and ab < 11a, no number or + = ... (ii)
satisfy this condition. 3(8V0 / 5) A 2(8V0 / 5)

Thus there are only 9 values (all are two Subtracting eq. (ii) from eq. (i), we have
digit number) of n that satisfy the
condition. æ1 5 ö æ 1 5ö
n1 ç - ÷ = n 2 ç - ÷ or n : n = 35 : 26
è 5 12 ø è 3 8ø 1 2
PHYSICS
67. (c) Shearing strain is created along the side
66. (a) Let the weight of the piston W and cross- surface of the punched disk. Note that the
sectional area of the cylinder be A. Let the forces exerted on the disk are exerted along
initial and the final pressures of the upper the circumference of the disk, and the total
and the lower regions be P1, P¢1 and P2, P¢2 force exerted on its center only.
respectively. Further, if initial temperature
D
be taken to be T0, then final temperature
will be 2T0.
From the F.B.D. of the piston, initially, F

P1A + W = P2A
n1RT0 W n 2 RT0
or + = ... (i)
h
5V0 A 3V0
EBD_7839
190 KVPY-SA

Let us assume that the shearing stress The equilibrium extended length of wire
along the side surface of the disk is uniform, = L + DL
then
MgL æ 1 Mg ö
= L+ = L ç1 +
F= ò dFmax
3pR Y 2 ÷
è 3 pYR 2 ø
surface
69. (b) As no external force acts in z-direction,
= ò s max dA = s max ò dA hence z-coordinate of the centre of mass of
surface surface the ball should be zero. To make
z-coordinate zero other ball should fall
æ Dö symmetrically with respect to z-axis.
= s max .2p çè ÷ø h
2 Hence z-coordinate of other ball = –5 m.
The balls do not have any external force in
8 æ1 -2 ö -2
= 3.5 ´10 ´ ç ´10 ÷ ´ 0.3 ´10 ´ 2p x-direction. Hence in x-direction the centre
è 2 ø
of mass should move with constant
4 4 velocity.
= 3.297 ´ 10 Þ h = 3.3 ´ 10 N
x-coordinate of centre of mass at t = 1.5 s,
68. (c) Consider a small element dx of radius r,
= 200 × 2 = 400 m
2R
r= x+R m1 x1 + m2 x2
L Hence, xCM =
m1 + m2
R

y
x
200 m/s
dx r

30 m x
5m
L
z 250 m

20 ´ 50 + 20 x2
3R 400 =
20 + 20
Mg
x2 = 800 – 250 = 550 m
At equilibrium change in length of the wire
Position of centre of mass at t = 2 s,
1 1
Mg dx h= ´ 10 ´ 2 2 = 20 m
ò dL = ò 2 2
0 é 2R ù
pê x + Rú y
ë L û Hence y-coordinate of centre of mass
= 30 – 20 = 10 m
Taking limit from 0 to L
m1 y1 + m2 y2
Hence yCM =
Mg é 1 L ù MgL m1 + m2
DL = ê - ´ =
py L 2 R ú 3pR 2 y
ê é 2 Rx + R ù ú 20 ´ 0 + 20 ´ y2
êë êë L úû úû Þ 10 = \ y2 = 20 m
0 20 + 20
SOLUTIONS – MOCK TEST-8 191

70. (a) If x1 is the overhang length on second 73. (d) If two chirality centres are created as a result
blocks, then, of an addition reaction four stereoisomers
can be obtained as products.
æl ö M l
M ç - x1 ÷ = ( x1) \ x1 =
è2 ø 2 3
Now of x2 is the overhang table then

æl ö 3M l
M ç - x2 ÷ = x2 \ x2 =
è 2 ø 2 5
Br CH3
l l 8l | |
Nowx = x1 + x2 = + = .
3 5 15 CH3 – CH – CH – CH2 – CH3
* *
2-Bromo-3-methyl pentane
CHEMISTRY (2 chiral centre)

71. (d) No. of stereoisomers = 2n = 22 = 4


where n = chiral centre

CH3 CH3

H Br Br H

H CH3 H3C H

C2H5 C2H5
(I) (II)

CH3 CH3

72. (d) At NTP 22400 c.c. of N2O Br H H Br


= 6.02 × 1023 molecules
H CH3 H3C H
6.02 ´ 10 23
\ 1 cc N2O = molecules
22400 C2H5 C2H5
23
3 ´ 6.02 ´ 10 (III) (IV)
= atoms
22400
1. 8
= ´ 1022 atoms 74. (c) PbCl2 is most ionic because on going down
224 the group the metallic character increases
No. of electrons in a molecule of and also the inert pair effect predominates.
N2O = 7 + 7 + 8 = 22 Between PbCl2 and PbCl4; PbCl2 is more
Hence no. of electrons ionic according to Fajan’s rule.
6.02 ´ 1023 75. (c) The b.p. of p-nitrophenol is higher than that
= ´ 22
22400 of o-nitrophenol because in p-nitrophenol
1.32´1023 there is intermolecular H-bonding but in
= electrons o-nitrophenol there is intramolecular
224
H-bonding.
EBD_7839
192 KVPY-SA

BIOLOGY 78. (c) It is the part of brain where the optic nerves
partially cross. The optic chiasma is located
76. (a) In oxidative decarboxylation, the transition at the bottom of the brain immediately
reaction converts the two molecules of the inferior to the hypothalamus.
3-carbon pyruvate into two molecules of 79. (b) The inhibitory gene ratio is 13:3
2-carbon molecule of acetyl CoA and two Complementary gene ratio is 9:7
molecules of carbon dioxide.
Recessive epistasis ratio is 9:3:4
77. (b) In grasshoppers, human and Drosophila, Dihybrid test cross ratio is 1:1:1
the sex determination is similar because all Dominant epistasis ratio is 12:3:1
the male individuals consist of one
80. (d) [H+] [OH–] = 10–14
X chromosome and the female individuals
1.3 × 10–4 × [OH–] = 10–14
consist of two X chromosomes. In
grasshopper and Drosophila, no 10 –14
[OH–] = M
Y chromosome is foun d. So, sex 1.3 ´ 10 –4
determination is done on the basis of = 0.769 ´ 10–10 M
presence or absence of X-chromosome. = 7.7 ´ 10–11 M
MOCK TEST-9
PART-I \ pq = 2005, – 2004
MATHEMATICS 4. (a)
1. (a) Q P(a) = P(b) = P(c) = 0 a, b, c are roots
x3 – 3x2 + 2x + 5 = (x – a)(x – b)(x – c)
Replacing x by 2
23 – 3(2)2 + 2(2) + 5 = (2 – a)(2 – b)(2 – c) 10 3 10 3
(2 – a)(2 – b)(2 – c) = 5
2. (d) Let the two digit number 10a + b
Then (10a + b)2 + (10b + a)2 ends with 3. 10 3
Or 100(a2 + b2) + 40ab + (a2 + b2) has unit
digit 3.
But here a2 + b2 will give us unit digit Let the side of eq D is a.
Square of any numbers ends with either 0, 1, 4,
5, 6 or 9 and unit digit after adding any two of 3 2 æ1 ö æ1 ö
\ . a = ç ´ (4) ´ a ÷ + ç 2 ´ (5) ´ a ÷
them is 3 then these two must be 4 & 9 4 è2 ø è ø
Hence unit digit of two squares must be end
with 4 & 9. æ1 ö
+ ç ´ (6) ´ a ÷
If square has unit digit 4 then unit digit of è2 ø
that no must be 2 or 8. And if square of a no
has unit digit 9 then the nos must has unit
3 2 1
digit 3 or 7. Þ . a = × a × (15) Þ a = 10 3
4 2
Hence numbers are 23, 27, 83, 87 or reverse
of these 32, 72, 38 or 78 for height of D;
Hence total 8 such numbers exists.
(10 3 ) = (5 3 )
2 2
3. (c) 2005 + p = q2 .....(1) + h2
2005 + q = p 2
.....(2)
From equation (1), (2), (p – q) = q2 – p2 Þ 300 = 75 + h2 Þ 225 = h2 Þ h = 15
Þ (p + q) = – 1 .....(3) 5. (d) (2!)x = 2x
(5!)y = 23y, 3y, 5y
also from eqn. (1) × (2),
(7!)2 = 24z, 32z, 5z, 7z
Þ (2005 + p) (2005 + q) = p2.q2
Hence (2!)x × (5!)y × (7!)z
Þ (2005)2 + 2005 (p + q) + pq = p2q2
+z
= 2x + 3y + 4z × 3 y+ × 5 y × 7 z
2z

from (3),
Þ p2q2 – pq – 2005 (– 1) – (2005)2 = 0 Since 100! = 297 × 348 × 524 × 716
Maximum value of z = 16
Þ (pq)2 – (pq) – 2004 × 2005 = 0
Maximum value of y + z = 24
Þ (pq)2 – (2005pq) + (2004pq)
Maximum value of z = x + 3y + 4z = 97;
– 2004 × 2005 = 0 hence for maximum value of x + y + z,
Þ (pq – 2005) (pq + 2004) = 0 y = 0, z = 0 and x = 97, then x + y + z = 97
Telegram @unacademyplusdiscounts

Join Us on Telegram for More Such Books

https://telegram.me/unacademyplusdiscounts

Join us from the above link or search ''unacademyplusdiscounts'' in Telegram


EBD_7839
194 KVPY-SA

Therefore, DFAC ~ DEBC (AA similarity)


4+6+8
6. (b) s= =9
2 FA AC
Þ = ....(2)
EB BC
1
D = 9.1.3.5 = 3 15 = .4.h But EB = DB (B is mid-points of DE)
2
FA AC
Þ =
3 DB BC
Þh = 15
2 Therefore, from (1) and (2) we have :
AC OA
A =
h1 BC OB
M N
8 OC - OA OA
i.e, = or
6 OB - OC OB
h
OB.OC – OA.OB = OA.OB – OA.OC
or OB.OC + OA.OC = 2OA.OB or
B 4 C
(OB + OA).OC = 2OA.OB
1 1 2
D 15 or + =
r= = OA OB OC
s 3
[Dividing both the sides by OA.OB.OC]
3 2 5 8. (d) Area of circle = p
h1 = h – 2r = 15 - 15 = 15
2 3 6 Area of square = p

DAMN ~ DABC

5
4. 15
h1 MN 4h1 20
= Þ MN = = 6 =
h 4 h 3 9
15
2

7. (d) In DAOF and DBOD


ÐO = ÐO (same angle)
and ÐA = ÐB (each 90°)
\ Side of square = p
Therefore, DAOF ~ DBOD (AA similarity)

OA FA p
So, = ....(1) Þ OM =
OB DB 2

Also, in DFAC and ÐEBC, ÐA = ÐB (each Now: MQ2 + OM2 = 12


= 90°) and ÐFCA = ÐECB (Vertically p
Þ MQ2 + =1
opposite angles). 4
SOLUTIONS – MOCK TEST-9 195

25 = 6y – 7
4-p
Þ MQ 2 =
4 32 16
y= =
6 3
PQ = 2MQ = 4 - p
9. (b) Here, area of square ABCD = 4cm2 2
æ 16 ö 20
According to question, Þ x 2 = ç ÷ + 42 =
è 3ø 3
Area of D ADE = Area of DBEC = 1
11. (d) Let the number of deer = x
4
Area of D DEC = =2 According to question
2
x 3æ x ö
+ +9= x
2 4 çè 2 ÷ø

x 3x x
x– - =9 Þ = 9 Þ x = 72
2 8 8
Difference between number of deer who
are grazing and those who are playing
= 36 – 27 = 9
So, it is a multiple of 9.
12. (a) Let the required square root be
2
Area of D DFC = =1 a+ b+ c
2
Area of D DGC = 1/2
Then 5 + 10 + 15 + 6
Area of D DHC = 1/4
æ5ö 15 3
1 1 = 5+2 ç ÷ +2 +2
\ Area of D KDC = = cm2 è2ø 4 2
4´2 8

( )
2
10. (c) Join OT which intersect = a+ b+ c
PQ at M = a + b + c + 2 ab + 2 ac + 2 bc
DPQT is an isosceles
Now equating the rational and irrational
Triangle and OT is angle bisector of PQ.
part we will get a + b + c = 5, ab = 5/2,
Þ OT ^ PQ bc = 15/4 and ac = 3/2
Þ PM = 4cm and OM = 3 cm Since in denominator of bc is 4 while that
Let PT = x, MT = y of other two terms is only 2 hence we can
In DPMT, by Pythagoras theorem assume that b = 3/2 and c = 5/2 then a 1
Hence required square roots is
x2 = y2 + 42 ... (i)
In DOPT, by Pythagoras theorem æ æ 3ö æ5öö
ç 1+ ç ÷ + ç ÷÷
(y + 3) = 5 x
2 2+ 2
... (ii) ç è 2ø è 2 ø ÷ø
è
Subtracting (i) from (ii), we get
EBD_7839
196 KVPY-SA

13. (c) Since N = (xyz – x)/99. Hence 396N will always As there is no slipping between any point
assume an integral value. of contact hence distance moved by the
14. (d) Since highest power of 7 in N! is k and that man is 2L.
in (N + 3)! is K + 2 it is possible only when
17. (b) According to the question,
N + 1, N + 2 or N + 3 is a multiple of 7^2 = 49.
There are 2 such numbers exist these are 49 Dt 1
t = (90 ± 1) or, =
and 98, two values of N is 48 and 97. t 90
Highest power of 7 in 46!, 47!, and 48! is 6
and that in 49!, 50!, and 51! is 8. Dl 0.1 Dg
l = (20 ± 0.1) or, = %=?
Highest power of 7 in 95!, 96!, and 97! is 14 l 20 g
and that in 98!, 99! And 100! is 16. Hence
total 6 numbers exist.
l
15. (d) Let number of girls is ‘g’ then As we know, t = 2p
g
Number of group having 4 boys and 1 girl
= (4C4) (gC1) = g
4 p 2l
And number 4 of groups having 3 boys and Þg=
t2
2 girls = (4C3) (gC2) = 2g (g – 1)
Thus, total number of tests is g + 2g(g – 1) = 66 Dg æ Dl Dt ö æ 0.1 1ö
or, = ±ç +2 ÷ = ç +2´ ÷
Or 2g2 – g – 66 = 0 only integral value of g is 6 g è l t ø è 20 90 ø
Hence total number of students is 5 + 6 = 11
= 0.027
PHYSICS
Dg
16. (a) Let v0 be the linear speed of the axis of the \ % = 2.7%
g
cylinder and w be its angular speed about
the axis. As it does not slip on the ground 18. (a) From 0 to 6 second the acceleration varies
v0 linearly with time, therefore we have,
hence w = . Where R is the radius of
R
the cylinder.
Speed of the topmost point is
v = v0 + wR = 2v0 da 5
= +
Since time taken by the axis to move a dt 6
distance L is equal to t = L/v0.
In the same interval of time distance moved
by the topmost point is
L
s = 2v0 ´ = 2L d é d 2s ù 5
v0 or, ê ú=+
dt ëê dt 2 ûú 6
v=v0+wR

d 3s 5
or 3 = +
dt 6
v=v0–wR
SOLUTIONS – MOCK TEST-9 197

Integrating equation (i) w.r.t. time, we get


1 2
Now,v0 sin q t = at
2
d 2s 5
2 = t + c1
dt 6 2
1 é 2v0 cos q ù
or v0 sin q = a ê ú
2 ë g û
ds 5t 2
\ = …(ii)
dt 12
a
\ tan q =
5 3 g
and s= .t …(iii)
36
4
The distance travelled from 0 to 6s, from tan q= = 0.4
10
equation (ii), we get
20. (d) For path ab : (DU)ab = 7000 m
5 By using DU = mCV DT
s1 = ´ (6)3 = 30 m
36
5
The velocity at t = 6 s, 7000 = m ´ R ´ 700 Þ m = 0.48
2
5 2 5
v= t = ´ 62 = 15 m/s For path ca :
12 12
( DQ)ca = ( DU )ca + ( DW )ca ...(i)
Now from, 6 s to 12 s
u = 15 m/s, a = 5m/s2 Q ( DU )ab + (DU )bc + ( DU )ca = 0

1 2 1 Q 7000 + 0 + ( DU )ca = 0 Þ ( DU )ca


2
\ s2 = ut + at = 15 ´ 6 + ´ 5 ´ 6 = –7000 m ...(ii)
2 2
= 180 m. Also ( DW )ca = P1 (V1 - V2 ) = mR (T1 - T2 )
Therefore total distance travelled on = 0.48 × 8.31 × (300 – 1000)
runway = – 2792.16 m ... (iii)
= 30 + 180 = 210 m. On solving equations (i), (ii) and (iii)
19. (d) If t is the time taken by ball to return the ( DQ )ca = -7000 - 2792.16
boy’s hand, then
= –9792.16 J » –9800 m
v0 cos q 21. (b) From geometry
v0
q
f
v0 sin q a
r

1 2
0 = v0 cos qt - gt r = f tan a
2
\pr2 = pf 2 tan 2 a
2v0 cos q
or t= or pr2 µ f 2
g
EBD_7839
198 KVPY-SA

22. (a) In fig. A force due to 5M, 2M cancel and From conservation of momentum, Pi = Pf
similarly in fig. B force due to 2M, 2M m B v 0 = (m A + m B )v
cancel, hence FA = FB. In fig. C also force
m Bv0
due to 5M cancel but net force will be more or v =
mA +mB
in comparison to fig. A or fig. B as man at
the centre is double similarly think for fig D.
2GM
23. (b) We choose ball and cart as our system, No 24. (a) Escape velocity = v =
R
external force acts on the system in
x-direction; therefore momentum along x- 2GM
axis is conserved. Þ v2 = .......(i)
R
The ball will continue to move upwards until v = (2GM)R–1
2

its velocity relative to the cart is zero.


r r r Differentiating both sides, we get,
i.e., v BA = v B - v A = 0
r r
or vB = vA dv 2GM dv GM
2v =– 2
Þv = 2 ......(ii)
When the ball reaches maximum height, the dR R dR R
cart and ball move horizontally with same
1 dv 1
velocity at the extreme position. Dividing (ii) by (i), =–
v dR 2R

(vA)i = v0 dv 1
A Þ × 100 = × 4% = 2%
v 2
\ If the radius decreases by 4%, escape
velocity will increase by 2%.
(vB)i= v0 25. (c) This graph suggest that when
Reference level
B u = – f, v = +¥
Initial position

v (cm)

(vA)f = v
A

(vBA)i = 0
f

h (vB)f =(vA)f =V –f u (cm)


B
Final position

When the object is moved further away


Pi = m B v B = m B v 0
from the lens, v decreases but remains
Pf = m B (v B )f + m A (v A )f positive. When u is at –¥, v = f.
= (m B + m A )v
SOLUTIONS – MOCK TEST-9 199

This is how image formation takes place v = (2pr)n (where n is frequency)


for different positions of the object in case
of a convex lens. = (2p ´ 9)(6) = 1.8p m / s
26. (c) Here, s = (13.8 ± 0.2) m t = (4.0 ± 0.3) s
s 13.8
Velocity, v = = = 3.45 ms–1
t 4.0
= 3.5 ms–1
(rounding off to two significant figures) N1

Dv æ Ds Dt ö
= ±ç + ÷
v è s t ø N2
mg
æ 0.2 0.3 ö
= ±ç + =±
( 0.8 + 4.14)
÷
è 13.8 4.0 ø 13.8 ´ 4.0
N1 Y
Dv 4.94
Þ =± = ±0.0895 N2
v 13.8 ´ 4.0
X
D v = ± 0.0895 × v = ± 0.0895 × 3.45
mg
= ± 0.3087 = ± 0.31
(rounding off to two significant figures)
Hence, v = (3.5 ± 0.31) ms–1

Dv Therefore,
% age error in velocity = ´ 100
v
(50)(1.8p ) 2
N2 = = 178 N
= ± 0.0895 × 100 = ± 8.95 % = ± 9% 9
27. (a) In x = A cos wt, the particle starts oscillating
N1 = mg = 490 N
from extreme position. So at t = 0, its
The magnitude of her weight is the
potential energy is maximum.
magnitude of the resultant force exerted on
28. (a) The woman experiences three forces : mg,
her by the chair.
her weight acting vertically downwards; N1,
reaction due to her weight; N = N12 + N 22 = 4902 + 1782 = 521 N
N2, horizontal reaction which provides the
centripetal acceleration. 29. (a) Angle of incidence is given by

From Newton’s second law,


cos (p–i) =
(6 )
3iˆ + 8 3 ˆj - 10kˆ .kˆ
20
mv2
SFx = N 2 =
r 1
– cos i = –
2
SFy = N1 - mg = 0
Ð i = 60 °
EBD_7839
200 KVPY-SA

sin q cos q
Þm³
z 2 - cos 2 q
\ Minimum coefficient of friction,
2 i
æ sin q cos q ö 1
m min = ç =
x 2 ÷
è 2 - cos q ømax 2 2
3
CHEMISTRY
r
31. (d) SnCl2 SnCl4
119 : 2 × 35.5 119 : 4 × 35.5
Chlorine ratio in both compounds is
= 2 × 35.5 : 4 × 35.5 = 1 : 2
From Snell’s law, 2 sin i = 3 sin r
32. (c) In sp2-hybridisation,
Ð r = 45° Hybridised orbital = 3 [3s-bonds]
30. (a) Consider a general position of rod as shown Unhybridised orbital = 1 [1p-bond]
in figure. Let applied force be F.
As the end is moved slowly so the rod has 33. (a) ˆˆ† M 2 + + 2X -
MX 2 ‡ˆˆ
s 2s
no acceleration. Applying Newton’s law in
x-direction Ksp = s × (2s)2 = 4s3
f - F sin q = 0 Þ f = F sin q ... (i) = 4 × (0.5 × 10–4)3 = 5 × 10–13
In y-direction, F cos q + N = mg ... (ii) 34. (d) The electron goes to 4p after filling up to
l
Making t A = 0 Þ F l - mg cos q = 0 3d.
2
35. (a) Due to hydrogen bonding between the two
OH groups, gauche conformation of
F
ethylene glycol (a) is the most stable
y
B conformation.

N H
O
mg
H O
A f x H

mg cos q H H
Þ F= ... (iii)
2 H
For no slipping
36. (b) In SO2 the oxidation state of S varies from
f F sin q
f £ mN Þ m ³ Þm³ –2 to + 6. Hence, it can behave as reducing
N mg - F cos q
as well as oxidising agent both.
SOLUTIONS – MOCK TEST-9 201

37. (d) In CCl4, dipole moment of each C–Cl bond Gram equivalent of H2SO4
is cancelled by dipole moment of the other 100 ´ 0.1
C–Cl bond. = = 0.01
1000
Hence solution will be neutral.
45. (b) Chiral conformation will not have plane of
symmetry. Since twist boat does not have
plane of symmetry it is chiral.

38. (a) Carbon atom which is connected with four BIOLOGY


different groups is chiral. 46. (d) Fats make more electrons available to the
electron transport system and more
39. (a) CO32 - + H 2 O ¾¾
® HCO3- + OH - hydrogen ions available for chemiosmosis.
Base Acid Acid Base
47. (a) A - Stele: It is the central core of the stem
40. (a) As the basicity of metal hydroxides
and root of a vascular plant, consisting of
increases down the group from Be to Ba,
the thermal stability of their carbonates also the vascular tissues (xylem and phloem)
increases in the same order. Further, group and associated supporting tissues.
1 compounds are more thermally stable than B - Endodermis: It is an inner layer of cells
group 2 because their hydroxide are much
basic than group 2 hydroxides, therefore, in the cortex of a root and of some stems,
the order of thermal stability is surrounding the stele.
BeCO3 < MgCO3< CaCO3< K2CO3. C - Casparian strip: It is a band of cell wall
41. (c) Sodium reacts vigorously with cold water material deposited on the radial and
while Mg reacts slowly with hot water. transverse walls of the endodermis, and is
Under normal conditions Zn and Fe do not
chemically different from the rest of the cell
react with water. Hence, the reactivity order
wall, which is made of lignin and without
is Na > Mg > Zn > Fe.
suberin whereas the casparian strip is made
42. (c) When double and triple bonds, both are
present. Then bond nearest to terminal of suberin and sometimes lignin.
carbon will be preferred. But, if both are D - Bark: It is the outermost layer of stems
equidistant from terminal carbon then and roots of woody plants. Plants with
double bond will be preferred over triple bark include trees, woody vines and
bond. shrubs.
CH 3 - CH = CH - C º CH 48. (a) As in heavy rainfall, minerals are also
5 4 3 2 1 drained along with water, hence it will not
Pent -3- en -1- yne make minerals more available to plants.
43. (c) Glucose is considered as a typical 49. (d) Glutamic acid is an amino acid formed by
carbohydrate which contains –CHO and hydrolysis of proteins. It is the only amino
acid, which is metabolised by the brain.
–OH group.
50. (b) Epinephrine is secreted by medulla of
0.62 adrenal gland along with closely related
44. (b) Gram equivalent of Na2CO3.H2O = = noradrenaline.
62 It is used in medicine as a heart stimulant
0.01(Mol. wt. of Na2CO3.H2O = 124) and to constrict blood vessels. Chemically,
epinephrine is a derivative of amino acid
(aminohydroxy phenyl-propionic acid).
EBD_7839
202 KVPY-SA

51. (c) Insulin is secreted by the Islets of Langerhans 56. (a) AIDS is characterised by reduction in the
in the pancreas when the blood sugar level is number of helper T- lymphocytes because
high. This increases the rate of glucose uptake of HIV infection. It suppresses human
from the blood into the muscle cells. immune system due to which any secondary
52. (c) The animal found by the boy belongs to infection may lead to death.
phylum mollusca. Mollusca is the second T-lymphocytes are the main cells of immune
largest phylum after arthropoda and include system.
predominantly marine animals. They are 57. (c) Primary respiratory substrate is
triploblastic, bilaterally symmetrical, carbohydrate and secondary respiratory
schizocoelic and unsegmented substrate is fat.
protostomes. They have moist skin, a 58. (a) When blood is shed, th e platelets
complete digestive tract, a ventral nerve disintegrate and liberate thromboplastin
cord, and had gone through torsion. which activates prothrombin to thrombin.
53. (d) Plants can absorb water through their entire 59. (c) Lipoproteins are conjugated proteins having
surface. However, water is found in the soil polypeptides in association with lipids.
and only positively geotropic part, i.e., root Immunoglobulins are the constituent of
system is specialised to absorb water. In antibodies. Interferons (INFs) are a group of
root system, the most efficient region of three vertebrate glycoproteins (i.e. a, b, g,).
water absorption is the root hair zone or Out of these three a and b are produced
zone of cell differentiation. within virally infected cells.
54. (c) ELISA test is a technique used to detect 60. (c) Chitinase is an enzyme that cleaves the
and quantitate extremely small amount of a glycosidic bonds in chitin, thereby breaking
protein, antibody or antigen with the help down the structural polysaccharide
of enzyme. The commonly used enzymes component of the hard outer covering of
are peroxidase and alkaline phosphatase. many animals and of the cell wall of fungi.
Southern blotting and DNA probes are used
PART-II
in molecular analysis of DNA. Catalase is
not involved in ELISA. MATHEMATICS
55. (d) Primary succession on rocks starts with 61. (c) CD || AE and CY || BA.
lichens like Rhizocarpon, Rinodina and Since triangle on the same base and
Lecanora. They produce some acids which between the same parallels are equal in area,
bring about weathering of rocks. These so we have
lichens are then replaced by foliose type ar (DABC) = ar (DABY)
of lichens. Due to deep depressions and Þ ar (DCBX) + ar (DABX)
retention of water by them, they form a fine = ar (DABX) + ar (DAXY)
thin soil layer on rock surface and thus Hence, ar (DCBX) = ar (DAXY)
there is a change in the habitat. [Cancelling ar (DABX) from both sides]
SOLUTIONS – MOCK TEST-9 203

log x h 5 – r PC
62. (a) 4096. ( f ( x, x )) 2 = x13 Þ = =
12 5 AC
2
Þ 4096. x log 2 x = x13
h 5–r 2r 5 – r
Þ = Þ =
Take log on base 2 Þ 12 + log32 x = 13log2x 12 5 12 5
Þ 10r = 60 – 12r
1
Þ log2x = 1, 3, – 4 \ x = 2, 8,
16

161
\ Sum =
16
63. (a)

30 60
Þp= and h =
11 11

Q BE is median, Volume of cylinder = pr2h


\a+ b= c+d ...(1) 22 æ 900 ö 60
= » 127.50
also, CF is median, 7 çè 121 ÷ø 11
\a+ c= b+d ...(2)
65. (b) Sum of 16 observations = 16 × 16 = 256
From eqn. (1) – (2),
Sum of resultant 18 observations
(a + b) – (a + c) = (c + d) – (b + d)
= 256 – 16 + (3 + 4+5) = 252
Þ b – c = c – b Þ 2b = 2c Þ b = c
252
\ ar DGBC = ar (AFGE) Mean of observations = = 14
18
64. (c) Let h be the height and r be the radius of
the cylinder. PHYSICS
Given h = 2r 66. (c) If x is the object distance, then image dis-
Since, DPSC ~ DAOC tance v = D – x. Thus
\ By similarity of two triangles, we have
v D-x
m= =
PS SC PC u x
= =
AO OC AC or D – x = m x
EBD_7839
204 KVPY-SA

Ray comes out from CD, when it get


D
\ x = (1 + m ) reflected from AD.

n1 sin a max
n2 = sin r1
mD
and D – x =
1+ m
-1 é m ù
1 1 1 Þ amax = sin ê 1 sin r1 ú
Now using lens formula, - = , we ë m2 û
v u f
Also r1 = 90° – q = 90° – C
have
æ 1 ö
1 1 1 or r1 = 90° – sin -1 ç
Þ - = è 2 m1 ÷ø
æ mD ö æ D ö f
+ç ÷ -ç ÷
è1+ m ø è 1+ m ø
æm ö
or r1 = 90° – sin -1 ç 2 ÷
After solving, we get è m1 ø

mD
f= . é
-1 m1 ì -1 m 2 ü
ù
(1+ m) 2 \amax= sin ê sin í90° – sin ýú
êë m 2 î m1 þúû
67. (b) In figure, "C" reaches the position where
p é ìï -1 æ m 2 ö üï ù
"A' already reaches after wt = and "A" -1 m1
2 = sin ê m cos ísin çè m ÷ø ý ú
reaches the position where "B" already ëê 2 îï 1 þ ï ûú

p 70. (c) Let q0 be the temperature of surroundings.


reaches after wt = .
2 According to Newton's law of cooling,
The rate of cooling µ excess temperature
68. (d) In case of destructive interference
Phase difference f = 180° or p Body cools 80°C to 64°C : Average rate of
So wave pair (i) and (ii) will produce 80 - 64
destructive interference. cooling = °C / min
5
Stationary or standing waves will produce
by equations (iii) and (iv) as two waves é 80 + 64 ù
Mean temperature = ê °C
travelling along the same line but in opposite ë 2 úû
direction.
é 80 + 64 ù
\ Excess temperature = ê - q0
ë 2 úû
69. (a) A µ2 D

r
µ1 80 - 64 é 80 + 64 ù
a max =K ê - q0 ú ... (i)
5 ë 2 û
B C where K is a constant of proportionality.
SOLUTIONS – MOCK TEST-9 205

Body cools 64°C to 52°C : Average rate of 72. (a)


64 - 52 OH
cooling =
5 Å
+
H [1, 2]-methyl shift
64 + 52 ¾®
Mean temperature =
2

\ 64 - 52 = K é 64 + 52 - q0 ù ...(ii)
5 ê 2 ú
ë û
Dividing eqn.(i) by (ii), we get

16 72 - q 0
= 73. (a) Given: T = 27°C = 27 + 273 = 300 K
12 58 - q0
V = 10.0 L
By solvin g, we get temperature of Mass of He = 0.4 g
surrounding, q0 = 16°C Mass of oxygen = 1.6 g
Let q0 be the temp. at the end of next 5 min- Mass of nitrogen = 1.4 g
utes (or after 15 minutes from start) nHe = 0.4/4 = 0.1
\Average rate of cooling nO = 1.6/32 = 0.05
2
52 - q nN = 1.4/28 = 0.05
= °C / min 2
5
52 + q ntotal = nHe + nO2 + nN = 0.1 + 0.05 + 0.05
2
Mean temperature =
2 ntotal = 0.2
52 - q é 52 + q ù n RT 0.2 ´ 0.082 ´ 300
\ =K ê - q0 ú ...(iii) P= = = 0.492 atm
5 ë 2 û V 10
Dividing eqn. (iii) by (i) and putting 74. (a) I 2 (s) + Cl 2 (g) ¾¾
® 2ICl(g)
q0 = 16°C DrH = [DH(I2(s) ® I2(g)) + DHI–I + DHCl–Cl]
52 + q – [DHI – Cl]
- 16
52 - q
= 2 = 62.76 +151.0 + 242.3 –2 × 211.3 = 33.46
16 72 - 16
33.46
By solving, we get q = 43°C Χf H°(ICl) < < 16.73 kJ/mol
2
CHEMISTRY
75. (a) CH3 – CH 2 – C º CH + 2Br2 ¾®
®
71. (c) In a period, the value of ionisation potential 1–Butyne
increases from left to right with breaks
where the atoms have somewhat stable Br Br
configuration. In this case, N has half filled CH3 CH2 C C H
stable orbitals. Hence it has highest
Br Br
ionisation energy. Thus, the correct order
is
B< C< O < N
EBD_7839
206 KVPY-SA

Since the molecule takes 2 moles of Br 2. 78. (a) Homologous structures are the structures
Therefore it is alkyne. Also it gives white of different organisms which have similar
ppt with Tollen’s reagent therefore acidic H basic structure but different functions.
is present. Hence, it is 1-Butyne. Such organisms have evolved from
common ancestor. For example, thorns and
BIOLOGY
tendrils are modified stem but perform
76. (b) The given sequence shows change in the different functions. Thorns are used to
species structure of an ecological
community over time hence it represents check transpiration and to protect the
an ecological succession. plants from grazing animals whereas
77. (a) After growing bacteria in culture tendrils are used for support and
containing radioactive adenine (A*), both attachment by climbing plants.
strands of bacterial DNA will have 79. (b) A protease is an enzyme that catalyses the
radioactive adenine. During one round of breakdown of proteins into smaller
replication in a medium containing polypeptides or single amino acids. So, on
nonradioactive adenine (A), both strands protease treatment, enzyme structure is
function as template strand for the destroyed as well as its activity. On heating,
synthesis of new strands. At the end, two enzyme may lose its activity due to its
DNA strands formed will have one of the denaturation at high temperature. On
parental strands with radioactive adenine freezing, enzyme activity is decreased but
and other newly synthesised strand with during thawing, enzyme activity is restored.
nonradioactive adenine (A). Such 80. (b) When the source of CO2 is eliminated, the
combination is present in DNA sequence reaction catalysed by RuBisCo is stopped.
of option (a). Options (b) and (d) have As a result, both production of 3PG and
radioactive adenine (A*) in both the the consumption of RuBP also stop. The
strands whereas option (c) has other reactions of the cycle can continue
nonradioactive adenine (A) in both the to synthesise RuBP until their substrates
strands. are used up. Therefore, 3PG levels will fall,
and RuBP levels will rise.
MOCK TEST-10

PART – I 3. (a) Q 49 a + 7b + c =286


MATHEMATICS Þ 7 (7a + b) + c = 286
Q c is single digit positive number
2f ( n ) + 1
1. (b) Given f (n + 1) = \ possible value of c = 6.
2
\ 7 (7a + b) = 280
and f (1) = 2 then f (101) = 2 \ 7a + b = 40
2f (1) + 1 5 also, b is a single digit positive number,
for n = 1, f (2) = = \ possible value of b = 5.
2 2
\ 7a = 35 Þ a = 5.
2f ( 2 ) + 1 \ we have a = 5, b = 5, c = 6.
for n = 2, f (3) = =3
2 \ 100a +10b + c = 500 + 50 + 6 = 556.

2f ( 3) + 1 6 + 1 7
f (4) = = = 4. (c)
2 2 2

5 7
So, f (1), f (2), f (3), f (4), ……, 2, ,3 ,
2 2

3+ n 3 + 101
f(n) = f(101) = = 52
2 2 DP 1
\ =
2. (c) From the given condition N + 648 and AP 3
N – 739 is a perfect cube.
ar DDPE 12 1
We can assume N + 648 = a3 and \ ar DDAB = =
N – 739 = b3
(1 + 3) 16
2

Or a3 – b3 = 1387 or (a – b)(a2 + ab + b2)


= 1387 = 1 × 1387 = 19 × 73 \ ar DDPE = 1 × R = R
Case (i) if (a – b)(a2 + ab + b2) = 1 × 1387 and ar DDAB = 16R
then a – b = 1 or a = 1 + b then
QC 1
(a2 + ab + b2) = 1387 also, =
BQ 3
Hence (1 + b)2 + b(1 + b) + b2 = 3b2 + 3b + 1
= 1387 or 3b(1 + b) = 1386 or b(1 + b) = 462
ar DBCD (1 + 3) 2 16
or b = 21 \ = =
ar DBQE 12 1
Hence b3 = 213 = 9261 hence N = 9261 + 739 =
10000, so statement (i) and (iii) are correct. \ ar DBCD = 16R
Case (ii) if we take (a – b)(a2 + ab + b2) ar of Trap ABCD = ar DBCD + ar DABD
= 19 + 73 then a = b + 19 and (a2 + ab + b2) = 16R + 16R = 32R
= 73 but a2 must be more than 192 hence ar DDPE R 1
this case is not possible. \ = =
ar of Trap ABCD 32 R 32
EBD_7839
208 KVPY-SA

5. (b) From the given condition N = K100 (100!) In DBDC and DBRF
Since 100! has the highest power of 2 and 5 ÐD = ÐR = 90º
as 97 and 24, but N is divisible by 10100 i.e.
& ÐB = ÐB (common)
2100 and 5100. Hence K must be the multiple of
2 and 5. \ DBDC ~ DBRF (by AA similarity)
For the minimum value K = 10, then N = 10100 Q F is mid point of BC
(1001). Hence the minimum number of zeros by converse of BPT, R is mid point of BD.
at the end of N is 124 also, E is mid point of AD, and R is mid
point of BD.
\ AE + RB = ED + DR
6. (c)
1
\ ER = AB = 4
2

Similarly, by BPT
1
FR = CD = 3
2
\ in DEFR, applying pythagoreous
theorem
Q NB || MX we get EF2 = 32 + 42
\ DTNB ~ DTMX Þ EF = 5

TB TN
\ = ...(1)
TX TM
also we have NX || MC
\ DTNX ~ DTMC

TN TX
\ = ...(2)
TM TC
From (1) and (2)

TB TX
We get, = Þ TX2 = TB . TC
TX TC
7. (b) Let E & F be the mid points of AD and BC. 8. (a)
Draw FR perpendicular to AB

Let the side of square be a.


Then, radius of inner circle = half of side
a
square =
2
SOLUTIONS – MOCK TEST-10 209

Therefore, the area of inner circle 1


[QBP = BC, as P is the mid-point of BC]
= p (radius ) 2 2
\ BPQR is a parallelogram.
2 2
æ aö a Since the diagonals of a parallelogram
= pç ÷ = p
è 2ø 4 always bisect each other
\ X is the mid-point of PR.
Since, the diagonal of the square = 2a Similarly, Y is the mid-point of PQ.
Then, radius of outer circle = half of In DPQR, XY is the line segment joining
2a the mid-points of sides PQ and PR.
diagonal of square =
2 1
\ XY = RQ
Hence, the ar ea of outer circle 2
But, in DABC, RQ is a line segment joining
2
æ 2a ö 2a 2 the mid-points of sides AB and AC.
= pç ÷ = p .
è 2 ø 4
1
\ RQ = BC
2
Hence, the required ratio of the areas of
inner circle to outer circle is = 1 1 æ1 ö 1
Hence, XY = RQ = çè BC÷ø = BC
2 2 2 4
a 2 2a 2 10. (b) Const: Join C to D,
: = 1: 2
4 4

A
9. (c)

Q AD is diameter,
\ ÐACD = 90°, also AB = BC
R Q \ ÐBCA = ÐBAC = 25°
Q ABCD is cyclic quad.
\ ÐBAD + ÐBCD = 180°
X Y Þ (25° + ÐCAD) + (25° + 90°) = 180°
Þ ÐCAD = 40°
B P C 11. (a) Let Speed of flight = x km/hr
Time duration of flight = y hours
Join R and Q. xy = 600 ...(1)
RQ is a line segment joining the mid-points
( x - 200) æçè y +
30 ö
of two sides AB and AC of DABC. ÷ = 600 ...(2)
60 ø
1 From eq. (1) & (2),
\ RQ || BC and RQ = BC
2 æ 1ö
Þ RQ || BP and RQ = BP xy = ( x - 200) ç y + ÷
è 2ø
EBD_7839
210 KVPY-SA

Þ xy = xy +
x
- 200 y - 100 (
Also 7 + 5 2 = )
2

(a + b )
3
x = a3 + 3ab + 3a 2 b + b b
Þ 200 y + 100 =
2
Equating rational and irrational parts
Þ x = 400y + 200
Put value of x in eq. (1) a 2 + 3ab = 7 and ( )
b 3a 2 + b = 5 2
(400y + 200)y = 600
On solving we will get a = 1, b = 2.
Þ (2y + 1)y = 3 Þ 2y2 + y – 3 = 0
Þ (2y + 3)(y – 1) = 0 (
Hence required 6th root is 1 + 2 )
3 13. (d) First consider the single digit number:
y = - , y = 1 hr.
2 122333…….99…..9 times
Here total number of digits used
-3
As y = is not possible, thus y = 1 hrs. = 1 + 2 + 3 + ………… + 9 = 45
2 Remaining digits = 1000 – 45 = 955
12. (a) In order to find 6th root first we will Now consider two digit numbers
calculate its square root and then cube 101010….10 times 111111…..11 times
root or vice– versa. (20 digits) (22 digits)
Let square root of 99 + 70 2 is p + q Let while writing down nth number we
get 955th digit then
Hence ( p + Ö q) 2 = 99 + 70 2 20 + 22 + ………… n terms e” 955’!
n/2[40 + (n – 1)2] = n(19 + n) e” 955
Or p + p + 2 pq = 99 + 70 2 From trial & error n = 20,n(n + 19)
Equating rational and irrational parts we = 20 × 39 = 780
will get p + q = 99 n = 21, n(n + 1) = 21 × 40 = 840
pq = 2450 n = 22, n(n + 1) = 22 × 41 = 902
On solving these two equations we will n = 23, n(n + 1) = 23 × 42 = 986
get p = 50, q = 49, Hence when n = 22 the number = 31 will
be written 31 times. We will use 902 digits
Hence square root of 99 + 70 2 hence after we will write 32 times.
= 49 + 50 = 7 + 5 2 Remain digits = 955 – 902 = 53
Now we have to find cube root of Hence to get 955 digits when we write 26
times 32 and one 3 of next 32.
7+5 2 Hence last 3 digit number will be 323
Let it be a + b then cube root of 7 – 5 14. (d) since p = 1! + (2 × 2!)+ (3 × 3!) +...+ (10 × 10!)
We can write 1! = 1 × 1! = (2 – 1)1!
2 is a – b While 2 × 2! = (3 – 1)2! = 3! – 2! and so on
Hence given expression is p = (2–1)1! +
(
Hence 3 7 + 5 2 7 - 5 2 )( ) (3–1)2! + (4–1)3! + …… + (11–1)10!
Or p = 2! –1! + 3! – 2! –2! + 4! –5! + - - -
(
= a+ b a- b )( ) 11! –10!= 11! –1! = 11! – 1
Or p + 2 = 11! + 1
= ( 49 - 50 )
1/ 3
= ( -1)
1/3
=–1 Hence (p + 2) leaves a reminder of 1 when
divided by 11!
= a 2 - b or a 2 = 1 - b
SOLUTIONS – MOCK TEST-10 211

15. (b) Let n(B) = x n(A) = x + 2 2


n( F ) = y n(C) = y + 3 On comparing with, a = -w y , we get
n(D) = z n(E) = z + 5
3rgA m
x + y + z + x + 2 + y + 3 + z + 5 = 40 w= and T = 2p
x + y + z = 15 but x ³ 1, y ³ 1 and z ³ 1. So
m 3rgA
total ways of distribution is 14C2 = 91 18. (a) According to Wein’s displacement law
PHYSICS lm × T = constant

r Here , l m3 < l m2 < l m1


16. (a) The velocity of wind, v = vx $i + vy $j Þ T3 > T2 > T1
Velocity of boat w.r.t. bank vb = 3 + 1 = 4 m/s
r r r
Now éë v w ùû b = v w – vb = (vx – 4) $i + vy $j

The temperature of Sun is higher than that


of welding arc which in turn is greater than
tungsten filament.
19. (b) The given rod system is equivalent to as
Given; vx – 4 = 0 Þ vx = 4 m/s. shown in figure.
r 0 r r
( )
And éëv w ùû = v w – vb = v x $i + v y $j - 3j$
b

(
= v x $i + v y - 3 $j )
vy - 3
Given; tan 45° = Þ vy = vx + 3 = 3 + 3 = 6 m/s
vx
17. (a) Suppose the liquid in left side limb is
trest = -2mg ´ é l sin qù / 2
displaced slightly by y, the liquid in right
limb will increase by y/2. The restoring force ê 2 ú
ë û

1
2 mg l(-q)
trest 2
and a = ;
é (2m) ( 2ù
l/ 2) ú
I
ê
ë 3 û

3 g
æ 3y ö = (-q)
F = –PA = –rg ç ÷ ´ 2 A = 3rgA( - y) . 2 2 l
è 2ø
Comparing with , a = -w2q we get
F
a= = 3rgA( - y ) / m
m 3g 2 2l
w= and T = 2p .
2 2l 3g
EBD_7839
212 KVPY-SA

V V ± DV M a 27T04 J
20. (d) R= Þ R ± DR =
I I ± DI Þ dt =
s 4pR 2 ´ 80T04

æ ö Solving we get,
æ DR ö V ç 1 ± DV / V ÷
R ç1 ± = ç
è R ÷ø I ç 1 ± DI
÷
÷
Time taken for the sphere to cool down
è I ø temperature 2T0,

Ma æ 16 ö
æ DR ö æ DV ö æ DI ö t= ln ç ÷
ç R ÷ =ç ÷ + ç ÷ = (3 + 3)% = 6% 16pR s è 3 ø
2
è ø è V ø è I ø
21. (d) The real gas cannot be liquefied above 25. (c) The resistance of the heater is
critical temperature by applying pressure.
l V 2 100 ´ 100
2 R= = = 10 W
22. (d) As, g = 4p P 100
T2
The power on which it operates is 62.5 m
Dg Dl DT
So, ´ 100 = ´ 100 + 2 ´ 100 \ V = R ´ P ' = 10 ´ 62.5 = 625 = 25
g l T
Since the voltage drop across the heater is
0.1 1 25m hence voltage drop across 10W resistor
= ´100 + 2 ´ ´ 100 = 2.72 ; 3%
20 90 is (100 – 25) = 75 m.
23. (a) R = (2 p r) (1/p) = 4 p A B Heater
Here 1 ohm and 3 ohm will now be in parallel R
\ Reff = (3/4) p Þ V/Reff = 8 A.
24. (c) In the given problem, fall in temperature of
100 V
sphere,
dT = (3T0 – 2T0) = T0 V 75
\ The current in AB = I = = = 7.5 A
Temperature of surrounding, Tsurr = T0 R 10
This current divides into two parts. Let I1
Initial temperature of sphere, Tinitial = 3T0
be the current that passes through the
Specific heat of the material of the sphere heater. Therefore
varies as, 25 = I1 × 10 Þ I1 = 2.5 A
C = aT3 per unit mass (a = a constant) Thus current through R is 5A.
Applying Ohm's law across R, we get
Applying formula,
25 = 5 × R Þ R = 5W
26. (b) In fig, C1 and C2 are IC (instantaneous
dT
=
sA 4
dt McJ
( 4
T - Tsurr ) centre of rotation) of the two cylinders. The
cylinder can be considered as rotating
about C1 and C2. In the absence of slipping
T0 s 4 pR 2 é
Þ = (3T )4 - ( T0 ) 4 ùúû
dt M a ( 3T )3 J êë 0
between the plank and the cylinders, points
0 A1 and A2 have the same velocity.
SOLUTIONS – MOCK TEST-10 213

Angular velocity of the larger cylinder is CHEMISTRY


2v v 31. (a) No. of molecules in different cases
=
4R 2R (a) Q 22.4 litre at STP contains
A2 A1 = 6.023 × 1023 molecules of H2
2v
R \ 15 litre at STP contains
2R v
15
= ´ 6.023 ´ 10 23
22.4
C1
= 4.03×1023 molecules of H2
C2 (b) Q 22.4 litre at STP contains
= 6.023×1023 molecules of N2
æ v ö
v CM = ( 2R ) ç =v
è 2R ÷ø \ 5 litre at STP contains
27. (a) According to given conditions TIR must 5
= ´ 6.023 ´ 10 23
take place at both the surfaces AB and AC. 22.4
Hence only option (a) is correct. = 1.344 × 1023 molecules of N2
28. (d) v2 = w2(A2 – x2) … (i) (c) Q 2 g of H2= 6.023×1023 molecules of H2
and a2 = (w2x)2 = w4x2 … (ii)
From above equations, we have 0.5
\ 0.5 g of H2= ´ 6.023 ´10 23
2
a2
v2 = - 2
+ w2 A2 Þ Y = mX + c = 1.505 × 1023 molecules of H2
w
It represents straight line with negative (d) Similarly 10 g of O2 gas
slope.
29. (b) Sound waves in air are longitudinal while 10
= ´ 6.023 ´ 10 23 molecules of O2
light waves are transverse. Sound waves 32
require material medium to propagate but = 1.88 × 1023 molecules of O2
transverse waves do not require any
material medium. Thus (a) will have maximum number of
30. (a) For whole system molecules.
f1 - f 2 = 3 (1)a ... (i) 32. (b) Due to mesomeric effect (+M) of –OH
group the electron density on benzene ring
increases. So the electrophile easily attack
on these electron rich center.
m
33. (b) During the dissolution of alkali metal
hydrides energy is released in large amount,
////////////////////////////////////// i.e., it is exothermic in nature.
f1 f2
34. (b) PbO2 is a powerful oxidizing agent and
For rear cylinder
liberate O2 when treated with acids.
100 – f1 = 0.5 (a) ... (ii)
For front cylinder 2PbO 2 + 4HNO 3 ¾¾
®
f2 = 0.5 (a) ... (iii) 2Pb(NO3 ) 2 + 2H 2 O + O 2 ­
From (i), (ii) and (iii)
100 = 4a 35. (d) Copper is more reactive than silver hence
a = 20 m/s2 displaces silver from silver nitrate solution.
Telegram @unacademyplusdiscounts

Join Us on Telegram for More Such Books

https://telegram.me/unacademyplusdiscounts

Join us from the above link or search ''unacademyplusdiscounts'' in Telegram


EBD_7839
214 KVPY-SA

36. (d) A compound is said to be aromatic if it is Þ P = 1.125 ´ .0821 ´ 273


planar and there is complete delocalization
P = 25.22 atm
of p-electrons, which is only possible if it is a
44. (b) Diacidic base has two replaceable hydroxyl
conjugated cyclic system and number of p
groups.
electrons used in delocalization is (4n + 2).
–OH groups present in compounds (a) and
1, 3, 5-heptatriene is not an aromatic
(c) represent alcoholic group, not base.
c om pou n d because comp l et e
45. (c) When gas is compressed its entropy
delocalization of p-electrons, is not
decreases, so, DS is negative.
possible in it.
37. (b) The sub-shells are 3d, 4d, 4p and 5s; among BIOLOGY
these 4d has highest energy as n + l value 46. (c) SA (sinoatrial) node lies in the wall of right
is maximum for it.
atrium near the opening of superior vena
38. (c) Draw a line at constant P parallel to volume
cava. It initiates the electrical impulse to
axis. Take volume corresponding to each
stimulate contraction. From SA node,
temperature.
From volume axis, V1 > V2 > V3 cardiac impulse travels to AV node
Hence, T1 > T2 > T3. (atrioventricular node, which lies between
39. (a) Higher the value of reduction potential right atrium and ventricle), then passes to
higher will be the oxidising power whereas AV bundle (also called bundle of His) and its
lower the value of reduction potential branches reaches to the Purkinje fibres in
higher will be the reducing power. ventricles. Purkinje fibres conducts the
40. (b) Eq. of acid = Eq of base, impulses five times more rapidly than
surrounding cells. It forms a pathway for
0.45 20 ´ 0.5
\ = , = E.Wt = 45 conduction of impulse that ensures that the
E.Wt 1000
heart muscle contracts in the most efficient
M.Wt 90 manner.
Basicity = = =2 47. (d) Peroxisomes contain oxidative enzymes
E.Wt 45
such as catalase.
41. (a) CaO is basic as it forms strong base
48. (d) For an action potential to occur in an axon,
Ca(OH)2 on reaction with water.
the membrane must be depolarised above a
CaO + H2O ¾® Ca(OH)2
certain level. This level is known as the
CO2 is acidic as it dissolves in water forming
unstable carbonic acid. threshold.
H2O + CO2 ¾® H2CO3 49. (c) In monocot stem, vascular bundles are
Silica (SiO2) is insoluble in water and acts scattered. Each bundle is surrounded by a
as a veryweak acid. thick-walled bundle sheath.
SnO2 is amphoteric as it reacts with both Due to disintegration of protoxylem
acid and base. elements, a lysigenous water cavity is
SnO2 + 2H2SO4 ¾® Sn(SO4)2 + 2H2O formed.
SnO2 + 2KOH ¾® K2SnO3 + H2O 50. (a) Excessive nutrients run off from the land
42. (b) For isomeric alkanes, the one having into water body. These nutrients such as
longest straight chain has highest b.p. nitrates, phosphates and urea act as
because of larger surface area. nutrients and accelerate the growth of algae
43. (a) Total moles that may form a mat on water surface. This
process of enrichment of water body with
4 2
= + = 1.125; PV = nRT nutrients is called entrophication.
32 2
SOLUTIONS – MOCK TEST-10 215

51. (b) Only 10 percent of the energy is transferred PART – II


between trophic levels. MATHEMATICS
52. (a) Vaccine contains dead, attenuated form or
antigen of a pathogen which can be injected 61. (a) Here in quadrilateral APCB, AB || CP
to provide immunity towards that pathogen. Since triangles on the same base and
Monoclonal antibodies are homogeneous between same parallel lines are equal in
immunological reagents of defined area.
specificity, so that these can be utilised for Þ Area (DAPC) = Area (DBPC) .....(i)
diagnosis and screening with certainty. Also in quadrilateral ADQC, AD || CQ
53. (b) Photosynthesis occurs most efficiently with \ Area (DDCQ) = Area (DACQ)
red or blue light because the chlorophyll or Area (DDPQ) + Area (DPCQ)
pigments can absorb light maximally at these = Area (DAPC) + Area (DPCQ)
wavelengths and the least at green light. Þ Area (DDPQ) = Area (DAPC) .....(ii)
54. (c) Atropine is a substance that blocks the
From (i) & (ii)
inhibitory effects of muscarinic receptors
Þ Area (DBPC) = Area (DDPQ)
in muscle tissue, especially the heart.
55. (c) Purkinje fibres are a vital component of the
system of blood circulation in humans.
These fibres are present at both ventricular
myocardium for the proper contraction of
ventricles. They receive conductive signals
originating at the atrioventricular node
(AVN), and simultaneously activate the left
and right ventricles by directly stimulating
the ventricular myocardium.
56. (c) Oxytocin stimulates muscle contractions
62. (c) Let t = 5x + 2, then A = {t : 0 £ t £ p}
during childbirth and milk letdown. \ f (t) = cos t which is bijective in [0, p].
57. (b) The enzyme salivary amylase present in Hence, f (x) is bijective.
saliva, converts starch into maltose. It is
the maltose that tastes sweet. Maltase 63. (d)
secreted by the intestine converts maltose
into glucose.
58. (a) Resting membrane potential is greater in a
large size nerve fibre than in a small size
nerve fibre. In larger nerve fibres, it can go
up to –80mV and in small size fibre it is
–60mV.
59. (d) Lactic acid is a three-carbon organic acid
formed in muscle resulting in fatigue. Construction: Join D to C.
60. (a) Inhaled air passes from bronchi into Q DP || QC
bronchioles that lead to alveolar ducts, \ ar DDPC = ar DQDP
which terminate at alveoli.
EBD_7839
216 KVPY-SA

(same base DP, and between two parallel Hence, the required volume of sphere
lines DP and QC)
3
Q D is mid point, 4 3 4 æ hö 4ph3
= pr = p ç ÷ =
\ DC is median, 3 3 è 3ø 81

1
\ ar (DBDC) = ar (DABC) 65. (a) There are 2n observations x1, x2, ..., x2n.
2
2n
1 x
Þ ar (DBDP) + ar (DDPC) =
2
ar (DABC) So, mean = å 2ni .
i =1

1 Let these observations be divided into two


\ ar (DBDP) + ar (DQDP) = ar (DABC)
2 parts x1, x2, ..., xn and xn+1, ..., x2n
Each in 1st part 5 is added, so total of first
1
\ ar (DBQP) = ar (DABC) n
2 part is å xi + 5n .
i =1
64. (d) Since vertical angle of the cone = 60º
In second part 3 is subtracted from each
P 2n
So, total of second part is å xi - 3n.
i = n +1
6 0°
Total of 2n terms are
l l
60° S h
r n 2n 2n
O
å xi + 5n + å xi - 3n = å xi + 2n
i =1 i =n +1 i =1
Q R
2n 2n
xi + 2 n x
Mean = å 2n
= å i +1
2n
i =1 i =1
3 2h
h= l Þl= So, it increase by 1.
2 3

Q ÐOPS = 30° PHYSICS


66. (b) Remote half from the sun is the arc B'A'B.
l h
\ In DOPS, PS = = ...(1) B
2 3

OS A A'
In DPOS, tan 30° = S C
PS

1 OS PS h h B'
Þ = Þ OS = = =
3 PS 3 3 3 3 Let t be the time of description of the arc
B'A'B, therefore rate of description of the
Therefore, the radius of sphere is
sectorial area SB'A'BS
h area SB ¢A¢BS
r = OS = =
3 t
SOLUTIONS – MOCK TEST-10 217

Since the whole area of the ellipse is -8(2 cos q - sin q)


described in a year, therefore rate of or = 0 or 2 cos q – sin q = 0
(2 sin q + cos q) 2
description of the sectorial area is
or tan q = 2
area of the ellipse From equation (i),
=
a year
8 8
vmin = = = 3.57 m/s
¢ ¢
Hence, area SB A BS = area of the ellipse æ 2 ö 1
2ç ÷ +
5
t a year è 5ø 5
t area SB¢A¢BS 68. (d) From, PV = nRT, we have
or =
year area of the ellipse PAVA n
Þ = A
PBVB nB
1
area of the ellipse + D SBB¢
= 2 æ 4T ö 4 3
pab
çè 8 + r ÷ø ´ 3 prA nA
A
1 1 Þ = n
pab + .2bae æ 4T ö 4 3 B
=
2 2 1 e
= + çè 8 + r ÷ø ´ 3 prB
pab 2 p B
nB
After substituting values, we get = 6.
æ ö 1 e 1 e nA
\ t = ç + ÷ year = year + year
è2 pø 2 p b c
Q æ S Dq ö æ 1 ö
69. (d) Let = ha ç
=
1
year + 2 days nearly A è h ÷ø çè rg ø÷
2
Solving e = 1/60 Using dimensional method
67. (c) Let the man starts crossing the road at an [MT–3] = [ML–1T–1]a [LT–2]b [M–1L2T2]c
angle q as shown in figure. For safe
or, [MT–3] = [Ma–cL–a + b + 2c T –a – 2b + 2c]
crossing the condition is that the man must
cross the road by the time the truck Equating powers and solving
describes the distance 4 + AC or 4 + 2cotq. we get, a = 1, b = 1, c = 0
4 + 2 cot q 2 / sin q Q S Dq
\ = or v \ =h
8 v A h
8
= ... (i) 70. (a) Let P be the common point of the two
2sin q + cos q parabolas where the two particles collide.
dv Let v1, v2 be the velocities of m1 and m2 at
For minimum v, =0 the time of collision.
dq

A C P
90°

2m Truck v0 v S

q
B
4m
EBD_7839
218 KVPY-SA

Since the two paths are parabolic, therefore Now substituting for v12 from (1), we have

2m m12 + m22 2m æ2 1ö
v12 = = v22 (at the point P) ... (i) =mç - ÷
R ( m1 + m2 )2 R è R a¢ ø

é m ù 1 ìï m2 + m22 üï 2 2m1m2 2
ê acc. = ú \ = í1 - 1 ý× = ×
(distance) 2 ûú a¢ ïî ( m1 + m2 ) ïþ R (m1 + m2 ) R
2 2
ëê
After collision the two masses combine (m1 + m2 )2
i.e. 2a¢ = .R
into one, let V be the velocity after 2 m1m2
impact of the single mass (m1 + m2 ).
By th e principle of conservation of ( m1 + m2 ) 2
That is, major axis = ×R
momentum the resultant momentum must 2m1m2
be the same after and before the impact : CHEMISTRY
2 2
\ ( m1 + m2 ) V = m12 v12 + m22 v22 ... (ii) 71. (a) Total energy (En) = K.E + P.E
Because v1 and v2 are at right angles. in first excited state
From (i) it is seen that v2 = v1 and then from
1 é Ze 2 ù 1 Ze 2 Ze2
(ii), we have E = mv 2 + ê - ú =+ -
2 êë r úû 2 r r
(m1 + m2 )2 V 2 = (m12 + m22 )v12
1 Ze2
m12 + m22 -3.4 eV = -
i.e. V 2 = v12 ... (iii) 2 r
( m1 + m2 ) 2
1 Ze2
It is seen from (iii) that after the collision \ K.E = = +3.4 eV
2 r
the (velocity)2 becomes less than v12, that
2m 72. (c)
is less than , which is the condition for
R
describing an ellipse.
So after the collision the path will be an
ellipse. Let 2a' be its major axis.
For the path to be an ellipse,

æ2 1 ö
v2 = m ç - ÷
è r a¢ ø
+
At the point P, just after the collision v = V C
H3
and r = R.

So, V 2 = m æç
2 1ö
- ÷
èR a¢ ø

Substituting for V2 from (3), we have

m12 + m22 æ2 1ö
v12 = m ç - ÷
( m1 + m2 ) 2 è R a¢ ø
SOLUTIONS – MOCK TEST-10 219

73. (d) Second ionization energy is amount of When plant cell ‘A’ with 11 atm osmotic
energy required to take out an electron from pressure is kept in intimate contact with
the monopositive cation. plant cell ‘B’ with 9 atm osmotic pressure,
water will move from plant cell ‘B’ to ‘A’.
® M 2 + (g) + 2e- ... (5)
M (g) ¾¾ 77. (d) Human beings have 44 autosomes and
2 sex chromosomes in each cell. Females
® M + (g) + e - ... (3)
M (g) ¾¾
have two X chromosomes (46, XX) while
Hence I.E. from (5) – I.E. from (3) will be males have one X chromosome and one Y
I.E.2 of M. chromosome (46, XY). Females receive one
74. (c) (i) [Cr(H 2 O)6 ]Cl3 : X chromosome from their mother and the
other X chromosome from their father.
x + 6 × 0 + (–1) × 3 =0
Males receive X chromosome from their
x =+3
mother and Y chromosome from father. So,
(ii) [Cr(C6H6)2]: the female has received her X chromosome
y+ 2× 0 = 0 having TDF gene from her father. Though
y=0 TDF gene is found on the Y chromosome
(iii) K2[Cr(CN)2(O)2 (O2)(NH3)] : but this gene is moved to X chromosome
2 × 1 + z + 2 × (–1) + 2 × (–2) + (–2) + 0 due to pairing (synapsis) of the X and Y
=0 ch romosomes and cr ossing over
z = +6 (recombination) between them.
The oxidation states of Cr in given 78. (b) Allele for black coat colour = XB
compounds are +3, 0 an d + 6 Allele for orange coat colour = Xb
respectively. Since males have only one X chromosome,
75. (c) Because of high electronegativity of the they can either have allele XB or Xb for
halogen atom, the carbon halogen (C – X) black or orange coat colour, respectively.
bond is highly polarised covalent bond. In cont ra st, fema les h ave t wo X
Thus, the carbon atom of the C – X bond ch romosomes so thr ee di fferent
becomes a good site for attack by genotypes - XBXB, XBXb, XbXb with black,
nucleophiles (electron rich species). calico and orange coat colour, respectively
Nucleophilic substitution reactions are the are possible. But, due to X inactivation,
most common reactions of alkyl halides. females heterozygous for alleles can have
BIOLOGY mosaic pattern i.e., patches of both black
and orange coat colour.
76. (b) Water moves from lower osmotic pressure
79. (d) Plant B is a long day plant because it needs
to higher osmotic pressure. So, when a
plant cell ‘A’ is kept in solution of 10 atm a minimum of 15.5 hrs of light to flower.
osmotic pressure, endosmosis will occur Plant A with critical day length of 15.5 hrs,
till equilibrium is achieved that is, both the requires 8.5 or more hours of darkness to
solution and plant cell has 11 atm osmotic flower. So, it is a short day plant. Plant C
pressure. When a plant cell ‘B’ is kept in and D are also short day plants because
solution of 8 atm osmotic pressure, they flower when they receive light less
endosmosis will occur till both solution and than critical day length of 10 hrs and
plant cell ‘B’ has 9 atm osmotic pressure.
9.5 hrs, respectively.
EBD_7839
220 KVPY-SA

80. (c) The steps in glycolysis are as follows:

Glucose
first
priming 1 ATP
reaction
ADP
Glucose 6-phosphate (a)

Fructose 6-phosphate (b)


second
priming 3 ATP
reaction ADP
Fructose 1, 6-bisphosphate (c)
cleavage
of 6-carbon
sugar
phosphate to 4
two 3-carbon
sugar
phosphates
Glyceraldehyde 3-phosphate (d)
+
Dihydroxyacetone phosphate (e)
5

Glyceraldehyde 3-phosphate (2) (f)


2P
oxidation and 2NAD+
6
phosphorylation +
2 NADH + H
1, 3-Bisphosphoglycerate (2) (g)
first ATP- 2ADP
forming reaction
7
(substrate-level
phosphorylation) 2 ATP
3-Phosphoglycerate (2)

2-Phosphoglycerate (2)

9 2H2O

Phosphoenol pyruvate (2) (h)


second ATP- 2ADP
forming reaction
10
(substrate-level 2 ATP
phosphorylation)
Pyruvate (2)
Telegram @unacademyplusdiscounts

Join Us on Telegram for More Such Books

https://telegram.me/unacademyplusdiscounts

Join us from the above link or search ''unacademyplusdiscounts'' in Telegram


Telegram @unacademyplusdiscounts

Join Us on Telegram for More Such Books

https://telegram.me/unacademyplusdiscounts

Join us from the above link or search ''unacademyplusdiscounts'' in Telegram

You might also like